You are on page 1of 277

NGUY

N THUY

THANH
B
`
AI T

A
.
P
TO

AN CAO C

P
Ta
.
p 1
Da
.
i so

tuye

n tnh
v`a H`nh ho
.
c gia

i tch
NH
`
A XU

T BA

N DA
.
I HO
.
C QU

C GIA H
`
A N

O
.
I
H`a No
.
i 2006
Mu
.
c lu
.
c
L`o
.
i noi da
`
u . . . . . . . . . . . . . . . . . . . . . . . . . . 4
1 So

ph u
.
c 6
1.1 D
-
i
.
nh ngha so ph u
.
c . . . . . . . . . . . . . . . . . . . . 6
1.2 Da
.
ng da
.
i so cu a so ph u
.
c . . . . . . . . . . . . . . . . . 8
1.3 Bie

u die

n h`nh ho
.
c. Modun v`a acgumen . . . . . . . . 13
1.4 Bie

u die

n so ph u
.
c du
.
o
.
i da
.
ng lu
.
o
.
.
ng giac . . . . . . . . 23
2 D
-
a th u
.
c v`a h`am h u
.
u ty 44
2.1 D
-
a th u
.
c . . . . . . . . . . . . . . . . . . . . . . . . . . 44
2.1.1 D
-
a th u
.
c tren tru
.
`o
.
ng so ph u
.
c C . . . . . . . . . 45
2.1.2 D
-
a th u
.
c tren tru
.
`o
.
ng so thu
.
.
c R . . . . . . . . . 46
2.2 Phan th u
.
c h u
.
u ty . . . . . . . . . . . . . . . . . . . . . 55
3 Ma tr a
.
n. D
-
i
.
nh th u
.
c 66
3.1 Ma tra
.
n . . . . . . . . . . . . . . . . . . . . . . . . . . 67
3.1.1 D
-
i
.
nh ngha ma tra
.
n . . . . . . . . . . . . . . . . 67
3.1.2 C ac phep toan tuye

n tnh tren ma tra


.
n . . . . . 69
3.1.3 Phep nhan cac ma tra
.
n . . . . . . . . . . . . . . 71
3.1.4 Phep chuye

n vi
.
ma tra
.
n . . . . . . . . . . . . . 72
3.2 D
-
i
.
nh th u
.
c . . . . . . . . . . . . . . . . . . . . . . . . . 85
3.2.1 Nghi
.
ch the

. . . . . . . . . . . . . . . . . . . . . 85
3.2.2 D
-
i
.
nh th u
.
c . . . . . . . . . . . . . . . . . . . . . 85
3.2.3 Tnh chat cu a di
.
nh th u
.
c . . . . . . . . . . . . . 88
2 MU
.
C LU
.
C
3.2.4 Phu
.
o
.
ng phap tnh di
.
nh th u
.
c . . . . . . . . . . . 89
3.3 Ha
.
ng cu a ma tra
.
n . . . . . . . . . . . . . . . . . . . . . 109
3.3.1 D
-
i
.
nh ngha . . . . . . . . . . . . . . . . . . . . 109
3.3.2 Phu
.
o
.
ng phap t`m ha
.
ng cu a ma tra
.
n . . . . . . 109
3.4 Ma tra
.
n nghi
.
ch da o . . . . . . . . . . . . . . . . . . . . 118
3.4.1 D
-
i
.
nh ngha . . . . . . . . . . . . . . . . . . . . 118
3.4.2 Phu
.
o
.
ng phap t`m ma tra
.
n nghi
.
ch da o . . . . . 119
4 He
.
phu
.
o
.
ng tr`nh tuye

n tnh 132
4.1 He
.
n phu
.
o
.
ng tr`nh vo
.
i n a

n co di
.
nh th u
.
c khac 0 . . . . 132
4.1.1 Phu
.
o
.
ng phap ma tra
.
n . . . . . . . . . . . . . . 133
4.1.2 Phu
.
o
.
ng phap Cramer . . . . . . . . . . . . . . 134
4.1.3 Phu
.
o
.
ng phap Gauss . . . . . . . . . . . . . . . 134
4.2 He
.
t` uy y cac phu
.
o
.
ng tr`nh tuye

n tnh . . . . . . . . . . 143
4.3 He
.
phu
.
o
.
ng tr`nh tuye

n tnh thua
`
n nhat . . . . . . . . . 165
5 Khong gian Euclide R
n
177
5.1 D
-
i
.
nh ngha khong gian n-chie
`
u v`a mo
.
t so khai nie
.
m co
.
ba n ve
`
vecto
.
. . . . . . . . . . . . . . . . . . . . . . . . 177
5.2 Co
.
so
.
. D
-
o

i co
.
so
.
. . . . . . . . . . . . . . . . . . . . . 188
5.3 Khong gian vecto
.
Euclid. Co
.
so
.
tru
.
.
c chua

n . . . . . . 201
5.4 Phep bie

n do

i tuye

n tnh . . . . . . . . . . . . . . . . . 213
5.4.1 D
-
i
.
nh ngha . . . . . . . . . . . . . . . . . . . . 213
5.4.2 Ma tra
.
n cu a phep bdtt . . . . . . . . . . . . . . 213
5.4.3 C ac phep toan . . . . . . . . . . . . . . . . . . . 215
5.4.4 Vecto
.
rieng v`a gia tri
.
rieng . . . . . . . . . . . . 216
6 Da
.
ng to`an phu
.
o
.
ng v`a u
.
ng du
.
ng de

nha
.
n da
.
ng du
.
`o
.
ng
v`a m a
.
t ba
.
c hai 236
6.1 Da
.
ng to`an phu
.
o
.
ng . . . . . . . . . . . . . . . . . . . . 236
6.1.1 Phu
.
o
.
ng phap Lagrange . . . . . . . . . . . . . . 237
6.1.2 Phu
.
o
.
ng phap Jacobi . . . . . . . . . . . . . . . 241
MU
.
C LU
.
C 3
6.1.3 Phu
.
o
.
ng phap bie

n do

i tru
.
.
c giao . . . . . . . . . 244
6.2 D
-
u
.
a phu
.
o
.
ng tr`nh to

ng quat cu a du
.
`o
.
ng ba
.
c hai v` a ma
.
t
ba
.
c hai ve
`
da
.
ng chnh ta

c . . . . . . . . . . . . . . . . 263
L`o
.
i noi da
`
u
Giao tr`nh B`ai ta
.
p toan cao cap n`ay du
.
o
.
.
c bien soa
.
n theo Chu
.
o
.
ng
tr`nh Toan cao cap cho sinh vien cac ng` anh Khoa ho
.
c Tu
.
.
nhien cu a
Da
.
i ho
.
c Quoc gia H`a N o
.
i v`a da du
.
o
.
.
c Da
.
i ho
.
c Quoc gia H`a N o
.
i thong
qua v`a ban h`anh.
Mu
.
c dch cu a giao tr`nh l`a gi up do
.
sinh vien cac ng`anh Khoa ho
.
c
Tu
.
.
nhien na

m v u
.
ng v`a va
.
n du
.
ng du
.
o
.
.
c c ac phu
.
o
.
ng phap gia i toan cao
cap. Mu
.
c tieu n`ay quye

t di
.
nh to`an bo
.
cau tr uc cu a giao tr`nh. Trong
mo

i mu
.
c, da
`
u tien ch ung toi tr`nh b`ay tom ta

t nh u
.
ng co
.
so
.
l y thuye

t
v`a lie
.
t ke nh u
.
ng cong th u
.
c ca
`
n thie

t. Tie

p do, trong pha


`
n C ac v du
.
ch ung toi quan tam da
.
c bie
.
t to
.
i vie
.
c gia i cac b`ai toan ma

u ba
`
ng cach
va
.
n du
.
ng cac kie

n th u
.
c l y thuye

t da tr`nh b`ay. Sau c` ung, l`a pha


`
n B` ai
ta
.
p. O
.

day, c ac b`ai ta
.
p du
.
o
.
.
c go
.
p th`anh t` u
.
ng nhom theo t` u
.
ng chu de
`
v`a du
.
o
.
.
c sa

p xe

p theo th u
.
tu
.
.
tang da
`
n ve
`
do
.
kho v`a mo

i nhom de
`
u
co nh u
.
ng ch da

n ve
`
phu
.
o
.
ng phap gia i. Ch ung toi hy vo
.
ng ra
`
ng vie
.
c
l`am quen vo
.
i l`o
.
i gia i chi tie

t trong pha
`
n C ac v du
.
se gi up ngu
.
`o
.
i ho
.
c
na

m du
.
o
.
.
c c ac phu
.
o
.
ng phap gia i toan co
.
ba n.
Giao tr`nh B`ai ta
.
p n`ay co the

su
.
du
.
ng du
.
o
.
i su
.
.
hu
.
o
.
ng da

n cu a
giao vien hoa
.
c tu
.
.
m`nh nghien c u
.
u v` cac b`ai ta
.
p de
`
u co dap so, mo
.
t
so co ch da

n v`a tru
.
o
.
c khi gia i cac b`ai ta
.
p n`ay da co pha
`
n Cac v du
.
tr`nh b`ay nh u
.
ng ch da

n ve
`
ma
.
t phu
.
o
.
ng phap gia i toan.
Tac gia giao tr`nh chan th`anh ca m o
.
n cac tha
`
y giao: TS. Le D`nh
Ph` ung v`a PGS. TS. Nguye

n Minh Tuan da do
.
c k y ba n tha o v`a dong
Co
.
so
.
l y thuye

t h`am bie

n ph u
.
c 5
gop nhie
`
u y kie

n qu y bau ve
`
cau tr uc v`a no
.
i dung v`a da gop y cho tac
gia ve
`
nh u
.
ng thie

u sot cu a ba n tha o giao tr`nh.


Mo
.
i xuat ba n la
`
n da
`
u, Giao tr`nh kho tranh khoi sai sot. Ch ung
toi rat chan th`anh mong du
.
o
.
.
c ba
.
n do
.
c vui l`ong ch ba o cho nh u
.
ng
thie

u sot cu a cuon sach de

giao tr`nh ng`ay du


.
o
.
.
c ho`an thie
.
n ho
.
n.
H`a N o
.
i, M` ua thu 2004
Tac gia
Chu
.
o
.
ng 1
So

ph u
.
c
1.1 D
-
i
.
nh ngha so ph u
.
c . . . . . . . . . . . . . . 6
1.2 Da
.
ng da
.
i so cu a so ph u
.
c . . . . . . . . . . . 8
1.3 Bie

u die

n h`nh ho
.
c. Modun v`a acgumen . 13
1.4 Bie

u die

n so ph u
.
c du
.
o
.
i da
.
ng lu
.
o
.
.
ng giac . 23
1.1 D
-
i
.
nh ngha so

ph u
.
c
Mo

i ca
.
p so thu
.
.
c c o th u
.
tu
.
.
(a; b) a R, b R du
.
o
.
.
c go
.
i l`a mo
.
t so
ph u
.
c ne

u tren ta
.
p ho
.
.
p cac c a
.
p do quan he
.
ba
`
ng nhau, phep co
.
ng v`a
phep nhan du
.
o
.
.
c du
.
a v`ao theo cac di
.
nh ngha sau day:
(I) Quan he
.
ba
`
ng nhau
(a
1
, b
1
) = (a
2
, b
2
)
_
_
_
a
1
= a
2
,
b
1
= b
2
.
(II) Phep co
.
ng
1.1. D
-
i
.
nh ngha so ph u
.
c 7
(a
1
, b
1
) + (a
2
, b
2
)
def
= (a
1
+a
2
, b
1
+b
2
).
1
(III) Phep nhan
(a
1
, b
1
)(a
2
, b
2
)
def
= (a
1
a
2
b
1
b
2
, a
1
b
2
+a
2
b
1
).
Ta
.
p ho
.
.
p so ph u
.
c du
.
o
.
.
c k y hie
.
u l`a C. Phep co
.
ng (II) v`a phep nhan
(III) trong C co tnh chat giao hoan, ke

t ho
.
.
p, lien he
.
vo
.
i nhau bo
.
i
lua
.
t ph an bo v`a mo
.
i pha
`
n tu
.
,= (0, 0) de
`
u c o pha
`
n tu
.
nghi
.
ch da o.
Ta
.
p ho
.
.
p C la
.
p th`anh mo
.
t tru
.
`o
.
ng (go
.
i l`a tru
.
`o
.
ng so ph u
.
c) vo
.
i pha
`
n
tu
.
khong l`a ca
.
p (0; 0) v`a pha
`
n tu
.
do
.
n vi
.
l`a ca
.
p (1; 0).

Ap du
.
ng quy
ta

c (III) ta c o: (0; 1)(0; 1) = (1, 0). Ne

u k y hie
.
u i = (0, 1) th`
i
2
= 1
Doi vo
.
i cac c a
.
p da
.
ng da
.
c bie
.
t (a, 0), a R theo (II) v`a (III) ta
co
(a, 0) + (b, 0) = (a +b, 0),
(a, 0)(b, 0) = (ab, 0).
T` u
.
do ve
`
ma
.
t da
.
i so cac ca
.
p da
.
ng (a, 0), a R khong co g` khac bie
.
t
vo
.
i so thu
.
.
c R: v` ch ung du
.
o
.
.
c co
.
ng v`a nhan nhu
.
nh u
.
ng so thu
.
.
c. Do
va
.
y ta c o the

do
`
ng nhat c ac ca
.
p da
.
ng (a; 0) vo
.
i so thu
.
.
c a:
(a; 0) a a R.
Da
.
c bie
.
t l`a (0; 0) 0; (1; 0) 1.
Doi vo
.
i so ph u
.
c z = (a, b):
1
+
So thu
.
.
c a du
.
o
.
.
c go
.
i l`a pha
`
n thu
.
.
c a = Re z, so thu
.
.
c b go
.
i l`a pha
`
n
a o v`a k y hie
.
u l`a b = Imz.
2
+
So ph u
.
c z = (a, b) go
.
i l`a so ph u
.
c lien ho
.
.
p vo
.
i so ph u
.
c z
1
def. l`a c ach vie

t ta

t cu a t` u
.
tie

ng Anh denition (di


.
nh ngha)
8 Chu
.
o
.
ng 1. So ph u
.
c
1.2 Da
.
ng da
.
i so

cu

a so

ph u
.
c
Mo
.
i so ph u
.
c z = (a; b) C de
`
u co the

vie

t du
.
o
.
i da
.
ng
z = a +ib. (1.1)
Tha
.
t va
.
y, z = (a, b) = (a, 0) + (0, b) = (a, 0) + (0, 1)(b, 0) = a +ib
Bie

u th u
.
c (1.1) go
.
i l`a da
.
ng da
.
i so cu a so ph u
.
c z = (a, b). T` u
.
(1.1)
v`a di
.
nh ngha so ph u
.
c lien ho
.
.
p ta c o z = a ib.
Du
.
o
.
i da
.
ng da
.
i so cac phep tnh tren ta
.
p ho
.
.
p so ph u
.
c du
.
o
.
.
c thu
.
.
c
hie
.
n theo cac quy ta

c sau.
Gia su
.
z
1
= a
1
+ib
1
, z
2
= a
2
+ib
2
. Khi do
(I) Phep c o
.
ng: z
1
z
2
= (a
1
a
2
) +i(b
1
b
2
).
(II) Phep nhan: z
1
z
2
= (a
1
a
2
b
1
b
2
) +i(a
1
b
2
+a
2
b
1
).
(III) Phep chia:
z
2
z
1
=
a
1
a
2
+b
1
b
2
a
2
1
+b
2
1
+i
a
1
b
2
a
2
b
1
a
2
1
+b
2
1

AC V

I DU
.
V du
.
1. 1
+
Tnh i
n
. T` u
.
do ch u
.
ng minh ra
`
ng
a) i
n
+i
n+1
+i
n+2
+i
n+3
= 0;
b) i i
2
i
99
i
100
= 1.
2
+
T`m so nguyen n ne

u:
a) (1 +i)
n
= (1 i)
n
;
b)
_
1 +i

2
_
n
+
_
1 i

2
_
n
= 0.
Gia i. 1
+
Ta co i
0
= 1, i
1
= i, i
2
= 1, i
3
= i, i
4
= 1, i
5
= i v`a
gia tri
.
l uy th` u
.
a ba

t da
`
u la
.
p la
.
i. Ta khai quat hoa. Gia su
.
n Z v`a
n = 4k +r, r Z, 0 r 3. Khi do
i
n
= i
4k+r
= i
4k
i
r
= (i
4
)
k
i
r
= i
r
1.2. Da
.
ng da
.
i so cu a so ph u
.
c 9
(v` i
4
= i). T` u
.
do, theo ke

t qua tren ta co
i
n
=
_

_
1 ne

u n = 4k,
i ne

u n = 4k + 1,
1 ne

u n = 4k + 2,
i ne

u n = 4k + 3.
(1.2)
T` u
.
(1.2) de

d`ang suy ra a) v`a b).


2
+
a) T` u
.
he
.
th u
.
c (1 +i)
n
= (1 i)
n
suy ra
_
1 +i
1 i
_
n
= 1.
Nhu
.
ng
1 +i
1 i
= i nen
_
1 +i
1 i
_
n
= i
n
= 1 n = 4k, k Z.
b) T` u
.
da

ng th u
.
c
_
1 +i

2
_
n
+
_
1 i

2
_
n
= 0 suy ra
`
ng
_
1 +i
1 i
_
n
= 1
v`a do do i
n
= 1 n = 4k + 2, k Z.
V du
.
2. Ch u
.
ng minh ra
`
ng ne

u n l`a bo
.
i cu a 3 th`
_
1 +i

3
2
_
n
+
_
1 i

3
2
_
n
= 2
v`a ne

u n khong chia he

t cho 3 th`
_
1 +i

3
2
_
n
+
_
1 i

3
2
_
n
= 1.
Gia i. 1
+
Ne

u n = 3m th`
S =
__
1 +i

3
2
_
3
_
m
+
__
1 i

3
2
_
3
_
m
=
_
1 + 3i

3 + 9 3i

3
8
_
m
+
_
1 3i

3 + 9 + 3i

3
8
_
m
= 1
m
+ 1
m
= 2.
10 Chu
.
o
.
ng 1. So ph u
.
c
2
+
Ne

u n = 3m+ 1 th`
S =
__
1 +i

3
2
_
3
_
m
_
1 +i

3
2
_
+
__
1 i

3
2
_
3
_
m
_
1 i

3
2
_
=
1 +i

3
2
+
1 i

3
2
= 1.
Tu
.
o
.
ng tu
.
.
ne

u n = 3m+ 2 ta c ung co S = 1.
V du
.
3. Tnh bie

u th u
.
c
=
_
1 +
1 +i
2
__
1 +
_
1 +i
2
_
2
__
1 +
_
1 +i
2
_
2
2_

_
1 +
_
1 +i
2
_
2
n_
.
Gia i. Nhan v`a chia bie

u th u
.
c da cho vo
.
i 1
1 +i
2
ta co
=
1
__
1 +i
2
_
2
n_
2
1
1 +i
2
=
1
_
1 +i
2
_
2
n+1
1
1 +i
2

Ta ca
`
n tnh
_
1 +i
2
_
2
n+1
=
__
1 +i
2
_
2
_
2
n
=
_
i
2
_
2
n
=
i
2
n
2
2
n
=
1
2
2
n

Do do
=
1
1
2
2
n
1
1 +i
2
=
2
_
1
1
2
2
n
_
1 i

1 +i
1 +i
=
_
1
1
2
2
n
_
(1 +i)
V du
.
4. Bie

u die

n so ph u
.
c

4 3i du
.
o
.
i da
.
ng da
.
i so.
Gia i. Theo di
.
nh ngha ta c a
`
n t`m so ph u
.
c w sao cho w
2
= 4 3i.
Ne

u w = a +bi, a, b R th`
4 3i = (a +bi)
2
= a
2
b
2
+ 2abi.
1.2. Da
.
ng da
.
i so cu a so ph u
.
c 11
T` u
.
do
a
2
b
2
= 4, (1.3)
2ab = 3. (1.4)
T` u
.
(1.4) ta co b =
3
2a
. The

v`ao (1.3) ta thu du


.
o
.
.
c
4u
2
16u 9 = 0, u = a
2

_
u
1
=
8 +

100
4
=
8 + 10
4
=
18
4
=
9
2
,
u
2
=
8

100
4
=
8 10
4
=
1
2

V` a R nen u 0 u =
9
2
v`a do va
.
y
a =
3

2
b =
1

T` u
.
do ta thu du
.
o
.
.
c
w
1,2
=
_
3

2
i
_

V du
.
5. Bie

u die

n so ph u
.
c
z =

5 + 12i

5 12i

5 + 12i +

5 12i
vo
.
i die
`
u kie
.
n l`a cac pha
`
n thu
.
.
c cu a

5 + 12i v`a

5 12i de
`
u am.
Gia i.

Ap du
.
ng phu
.
o
.
ng phap gia i trong v du
.
4 ta co

5 + 12i = x +iy 5 + 12i = x


2
y
2
2xyi

_
_
_
x
2
y
2
= 5,
2xy = 12.
12 Chu
.
o
.
ng 1. So ph u
.
c
He
.
n`ay co hai nghie
.
m l`a (3; 2) v`a (3; 2). Theo die
`
u kie
.
n, pha
`
n
thu
.
.
c cu a

5 + 12i am nen ta co

5 + 12i = 3 2i. Tu
.
o
.
ng tu
.
.
ta
t`m du
.
o
.
.
c

5 12i = 3 + 2i. Nhu


.
va
.
y
z =
3 2i (3 + 2i)
3 2i + (3 + 2i)
=
2
3
i
V du
.
6. Gia su
.
z = a +ib, z = 1. Ch u
.
ng minh ra
`
ng w =
z 1
z + 1
l`a
so thua
`
n a o khi v`a ch khi a
2
+b
2
= 1.
Gia i. Ta co
w =
(a 1) +ib
(a + 1) +ib
=
a
2
+b
2
1
(a + 1)
2
+b
2
+i
2b
(a + 1)
2
+b
2

T` u
.
do suy ra
`
ng w thua
`
n a o khi v`a ch khi
a
2
+b
2
1
(a + 1)
2
+b
2
= 0 a
2
+b
2
= 1.
B
`
AI T

A
.
P
Tnh
1.
(1 +i)
8
1
(1 i)
8
+ 1
(DS.
15
17
)
2.
(1 + 2i)
3
+ (1 2i)
3
(2 i)
2
(2 +i)
2
(DS.
11
4
i)
3.
(3 4i)(2 i)
2 +i

(3 + 4i)(2 +i)
2 i
(DS.
14
5
)
4.
_
1 +
1 i

2
__
1 +
_
1 i

2
_
2
__
1 +
_
1 i

2
_
2
2_

_
1 +
_
1 i

2
_
2
n_
.
(DS. 0)
Ch da

n.

Ap du
.
ng cach gia i v du
.
3.
5. Ch u
.
ng minh ra
`
ng
a) z
1
+z
2
= z
1
+z
2
; b) z
1
z
2
= z
1
z
2
; c)
_
z
1
z
2
_
=
z
1
z
2
;
1.3. Bie

u die

n h`nh ho
.
c. Modun v`a acgumen 13
d) z
n
= (z)
n
; e) z +z = 2Re z; g) z z = 2Imz.
6. Vo
.
i gia tri
.
thu
.
.
c n`ao cu a x v`a y th` cac ca
.
p so sau day l`a cac c a
.
p
so ph u
.
c lien ho
.
.
p:
1) y
2
2y +xy x +y + (x +y)i v`a y
2
+ 2y + 11 4i;
2) x +y
2
+ 1 + 4i v`a ixy
2
+iy
2
3 ?
(DS. 1) x
1
= 1, y
1
= 3; x
2
= 9, y
2
= 5; 2) x
1,2
= 5, y
1,2
= 5)
7. Ch u
.
ng minh ra
`
ng z
1
v`a z
2
l`a nh u
.
ng so ph u
.
c lien ho
.
.
p khi v`a ch
khi z
1
+z
2
v`a z
1
z
2
l`a nh u
.
ng so thu
.
.
c.
8. Tnh:
1)

5 12i. (DS. (2 3i))


2)

24 + 10i. (DS. (5 +i))


3)

24 10i. (DS. (5 i))


4)
_
1 +i

3 +
_
1 i

3. (DS.

6, i

2)
9. Ch u
.
ng minh ra
`
ng
1) 1 C
2
8
+C
4
8
C
6
8
+C
8
8
= 16;
2) 1 C
2
9
+C
4
9
C
6
9
+C
8
9
= 16;
3) C
1
9
C
3
9
+C
5
9
C
7
9
+C
9
9
= 16.
Ch da

n.

Ap du
.
ng cong th u
.
c nhi
.
th u
.
c Newton doi vo
.
i (1 +i)
8
v`a
(1 +i)
9
.
1.3 Bie

u die

n h`nh ho
.
c. Modun v`a acgu-
men
Mo

i so ph u
.
c z = a + ib co the

da
.
t tu
.
o
.
ng u
.
ng vo
.
i die

m M(a; b) cu a
ma
.
t pha

ng to
.
a do
.
v`a ngu
.
o
.
.
c la
.
i mo

i die

m M(a; b) cu a ma
.
t pha

ng de
`
u
tu
.
o
.
ng u
.
ng vo
.
i so ph u
.
c z = a + ib. Phep tu
.
o
.
ng u
.
ng du
.
o
.
.
c xac la
.
p l`a
do
.
n tri
.
mo
.
t - mo
.
t. Phep tu
.
o
.
ng u
.
ng do cho phep ta xem cac so ph u
.
c
nhu
.
l`a cac die

m cu a ma
.
t pha

ng to
.
a do
.
. Ma
.
t pha

ng do du
.
o
.
.
c go
.
i l`a
ma
.
t pha

ng ph u
.
c. Tru
.
c ho`anh cu a no du
.
o
.
.
c go
.
i l`a Tru
.
c thu
.
.
c, tru
.
c tung
14 Chu
.
o
.
ng 1. So ph u
.
c
du
.
o
.
.
c go
.
i l`a Tru
.
c a o. Thong thu
.
`o
.
ng so ph u
.
c z = a + ib co the

xem
nhu
.
vecto
.

OM. Mo

i vecto
.
cu a ma
.
t pha

ng vo
.
i die

m da
`
u O(0, 0) v`a
die

m cuoi ta
.
i die

m M(a; b) de
`
u tu
.
o
.
ng u
.
ng vo
.
i so ph u
.
c z = a + ib v`a
ngu
.
o
.
.
c la
.
i.
Su
.
.
tu
.
o
.
ng u
.
ng du
.
o
.
.
c xac la
.
p gi u
.
a ta
.
p ho
.
.
p so ph u
.
c C vo
.
i ta
.
p ho
.
.
p
cac die

m hay cac vecto


.
ma
.
t pha

ng cho phep go
.
i cac so ph u
.
c l`a die

m
hay vecto
.
.
Vo
.
i phep bie

u die

n h`nh ho
.
c so ph u
.
c, cac phep toan c o
.
ng v`a tr` u
.
cac so ph u
.
c du
.
o
.
.
c thu
.
.
c hie
.
n theo quy ta

c c o
.
ng v`a tr` u
.
cac vecto
.
.
Gia su
.
z C. Khi do do
.
d`ai cu a vecto
.
tu
.
o
.
ng u
.
ng vo
.
i so ph u
.
c z
du
.
o
.
.
c go
.
i l`a modun cu a no.
Ne

u z = a +ib th`
r = [z[ =

a
2
+b
2
=

z z.
Goc gi u
.
a hu
.
o
.
ng du
.
o
.
ng cu a tru
.
c thu
.
.
c v`a vecto
.
z (du
.
o
.
.
c xem l`a goc
du
.
o
.
ng ne

u no c o di
.
nh hu
.
o
.
ng ngu
.
o
.
.
c chie
`
u kim do
`
ng ho
`
) du
.
o
.
.
c go
.
i l`a
acgumen cu a so z ,= 0. Doi vo
.
i so z = 0 acgumen khong xac di
.
nh.
Khac vo
.
i modun, acgumen cu a so ph u
.
c xac di
.
nh khong do
.
n tri
.
, no
xac di
.
nh vo
.
i su
.
.
sai khac mo
.
t so ha
.
ng bo
.
i nguyen cu a 2 v`a
Arg z = arg z + 2k, k Z,
trong do arg z l`a gia tri
.
chnh cu a acgumen du
.
o
.
.
c xac di
.
nh bo
.
i die
`
u
kie
.
n < arg z hoa
.
c 0 arg z < 2.
Pha
`
n thu
.
.
c v`a pha
`
n a o cu a so ph u
.
c z = a +ib du
.
o
.
.
c bie

u die

n qua
modun v`a acgument cu a no nhu
.
sau
_
_
_
a = r cos ,
y = r sin.
1.3. Bie

u die

n h`nh ho
.
c. Modun v`a acgumen 15
Nhu
.
va
.
y, acgumen cu a so ph u
.
c co the

t`m t` u
.
he
.
phu
.
o
.
ng tr`nh
_

_
cos =
a

a
2
+b
2
,
sin =
b

a
2
+b
2

AC V

I DU
.
V du
.
1. T`m modun cu a so z =
x
2
y
2
+ 2xyi
xy

2 +i
_
x
4
+y
4

Gia i. Ta co
[z[ =
_
(x
2
y
2
)
2
+ (2xy)
2
_
(xy

2)
2
+ (
_
x
4
+y
4
)
2
=
x
2
+y
2
x
2
+y
2
= 1.
V du
.
2. Ch u
.
ng minh ra
`
ng z
1
, z
2
C ta de
`
u co:
(i) [z
1
+z
2
[ [z
1
[ +[z
2
[; (ii) [z
1
z
2
[ [z
1
[ +[z
2
[;
(iii) [z
1
+z
2
[ [z
1
[ [z
2
[; (iv) z
1
z
2
[ [z
1
[ [z
2
.
Gia i. (i) Ta co
[z
1
+z
2
[
2
= (z
1
+z
2
)(z
1
+z
2
) = [z
1
[
2
+[z
2
[
2
+ 2Re(z
1
z
2
).
V` [z
1
z
2
[ Re(z
1
z
2
) [z
1
z
2
[ nen
[z
1
+z
2
[
2
[z
1
[
2
+[z
2
[
2
+ 2[z
1
[[z
2
[ = ([z
1
[ +[z
2
[)
2
[z
1
+z
2
[ [z
1
[ +[z
2
[.
(ii) V` [z
2
[ = [ z
2
[ nen
[z
1
z
2
[ = [z
1
+ (z
2
)[ [z
1
[ +[ z
2
[ = [z
1
[ +[z
2
[.
(iii)

Ap du
.
ng (ii) cho z
1
= (z
1
+z
2
) z
2
v`a thu du
.
o
.
.
c
[z
1
[ [z
1
+z
2
[ +[z
2
[ [z
1
+z
2
[ [z
1
[ [z
2
[.
16 Chu
.
o
.
ng 1. So ph u
.
c
(iv) [z
1
z
2
[ = [z
1
+ (z
2
)[ [z
1
[ [ z
2
[ = [z
1
[ [z
2
[.
Nha
.
n xet. Cac bat da

ng th u
.
c (iii) v`a (iv) c`on c o the

vie

t du
.
o
.
i
da
.
ng
(iii)

. [z
1
+z
2
[

[z
1
[ [z
2
[

; (iv)

. [z
1
z
2
[

[z
1
[ [z
2
[

.
Tha
.
t va
.
y ta c o [z
1
+z
2
[ [z
1
[ [z
2
[ v`a [z
1
+z
2
[ [z
2
[ [z
1
[. C ac
ve

pha i khac nhau ve


`
dau do do ne

u lay ve

pha i du
.
o
.
ng th` thu du
.
o
.
.
c
(iii)

. Bat da

ng th u
.
c (iv)

thu du
.
o
.
.
c t` u
.
(iii)

ba
`
ng cach thay z
2
bo
.
i
z
2
.
V du
.
3. Ch u
.
ng minh do
`
ng nhat th u
.
c
[z
1
+z
2
[
2
+[z
1
z
2
[
2
= 2([z
1
[
2
+[z
2
[
2
).
Gia i thch y ngha h`nh ho
.
c cu a he
.
th u
.
c da ch u
.
ng minh.
Gia i. Gia su
.
z
1
= x
1
+iy
1
, z
2
= x
2
+iy
2
. Khi do
z
1
+z
2
= x
1
+x
2
+i(y
1
+y
2
),
z
1
z
2
= x
1
x
2
+i(y
1
y
2
),
[z
1
+z
2
[
2
= (x
1
+x
2
)
2
+ (y
1
+y
2
)
2
,
[z
1
z
2
[
2
= (x
1
x
2
)
2
+ (y
1
y
2
)
2
.
T` u
.
do thu du
.
o
.
.
c
[z
1
+z
2
[
2
+[z
1
z
2
[
2
= 2(x
2
1
+y
1
)
2
+ 2(x
2
2
+y
2
2
) = 2([z
1
[
2
+[z
2
[
2
).
T` u
.
he
.
th u
.
c da ch u
.
ng minh suy ra
`
ng trong mo

i h`nh b`nh h`anh to

ng c ac
b`nh phu
.
o
.
ng do
.
d`ai cu a cac du
.
`o
.
ng cheo ba
`
ng to

ng cac b`nh phu


.
o
.
ng
do
.
d`ai cu a cac ca
.
nh cu a no.
V du
.
4. Ch u
.
ng minh ra
`
ng ne

u [z
1
[ = [z
2
[ = [z
3
[ th`
arg
z
3
z
2
z
3
z
1
=
1
2
arg
z
2
z
1

Gia i. Theo gia thie

t, c ac die

m z
1
, z
2
v`a z
3
na
`
m tren du
.
`o
.
ng tr`on
n`ao do vo
.
i tam ta
.
i goc to
.
a do
.
. Ta xet cac vecto
.
z
3
z
2
; z
3
z
1
, z
1
v`a
z
2
(hay ve h`nh).
1.3. Bie

u die

n h`nh ho
.
c. Modun v`a acgumen 17
Ba
`
ng nh u
.
ng nguyen do h`nh ho
.
c, de

thay ra
`
ng
arg
z
3
z
2
z
3
z
1
= arg(z
3
z
2
) arg(z
3
z
1
)
v`a goc n`ay nh`n cung tr`on noi die

m z
1
v`a z
2
v`a goc o
.
tam
arg
z
2
z
1
= argz
2
argz
1
c ung cha

n chnh cung tr`on do. Theo di


.
nh l y quen thuo
.
c cu a h`nh ho
.
c
so
.
cap ta c o
arg
z
3
z
2
z
3
z
1
=
1
2
arg
z
2
z
1

V du
.
5. Ch u
.
ng minh ra
`
ng ne

u [z
1
[ = [z
2
[ = [z
3
[ = 1 v`a z
1
+z
2
+z
3
= 0
th` c ac die

m z
1
, z
2
v`a z
3
l`a cac d nh cu a tam giac de
`
u no
.
i tie

p trong
du
.
`o
.
ng tr`on do
.
n vi
.
.
Gia i. Theo gia thie

t, ba die

m z
1
, z
2
v`a z
3
na
`
m tren du
.
`o
.
ng tr`on
do
.
n vi
.
. Ta t`m do
.
d`ai cu a cac ca
.
nh tam giac.
1
+
T`m do
.
d`ai [z
1
z
2
[. Ta co
[z
1
z
2
[
2
= (x
1
x
2
)
2
+ (y
1
y
2
)
2
= x
2
1
+y
2
1
+x
2
2
+y
2
2
(2x
1
x
2
+ 2y
1
y
2
)
= 2(x
2
1
+y
2
1
) + 2(x
2
2
+y
2
2
) [(x
1
+x
2
)
2
+ (y
1
+y
2
)
2
]
= 2[z
1
[
2
+ 2[z
2
[
2
2[z
1
+z
2
[
2
.
Nhu
.
ng z
1
+z
2
= z
3
v`a [z
1
+z
2
[ = [z
3
[. Do do
[z
1
z
2
[
2
= 2[z
1
[
2
+ 2[z
2
[
2
[z
3
[
2
= 2 1 + 2 1 1 = 3
v`a t` u
.
do
[z
1
z
2
[ =

3 .
2
+
Tu
.
o
.
ng tu
.
.
ta c o [z
2
z
3
[ =

3, [z
3
z
1
[ =

3. T` u
.
do suy ra
tam giac vo
.
i d nh z
1
, z
2
, z
3
l`a tam gi ac de
`
u.
18 Chu
.
o
.
ng 1. So ph u
.
c
V du
.
6. Vo
.
i die
`
u kie
.
n n`ao th` ba die

m khac nhau t` u
.
ng doi mo
.
t z
1
,
z
2
, z
3
na
`
m tren mo
.
t du
.
`o
.
ng tha

ng.
Gia i. 1
+
Ne

u cac die

m z
1
, z
2
, z
3
na
`
m tren du
.
`o
.
ng tha

ng cho tru
.
o
.
c
th` vecto
.
di t` u
.
z
2
de

n z
1
co hu
.
o
.
ng nhu
.
cu a vecto
.
di t` u
.
die

m z
3
de

n
z
1
hoa
.
c c o hu
.
o
.
ng ngu
.
o
.
.
c la
.
i. Die
`
u do co ngha l`a cac goc nghieng cu a
cac vecto
.
n`ay doi vo
.
i tru
.
c thu
.
.
c hoa
.
c nhu
.
nhau hoa
.
c sai khac goc .
Nhu
.
ng khi do ta co
arg(z
1
z
2
) = arg(z
1
z
3
) +k, k = 0, 1.
T` u
.
do suy ra
arg
z
1
z
2
z
1
z
3
= arg(z
1
z
2
) arg(z
1
z
3
) = k, k = 0, 1.
Nhu
.
va
.
y so ph u
.
c
z
1
z
2
z
1
z
3
co acgumen ba
`
ng 0 hoa
.
c ba
`
ng , t u
.
c l`a so
z
1
z
2
z
1
z
3
l`a so thu
.
.
c. Die
`
u kie
.
n thu du
.
o
.
.
c l`a die
`
u kie
.
n ca
`
n.
2
+
Ta ch u
.
ng minh ra
`
ng do c ung l`a die
`
u kie
.
n du . Gia su
.

z
1
z
2
z
1
z
3
= , R.
Khi do Im
z
1
z
2
z
1
z
3
= 0. He
.
th u
.
c n`ay tu
.
o
.
ng du
.
o
.
ng vo
.
i he
.
th u
.
c
y
1
y
3
y
1
y
2
=
x
1
x
3
x
1
x
2
(1.5)
Phu
.
o
.
ng tr`nh du
.
`o
.
ng th a

ng qua die

m (x
1
, y
1
) v`a (x
2
, y
2
) c o da
.
ng
y y
1
y
2
y
1
=
x x
1
x
2
x
1
(1.6)
T` u
.
(1.5) v`a (1.6) suy ra die

m (x
3
, y
3
) na
`
m tren du
.
`o
.
ng tha

ng do.
V du
.
7. Xac di
.
nh ta
.
p ho
.
.
p die

m tren ma
.
t pha

ng ph u
.
c thoa man cac
die
`
u kie
.
n:
1.3. Bie

u die

n h`nh ho
.
c. Modun v`a acgumen 19
1) [z 2[ +[z + 2[ = 5;
2) [z 2[ [z + 2[ > 3;
3) Re z c;
4) Imz < 0.
Gia i. 1) Da

ng th u
.
c [z 2[ + [z + 2[ = 5 xac di
.
nh qu y tch nh u
.
ng
die

m cu a ma
.
t pha

ng m`a to

ng khoa ng cach t` u
.
do de

n hai die

m cho
tru
.
o
.
c F
1
= 2 v`a F
2
= +2 l`a ha
`
ng so ba
`
ng 5. Theo di
.
nh ngha trong
h`nh ho
.
c gia i tch do l`a du
.
`o
.
ng ellip vo
.
i ban tru
.
c lo
.
n ba
`
ng
5
2
v`a tieu
die

m 2.
2) Qu y tch cac die

m cu a ma
.
t pha

ng C thoa man die


`
u kie
.
n

[z 2[ [z + 2[

= 3 l`a du
.
`o
.
ng hypecb on. Da

ng th u
.
c
[z 2[ [z + 2[ = 3
xac di
.
nh nhanh ben trai cu a du
.
`o
.
ng hypecb on v`a bat da

ng th u
.
c
[z 2[ [z + 2[ > 3 xac di
.
nh ph a
`
n trong cu a nhanh do.
3) Rez c x c. Do l`a nu
.
a ma
.
t pha

ng ben pha i du
.
`o
.
ng tha

ng
x = c (ke

ca du
.
`o
.
ng tha

ng x = c).
4) V` Imz = y Imz < c y < c. Do l`a nu
.
a ma
.
t pha

ng du
.
o
.
i
du
.
`o
.
ng tha

ng y = c (khong ke

du
.
`o
.
ng tha

ng do).
V du
.
8. Xac di
.
nh ta
.
p ho
.
.
p die

m tren ma
.
t pha

ng ph u
.
c C du
.
o
.
.
c cho
bo
.
i die
`
u kie
.
n:
1) [z[ = Rez + 1;
2) [z 1[ 2[z i[;
3) [z 2 +i[u
2
2[z 2 +i[u + 1 > 0 u R.
4) log
3
(2 +[z
2
+i[) + log
27
1
(2 +[z
2
i[)
3
= 0.
Gia i. 1) Gia su
.
z = x +iy. Khi do t` u
.
die
`
u kie
.
n
[z[ = Rez + 1
_
x
2
+y
2
= x + 1 y
2
= 2x + 1.
20 Chu
.
o
.
ng 1. So ph u
.
c
Do l`a phu
.
o
.
ng tr`nh parabon vo
.
i d nh ta
.
i die

m
_

1
2
; 0
_
vo
.
i tru
.
c doi
x u
.
ng l`a tia
=
_
(x, y) R
2
: x
1
2
, y = 0
_
.
2) Gia su
.
z = x +iy. Khi do t` u
.
die
`
u kie
.
n da cho suy ra:
[x 1 +iy[ 2[x +i(y 1)[

_
(x 1)
2
+y
2
2
_
x
2
+ (y 1)
2

_
x +
1
3
_
2
+
_
y
4
3
_
2

8
9

T` u
.
do suy ra ra
`
ng die
`
u kie
.
n da cho xac di
.
nh h`nh tr`on tamz
0
=
1
3
+i
4
3
v`a ban knh
2

2
3
.
3) V` tam th u
.
c ba
.
c hai (doi vo
.
i u) o
.
ve

trai cu a die
`
u kie
.
n da cho
du
.
o
.
ng u R nen bie
.
t so cu a no am, t u
.
c l`a
[z 2 +i[
2
[z 2 +i[ < 0
[z 2 +i[ < 1.
Do l`a h`nh tr`on vo
.
i tam ta
.
i z
0
= 2 i v`a ban knh ba
`
ng 1.
4) T` u
.
die
`
u kie
.
n da cho ta thu du
.
o
.
.
c
log
3
2 +[z
2
+i[
2 +[z
2
i[
= 0

2 +[z
2
+i[
2 +[z
2
i[
= 1 v`a [z
2
+i[ = [z
2
i[.
T` u
.
do suy ra
`
ng z
2
l`a so thu
.
.
c bat k` y. Nhu
.
ng khi do z l`a so thu
.
.
c bat
k` y hoa
.
c so thua
`
n a o bat k` y. Nhu
.
va
.
y ch co cac die

m na
`
m tren cac
tru
.
c to
.
a do
.
l`a thoa man die
`
u kie
.
n da cho.
B
`
AI T

A
.
P
1.3. Bie

u die

n h`nh ho
.
c. Modun v`a acgumen 21
1. Ch u
.
ng minh ra
`
ng
1) [z
1
z
2
[ = [z
1
[ [z
2
[;
2) [z
1
z
2
[ [z
1
[ +[z
2
[;
3) [z
1
z
2
[

[z
1
[ [z
2
[

.
2. Xuat phat t` u
.
cac bie

u die

n h`nh ho
.
c, ch u
.
ng minh:
1)

z
[z[
1

[argz[;
2) [z 1[

[z[ 1

+[z[[argz[.
3. Ch u
.
ng minh ra
`
ng ne

u gia tri
.
chnh argz = arg(a + ib) thoa man
die
`
u kie
.
n < argz th` no du
.
o
.
.
c tnh theo c ong th u
.
c
arg(a +ib) =
_

_
arctg
b
a
ne

u a > 0,
arctg
b
a
+ ne

u a < 0, b 0,
arctg
b
a
ne

u a < 0, b < 0.
4. Ch u
.
ng minh ra
`
ng ne

u gia tri
.
chnh arg(a +ib) thoa man die
`
u kie
.
n
0 arg(a +ib) < 2 th`
arg(a +ib) =
_

_
arctg
b
a
ne

u a > 0, b > 0,
arctg
b
a
+ 2 ne

u a > 0, b < 0,
arctg
b
a
+ ne

u a < 0.
Ch da

n. Lu
.
u y ra
`
ng gia tri
.
chnh cu a arctg
b
a

_


2
,

2
_
.
5. Ch u
.
ng minh ra
`
ng [a +b[
2
+[a b[
2
= 4[a[
2
ne

u [a[ = [b[.
6. Ch u
.
ng minh do
`
ng nhat th u
.
c
[1 ab[
2
[a b[
2
= (1 +[ab[)
2
([a[ +[b[)
2
, a C, b C.
Ch da

n. Su
.
du
.
ng he
.
th u
.
c [z[
2
= zz.
22 Chu
.
o
.
ng 1. So ph u
.
c
7. Ch u
.
ng minh do
`
ng nhat th u
.
c
1) [a +b[
2
= ([a[ +[b[)
2
2
_
[ab[ Re(ab)

.
2) [ab + 1[
2
+[a b[
2
= ([a[
2
+ 1)([b[
2
+ 1).
8. Ch u
.
ng minh ra
`
ng mo
.
i so ph u
.
c z ,= 1 v`a [z[ = 1 de
`
u co the

bie

u
die

n du
.
o
.
i da
.
ng
z =
1 +ti
1 ti
, t R.
Ch da

n. Bie

u die

n t qua z v`a ch u
.
ng minh t = t.
9. Ch u
.
ng minh ra
`
ng ne

u Rea 0 th` [1 +a[


1 +[a[

Ch da

n. Co the

ch u
.
ng minh ba
`
ng pha n ch u
.
ng.
10. Trong c ac so ph u
.
c thoa man die
`
u kie
.
n
[z 25i[ 15
hay t`m so co acgument du
.
o
.
ng nho nhat.
11. T`m acgumen cu a cac so ph u
.
c sau day
1) cos

6
i sin

6
(DS.

6
)
2) cos

3
+i sin

3
(DS.
2
3
)
3) cos i sin. (DS. )
4) cos i sin. (DS. +)
5) sin +i cos . (DS.

2
)
6) sin i cos . (DS.

2
)
7) sin i cos . (DS.
_


2

_
)
1.4. Bie

u die

n so ph u
.
c du
.
o
.
i da
.
ng lu
.
o
.
.
ng giac 23
1.4 Bie

u die

n so

ph u
.
c du
.
o
.
i da
.
ng lu
.
o
.
.
ng
giac
Mo
.
i so ph u
.
c z = a +ib ,= 0 de
`
u bie

u die

n du
.
o
.
.
c du
.
o
.
i da
.
ng
z = a +ib = r(cos +i sin) (1.7)
trong do r = [z[ =

a
2
+b
2
, l` a mo
.
t trong cac acgumen cu a no.
Phep bie

u die

n do du
.
o
.
.
c go
.
i l` a da
.
ng lu
.
o
.
.
ng giac cu a so ph u
.
c z. De

chuye

n t` u
.
da
.
ng da
.
i so sang da
.
ng lu
.
o
.
.
ng giac ta ch ca
`
n t`m modun
v`a mo
.
t trong c ac acgument cu a no. V` modun v`a acgumen cu a to

ng
(hie
.
u) hai so ph u
.
c kho co the

bie

u die

n qua modun v`a acgumen cu a


cac so ha
.
ng nen phep c o
.
ng v`a phep tr` u
.
du
.
o
.
i da
.
ng lu
.
o
.
.
ng giac l`a khong
kha thi. Ngu
.
o
.
.
c la
.
i, phep nhan, phep chia, phep nang len l uy th` u
.
a v`a
khai can du
.
o
.
.
c thu
.
.
c hie
.
n rat tie
.
n lo
.
.
i du
.
o
.
i da
.
ng lu
.
o
.
.
ng giac.
Gia su
.
z
1
= r
1
(cos
1
+ i sin
1
), z
2
= r
2
(cos
2
+ i sin
2
),
z = r(cos +i sin). Khi do
1
+
z
1
z
2
= r
1
r
2
[cos(
1
+
2
) +i sin(
1
+
2
)]
2
+
z
1
z
2
=
r
1
r
2
[cos(
1

2
) +i sin(
1

2
)], r
2
,= 0.
3
+
z
n
= r
n
[cos n +i sinn], n Z.
4
+
w
k
=
n

r
_
cos
+ 2k
n
+i sin
+ 2k
n
_
, k = 0, n 1.
T` u
.
3
+
suy ra
[cos +i sin]
n
= cos n +i sinn. (1.8)
Cong th u
.
c (1.8) du
.
o
.
.
c go
.
i l`a cong th u
.
c Moivre.
Phep toan nang so e len lu y th` u
.
a ph u
.
c z = x+iy du
.
o
.
.
c di
.
nh ngha
bo
.
i cong th u
.
c
e
z
= e
x+iy
def
= e
x
(cos y +i siny). (1.9)
Cha

ng ha
.
n
24 Chu
.
o
.
ng 1. So ph u
.
c
e
1+i
= e(cos 1 +i sin1),
e
i/2
= cos

2
+i sin

2
= i,
e
i
= cos +i sin = 1.
T` u
.
(1.9) khi z = i ta thu du
.
o
.
.
c c ong th u
.
c
e
i
= cos +i sin (1.10)
go
.
i l`a cong th u
.
c Euler.
Mo
.
i so ph u
.
c z ,= 0 de
`
u co the

bie

u die

n du
.
o
.
i da
.
ng
z = re
i
, (1.11)
trong do r = [z[, l`a mo
.
t trong cac acgumen cu a no. Phep bie

u die

n
(1.11) du
.
o
.
.
c go
.
i l`a da
.
ng m u cu a s o ph u
.
c. C ung nhu
.
doi vo
.
i da
.
ng lu
.
o
.
.
ng
giac ta co:
1/ ne

u z
1
= r
1
e
i
1
, z
2
= r
2
e
i
2
th`
z
1
z
2
= r
1
r
2
e
i(
1
+
2
)
, (1.12)
z
1
/z
2
=
r
1
r
2
e
i(
1

2
)
, (1.13)
2/ ne

u z = re
i
th`
z
n
= r
n
e
in
, (1.14)
n

z =
n

re
i
+2k
n
, k = 0, n 1 (1.15)
C

AC V

I DU
.
V du
.
1. Bie

u die

n cac so ph u
.
c sau day du
.
o
.
i da
.
ng lu
.
o
.
.
ng giac
1) 1 +i

3; 2) 2 +

3 +i.
Gia i. 1) T`m modun v`a acgumen cu a so ph u
.
c da cho:
r =
_
(1)
2
+ (

3)
2
= 2; tg =

3 .
1.4. Bie

u die

n so ph u
.
c du
.
o
.
i da
.
ng lu
.
o
.
.
ng giac 25
T` u
.
do ho a
.
c = /3, ho a
.
c =

3
+ =
2
3
. V` so ph u
.
c da
cho thuo
.
c goc pha
`
n tu
.
II nen ta cho
.
n =
2
3
. T` u
.
do 1 + i

3 =
2
_
cos
_
2
3
_
i sin
_
2
3
__
.
2) T`m modun v`a acgumen:
[2 +

3 +i[ =
_
(2 +

3)
2
+ 1 =
_
8 + 4

3 = 2
_
2 +

3.
Ne

u = arg(2 +

3 +i) th`
cos =
2 +

3
2
_
2 +

3
=
_
2 +

3
2
=

_1 +

3
2
2
=

_
1 + cos

6
2
= cos

12

T` u
.
do suy ra
`
ng
2

3 +i = 2
_
2 +

3
_
cos

12
+i sin

12
_

V du
.
2. Bie

u die

n cac so ph u
.
c du
.
o
.
i da
.
ng lu
.
o
.
.
ng giac
1) 1 + cos +i sin , < < .
2) 1 + cos +i sin , < < 2.
3) w =
1 + cos +i sin
1 + cos i sin
, 0 < <

2

Gia i. 1) Ta c o
[z[ =
_
2(1 + cos ) = 2

cos

2

= 2 cos

2
v` < <

2
<

2
<

2
cos

2
> 0.
Gia su
.
= argz. Khi do
cos =
1 + cos
2 cos

2
= cos

2
,
sin =
sin
2 cos

2
= sin

2

_

_
z = 2 cos

2
_
cos

2
+i sin

2
_
.
26 Chu
.
o
.
ng 1. So ph u
.
c
2) Trong tru
.
`o
.
ng ho
.
.
p n`ay ta c o
r = [z[ =
_
2(1 + cos ) = 2

cos

2

= 2 cos

2
v`

2
<

2
< . Gia su
.
= argz. Khi do
cos =
1 + cos
2 cos

2
= cos

2
= cos
_

2

_
,
sin =
sin
2 cos

2
= sin

2
= sin
_

2

_
.
T` u
.
do suy ra
`
ng
1 + cos +i sin = 2 cos

2
_
cos
_

2

_
+i sin
_

2

__
.
3) Tru
.
o
.
c he

t nha
.
n xet ra
`
ng [w[ = 1 v` tu
.
so v`a ma

u so cu a no co
modun ba
`
ng nhau. Ta t`m da
.
ng lu
.
o
.
.
ng giac cu a tu
.
so v`a ma

u so.
Xet tu
.
so: z
1
= 1 + cos +i sin ,
_
0,

2
_
[z
1
[ =
_
2(1 + cos ) ,

1
= argz
1
= arctg
sin
1 + cos
= arctg
_
tg

2
_
=

2

_


2
,

2
_
.
Tu
.
o
.
ng tu
.
.
, doi vo
.
i ma

u so
z
2
= 1 + cos i sin
ta co
[z
2
[ =
_
2(1 + cos ) ,

2
= argz
2
= arctg
_
sin
1 + cos
_
= arctg
_
tg

2
_
= arctg
_
tg
_


2
__
=

2

_


2
,

2
_
.
1.4. Bie

u die

n so ph u
.
c du
.
o
.
i da
.
ng lu
.
o
.
.
ng giac 27
T` u
.
do thu du
.
o
.
.
c
z
2
=
_
2(1 + cos )
_
cos
_


2
_
+i sin
_


2
__
v`a do va
.
y
w =
_
2(1 + cos )
_
2(1 + cos )

_
cos

2
+i sin

2
_
cos
_


2
_
+i sin
_


2
_
= cos +i sin.
V du
.
3. 1) Tnh (

3 +i)
126
2) Tnh acgumen cu a so ph u
.
c sau
w = z
4
z
2
ne

u argz = v`a [z[ = 1.


Gia i. 1) Ta c o

3 +i = 2
_
cos

6
+i sin

6
_
. T` u
.
do ap du
.
ng cong
th u
.
c Moivre ta thu du
.
o
.
.
c:
(

3 +i)
126
= 2
126
_
cos
126
6
+i sin
126
6
_
= 2
126
[cos +i sin] = 2
126
.
2) Ta c o
w = z
4
z
2
= cos 4 +i sin4 [cos 2 i sin 2]
= cos 4 cos 2 +i(sin4 + sin2)
= 2 sin 3sin + 2i sin 3cos
= 2 sin 3[sin +i cos ].
(i) Ne

u sin3 > 0 (t u
.
c l`a khi
2k
3
< <
(2k + 1)
3
, k Z) th`
w = 2 sin 3
_
cos
_

2
+
_
+i sin
_

2
+
__
.
(ii) Ne

u sin3 < 0 (t u
.
c l`a khi
(2k 1)
3
< <
2k
3
, k Z) th`
w = (2 sin3)[sin i cos ].
28 Chu
.
o
.
ng 1. So ph u
.
c
Ta t`m da
.
ng lu
.
o
.
.
ng giac cu a v = sin i cos . Hie

n nhien [v[ = 1.
Ta tnh argv
argv = arctg
_
cos
sin
_
= arctg(cotg)
= arctg
_
tg
_

2

__
= arctg
_
tg
_


2
__
=

2

Nhu
.
va
.
y ne

u sin 3 < 0 th`


w = (2 sin 3)
_
cos
_


2
_
+i sin
_


2
__
.
(iii) Ne

u sin 3 = 0 =
k
3
w = 0.
Nhu
.
va
.
y
argw =
_

2
+ ne

u
2k
3
< <
(2k + 1)
3
,
khong x ac di
.
nh ne

u =
k
3
,


2
ne

u
(2k 1)
3
< <
2k
3

V du
.
4. Ch u
.
ng minh ra
`
ng
1) cos

9
+ cos
3
9
+ cos
5
9
+ cos
7
9
=
1
2
.
2) cos + cos( +) + cos( + 2) + + cos( +n)
=
sin
(n + 1)
2
cos
_
+
n
2
_
sin

2

Gia i. 1) Da
.
t
S = cos

9
+ cos
3
9
+ + cos
7
9
,
T = sin

9
+ sin
3
9
+ + sin
7
9
,
z = cos

9
+i sin

9
.
1.4. Bie

u die

n so ph u
.
c du
.
o
.
i da
.
ng lu
.
o
.
.
ng giac 29
Khi do
S +iT = z +z
3
+z
5
+z
7
=
z(1 z
8
)
1 z
2
=
z z
9
1 z
2
=
z + 1
1 z
2
=
1
1 z
=
1
_
1 cos

9
_
i sin

9
=
_
1 cos

9
_
+i sin

9
_
1 cos

9
_
2
+ sin
2

9
=
1
2
+
sin

9
2
_
1 cos

9
_
Do do S =
1
2

2) Tu
.
o
.
ng tu
.
.
nhu
.
trong 1) ta k y hie
.
u
S = cos + cos( +) + + cos( +n),
T = sin + sin( +) + + sin( +n),
z = cos +i sin , c = cos +i sin.
Khi do
S +iT = c +cz + +cz
n
=
c(1 z
n+1
)
1 z
=
(cos +i sin)[1 cos(n + 1) i sin(n + 1)]
1 cos i sin
=
(cos +i sin)2 sin
(n + 1)
2
_
cos
(n + 1)
2
+i sin
(n + 1)
2
_
2 sin

2
_
cos

2
+i sin

2
_
=
sin
(n + 1)
2
cos
_
+
n
2
_
sin

2
+
sin
(n + 1)
2
sin
_
+
n
2
sin

2
i.
T` u
.
do so sanh pha
`
n thu
.
.
c v`a ph a
`
n a o ta thu du
.
o
.
.
c ke

t qua .
Ba
`
ng phu
.
o
.
ng phap tu
.
o
.
ng tu
.
.
ta co the

tnh c ac to

ng da
.
ng
a
1
sinb
1
+a
2
sinb
2
+ +a
n
sinb
n
,
a
1
cos b
1
+a
2
cos b
2
+ +a
n
cos b
n
30 Chu
.
o
.
ng 1. So ph u
.
c
ne

u cac acgumen b
1
, b
2
, . . . , b
n
la
.
p nen c ap so co
.
ng c` on cac he
.
so
a
1
, a
2
, . . . , a
n
la
.
p nen cap so nhan.
V du
.
5. Tnh to

ng
1) S
n
= 1 +a cos +a
2
cos 2 + +a
n
cos n;
2) T
n
= a sin +a
2
sin2 + +a
n
sinn.
Gia i. Ta la
.
p bie

u th u
.
c S
n
+iT
n
v`a thu du
.
o
.
.
c
= S
n
+iT
n
= 1 +a(cos +i sin) +a
2
(cos 2 +i sin2) +. . .
+a
n
(cos n +i sinn).
Da
.
t z = cos +i sin v`a ap du
.
ng cong th u
.
c Moivre ta c o:
= 1 +az +a
2
z
2
+ +a
n
z
n
=
a
n+1
z
n+1
1
az 1
(nhan tu
.
so v`a ma

u so vo
.
i
a
z
1)
=
a
n+2
z
n
a
n+1
z
n+1

a
2
+ 1
a
2
a
_
z +
1
z
_
+ 1
(do z +
1
z
= 2 cos )
=
a
n+2
(cos n +i sinn) a
n+1
[cos(n + 1) +i sin(n + 1)]
a
2
2a cos + 1
+
a cos +ai sin + 1
a
2
2a cos + 1
=
a
n+2
cos n a
n+1
cos(n + 1) a cos + 1
a
2
2a cos + 1
+
+i
a
n+2
sinn a
n+1
sin(n + 1) +a sin
a
2
2a cos + 1

Ba
`
ng cach so sanh pha
`
n thu
.
.
c v`a pha
`
n a o ta thu du
.
o
.
.
c c ac ke

t qua ca
`
n
du
.
o
.
.
c tnh.
V du
.
6. 1) Bie

u die

n tg5 qua tg.


1.4. Bie

u die

n so ph u
.
c du
.
o
.
i da
.
ng lu
.
o
.
.
ng giac 31
2) Bie

u die

n tuye

n tnh sin
5
qua cac h`am sin cu a goc bo
.
i cu a .
3) Bie

u die

n cos
4
v`a sin
4
cos
3
qua h`am cosin cu a c ac goc bo
.
i.
Gia i. 1) V` tg5 =
sin 5
cos 5
nen ta c a
`
n bie

u die

n sin 5 v`a cos 5


qua sin v`a cos . Theo cong th u
.
c Moivre ta c o
cos 5 +i sin5 = (cos +i sin )
5
= sin
5
+ 5i cos
4
sin
10 cos
3
sin
2
10i cos
2
sin
3

+ 5 cos sin
4
+i sin
5
.
Tach pha
`
n thu
.
.
c v`a pha
`
n a o ta thu du
.
o
.
.
c bie

u th u
.
c doi vo
.
i sin5 v`a
cos 5 v`a t` u
.
do
tg5 =
5 cos
4
sin 10 cos
2
sin
3
+ sin
5

cos
5
10 cos
3
sin
2
+ 5 cos sin
4

(chia tu
.
so v`a ma

u so cho cos
5
)
=
5tg 10tg
3
+ tg
5

1 10tg
2
+ 5tg
4


2) Da
.
t z = cos + i sin. Khi do z
1
= cos i sin v`a theo
cong th u
.
c Moivre:
z
k
= cos k +i sin k, z
k
= cos k i sink.
Do do
cos =
z +z
1
2
, sin =
z z
1
2i
z
k
+z
k
= 2 cos k, z
k
z
k
= 2i sink.

Ap du
.
ng cac ke

t qua n`ay ta co
sin
5
=
_
z z
1
2i
_
5
=
z
5
5z
3
+ 10z 10z
1
+ 5z
3
z
5
32i
=
(z
5
z
5
) 5(z
3
z
3
) + 10(z z
1
)
32i
=
2i sin5 10i sin 3 + 20i sin
32i
=
sin5 5 sin 3 + 10 sin
16

32 Chu
.
o
.
ng 1. So ph u
.
c
3) Tu
.
o
.
ng tu
.
.
nhu
.
trong pha
`
n 2) hoa
.
c gia i theo cach sau day
1
+
cos
4
=
_
e
i
+e
i
2
_
4
=
1
16
_
e
4i
+ 4e
2i
+ 6 + 4e
2i
+e
4i

=
1
8
_
e
4i
+e
4i
2
_
+
1
2
_
e
2i
+e
2i
2
_
+
3
8
=
3
8
+
1
2
cos 2 +
1
8
cos 4.
2
+
sin
4
cos
3
=
_
e
i
e
i
2i
_
4
_
e
i
+e
i
2
_
3
=
1
128
_
e
2i
e
2i
_
3
_
e
i
e
i
_
=
1
128
_
e
6i
3e
2i
+ 3e
2i
e
6i
_
e
i
e
i

=
1
128
_
e
7i
e
5i
3e
3i
+ 3e
i
+ 3e

i
3e
3i
e
5i
+e
7i
_
=
3
64
cos
3
64
cos 3
1
64
cos 5
1
64
cos 7.
V du
.
7. 1) Gia i cac phu
.
o
.
ng tr`nh
1
+
(x + 1)
n
(x 1)
n
= 0
2
+
(x +i)
n
+ (x i)
n
= 0, n > 1.
2) Ch u
.
ng minh ra
`
ng mo
.
i nghie
.
m cu a phu
.
o
.
ng tr`nh
_
1 +ix
1 ix
_
n
=
1 +ai
1 ai
, n N, a R
de
`
u l`a nghie
.
m thu
.
.
c khac nhau.
Gia i. 1) Gia i phu
.
o
.
ng tr`nh
1
+
Chia hai ve

cu a phu
.
o
.
ng tr`nh cho (x 1)
n
ta du
.
o
.
.
c
_
x + 1
x 1
_
n
= 1
x + 1
x 1
=
n

1 = cos
2k
n
+i sin
2k
n
=
k
,
k = 0, 1, . . . , n 1.
1.4. Bie

u die

n so ph u
.
c du
.
o
.
i da
.
ng lu
.
o
.
.
ng giac 33
T` u
.
do suy ra
`
ng
x + 1 =
k
(x 1) x(
k
1) = 1 +
k
.
Khi k = 0
0
= 1. Do do vo
.
i k = 0 phu
.
o
.
ng tr`nh vo nghie
.
m. Vo
.
i
k = 1, n 1 ta c o
x =

k
+ 1

k
1
=
(
k
+ 1)(
k
1)

k
1)(
k
1)
=

k

k
+
k

k
1

k

k

k
1
=
2i sin
2k
n
2 2 cos
2k
n
= i
sin
2k
n
1 cos
2k
n
= icotg
k
n
, k = 1, 2, . . . , n 1.
2
+
C ung nhu
.
tren, t` u
.
phu
.
o
.
ng tr`nh da cho ta co
_
x +i
x i
_
n
= 1
x +i
x i
=
n

1 = cos
+ 2k
n
+i sin
+ 2k
n
hay l`a
x +i
x i
= cos
(2k + 1)
n
+i sin
(2k + 1)
n
= cos +i sin , =
(2k + 1)
n

Ta bie

n do

i phu
.
o
.
ng tr`nh:
x +i
x i
1 = cos +i sin 1

2i
x i
= 2i sin

2
cos

2
2 sin
2

2

1
x i
= sin

2
_
cos

2

1
i
sin

2
_
= sin

2
_
cos

2
+i sin

2
_
.
34 Chu
.
o
.
ng 1. So ph u
.
c
T` u
.
do suy ra
x i =
1
sin

2
_
cos

2
+i sin

2
_
=
cos

2
i sin

2
sin

2
= cotg

2
i.
Nhu
.
va
.
y
x i = cotg

2
i x = cotg

2
= cotg
(2k + 1)
2n
, k = 0, n 1.
2) Ta xet ve

pha i cu a phu
.
o
.
ng tr`nh da cho. Ta co

1 +ai
1 ai

= 1
1 +ai
1 ai
= cos +i sin
v`a t` u
.
do
1 +xi
1 xi
=
n
_
1 +ai
1 ai
= cos
+ 2k
n
+i sin
+ 2k
n
, k = 0, n 1.
T` u
.
do ne

u da
.
t =
+ 2k
n
th`
x =
cos 1 +i sin
i[cos + 1 +i sin]
= tg

2
= tg
+ 2k
2n
, k = 0, n 1.
Ro r`ang do l`a nh u
.
ng nghie
.
m thu
.
.
c khac nhau.
V du
.
8. Bie

u die

n cac so ph u
.
c sau day du
.
o
.
i da
.
ng m u:
1) z =
(

3 +i)
_
cos

12
i sin

12
_
1 i

2) z =
_

3 +i.
1.4. Bie

u die

n so ph u
.
c du
.
o
.
i da
.
ng lu
.
o
.
.
ng giac 35
Gia i. 1) Da
.
t z
1
=

3 + i, z
2
= cos

12
i sin

12
, z
3
= 1 i v`a
bie

u die

n cac so ph u
.
c do du
.
o
.
i da
.
ng m u. Ta co
z
1
= 2e
5
6
i
;
z
2
= cos

12
i sin

12
= cos
_


12
_
+i sin
_


12
_
= e


12
i
;
z
3
=

2e

4
i
.
T` u
.
do thu du
.
o
.
.
c
z =
2e
5
6
i
e


12
i

2e

4
i
=

2e
i
.
2) Tru
.
o
.
c he

t bie

u die

n so ph u
.
c z
1
=

3 +i du
.
o
.
i da
.
ng m u. Ta co
[z
1
[ = 2; = arg(

3 +i) =

6
,
do do

3 +i = 2e

6
i
. T` u
.
do thu du
.
o
.
.
c
w
k
=
4
_

3 +i =
4

2e
i
(

6
+2k)
4
=
4

2e
i
(12k+1)
24
, k = 0, 3.
V du
.
9. Tnh cac gia tri
.
1) can ba
.
c 3: w =
3

2 + 2i
2) can ba
.
c 4: w =
4

4
3) can ba
.
c 5: w =
5

3 i
8 + 8i
.
Gia i. Phu
.
o
.
ng phap tot nhat de

tnh gia tri


.
cac c an th u
.
c l`a bie

u
die

n so ph u
.
c du
.
o
.
i dau can du
.
o
.
i da
.
ng lu
.
o
.
.
ng giac (hoa
.
c da
.
ng m u) ro
`
i
ap du
.
ng cac cong th u
.
c tu
.
o
.
ng u
.
ng.
1) Bie

u die

n z = 2 + 2i du
.
o
.
i da
.
ng lu
.
o
.
.
ng giac. Ta co
r = [z[ =

8 = 2

2; = arg(2 + 2i) =
3
4

36 Chu
.
o
.
ng 1. So ph u
.
c
Do do
w
k
=
3
_

8
_
cos
3
4
+ 2k
3
+i sin
3
4
+ 2k
3
_
, k = 0, 2.
T` u
.
do
w
0
=

2
_
cos

4
+i sin

4
_
= 1 +i,
w
1
=

2
_
cos
11
12
+i sin
11
12
_
,
w
2
=

2
_
cos
19
12
+i sin
19
12
_
.
2) Ta c o
4 = 4[cos +i sin ]
v`a do do
w
k
=
4

4
_
cos
+ 2k
4
+i sin
+ 2k
4
_
, k = 0, 3.
T` u
.
do
w
0
=

2
_
cos

4
+i sin

4
_
= 1 +i,
w
1
=

2
_
cos
3
4
+i sin
3
4
_
= 1 +i,
w
2
=

2
_
cos
5
4
+i sin
5
4
_
= 1 i,
w
3
=

2
_
cos
7
4
+i sin
7
4
_
= 1 i.
3) Da
.
t
z =

3 i
8 + 8i

Khi do [z[ =

3 + 1

64 + 64
=
1
4

2
. Ta tnh argz. Ta co
argz = arg(

3 i) arg(8 + 8i) =

6


4
=
5
12

1.4. Bie

u die

n so ph u
.
c du
.
o
.
i da
.
ng lu
.
o
.
.
ng giac 37
Do va
.
y
w
k
=
5

_
1
4

2
_
_
cos

5
12
+ 2k
5
+i sin

5
12
+ 2k
5
_
=
1

2
_
cos
_


12
+
2k
5
_
+i sin
_


12
+
2k
5
__
, k = 0, 4.
V du
.
10. 1) Tnh to

ng mo
.
i can ba
.
c n cu a 1.
2) Tnh to

ng 1 + 2 + 3
2
+ + n
n1
, trong do l`a can ba
.
c n
cu a do
.
n vi
.
.
3) Tnh to

ng cac lu y th` u
.
a ba
.
c k cu a mo
.
i can ba
.
c n cu a so ph u
.
c .
Gia i. 1) Da
`
u tien ta vie

t c ac can ba
.
c n cu a 1. Ta co

k
=
n

1 = cos
2k
n
+i sin
2k
n
, k = 0, n 1.
T` u
.
do

0
= 1,
1
= = cos
2
n
+i sin
2
n
,

k
= cos
2k
n
+i sin
2k
n
=
_
cos
2
n
+i sin
2
n
_
k
=
k
, k = 1, 2, . . . , n 1.
Nhu
.
va
.
y mo
.
i nghie
.
m cu a can ba
.
c n cu a 1 co the

vie

t du
.
o
.
i da
.
ng
1, ,
2
, . . . ,
n1
.
Bay gi`o
.
ta tnh
S = 1 + +
2
+ +
n1
=
1
n
1

Ne

u n > 1 th`
n
= 1 v`a do do
S =
1
n
1
= 0.
38 Chu
.
o
.
ng 1. So ph u
.
c
2) Ta k y hie
.
u to

ng c a
`
n tnh l`a S. Ta xet bie

u th u
.
c
(1 )S = S S = 1 + 2 + 3
2
+ +n
n1
2
2
(n 1)
n1
n
n
= 1 + +
2
+ +
n1
. .
0(=1)
n
n
= n
v`
n
= 1.
Nhu
.
va
.
y
(1 )S = n S =
n
1
ne

u ,= 1.
Ne

u = 1 th`
S = 1 + 2 + +n =
n(n + 1)
2

3) Gia su
.

0
l`a mo
.
t trong cac gia tri
.
can cu a . Khi do (vo
.
i
,= 0) mo
.
i c an ba
.
c n cu a co the

bie

u die

n du
.
o
.
i da
.
ng tch
0

k
,
k = 1, 2, . . . , n 1, trong do
k
= cos
2k
n
+ i sin
2k
n
l`a can ba
.
c n
cu a 1.
T` u
.
do to

ng ca
`
n t`m S ba
`
ng
S =
k
0
+ (
0

1
)
k
+ (
0

2
)
k
+ + (
0

n1
)
k
=
k
0
(1 +
k
1
+
k
2
+ +
k
n1
)
_

k
m
=
_
cos
2m
n
+i sin
2m
n
_
k
=
_
cos
2
n
+i sin
2
n
_
mk
_
=
k
0
_
1 +
k
1
+
2k
1
+ +
(n1)k
1
_
.
Bie

u th u
.
c trong dau ngoa
.
c vuong l`a c ap so nhan. Ne

u
k
1
,= 1, t u
.
c l`a
k khong chia he

t cho n th`
S =
k
0
1
nk
1
1
k
1
=
k
0
1 1
1
k
1
= 0 (v`
n
1
= 1).
1.4. Bie

u die

n so ph u
.
c du
.
o
.
i da
.
ng lu
.
o
.
.
ng giac 39
Ne

u
k
1
= 1 t u
.
c l`a k chia he

t cho n, k = nq th`
S =
nq
0
[1 + 1 + + 1] =
nq
0
n = n
q
(v`
n
0
= ).
Nhu
.
va
.
y
S =
_
_
_
0 ne

u k chia he

t cho n;
n
q
ne

u k = nq, q Z.
B
`
AI T

A
.
P
1. Bie

u die

n cac so ph u
.
c sau day du
.
o
.
i da
.
ng lu
.
o
.
.
ng giac
1) 1 +i

3 (DS. 2
_
cos
2
3
+i sin
2
3
_
)
2)

3 i (DS. 2
_
cos
11
6
+i sin
11
6
_
)
3)

3 i (DS. 2
_
cos
7
6
+i sin
7
6
_
)
4)

3
2
+
i
2
(DS. cos

6
+i sin

6
)
5)

3
2
+
1
2
i (DS. cos
5
6
+i sin
5
6
)
6)
1
2
i

3
2
(DS. cos
5
3
+i sin
5
3
)
7)
1
2
i

3
2
(DS. cos
4
3
+i sin
4
3
)
8) 2 +

3 i (DS. 2
_
2 +

3
_
cos
23
12
+i sin
23
12
_
)
9) 2

3 i (DS. 2
_
2

3
_
cos
19
12
+i sin
19
12
_
)
2. Bie

u die

n cac so ph u
.
c sau day du
.
o
.
i da
.
ng lu
.
o
.
.
ng giac
1) cos +i sin (DS. cos( ) +i sin( ))
2) sin +i cos (DS. cos
_

2
+
_
+i sin
_

2
+))
40 Chu
.
o
.
ng 1. So ph u
.
c
3) cos i sin (DS. cos() +i sin())
4) cos i sin (DS. cos( +) +i sin( +))
Ba
`
ng cach da
.
t = + 2k, trong do 0 < 2, ta c o:
5) 1+cos +i sin (DS. 2 cos

2
_
cos

2
+i sin

2
_
vo
.
i 0 < ;
2 cos

2
_
cos
+ 2
2
+ i sin
+ 2
2
_
vo
.
i < 2)
6) 1 cos +i sin (DS. 2 sin

2
_
cos

2
+i sin

2
_
)
7) sin +i(1 + cos )
(DS. 2 cos

2
_
cos

2
+i sin

2
_
vo
.
i 0 < ;
2 cos

2
_
cos
3
2
+i sin
3
2
_
vo
.
i < 2)
8) sin +i(1 + cos )
(DS. 2 cos

2
_
cos
+
2
+i sin
+
2
_
vo
.
i 0 < ;
2 cos

2
_
cos
3 +
2
+ i sin
3 +
2
_
vo
.
i < 2)
3. Tnh:
1)
_
cos

6
i sin

6
_
100
(DS.
1
2
i

3
2
)
2)
_
4

3 +i
_
12
(DS. 2
12
)
3)
(

3 + i)
6
(1 +i)
8
(1 +i)
4
(DS. 3, 2)
4)
(i

3)
15
(1 i)
20
+
(i +

3)
15
(1 +i)
20
(DS. 64i)
5)
(1 +i)
100
(1 i)
96
+ (1 +i)
96
(DS. 2)
6)
(1 +icotg)
5
1 icotg)
5
(DS. cos( 10) +i sin( 10))
7)
(1 i

3)(cos +i sin )
2(1 i)(cos i sin)
(DS.

2
2
_
cos
_
6

12
_
+i sin
_
6

12
__
)
1.4. Bie

u die

n so ph u
.
c du
.
o
.
i da
.
ng lu
.
o
.
.
ng giac 41
8)
(1 +i

3)
3n
(1 +i)
4n
(DS. 2)
4. Ch u
.
ng minh ra
`
ng z +
1
z
= 2 cos z
n
+
1
z
n
= 2 cos n.
5. Hay bie

u die

n cac h`am sau d ay qua sin v`a cos


1) sin 3 (DS. 3 cos
2
sin sin
3
)
2) cos 3 (DS. cos
3
3 cos sin
2
)
3) sin 4 (DS. 4 cos
3
sin 4 cos sin
3
)
4) cos 4 (DS. cos
4
6 cos
2
sin
2
+ sin
4
)
6. Hay bie

u die

n cac h`am sau qua tgx


1) tg4 (DS.
4tg 4tg
3

1 6tg
2
+ tg
4

)
2) tg6 (DS.
6tg 20tg
3
+ 6tg
5

1 15tg
2
+ 15tg
4
tg
6

)
7. Ch u
.
ng minh ra
`
ng
1 C
2
n
+C
4
n
C
6
n
+. . . = 2
n
2
cos
n
4

C
1
n
C
3
n
+C
5
n
C
7
n
+. . . = 2
n
2
sin
n
4

Ch da

n. Tnh (1 + i)
n
ba
`
ng cach su
.
du
.
ng cong th u
.
c Moivre v`a
su
.
du
.
ng cong th u
.
c nhi
.
th u
.
c Newton ro
`
i so sanh pha
`
n thu
.
.
c v`a pha
`
n
a o cac so thu du
.
o
.
.
c.
8. Ch u
.
ng minh ra
`
ng
1) cos

5
+ cos
3
5
=
1
2
2) cos

7
+ cos
3
7
+ cos
5
7
=
1
2
3) cos
2
5
+ cos
4
5
=
1
2
4) cos
2
7
+ cos
4
7
+ cos
6
7
=
1
2
5) cos
2
9
+ cos
4
9
+ cos
6
9
+ cos
8
9
=
1
2
42 Chu
.
o
.
ng 1. So ph u
.
c
9. Gia i phu
.
o
.
ng tr`nh
_
i x
i +x
_
n
=
cotg +i
cotg i
, n N, R.
(DS. x = tg
+k
n
, k = 0, n 1)
10. Ch u
.
ng minh ra
`
ng ne

u A l`a so ph u
.
c c o modun = 1 th` mo
.
i nghie
.
m
cu a phu
.
o
.
ng tr`nh
_
1 +ix
1 ix
_
n
= A
de
`
u l`a nghie
.
m thu
.
.
c v`a khac nhau.
11. Gia i phu
.
o
.
ng tr`nh
x
n
nax
n1
C
2
n
a
2
x
n2
a
n
= 0.
(DS. x
k
=
a

2 1
, k = 0, n 1)
Ch da

n. D` ung cong th u
.
c nhi
.
th u
.
c Newton de

du
.
a phu
.
o
.
ng tr`nh
ve
`
da
.
ng x
n
= (x +a)
n
x
n
.
12. Gia i phu
.
o
.
ng tr`nh
x
5
+x
4
+x
3
+x
2
+x + 1 = 0.
(DS. x
k
= cos
k
3
+i sin
k
3
, k = 1, 2, 3, 4, 5)
13. Gia i phu
.
o
.
ng tr`nh
x
5
+x
4
+
2
x
3
+
3
x
2
+
4
x +
5
= 0, C, ,= 0.
(DS. x
k
=
_
cos
k
3
+i sin
k
3
_
, k = 1, 2, 3, 4, 5)
Ch da

n. Ve

trai l`a to

ng cap so nhan vo
.
i cong bo
.
i ba
`
ng

x
.
14. Gia su
.
n N, n > 1, c ,= 0, c R. Gia i cac phu
.
o
.
ng tr`nh sau
day
1.4. Bie

u die

n so ph u
.
c du
.
o
.
i da
.
ng lu
.
o
.
.
ng giac 43
1) (x +c)
n
(x c)
n
= 0 (DS. x = ccotg
k
n
, k = 1, n 1)
2) (x +ci)
n
(x ci)
n
= 0 (DS. x = cicotg
k
n
, k = 1, n 1)
3) (x +ci)
n
+i(x ci)
n
= 0
(DS. x = cicotg
(3 + 4k)
4n
, k = 0, n 1)
4) (x +ci)
n
(cos +i sin)(x ci)
n
= 0, ,= 2k.
(DS. x = cicotg
+ 2k
2n
, k = 0, n 1)
15. Tnh
D
n
(x) =
1
2
_
1
2
+ cos x + cos 2x + + cos nx
_
.
(DS. D
n
(x) =
1
2
sin
2n + 1
2
x
2 sin
x
2
)
16. 1) Bie

u die

n cos 5x v`a sin5x qua cos x v`a sinx.


2) Tnh cos
2
5
v`a sin
2
5
.
(DS. 1) cos 5x = cos
5
x 10 cos
3
xsin
2
x + 5 cos xsin
4
x,
sin 5x = 5 cos
4
xsinx 10 cos
2
xsin
3
x + sin
5
x.
2) sin
2
5
=
_
10 + 2

5
4
, cos
2
5
=

5 1
4
)
Ch da

n. De

tnh sin
2
5
ca
`
n su
.
du
.
ng bie

u th u
.
c cu a sin5x.
Chu
.
o
.
ng 2
D
-
a th u
.
c v`a h`am h u
.
u ty

2.1 D
-
a th u
.
c . . . . . . . . . . . . . . . . . . . . . 44
2.1.1 D
-
a th u
.
c tren tru
.
`o
.
ng so ph u
.
c C . . . . . . . 45
2.1.2 D
-
a th u
.
c tren tru
.
`o
.
ng so thu
.
.
c R . . . . . . . 46
2.2 Phan th u
.
c h u
.
u ty . . . . . . . . . . . . . . . 55
2.1 D
-
a th u
.
c
Da th u
.
c mo
.
t bie

n vo
.
i he
.
so thuo
.
c tru
.
`o
.
ng so T du
.
o
.
.
c bie

u die

n do
.
n tri
.
du
.
o
.
i da
.
ng to

ng h u
.
u ha
.
n
Q(x) = a
0
z
n
+a
1
z
n1
+ +a
n1
z +a
n
(2.1)
trong do z l`a bie

n, a
0
, a
1
, . . . , a
n
l`a cac so; v`a mo

i to

ng da
.
ng (2.1) de
`
u
l`a da th u
.
c.
K y hie
.
u: Q(z) T[z].
Ne

u a
0
, a
1
, . . . , a
n
C th` ngu
.
`o
.
i ta noi ra
`
ng Q(z) l`a da th u
.
c tren
tru
.
`o
.
ng so ph u
.
c: Q(z) C[z]. Ne

u a
0
, a
1
, . . . , a
n
R th` Q(z) l`a da
th u
.
c tren tru
.
`o
.
ng so thu
.
.
c: Q(z) R[z].
2.1. D
-
a th u
.
c 45
Ne

u Q(z) ,= 0 th` ba
.
c cu a no (k y hie
.
u degQ(z)) l`a so m u cao nhat
cu a mo
.
i lu y th` u
.
a cu a cac so ha
.
ng ,= 0 cu a da th u
.
c v`a he
.
so cu a so
ha
.
ng co lu y th` u
.
a cao nhat do go
.
i l`a he
.
s o cao nh at.
Ne

u P(z) v`a Q(z) T[z] l`a c a


.
p da th u
.
c v`a Q(z) ,= 0 th` to
`
n ta
.
i
ca
.
p da th u
.
c h(z) v`a r(z) T[z] sao cho
1
+
P = Qh +r,
2
+
hoa
.
c r(z) = 0, hoa
.
c degr < degQ.
D
-
i
.
nh l y Bezout. Pha
`
n du
.
cu a phep chia da th u
.
c P(z) cho nhi
.
th u
.
c
z l` a ha
`
ng P() (r = P()).
2.1.1 D
-
a th u
.
c tren tru
.
`o
.
ng s o

ph u
.
c C
Gia su
.
Q(z) C[z]. Ne

u thay z bo
.
i so th` ta thu du
.
o
.
.
c so ph u
.
c
Q() = a
0

n
+a
1

n1
+ +a
n1
+a
n
.
D
-
i
.
nh ngha 2.1.1. Ne

u Q() = 0 th` so z = du
.
o
.
.
c go
.
i l`a nghie
.
m
cu a da th u
.
c Q(z) hay cu a phu
.
o
.
ng tr`nh da
.
i so Q(z) = 0.
D
-
i
.
nh l y Descate. Da th u
.
c Q(z) chia he

t cho nhi
.
th u
.
c z khi v` a
ch khi l`a nghie
.
m cu a da th u
.
c P(z) (t u
.
c l`a P() = 0).
D
-
i
.
nh ngha 2.1.2. So ph u
.
c l`a nghie
.
m bo
.
i m cu a da th u
.
c Q(z)
ne

u v`a ch ne

u Q(z) chia he

t cho (z )
m
nhu
.
ng khong chia he

t cho
(z )
m+1
. So m du
.
o
.
.
c go
.
i l`a bo
.
i cu a nghie
.
m . Khi m = 1, so go
.
i
l`a nghie
.
m do
.
n cu a Q(z).
Trong tie

t 2.1.1 ta bie

t ra
`
ng ta
.
p ho
.
.
p so ph u
.
c C du
.
o
.
.
c la
.
p nen ba
`
ng
cach ghep them v`ao cho ta
.
p ho
.
.
p so thu
.
.
c R mo
.
t nghie
.
m a o x = i cu a
phu
.
o
.
ng tr`nh x
2
+ 1 = 0 v` a mo
.
t khi da ghep i v`ao th` mo
.
i phu
.
o
.
ng
tr`nh da th u
.
c de
`
u c o nghie
.
m ph u
.
c thu
.
.
c su
.
.
. Do do khong c a
`
n pha i
sang ta
.
o them cac so mo
.
i de

gia i phu
.
o
.
ng tr`nh (v` the

C c`on du
.
o
.
.
c go
.
i
l`a tru
.
`o
.
ng dong da
.
i so).
D
-
i
.
nh l y Gauss (di
.
nh l y co
.
ba n cu a da
.
i so).
46 Chu
.
o
.
ng 2. D
-
a th u
.
c v`a h`am h u
.
u ty
Mo
.
i da th u
.
c da
.
i s o ba
.
c n (n 1) tren tru
.
` o
.
ng s o ph u
.
c de
`
u co t
nhat mo
.
t nghie
.
m ph u
.
c.
T` u
.
di
.
nh l y Gauss r ut ra cac he
.
qua sau.
1
+
Mo
.
i da th u
.
c ba
.
c n (n 1) tren tru
.
`o
.
ng so ph u
.
c de
`
u co d ung n
nghie
.
m ne

u mo

i nghie
.
m du
.
o
.
.
c tnh mo
.
t so la
`
n ba
`
ng bo
.
i cu a no, t u
.
c l`a
Q(x) = a
0
(z
1
)
m
1
(z
2
)
m
2
(z
k
)
m
k
, (2.2)
trong do
i
,=
j
i ,= j v`a m
1
+m
2
+ +m
k
= n.
Da th u
.
c (2.1) vo
.
i he
.
so cao nhat a
0
= 1 du
.
o
.
.
c go
.
i l`a da th u
.
c thu
go
.
n.
2
+
Ne

u z
0
l`a nghie
.
m bo
.
i m cu a da th u
.
c Q(z) th` so ph u
.
c lien ho
.
.
p
vo
.
i n o z
0
l`a nghie
.
m bo
.
i m cu a da th u
.
c lien ho
.
.
p Q(z), trong do da
th u
.
c Q(z) du
.
o
.
.
c xac di
.
nh bo
.
i
Q(z)
def
= a
0
z
n
+a
1
z
n1
+ +a
n1
z +a
n
. (2.3)
2.1.2 D
-
a th u
.
c tren tru
.
`o
.
ng s o

thu
.
.
c R
Gia su
.

Q(z) = z
n
+a
1
z
n1
+ +a
n1
z +a
n
(2.4)
l`a da th u
.
c quy go
.
n vo
.
i he
.
so thu
.
.
c a
1
, a
2
, . . . , a
n
.
Da th u
.
c n`ay co tnh chat da
.
c bie
.
t sau day.
D
-
i
.
nh l y 2.1.1. Ne

u so ph u
.
c l`a nghie
.
m bo
.
i m cu a da th u
.
c (2.4) vo
.
i
he
.
so thu
.
.
c th` s o ph u
.
c lien ho
.
.
p vo
.
i no c ung l`a nghie
.
m bo
.
i m cu a
da th u
.
c do.
Su
.
du
.
ng di
.
nh l y tren day ta c o the

t`m khai trie

n da th u
.
c vo
.
i he
.
so thu
.
.
c Q(z) th`anh tch cac th` u
.
a so. Ve
`
sau ta thu
.
`o
.
ng ch xet da
th u
.
c vo
.
i he
.
so thu
.
.
c vo
.
i bie

n ch nha
.
n gia tri
.
thu
.
.
c nen bie

n do ta k y
hie
.
u l`a x thay cho z.
2.1. D
-
a th u
.
c 47
D
-
i
.
nh l y 2.1.2. Gia su
.
da th u
.
c Q(x) c o c ac nghie
.
m thu
.
.
c b
1
, b
2
, . . . , b
m
vo
.
i bo
.
i tu
.
o
.
ng u
.
ng
1
,
2
, . . . ,
m
v`a c ac c a
.
p nghie
.
m ph u
.
c lien ho
.
.
p a
1
v`a a
1
, a
2
v`a a
2
, . . . , a
n
v`a a
n
vo
.
i bo
.
i tu
.
o
.
ng u
.
ng
1
,
2
, . . . ,
n
. Khi do
Q(x) = (x b
1
)

1
(x b
2
)

2
(x b
m
)
m
(x
2
+p
1
x +q
1
)

(x
2
+p
2
x +q
2
)

2
(x
2
+p
n
x +q
b
)
n
. (2.5)
D
-
i
.
nh l y 2.1.3. Ne

u da th u
.
c Q(x) = x
n
+a
1
x
n1
+ +a
n1
x +a
n
vo
.
i he
.
s o nguyen v`a vo
.
i he
.
s o cao nh at b a
`
ng 1 co nghie
.
m h u
.
u ty th`
nghie
.
m do l`a so nguyen.
Doi vo
.
i da th u
.
c vo
.
i he
.
so h u
.
u ty ta co
D
-
i
.
nh l y 2.1.4. Ne

u phan so toi gia n



m
(, m Z, m > 0) l` a nghie
.
m
h u
.
u ty cu a phu
.
o
.
ng tr`nh vo
.
i he
.
s o h u
.
u ty a
0
x
n
+a
1
x
n1
+ +a
n1
x+
a
n
= 0 th` l`a u
.
o
.
c cu a so ha
.
ng tu
.
.
do a
n
v` a m l` a u
.
o
.
c cu a he
.
s o cao
nhat a
0
.
C

AC V

I DU
.
V du
.
1. Gia su
.
P(z) = a
0
z
n
+ a
1
z
n1
+ + a
n1
z + a
n
. Ch u
.
ng
minh ra
`
ng:
1
+
Ne

u P(z) C[z] th` P(z) = P(z).


2
+
Ne

u P(z) R[z] th` P(z) = P(z).


Gia i. 1
+

Ap du
.
ng cac tnh chat cu a phep toan lay lien ho
.
.
p ta thu
du
.
o
.
.
c
p(Z) = a
0
z
n
+a
1
z
n1
+ +a
n1
z +a
n
= a
0
z
n
+a
1
z
n1
+ +a
n1
z +a
n
= a
0
(z)
n
+a
1
(z)
n1
+ +a
n1
z +a
n
= P(z).
48 Chu
.
o
.
ng 2. D
-
a th u
.
c v`a h`am h u
.
u ty
2
+
Gia su
.
P(z) R[z]. Khi do
P(z) = a
0
z
n
+a
1
z
n1
+ +a
n1
z +a
n
= a
0
z
n
+a
1
z
n1
+ +a
n1
z +a
n
= a
0
(z)
n
+a
1
(z)
n1
+ +a
n1
z +a
n
= a
0
(z)
n
+a
1
(z)
n1
+ +a
n1
z +a
n
= P(z).
T` u
.
do c ung thu du
.
o
.
.
c P(z) = P(z) v` P(z) = P(z).
V du
.
2. Ch u
.
ng minh ra
`
ng ne

u a l`a nghie
.
m bo
.
i m cu a da th u
.
c
P(z) = a
0
z
n
+a
1
z
n1
+ +a
n1
z +a
n
, a
0
,= 0
th` so ph u
.
c lien ho
.
.
p a l`a nghie
.
m bo
.
i m cu a da th u
.
c
P(z) = a
0
z
n
+a
1
z
n1
+ +a
n1
z +a
n
(go
.
i l`a da th u
.
c lien ho
.
.
p ph u
.
c vo
.
i da th u
.
c P(z)).
Gia i. T` u
.
v du
.
1 ta co
P(z) = P(z). (2.6)
V` a l`a nghie
.
m bo
.
i m cu a P(z) nen
P(z) = (z a)
m
Q(z), Q(a) ,= 0 (2.7)
trong do Q(z) l`a da th u
.
c ba
.
c n m. T` u
.
(2.6) v`a (2.7) suy ra
P(z) = P(z) = (z a)
m
Q(z) = (z a)
m
Q(z). (2.8)
Ta c` on ca
`
n ch u
.
ng minh ra
`
ng Q(a) ,= 0. Tha
.
t va
.
y, ne

u Q(a) = 0 th`
ba
`
ng cach lay lien ho
.
.
p ph u
.
c mo
.
t la
`
n n u
.
a ta c o
Q(a) = Q(a) = 0 Q(a) = 0.
Die
`
u n`ay vo l y. B a
`
ng cach da
.
t t = z, t` u
.
(2.8) thu du
.
o
.
.
c
P(t) = (t a)
m
Q(t), Q(a) ,= 0.
2.1. D
-
a th u
.
c 49
Da

ng th u
.
c n`ay ch u
.
ng to ra
`
ng t = a l`a nghie
.
m bo
.
i m cu a da th u
.
c
P(t).
V du
.
3. Ch u
.
ng minh ra
`
ng ne

u a l`a nghie
.
m bo
.
i m cu a da th u
.
c vo
.
i
he
.
so thu
.
.
c P(z) = a
0
z
n
+ a
1
z
n1
+ +a
n
(a
0
,= 0) th` so ph u
.
c lien
ho
.
.
p a c ung l`a nghie
.
m bo
.
i m cu a chnh da th u
.
c do.
Gia i. T` u
.
v du
.
1, 2
+
ta co
P(z) = P(z) (2.9)
v`a do a l`a nghie
.
m bo
.
i m cu a no nen
P(z) = (z a)
m
Q(z) (2.10)
trong do Q(z) l`a da th u
.
c ba
.
c n m v`a Q(a) ,= 0.
Ta ca
`
n ch u
.
ng minh ra
`
ng
P(z) = (z a)
m
Q(z), Q(a) ,= 0. (2.11)
Tha
.
t va
.
y t` u
.
(2.9) v`a (2.10) ta c o
P(z) = (z a)
m
Q(z) = (z a)
m
Q(z)
=
_
(z a)

m
Q(z) = (z a)
m
Q(z)
v` theo (2.9)
Q(z) = Q(z) Q(z) = Q(z).
Ta c` on ca
`
n ch u
.
ng minh Q(a) ,= 0. Tha
.
t va
.
y v` Q(a) ,= 0 nen
Q(a) ,= 0 v`a do do Q(a) ,= 0 v` doi vo
.
i da th u
.
c vo
.
i he
.
so thu
.
.
c th`
Q(t) = Q(t).
V du
.
4. Gia i phu
.
o
.
ng tr`nh z
3
4z
2
+ 4z 3 = 0.
Gia i. T` u
.
di
.
nh l y 4 suy ra
`
ng cac nghie
.
m nguyen cu a phu
.
o
.
ng tr`nh
vo
.
i he
.
s o nguyen de
`
u l`a u
.
o
.
c cu a so ha
.
ng tu
.
.
do a = 3. So ha
.
ng tu
.
.
do
50 Chu
.
o
.
ng 2. D
-
a th u
.
c v`a h`am h u
.
u ty
a = 3 co cac u
.
o
.
c l`a 1, 3. Ba
`
ng cach kie

m tra ta thu du
.
o
.
.
c z
0
= 3
l`a nghie
.
m nguyen. T` u
.
do
z
3
4z
2
+ 4z 3 = (z 3)(z
2
z + 1)
= (z 3)(z
1
2
+i

3
2
__
z
1
2
i

3
2
_
hay l`a phu
.
o
.
ng tr`nh da cho c o ba nghie
.
m l`a
z
0
= 3, z
1
=
1
2
i

3
2
; z
2
=
1
2
+i

3
2

V du
.
5. Bie

u die

n da th u
.
c P
6
(z) = z
6
3z
4
+ 4z
2
12 du
.
o
.
i da
.
ng:
1
+
tch cac th` u
.
a so tuye

n tnh;
2
+
tch c ac th` u
.
a so tuye

n tnh vo
.
i tam th u
.
c ba
.
c hai vo
.
i he
.
so
thu
.
.
c.
Gia i. Ta t`m mo
.
i nghie
.
m cu a da th u
.
c P(z). V`
z
6
3z
4
+ 4z
2
12 = (z
2
3)(z
4
+ 4)
nen ro r`ang l`a
z
1
=

3, z
2
=

3, z
3
= 1 +i,
z
4
= 1 i, z
5
= 1 +i, z
6
= 1 i.
T` u
.
do
1
+
P
6
(z) = (z

3)(z +

3)(z 1i)(z 1+i)(z +1i)(z +1+i)


2
+
Ba
`
ng cach nhan cac ca
.
p nhi
.
th u
.
c tuye

n tnh tu
.
o
.
ng u
.
ng vo
.
i cac
nghie
.
m ph u
.
c lien ho
.
.
p vo
.
i nhau ta thu du
.
o
.
.
c
P
6
(z) = (z

3)(z +

3)(z
2
2z + 2)(z
2
+ 2z + 2).
V du
.
6. T`m da th u
.
c he
.
so thu
.
.
c c o lu y th` u
.
a thap nhat sao cho c ac
so z
1
= 3, z
2
= 2 i l`a nghie
.
m cu a no.
2.1. D
-
a th u
.
c 51
Gia i. V` da th u
.
c ch co he
.
so thu
.
.
c nen c ac nghie
.
m ph u
.
c xuat hie
.
n
t` u
.
ng ca
.
p lien ho
.
.
p ph u
.
c, ngha l`a ne

u z
2
= 2 i l`a nghie
.
m cu a no th`
z
2
= 2 +i c ung l`a nghie
.
m cu a no. Do do
P(z) = (z 3)(z 2 +i)(z 2 i) = z
3
7z
2
+ 17z 15.
V du
.
7. Phan tch da th u
.
c
(x + 1)
n
(x 1)
n
th`anh cac th` u
.
a so tuye

n tnh.
Gia i. Ta co
P(x) = (x + 1)
n
(x 1)
n
= [x
n
+nx
n1
+. . . ] [x
n
nx
n1
+. . . ] = 2nx
n1
+. . .
Nhu
.
va
.
y P(x) l`a da th u
.
c ba
.
c n 1 vo
.
i he
.
so cao nhat ba
`
ng 2n. Doi
vo
.
i da th u
.
c n`ay ta da bie

t (1) nghie
.
m cu a no:
x
k
= icotg
k
n
, k = 1, 2, . . . , n 1.
Do do
(x + 1)
n
(x 1)
n
= 2n
_
x icotg

n
__
x icotg
2
n
_

_
x icotg
(n 1)
n
_
.
Khi phan tch da th u
.
c tren tru
.
`o
.
ng T th`anh th` u
.
a so ta thu
.
`o
.
ng
ga
.
p nh u
.
ng da th u
.
c khong the

phan tch th`anh tch hai da th u


.
c c o ba
.
c
thap ho
.
n tren c` ung tru
.
`o
.
ng T do. Nh u
.
ng da th u
.
c n`ay du
.
o
.
.
c go
.
i l`a da
th u
.
c bat kha quy.
Cha

ng ha
.
n: da th u
.
c x
2
2 l` a kha quy tren tru
.
`o
.
ng so thu
.
.
c v`:
x
2
2 = (x

2)(x +

2)
52 Chu
.
o
.
ng 2. D
-
a th u
.
c v`a h`am h u
.
u ty
nhu
.
ng bat kha quy tren tru
.
`o
.
ng so h u
.
u ty . Th a
.
t va
.
y, ne

u
x
2
2 = (ax +b)(cx +d); a, b, c, d Q
th` ba
`
ng cach da
.
t x =
b
a
ta co
b
2
a
2
2 = 0

2 =
b
a
v`a

2 l`a so h u
.
u ty . V o l y.
V du
.
8. Phan tch da th u
.
c x
n
1 th`anh tch cac da th u
.
c bat kha
quy tren R.
Gia i. Da
`
u tien ta khai trie

n da th u
.
c da cho th`anh tch cac th` u
.
a
so tuye

n tnh
x
n
1 = (x
0
)(x
1
) (x
n1
),

k
= cos
2k
n
+i sin
2k
n
, k = 0, n 1
v`a tach ra cac nhi
.
th u
.
c thu
.
.
c. Ta c o

k
R ne

u sin
2k
n
= 0 2k
.
.
. n, 0 k < n 1.
T` u
.
do
1
+
Ne

u n l`a so le th` die


`
u do (2k
.
.
. n) ch xa

y ra khi k = 0 (v` k < n)


v`a khi do
0
= 1.
2
+
Ne

u n l`a so cha

n (n = 2m) th` nghie


.
m
k
ch thu
.
.
c khi k = 0
v`a k = m. Do do
0
= 1,
m
= 1. Doi vo
.
i cac gia tri
.
k c`on la
.
i
k
khong l`a so thu
.
.
c. Doi vo
.
i cac gia tri
.
k n`ay ta co
sin
2(n k)
n
= sin
_
2
2k
n
_
= sin
2k
n
v`a do do

nk
=
k

1
=
n1
,
2
=
n2
, . . .
2.1. D
-
a th u
.
c 53
Ma
.
t kh ac
(x
k
)(x
k
) = x
2
(
k
+
k
)x +
k

k
= x
2
x 2 cos
2k
n
+ 1.
Do do
x
n
1 =
_

_
(x 1)
n1
2

k=1
_
x
2
x 2 cos
2k
n
+ 1
_
ne

u n l`a so le,
(x 1)(x + 1)
n2
2

k=1
_
x
2
x 2 cos
2k
n
+ 1
_
ne

u n l`a so cha

n.
B
`
AI T

A
.
P
1. Ch u
.
ng minh ra
`
ng so z
0
= 1 +i l`a nghie
.
m cu a da th u
.
c
P
4
(z) = 3z
4
5z
3
+ 3z
2
+ 4z 2.
T`m cac nghie
.
m c`on la
.
i.
(DS. z
1
= 1 i, z
2
=
1 +

13
6
, z
3
=
1

13
6
)
2. Ch u
.
ng minh ra
`
ng so z
0
= i l`a nghie
.
m cu a da th u
.
c
P
4
(z) = z
4
+z
3
+ 2z
2
+z + 1.
T`m cac nghie
.
m c`on la
.
i.
(DS. z
1
= i, z
2
=
1 +

3i
2
, z
3
=
1 i

3
2
)
3. Xac di
.
nh bo
.
i cu a nghie
.
m z
0
= 1 cu a da th u
.
c
P
4
(z) = z
4
5z
3
+ 9z
2
7z + 2. (DS. 3)
4. Xac di
.
nh bo
.
i cu a nghie
.
m z
0
= 2 cu a da th u
.
c
P
5
(z) = z
5
5z
4
+ 7z
3
2z
2
+ 4z 8. (DS. 3)
54 Chu
.
o
.
ng 2. D
-
a th u
.
c v`a h`am h u
.
u ty
5. T`m da th u
.
c he
.
so thu
.
.
c c o lu y th` u
.
a thap nhat sao cho so z
1
= i l`a
nghie
.
m kep v`a z
2
= 1 i l`a nghie
.
m do
.
n cu a no.
(DS. z
6
+ 2z
5
+ 4z
4
+ 4z
3
+ 5z
2
+ 2z + 2)
6. Phan tch c ac da th u
.
c da cho th`anh tch cac th` u
.
a so tuye

n tnh
1) z
3
6z
2
+ 11z 6 (DS. (z 1)(z 2)(z 3))
2) 6z
4
11z
3
z
2
4
(DS. 6(z 2)
_
z +
2
3
__
z
1 +i

3
2
__
z
1 i

3
2
_
.
3) 3z
4
23z
2
36 (DS. 3(z 3)(z +3)
_
z i
2

3
__
z +i
2

3
_
)
4) z
n
1 (DS. (z
0
)(z
1
) (z
n1
),

k
= cos
2k
n
+i sin
2k
n
, k = 0, n 1)
5) z
4
+ 4 (DS. (z 1 i)(z 1 +i)(z + 1 i)(z + 1 +i))
6) z
4
+ 16
(DS. (z

2(1 +i))(z

2(1 i))(z +

2(1 +i))(z +

2(1 i)))
7) z
4
+ 8z
3
+ 8z 1
(DS. (z i)(z +i)(z + 4

17)(z + 4 +

17))
8) z
3
+z + 2 (DS. (z + 1)
_
z
1 +i

7
2
__
z
1 i

7
2
_
)
7. Phan tch cac da th u
.
c tren tru
.
`o
.
ng so thu
.
.
c th`anh cac da th u
.
c bat
kha quy tren c` ung tru
.
`o
.
ng do.
1) x
3
+x + 2 (DS. (x + 1)(x
2
x + 2))
2) x
4
+ 16 (DS. (x
2
2x

2 + 4)(x
2
+ 2

2x + 4))
3) x
4
+ 8x
3
+ 8x 1 (DS. (x
2
+ 1)(x + 4

17)(x + 4 +

17))
4) x
4
+ 2x
3
+ 3x
2
+ 2x 3
(DS.
_
x

5 1
2
__
x +

5 + 1
2
_
(x
2
+x + 3))
5) x
10
2x
5
+ 2 (DS.
4

k=0
_
x
2
2
10

2 cos
8k + 1
20
+
5

2
_
)
6) x
4
+x
3
+x
2
+x + 1
2.2. Phan th u
.
c h u
.
u ty 55
(DS.
_
x
2

5 1
2
x + 1
__
x
2
+

5 + 1
2
x + 1
_
)
Ch da

n. Da
.
t x
2
l`am th` u
.
a so chung ro
`
i d` ung phep do

i bie

n y =
x +
1
x
7) x
2n
1 (DS. (x
2
1)
n1

k=1
(x
2
2xcos
k
n
+ 1))
8) x
2n+1
1 (DS. (x 1)
n

k=1
_
x
2
2xcos
2k
2n + 1
+ 1
_
)
2.2 Phan th u
.
c h u
.
u ty

Mo
.
t h`am so xac di
.
nh du
.
o
.
i da
.
ng thu
.
o
.
ng cu a hai da th u
.
c da
.
i so ta
.
i
nh u
.
ng die

m m`a ma

u so khong trie
.
t tieu go
.
i l`a phan th u
.
c h u
.
u ty .
1(x) =
P(x)
Q(x)
, Q(x) ,= 0.
Ne

u degP < degQ th` R(x) go


.
i l`a phan th u
.
c h u
.
u ty thu
.
.
c su
.
.
. Ne

u
degP degQ th` R(x) du
.
o
.
.
c go
.
i l`a phan th u
.
c h u
.
u ty khong thu
.
.
c su
.
.
.
Ne

u degP degQ th` ba


`
ng cach thu
.
.
c hie
.
n phep chia P(x) cho
Q(x) ta thu du
.
o
.
.
c
P(x)
Q(x)
= W(x) +
P
1
(x)
Q(x)
(2.12)
trong do W(x) l`a da th u
.
c, c`on
P
1
(x)
Q(x)
l`a phan th u
.
c h u
.
u ty thu
.
.
c su
.
.
.
Ve
`
sau ta ch xet cac phan th u
.
c h u
.
u ty l`a thu
.
o
.
ng cu a hai da th u
.
c
da
.
i so vo
.
i he
.
so thu
.
.
c (phan th u
.
c nhu
.
va
.
y du
.
o
.
.
c go
.
i l`a phan th u
.
c h u
.
u
ty vo
.
i he
.
so thu
.
.
c).
Phan th u
.
c thu
.
.
c do
.
n gia n nhat (c`on go
.
i l`a phan th u
.
c co
.
ba n) l`a
nh u
.
ng phan th u
.
c du
.
o
.
.
c bie

u die

n toi gia n bo
.
i mo
.
t trong hai da
.
ng sau
day
I.
A
(x )
m
; II.
Bx +C
(x
2
+px +q)
m
; A, B, C, p, q R.
56 Chu
.
o
.
ng 2. D
-
a th u
.
c v`a h`am h u
.
u ty
T` u
.
di
.
nh l y Gauss v` a cac he
.
qua cu a no ta c o
D
-
i
.
nh l y. Mo
.
i ph an th u
.
c h u
.
u ty thu
.
.
c su
.
.
P(x)
Q(x)
he
.
s o thu
.
.
c vo
.
i ma

u
s o co da
.
ng
Q(x) = (x )
r
(x )
s
(x
2
+p
1
x +q
1
)
m

(x
2
+p
2
x +q
2
)

(x
2
+p
s
x +q
s
)
n
(2.13)
de
`
u c o the

bie

u die

n du
.
o
.
i da
.
ng to

ng h u
.
u ha
.
n c ac phan th u
.
c co
.
ba n
da
.
ng I v` a II
P(x)
Q(x)
=
A
(x )
r
+
B
(x )
r1
+ +
C
x
+
+
D
(x )
s
+
E
(x )
s1
+ +
F
x
+
. . . . . . . . . . . . . . . . . . . . . . . .
+
Gx +H
(x
2
+p
1
x +q
1
)
m
+
Ix +H
(x
2
+p
1
x +q
1
)
m1
+ +
Lx +M
x
2
+p
1
x +q
1
+
. . . . . . . . . . . . . . . . . . . . . . . .
+
Nx +P
(x
2
+p
s
x +q
s
)
n
+
Qx +R
(x
2
+p
s
x +q
s
)
n1
+ +
Sx +T
x
2
+p
s
x +q
s
,
(2.14)
trong do A, B, . . . l`a nh u
.
ng ha
`
ng so thu
.
.
c.
Nhu
.
va
.
y cac phan th u
.
c co
.
ba n o
.
ve

pha i cu a (2.14) sa

p xe

p theo
t` u
.
ng nhom tu
.
o
.
ng u
.
ng vo
.
i cac th` u
.
a so o
.
ve

pha i cu a (2.13), trong do


so so ha
.
ng cu a mo

i nhom ba
`
ng so m u cu a lu y th` u
.
a cu a th` u
.
a so tu
.
o
.
ng
u
.
ng.
Ca
`
n lu
.
u y ra
`
ng khi khai trie

n phan th u
.
c cu
.
the

theo cong th u
.
c
(2.14) mo
.
t so he
.
so co the

ba
`
ng 0 v`a do do so so ha
.
ng trong mo

i nhom
co the

be ho
.
n so m u cu a th` u
.
a so tu
.
o
.
ng u
.
ng.
Trong thu
.
.
c h`anh, de

tnh cac he
.
so A, B, . . . ta se su
.
du
.
ng cac
phu
.
o
.
ng phap sau.
2.2. Phan th u
.
c h u
.
u ty 57
I. Gia su
.
da th u
.
c Q(x) ch co c ac nghie
.
m thu
.
.
c do
.
n, t u
.
c l`a
Q(x) =
n

j=1
(x a
j
), a
i
,= a
j
i ,= j.
Khi do
P(x)
Q(x)
=
n

j=1
A
j
x a
j
(2.15)
De

xac di
.
nh A
k
ta nhan hai ve

cu a (2.15) vo
.
i x a
k
v`a thu du
.
o
.
.
c
P(x)
n

j=1
j=k
(x a
j
)
= A
k
+
_
A
1
x a
1
+ +
A
k1
x a
k1
+
A
k+1
x a
k+1
+ +
A
n
x a
n
_
(x a
k
). (2.16)
Thay x = a
k
v`ao (2.16) ta c o
A
k
=
P(a
k
)
n

j=1
j=k
(a
k
a
j
)
(2.17)
Nhu
.
va
.
y de

tnh he
.
so A
k
cu a phan th u
.
c
A
k
x a
k
ta xoa th` u
.
a so
(x a
k
) khoi ma

u so cu a
P(x)
Q(x)
v`a tie

p theo l`a thay x = a


k
v`ao bie

u
th u
.
c c` on la
.
i. V` va
.
y phu
.
o
.
ng ph ap n`ay du
.
o
.
.
c go
.
i l`a phu
.
o
.
ng phap x oa.
II. Ne

u Q(x) co nghie
.
m bo
.
i th` cong th u
.
c (2.17) khong c`on su
.
du
.
ng
du
.
o
.
.
c. Gia su
.
Q(x) = g
m
, trong do hoa
.
c g = x hoa
.
c g l`a tch cac
th` u
.
a so l`a tam th u
.
c ba
.
c hai vo
.
i hai bie
.
t so am. Trong tru
.
`o
.
ng ho
.
.
p
n`ay ta ca
`
n khai trie

n P(x) theo c ac lu y th` u


.
a cu a g:
P(x) = a
0
+a
1
g +a
2
g
2
+. . .
58 Chu
.
o
.
ng 2. D
-
a th u
.
c v`a h`am h u
.
u ty
trong do a
0
, a
1
, . . . l`a h a
`
ng so ne

u g = x v`a l`a da th u
.
c ba
.
c khong
vu
.
o
.
.
t qua 1 trong tru
.
`o
.
ng ho
.
.
p th u
.
hai (trong tru
.
`o
.
ng ho
.
.
p n`ay ta ca
`
n
thu
.
.
c hie
.
n theo quy ta

c phep chia c o du
.
).
III. Doi vo
.
i tru
.
`o
.
ng ho
.
.
p to

ng quat, ta nhan hai ve

cu a (2.14) vo
.
i
da th u
.
c Q(z) v`a sa

p xe

p cac so ha
.
ng o
.
ve

pha i da

ng th u
.
c thu du
.
o
.
.
c
th`anh da th u
.
c v`a thu du
.
o
.
.
c d o
`
ng nhat th u
.
c gi u
.
a hai da th u
.
c: mo
.
t da
th u
.
c l`a P(x), c`on da th u
.
c kia l`a da th u
.
c vo
.
i he
.
so A, B, . . . chu
.
a du
.
o
.
.
c
xac di
.
nh. C an ba
`
ng cac he
.
so cu a cac lu y th` u
.
a c` ung ba
.
c ta thu du
.
o
.
.
c
he
.
phu
.
o
.
ng tr`nh tuye

n tnh vo
.
i a

n l`a A, B, . . . .
Gia i he
.
do, ta t`m du
.
o
.
.
c cac he
.
so A, B, . . . Phu
.
o
.
ng phap n`ay go
.
i
l`a phu
.
o
.
ng phap he
.
s o b at di
.
nh.
Ta co the

xac di
.
nh he
.
so ba
`
ng cach khac l`a cho bie

n x trong do
`
ng
nhat th u
.
c nh u
.
ng tri
.
so t` uy y (cha

ng ha
.
n cac gia tri
.
do l`a nghie
.
m thu
.
.
c
cu a ma

u so).
C

AC V

I DU
.
V du
.
1. Khai trie

n cac phan th u
.
c h u
.
u ty sau th`anh to

ng cac phan
th u
.
c co
.
ba n
1)
2x
3
+ 4x
2
+x + 2
(x 1)
2
(x
2
+x + 1)
, 2)
x
2
2x
(x 1)
2
(x
2
+ 1)
2

Gia i. 1) V` tam th u
.
c ba
.
c hai x
2
+x+1 khong co nghie
.
m thu
.
.
c nen
R
1
(x) =
2x
3
+ 4x
2
+x + 2
(x 1)
2
(x
2
+x + 1)
=
B
1
(x 1)
+
B
2
(x 1)
2
+
Mx +N
x
2
+x + 1

Quy do
`
ng ma

u so ta co
2x
3
+ 4x
2
+x + 2
(x 1)
2
(x
2
+x + 1)
=
B
1
(x
3
1) +B
2
(x
2
+x + 1) + (Mx +N)(x
2
2x + 1)
(x 1)
2
(x
2
+x + 1)

2.2. Phan th u
.
c h u
.
u ty 59
Can ba
`
ng he
.
so cu a x
0
, x
1
, x
2
v`a x
3
trong cac tu
.
so ta thu du
.
o
.
.
c he
.
phu
.
o
.
ng tr`nh
x
3

B
1
+B
2
+N = 2,
x
2

B
2
+M 2N = 1,
x
1

B
2
+N 2M = 4,
x
0

B
1
+M = 2.
Gia i he
.
phu
.
o
.
ng tr`nh ta c o B
1
= 2, B
2
= 3, M = 0, N = 1. T` u
.
do
R
1
(x) =
2
x 1
+
3
(x 1)
2
+
1
x
2
+x + 1

2) Ta c o
R
2
=
x
2
2x
(x 1)
2
(x
2
+ 1)
2
=
A
1
x 1
+
A
2
(x 1)
2
+
M
1
x +N
1
x
2
+ 1
+
M
2
x +N
2
(x
2
+ 1)
2

Quy do
`
ng ma

u so v`a can ba
`
ng cac tu
.
so ta c o
x
2
2x = A
1
(x 1)(x
2
+ 1)
2
+A
2
(x
2
+ 1)
2
+ (M
1
x +N
1
)(x 1)
2
(x
2
+ 1)
+ (M
2
x +N
2
)(x 1)
2
.
So sanh cac he
.
so cu a cac lu y th` u
.
a c` ung ba
.
c o
.
hai ve

ta thu du
.
o
.
.
c
x
5

A
1
+M
1
= 0,
x
4

A
1
+A
2
2M
1
+N
1
= 0,
x
3

2A
1
+ 2M
1
2N
1
+M
2
= 0,
x
2

2A
1
+ 2A
2
2M
1
+ 2N
1
+ 2N
1
2M
2
+N
2
= 1,
x
1

A
1
+M
1
2N
1
+M
2
2N
2
= 2,
x
0

A
1
+A
2
+N
1
+N
2
= 0.
T` u
.
do suy ra
A
1
=
1
2
, A
2
=
1
4
, M
1
=
1
2
,
N
1
=
1
4
, M
2
=
1
2
, N
2
= 1
60 Chu
.
o
.
ng 2. D
-
a th u
.
c v`a h`am h u
.
u ty
v`a do va
.
y
x
2
2x
(x 1)
2
(x
2
+ 1)
2
=
1
2
x 1
+

1
4
(x 1)
2
+

1
2
x
1
4
x
2
+ 1
+

1
2
x + 1
(x
2
+ 1)
2

V du
.
2. C ung hoi nhu
.
tren
1) R
1
(x) =
x
4
x
4
+ 5x
2
+ 1
; 2) R
2
(x) =
1
x
4
+ 1

Gia i. 1) R
1
(x) l`a phan th u
.
c h u
.
u ty khong thu
.
.
c su
.
.
nen da
`
u tien
ca
`
n thu
.
.
c hie
.
n phep chia:
x
4
x
4
+ 5x
2
+ 4
= 1
5x
2
+ 4
x
4
+ 5x
2
+ 4
= 1 +R
3
(x).
Ch u y ra
`
ng x
4
+ 5x
2
+ 4 = (x
2
+ 1)(x
2
+ 4), do do
R
3
=
5x
2
+ 4
(x
2
+ 1)(x
2
+ 4)
=
M
1
x +N
1
x
2
+ 1
+
M
2
x +N
2
x
2
+ 4

Quy do
`
ng ma

u so v`a so sanh hai tu


.
so ta thu du
.
o
.
.
c
5x
2
4 = (M
1
x +N
1
)(x
2
+ 4) + (M
2
x +N
2
)(x
2
+ 1)
v`a tie

p theo l`a can ba


`
ng cac he
.
so cu a cac lu y th` u
.
a c` ung ba
.
c cu a x ta
thu du
.
o
.
.
c he
.
phu
.
o
.
ng tr`nh
x
3

M
1
+M
2
= 0,
x
2

N
1
+N
2
= 5,

M
1
= M
2
= 0, N
1
=
1
3
, N
2
=
16
3

x
1

4M
1
+N 2 = 0,
x
0

4N
1
+N 2 = 4
Va
.
y
R
1
(x) = 1 +
1
3

1
x
2
+ 1

16
3

1
x
2
+ 4

2.2. Phan th u
.
c h u
.
u ty 61
2) V` x
4
+1 = (x
2
+1)
2
2x
2
= (x
2
+

2x+1)(x
2

2x+1) nen
R
2
=
1
x
4
+ 1
=
M
1
x +N
1
x
2
+

2x + 1
+
M
2
x +N
2
x
2

2x + 1

T` u
.
do
`
ng nhat th u
.
c
1 (M
1
x +N
1
)(x
2

2x + 1) + (M + 2x +N
2
)(x
2
+

2x + 1),
tie

n h`anh tu
.
o
.
ng tu
.
.
nhu
.
tren ta co
M
1
= M
2
=
1
2

2
, N
1
= N
2
=
1
2

Do do
1
x
4
+ 1
=
1
2

2
x +

2
x
2
+

2x + 1

1
2

2
x

2
x
2

2x + 1

V du
.
3. T`m khai trie

n phan th u
.
c
1) R
1
(x) =
x + 1
(x 1)(x 2)x
; 2) R
2
(x) =
x
2
+ 2x + 6
(x 1)(x 2)(x 4)

Gia i. 1) V` ma

u so ch co nghie
.
m do
.
n 0, 1, 2 nen
x + 1
x(x 1)(x 2)
=
A
1
x
+
A
2
x 1
+
A
2
x 2

Ap du
.
ng cong th u
.
c (2.17) ta du
.
o
.
.
c
A
1
=
x + 1

x=0
(x 1)(x 2)

x=0
=
1
2
;
A
2
=
x + 1
x(x 2)

x=1
= 2, A
3
=
x + 1
x(x 1)

x=2
=
3
2

Va
.
y
R
1
(x) =
1
2x
+
2
x 1
+
3
2(x 2)

62 Chu
.
o
.
ng 2. D
-
a th u
.
c v`a h`am h u
.
u ty
2) Tu
.
o
.
ng tu
.
.
ta co
R
2
(x) =
x
2
+ 2x + 6
(x 1)(x 2)(x 4)
=
A
1
x 1
+
B
x 2
+
C
x 3
V` ma

u so cu a R
2
(x) ch co nghie
.
m do
.
n nen
A =
x
2
+ 2x + 6
(x 2)(x 4)

x=1
= 3,
B =
x
2
+ 2x + 6
(x 1)(x 4)

x=2
= 7,
C =
x
2
+ 2x + 6
(x 1)(x 2)

x=4
= 5.
Do do
R
2
(x) =
3
x 1

7
x 2
+
5
x 4

Nha
.
n xet. Trong mo
.
t so tru
.
`o
.
ng ho
.
.
p da
.
c bie
.
t, vie
.
c khai trie

n phan
th u
.
c h u
.
u ty co the

thu du
.
o
.
.
c do
.
n gia n ho
.
n v`a nhanh ho
.
n. Ch a

ng ha
.
n,
de

khai trie

n phan th u
.
c
1
x
2
(1 +x
2
)
2
th`anh to

ng cac phan th u
.
c co
.
ba n
ta co the

thu
.
.
c hie
.
n nhu
.
sau:
1
x
2
(x
2
+ 1)
2
=
(1 +x
2
) x
2
x
2
(x
2
+ 1)
2
=
1
x
2
(x
2
+ 1)

1
(x
2
+ 1)
2
=
(1 +x
2
) x
2
x
2
(x
2
+ 1)

1
(x
2
+ 1)
2
=
1
x
2

1
x
2
+ 1

1
(x
2
+ 1)
2

V du
.
4. Khai trie

n cac phan th u
.
c h u
.
u ty sau:
1)
x
4
+ 5x
3
+ 5x
2
3x + 1
(x + 2)
5
; 2)
x
5
+ 3x
4
+x
3
2x
2
+ 2x + 3
(x
2
+x + 1)
3

Gia i. 1) Da
.
t g = (x + 2). Khi do ba
`
ng cach khai trie

n tu
.
so theo
cac lu y th` u
.
a cu a x+2 ba
`
ng cach ap du
.
ng cong th u
.
c nhi
.
th u
.
c Newton
2.2. Phan th u
.
c h u
.
u ty 63
ta thu du
.
o
.
.
c
x
4
+ 5x
3
+ 5x
2
3x + 1
(x + 2)
5
=
=
[(x + 2) 2]
4
+ 5[(x + 2) 2]
3
+ 5[(x + 2) 2]
2
3[(x + 2) 2)] + 1
(x + 2)
5
=
3 + 5g g
2
3g
3
+g
4
g
5
=
3
g
5
+
5
g
4

1
g
3

3
g
2
+
1
g
=
3
(x + 2)
5
+
5
(x + 2)
4

1
(x + 2)
3

3
(x + 2)
3
+
1
x + 2

2) Da
.
t g = x
2
+ x + 1. Do l`a tam th u
.
c ba
.
c hai khong co nghie
.
m
thu
.
.
c.

Ap du
.
ng thua
.
t toan chia c o du
.
ta co
P(x) = x
5
+ 3x
4
+x
3
2x
2
+ 2x + 3
= (x
2
+x + 1)(x
3
+ 2x
2
2x 2) + 6x + 5
t u
.
c l` a
P = g q
1
+r
1
, q
1
= x
3
+ 2x
2
2x 2, r
1
= 6x + 5.
Ta la
.
i chia q
1
cho g v`a thu du
.
o
.
.
c
q
1
= gq
2
+r
2
, degq
2
< deg(g)
q
2
= x + 1, r
2
= 4x 3.
Nhu
.
va
.
y
P = gq
1
+r
1
= r
1
+g(r
2
+gq
2
)
= r
1
+r
2
g +q
2
g
2
.
T` u
.
do suy ra
P
g
3
=
r
1
g
3
+
r
2
g
3
+q
2

1
g
=
6x + 5
(x
2
+x + 1)
3

4x + 3
(x
2
+x + 1)
2
+
x + 1
x
2
+x + 1

64 Chu
.
o
.
ng 2. D
-
a th u
.
c v`a h`am h u
.
u ty
B
`
AI T

A
.
P
Trong c ac b`ai toan sau day, hay khai trie

n phan th u
.
c h u
.
u ty da
cho th`anh to

ng h u
.
u ha
.
n cac phan th u
.
c co
.
ba n thu
.
.
c.
1.
2x 3
x(x
2
1)(x
2
4)
(DS.
3
4x
+
1
6(x 1)
+
5
6(x + 1)
+
1
24(x 2)

7
24(x + 2)
)
2.
x + 1
x
3
1
(DS.
2
3(x 1)

2x + 1
3(x
2
+x + 1)
)
3.
1
x
3
(x 1)
4
(DS.
10
x
+
4
x
2
+
1
x
3

10
x 1
+
6
(x 1)
2

3
(x 1)
3
+
1
(x 1)
4
)
4.
1
(x
4
1)
2
(DS.
3
16(x 1)
+
1
16(x 1)
2
+
3
16(x + 1)
+
1
16(x + 1)
2
+
1
4(x
2
+ 1)
+
1
4(x
2
+ 1)
2
)
5.
2x 1
(x + 1)
3
(x
2
+x + 1)
(DS.
2
x + 1

1
(x + 1)
2

3
(x + 1)
3

2x 1
x
2
+x + 1
)
6.
1
x(x
2
+ 1)
3
(DS.
1
x
+
x
(x
2
+ 1)
3

x
(x
2
+ 1)
2

x
x
2
+ 1
)
7.
x
2
+ 3x + 1
x
4
(x
2
+ 1)
(DS.
1
x
4
+
3
x
3

3
x
+
3x
x
2
+ 1
)
8.
x
5
+ 3x
3
x
2
+ 4x 2
(x
2
+ 1)
3
(DS.
2x 1
(x
2
+ 1)
3
+
x 1
(x
2
+ 1)
2
+
x
x
2
+ 1
)
9.
x
5
+ 2x
3
6x
2
3x 9
(x
2
+x + 2)
3
(DS.
1
(x
2
+x + 2)
3
+
x 1
(x
2
+x + 2)
2
+
x 2
x
2
+x + 2
)
10.
2x 1
x(x + 1)
2
(x
2
+x + 1)
2
2.2. Phan th u
.
c h u
.
u ty 65
(DS.
1
x
+
7
x + 1
+
3
(x + 1)
2

6x + 2
x
2
+x + 1

3x + 2
(x
2
+x + 1)
2
)
11.
x
2
(x
2
+ 1)(x
2
+x + 1)
2
(DS.
1
x
2
+ 1
+
1
x
2
+x + 1

x
(x
2
+x + 1)
2
)
12.
1
x
5
x
4
+x
3
x
2
+x 1
(DS.
1
3(x 1)

1
6
2x + 1
x
2
+x + 1

1
2(x
2
x + 1)
)
Chu
.
o
.
ng 3
Ma tra
.
n. D
-
i
.
nh th u
.
c
3.1 Ma tra
.
n . . . . . . . . . . . . . . . . . . . . . 67
3.1.1 D
-
i
.
nh ngha ma tra
.
n . . . . . . . . . . . . . 67
3.1.2 Cac phep toan tuye

n tnh tren ma tra


.
n . . 69
3.1.3 Phep nhan cac ma tra
.
n . . . . . . . . . . . 71
3.1.4 Phep chuye

n vi
.
ma tra
.
n . . . . . . . . . . . 72
3.2 D
-
i
.
nh th u
.
c . . . . . . . . . . . . . . . . . . . . 85
3.2.1 Nghi
.
ch the

. . . . . . . . . . . . . . . . . . 85
3.2.2 D
-
i
.
nh th u
.
c . . . . . . . . . . . . . . . . . . . 85
3.2.3 Tnh chat cu a di
.
nh th u
.
c . . . . . . . . . . . 88
3.2.4 Phu
.
o
.
ng phap tnh di
.
nh th u
.
c . . . . . . . . 89
3.3 Ha
.
ng cu a ma tra
.
n . . . . . . . . . . . . . . . 109
3.3.1 D
-
i
.
nh ngha . . . . . . . . . . . . . . . . . . 109
3.3.2 Phu
.
o
.
ng phap t`m ha
.
ng cu a ma tra
.
n . . . . 109
3.4 Ma tra
.
n nghi
.
ch da o . . . . . . . . . . . . . . 118
3.4.1 D
-
i
.
nh ngha . . . . . . . . . . . . . . . . . . 118
3.1. Ma tra
.
n 67
3.4.2 Phu
.
o
.
ng phap t`m ma tra
.
n nghi
.
ch dao . . . 119
3.1 Ma tra
.
n
Gia su
.
T l`a tru
.
`o
.
ng so n`ao do (T = R, C).
3.1.1 D
-
i
.
nh ngha ma tra
.
n
Ta xet ba ng h`nh ch u
.
nha
.
t la
.
p nen t` u
.
mn so cu a T:
a
11
a
12
. . . a
1n
a
21
a
22
. . . a
2n
.
.
.
.
.
.
.
.
.
.
.
.
a
m1
a
m2
. . . a
mn
Ba ng so n`ay du
.
o
.
.
c go
.
i l`a ma tra
.
n (hay chnh xac ho
.
n: ma tr a
.
n s o)
kch thu
.
o
.
c m n. C ac so a
ij
, i = 1, m, j = 1, n du
.
o
.
.
c go
.
i l`a pha
`
n
tu
.
cu a ma tra
.
n, trong do i ch s o hie
.
u h`ang, j ch so hie
.
u co
.
t cu a ma
tra
.
n.
K y hie
.
u: c o the

d` ung mo
.
t trong cac k y hie
.
u
A =
_

_
a
11
a
12
. . . a
1n
a
21
a
22
. . . a
2n
.
.
.
.
.
.
.
.
.
.
.
.
a
m1
a
m2
. . . a
mn
_

_
, hay
_
_
_
_
_
_
a
11
a
12
. . . a
1n
a
21
a
22
. . . a
2n
.
.
.
.
.
.
.
.
.
.
.
.
a
m1
a
m2
. . . a
mn
_
_
_
_
_
_
hay
_
_
_
_
_
_
_
_
_
_
a
11
a
12
. . . a
1n
a
21
a
22
. . . a
2n
.
.
.
.
.
.
.
.
.
.
.
.
a
m1
a
m2
. . . a
mn
_
_
_
_
_
_
_
_
_
_
68 Chu
.
o
.
ng 3. Ma tra
.
n. D
-
i
.
nh th u
.
c
hay nga

n go
.
n ho
.
n
A =
_
_
a
ij
_
_
mn
=
_
a
ij
_
mn
=
_
a
ij

mn
.
Ta
.
p ho
.
.
p mo
.
i (mn)-ma tra
.
n du
.
o
.
.
c k y hie
.
u l`a /(mn).
Ne

u m = n th` ma tra
.
n A =
_
_
a
ij
_
_
mn
du
.
o
.
.
c go
.
i l`a ma tr a
.
n vu ong
cap n (thu
.
`o
.
ng k y hie
.
u: A =
_
_
a
ij
_
_
nn
=
_
_
a
ij
_
_
n
1
). Doi v o
.
i ma tra
.
n
vuong A =
_
_
a
ij
_
_
n
1
cac pha
`
n tu
.
a
ii
, i = 1, n du
.
o
.
.
c go
.
i l`a nh u
.
ng pha
`
n
tu
.
du
.
`o
.
ng cheo. C ac pha
`
n tu
.
n`ay la
.
p th`anh du
.
`o
.
ng cheo chnh cu a ma
tra
.
n vuong.
Ma tra
.
n vuong m`a mo
.
i pha
`
n tu
.
khong na
`
m tren du
.
`o
.
ng cheo chnh
de
`
u ba
`
ng 0 (t u
.
c l`a a
ij
= 0 i ,= j) go
.
i l`a ma tr a
.
n du
.
` o
.
ng cheo:
A =
_

_
d
1
d
2
_
.
.
.
_
.
.
.
d
n
_

_
= diag[d
1
d
2
. . . d
n
].
Ne

u trong ma tra
.
n du
.
`o
.
ng cheo A mo
.
i pha
`
n tu
.
d
1
= d
2
= = d
n
= 1
th` ma tra
.
n do du
.
o
.
.
c go
.
i l`a ma tra
.
n do
.
n vi
.
cap n v`a k y hie
.
u:
E
n
= E =
_

_
1
1 _
.
.
.
_
.
.
.
1
_

_
.
Nhu
.
va
.
y E
n
=
_
_

ij
_
_
n
1
, trong do
ij
=
_
_
_
0 ne

u i ,= j
1 ne

u i = j.
3.1. Ma tra
.
n 69
Sau c` ung, (mn)-ma tra
.
n da
.
ng
O
mn
=
_

_
0 0 . . . 0
0 0 . . . 0
.
.
.
.
.
.
.
.
.
.
.
.
0 0 . . . 0
_

_
go
.
i l`a ma tra
.
n - khong kch thu
.
o
.
c mn. Ne

u m = n th` k y hie
.
u O
n
hay O
n
1
.
Nha
.
n xet. 1) Ta nhan ma
.
nh: ma tra
.
n A =
_
_
a
ij
_
_
mn
khong pha i l`a
mo
.
t so, no l`a mo
.
t Ba ng cac s o.
2) Ma tra
.
n kch thu
.
o
.
c (1 n) go
.
i l`a ma tra
.
n h`ang
_
a
1
, a
2
, . . . , a
n
_
c`on ma tra
.
n (m1) go
.
i l`a ma tra
.
n co
.
t
_

_
a
1
a
2
.
.
.
a
m
_

_
3.1.2 Cac phep toan tuye

n tnh tren ma tra


.
n
Gia su
.
mo
.
i ma tra
.
n du
.
o
.
.
c xet l`a tren c` ung mo
.
t tru
.
`o
.
ng T (= R, C).
Cac phep toan tuye

n tnh tren ta
.
p ho
.
.
p cac ma tra
.
n l`a phep co
.
ng cac
ma tra
.
n (ch doi vo
.
i cac ma tra
.
n c` ung kch thu
.
o
.
c!) v`a phep nhan ma
tra
.
n vo
.
i mo
.
t so v`a ch ung du
.
o
.
.
c di
.
nh ngha nh`o
.
cac phep toan tren c ac
pha
`
n tu
.
cu a ch ung.
1. Cho A =
_
a
ij

mn
, B =
_
b
ij

mn
. Ma tra
.
n C =
_
c
ij

mn
du
.
o
.
.
c
go
.
i l`a to

ng cu a A v`a B ne

u
c
ij
= a
ij
+b
ij
i = 1, m, j = 1, n
70 Chu
.
o
.
ng 3. Ma tra
.
n. D
-
i
.
nh th u
.
c
v`a k y hie
.
u
C = A+B
_
[c
ij
] = [a
ij
+b
ij
], i = 1, m, j = 1, n
_
.
2. Gia su
.
A =
_
a
ij

mn
v`a T. Ma tra
.
n C =
_
c
ij

mn
du
.
o
.
.
c go
.
i
l`a tch cu a ma tra
.
n A vo
.
i so ne

u
c
ij
= a
ij
i = 1, m, j = 1, n
v`a k y hie
.
u
C = A
_
A =
_
a
ij

mn
).
Tru
.
`o
.
ng ho
.
.
p da
.
c bie
.
t khi = 1 ta vie

t (1)A = A v`a go
.
i A
l`a ma tra
.
n doi cu a ma tra
.
n A.
Cac phep toan tuye

n tnh tren ta
.
p ho
.
.
p ma tra
.
n /(mn) c o cac
tnh chat sau day.
Gia su
.
A, B, C /(mn) v`a , T. Khi do
I. A+B = B +A (lua
.
t giao hoan).
II. A+ (B +C) = (A+B) +C (lua
.
t ke

t ho
.
.
p).
III. A+O
mn
= A.
IV. A+ (A) = O
mn
.
V. 1 A = A.
VI. (A) = ()A - lua
.
t ke

t ho
.
.
p doi vo
.
i phep nhan cac so.
VII. (A+B) = A+B - lua
.
t phan bo cu a phep nhan vo
.
i mo
.
t
so doi vo
.
i phep co
.
ng ma tra
.
n.
VIII. ( +)A = A+ A - lua
.
t phan bo cu a phep nhan vo
.
i ma
tra
.
n doi vo
.
i phep co
.
ng cac so.
Hie
.
u cac ma tra
.
n AB co the

di
.
nh ngha nhu
.
sau
AB
def
= A+ (B).
3.1. Ma tra
.
n 71
3.1.3 Phep nhan cac ma tra
.
n
Ma tra
.
n A du
.
o
.
.
c go
.
i l`a tu
.
o
.
ng thch vo
.
i ma tra
.
n B ne

u so co
.
t cu a ma
tra
.
n A ba
`
ng so h`ang cu a ma tra
.
n B (t` u
.
su
.
.
tu
.
o
.
ng thch cu a A vo
.
i B
noi chung khong suy ra du
.
o
.
.
c ra
`
ng ma tra
.
n B tu
.
o
.
ng thch vo
.
i ma tra
.
n
A).
Cho ma tra
.
n A =
_
a
ij

mn
v` a B =
_
b
ij

np
. Ma tra
.
n C =
_
c
ij

mp
du
.
o
.
.
c go
.
i l`a tch cu a ma tra
.
n A vo
.
i ma tra
.
n B ne

u
c
ij
=
n

s=1
a
is
b
sj
. (3.1)
K y hie
.
u C = AB v`a noi ra
`
ng nhan ben pha i ma tra
.
n A vo
.
i ma
tra
.
n B hay nhan ben trai ma tra
.
n B vo
.
i ma tra
.
n A.
T` u
.
(3.1) suy ra quy ta

c t`m cac so ha
.
ng cu a tch cac ma tra
.
n:
pha
`
n tu
.
c
ij
d u
.
ng o
.
vi
.
tr giao cu a h`ang th u
.
i v`a co
.
t th u
.
j cu a ma
tra
.
n C = AB ba
`
ng to

ng cac tch cu a cac pha


`
n tu
.
h`ang th u
.
i cu a ma
tra
.
n A nhan vo
.
i cac pha
`
n tu
.
tu
.
o
.
ng u
.
ng cu a co
.
t th u
.
j cu a ma tra
.
n
B.
_

_
a
11
a
12
. . . a
1n
. . . . . . . . . . . .
a
i1
a
i2
. . . a
in
. . . . . . . . . . . .
a
m1
a
m2
. . . a
mn
_

_
b
11
.
.
.
b
n1
_

_
b
ij
.
.
.
b
ij
_

_
b
1p
.
.
.
b
np
_

_
=
_

_
c
11
.
.
. c
1p
. . . c
ij
. . .
c
m1
.
.
. c
mp
_

_
Ch u y. 1) Noi chung phep nhan ma tra
.
n khong co tnh chat giao
hoan.
2) Tch hai ma tra
.
n khac 0 c o the

ba
`
ng ma tra
.
n khong.
3) V o
.
i die
`
u kie
.
n cac phep to an du
.
o
.
.
c vie

t ra co ngha, phep nhan


ma tra
.
n co cac tnh chat sau
I. (AB)C = A(BC) - lua
.
t ke

t ho
.
.
p.
II. (AB) = (A)B = A(B), T.
III. (A+B)C = AC +BC (lua
.
t phan bo phep nhan ben pha i
72 Chu
.
o
.
ng 3. Ma tra
.
n. D
-
i
.
nh th u
.
c
doi vo
.
i phep co
.
ng ma tra
.
n).
IV. C(A+B) = CA+CB (lua
.
t phan bo phep nhan ben trai
doi vo
.
i phep co
.
ng ma tra
.
n).
3.1.4 Phep chuye

n vi
.
ma tra
.
n
Phep toan tren c ac ma tra
.
n m`a trong do cac h`ang chuye

n th`anh cac
co
.
t c` on cac co
.
t chuye

n th`anh cac h`ang du


.
o
.
.
c go
.
i l`a phep chuye

n vi
.
ma
tra
.
n.
Cho ma tra
.
n A =
_
a
ij

mn
. Ma tra
.
n thu du
.
o
.
.
c t` u
.
ma tra
.
n A ba
`
ng
phep chuye

n vi
.
ma tra
.
n du
.
o
.
.
c go
.
i l`a ma tr a
.
n chuye

n vi
.
doi vo
.
i ma tra
.
n
A v`a du
.
o
.
.
c k y hie
.
u l`a A
T
. Nhu
.
va
.
y: A
T
l`a (n m)-ma tra
.
n.
Ma tra
.
n vuong du
.
o
.
.
c go
.
i l`a ma tr a
.
n doi x u
.
ng ne

u A
T
= A v`a du
.
o
.
.
c
go
.
i l`a ma tra
.
n pha n x u
.
ng ne

u A
T
= A. Nhu
.
va
.
y ne

u A =
_
a
ij

n
1
l`a
ma tra
.
n doi x u
.
ng th` a
ij
= a
ji
i, j = 1, n v`a ne

u A pha n x u
.
ng th`
a
ij
= a
ji
. Do do cac pha
`
n tu
.
tren du
.
`o
.
ng cheo chnh cu a ma tra
.
n
pha n x u
.
ng l`a ba
`
ng 0.
C

AC V

I DU
.
V du
.
1. 1) C o
.
ng cac ma tra
.
n
_
1 2
3 4
_
v`a
_
5 6
7 8
_
.
2) Nhan ma tra
.
n A =
_
1 2 1
4 0 1
_
vo
.
i so = 3.
Gia i. 1) Hai ma tra
.
n da cho co c` ung kch thu
.
o
.
c nen co the

co
.
ng
vo
.
i nhau. Theo di
.
nh ngha phep co
.
ng cac ma tra
.
n ta c o
_
1 2
3 4
_
+
_
5 6
7 8
_
=
_
1 + 5 2 + 6
3 + 7 4 + 8
_
=
_
6 8
10 12
_
.
2) A = 3
_
1 2 1
4 0 1
_
=
_
1 3 2 3 1 3
4 3 0 3 1 3
_
=
3.1. Ma tra
.
n 73
_
3 6 3
12 0 3
_
.
V du
.
2. Trong tru
.
`o
.
ng ho
.
.
p n`ao th`:
1) co the

nhan ben pha i mo


.
t ma tra
.
n h`ang vo
.
i mo
.
t ma tra
.
n co
.
t ?
2) co the

nhan ben pha i mo


.
t ma tra
.
n co
.
t vo
.
i mo
.
t ma tra
.
n h`ang ?
Gia i. 1) Ma tra
.
n h`ang l`a ma tra
.
n kch thu
.
o
.
c (1 n) c` on ma tra
.
n
co
.
t l`a ma tra
.
n kch thu
.
o
.
c (m1). Phep nhan ma tra
.
n h`ang (1 n)
vo
.
i ma tra
.
n co
.
t (m1) ch co the

ne

u n = m:
1 n n 1 = 1 1
t u
.
c l` a ke

t qua phep nhan l`a mo


.
t so, cu
.
the

l`a
_
a
1
a
2
. . . a
n
_
_

_
b
1
b
2
.
.
.
b
n
_

_
=
_
a
1
b
1
+a
2
b
2
+ +a
n
b
n
_
= c.
2) Ma tra
.
n co
.
t A
A =
_

_
a
1
a
2
.
.
.
a
m
_

_
l`a ma tra
.
n kch thu
.
o
.
c (m1). Ma tra
.
n n`ay tu
.
o
.
ng thch vo
.
i ma tra
.
n
kch thu
.
o
.
c (1 n), t u
.
c l`a ma tra
.
n h`ang. Nhu
.
va
.
y phep nhan da neu
luon luon thu
.
.
c hie
.
n du
.
o
.
.
c, cu
.
the

l`a
_

_
a
1
a
2
.
.
.
a
m
_

_
_
b
1
b
2
. . . b
n
_
=
_

_
a
1
b
1
a
1
b
2
. . . a
1
b
n
a
2
b
1
a
2
b
2
. . . a
2
b
n
.
.
.
.
.
.
.
.
.
.
.
.
a
m
b
1
a
m
b
2
. . . a
m
b
n
_

_
.
74 Chu
.
o
.
ng 3. Ma tra
.
n. D
-
i
.
nh th u
.
c
V du
.
3. Tnh AB v`a BA ne

u
1) A =
_
3 2 1
0 1 2
_
, B =
_

_
1
3
3
_

_.
2) A =
_
1 4 1
2 0 1
_
, B =
_

_
1 0
1 3
1 1
_

_
.
Gia i. 1) Theo quy ta

c nhan cac ma tra


.
n ta c o
AB =
_
3 2 1
0 1 2
_
_

_
1
3
3
_

_ =
_
3 1 + 2 3 + 1 3
0 1 + 1 3 + 2 3
_
=
_
12
9
_
.
Tch BA khong to
`
n ta
.
i v` ma tra
.
n B khong tu
.
o
.
ng thch vo
.
i ma
tra
.
n A.
2) Ta c o ma tra
.
n A tu
.
o
.
ng thch vo
.
i ma tra
.
n B. Do do
AB =
_
1 4 1
2 0 1
_
_

_
2 0
1 3
1 1
_

_
=
_
1 (2) + 4 1 + (1)(1) 1 0 + 4 3 + (1) 1
2 (2) + 0 1 + (1) (1) 2 0 + 0 3 + 1 1
_
=
_
3 11
5 1
_
.
Tu
.
o
.
ng tu
.
.
, ma tra
.
n B tu
.
o
.
ng thch vo
.
i ma tra
.
n A v`a
BA =
_

_
2 8 2
7 4 2
1 4 2
_

_
.
V du
.
4. 1) Cho ma tra
.
n A =
_
0 1
0 0
_
. T`m mo
.
i ma tra
.
n X giao
hoan vo
.
i A (AX = XA).
3.1. Ma tra
.
n 75
2) T`m mo
.
i ma tra
.
n giao ho an vo
.
i ma tra
.
n A =
_
1 2
1 1
_
.
3) Tnh tch
_
1 1
0 0
_ _
1 1
1 1
_
.
Gia i. 1) V` A l`a ma tra
.
n c ap 2 nen de

cac tch AX v`a XA xac


di
.
nh, ma tra
.
n X c ung pha i l`a ma tra
.
n cap 2. Gia su
.
A =
_


_
.
Khi do
AX =
_
0 1
0 0
_ _


_
=
_

0 0
_
,
XA =
_


_ _
0 1
0 0
_
=
_
0
0
_
.
T` u
.
do ne

u AX = XA = 0, = . Do do mo
.
i ma tra
.
n hoan vi
.
vo
.
i ma tra
.
n da cho de
`
u co da
.
ng
X =
_

0
_
.
2) Tu
.
o
.
ng tu
.
.
nhu
.
tren, gia su
.
X =
_
x y
u v
_
l`a ma tra
.
n giao hoan
76 Chu
.
o
.
ng 3. Ma tra
.
n. D
-
i
.
nh th u
.
c
vo
.
i ma tra
.
n A =
_
1 2
1 1
_
. Khi do
_
1 2
1 1
_ _
x y
u v
_
=
_
x y
u v
_ _
1 2
1 1
_

_
x + 2u y + 2v
x u y v
_
=
_
x y 2x y
u v 2u v
_

_
x + 2u = x y
x u = u v
y + 2v = 2x y
y v = 2u v

_
_
_
x = u 2v
y = 2u
; u, v t` uy y.
Va
.
y ta thu du
.
o
.
.
c
X =
_
u 2v 2u
u v
_
, u, v t` uy y.
3) De

d`ang thay ra
`
ng
_
1 1
0 0
_ _
1 1
1 1
_
=
_
0 0
0 0
_
. T` u
.
v du
.
n`ay
suy ra ra
`
ng doi v o
.
i c ac ma tra
.
n ne

u AB = O th` khong nhat thie

t
A = O hoa
.
c B = O.
V du
.
5. Ma tra
.
n S = E
n
, trong do E
n
l`a ma tra
.
n do
.
n vi
.
cap n v`a
l`a mo
.
t so du
.
o
.
.
c go
.
i l`a ma tr a
.
n v o hu
.
o
.
ng. Ch u
.
ng to ra
`
ng ma tra
.
n
vo hu
.
o
.
ng hoan vi
.
vo
.
i mo
.
i ma tra
.
n vuong c` ung cap.
Gia i.

Ap du
.
ng cac tnh chat cu a ma tra
.
n do
.
n vi
.
ta co
SA = (E
n
)A = (E
n
A) = A;
AS = A(E
n
) = (AE
n
) = A,
t u
.
c l`a AS = SA doi vo
.
i mo
.
i ma tra
.
n vuong A cap n.
Cho A l`a ma tra
.
n vuong, k l`a so tu
.
.
nhien lo
.
n ho
.
n 1. Khi do tch
k ma tra
.
n A du
.
o
.
.
c go
.
i l`a lu y th` u
.
a ba
.
c k cu a A v`a k y hie
.
u A
k
. Theo
3.1. Ma tra
.
n 77
di
.
nh ngha A
0
= E. Nhu
.
va
.
y
A
k
def
= AAA A
. .
k la `n
A

= E.
V du
.
6. T`m mo
.
i lu y th` u
.
a cu a ma tra
.
n
A =
_

_
0 1 0 0
0 0 1 0
0 0 0 1
0 0 0 0
_

_
.
Gia i. Ta co
A
2
=
_

_
0 1 0 0
0 0 1 0
0 0 0 1
0 0 0 0
_

_
_

_
0 1 0 0
0 0 1 0
0 0 0 1
0 0 0 0
_

_
=
_

_
0 0 1 0
0 0 0 1
0 0 0 0
0 0 0 0
_

_
,
v`a de

thay ra
`
ng
A
3
= A
2
A =
_

_
0 0 1 0
0 0 0 1
0 0 0 0
0 0 0 0
_

_
_

_
0 1 0 0
0 0 1 0
0 0 0 1
0 0 0 0
_

_
=
_

_
0 0 0 1
0 0 0 0
0 0 0 0
0 0 0 0
_

_
,
A
4
=
_

_
0 0 0 1
0 0 0 0
0 0 0 0
0 0 0 0
_

_
_

_
0 1 0 0
0 0 1 0
0 0 0 1
0 0 0 0
_

_
=
_

_
0 0 0 0
0 0 0 0
0 0 0 0
0 0 0 0
_

_
.
Cac l uy th` u
.
a tie

p theo cu a ma tra
.
n A de
`
u ba
`
ng 0.
V du
.
7. Gia su
.

J =
_
0 1
1 0
_
, E = E
22
.
78 Chu
.
o
.
ng 3. Ma tra
.
n. D
-
i
.
nh th u
.
c
Ch u
.
ng minh ra
`
ng
1) J
2
= E.
2) Ma tra
.
n da
.
ng Z = E + J =
_


_
du
.
o
.
.
c co
.
ng v`a nhan
vo
.
i nhau tu
.
o
.
ng tu
.
.
nhu
.
cac so ph u
.
c da
.
ng
Z = +i.
Gia i. 1) Ta c o
J
2
=
_
0 1
1 0
_ _
0 1
1 0
_
=
_
1 0
0 1
_
= E.
2) Xet Z
1
=
1
E +
1
J, Z
2
=
2
E +
2
J. Khi do theo di
.
nh ngha
cac phep toan tuye

n tnh tren ma tra


.
n c` ung cac tnh chat cu a ch ung,
mo
.
t ma
.
t ta co
Z
1
+Z
2
= (
1
+
2
)E + (
1
+
2
)J
v`a ma
.
t khac
Z
1
+Z
2
=
_

1

1

1

1
_
+
_

2

2

2

2
_
=
_

1
+
2

1
+
2
(
1
+
2
)
1
+
2
_
= (
1
+
2
)E + (
1
+
2
)J.
Doi vo
.
i phep nhan su
.
.
l y gia i c ung tu
.
o
.
ng tu
.
.
.
V du
.
8. Tnh A
n
ne

u:
1) A =
_
3 1
0 3
_
; 2) A =
_
4 1
0 3
_
.
Gia i. Du
.
.
a v`ao tnh chat cu a ma tra
.
n vo hu
.
o
.
ng: ma tra
.
n vo hu
.
o
.
ng
hoan vi
.
vo
.
i mo
.
i ma tra
.
n c` ung c ap, ta se bie

u die

n ma tra
.
n da cho th`anh
3.1. Ma tra
.
n 79
to

ng ma tra
.
n vo hu
.
o
.
ng co
.
ng vo
.
i ma tra
.
n da
.
ng da
.
c bie
.
t m`a phep nang
len l uy th` u
.
a du
.
o
.
.
c thu
.
.
c hie
.
n do
.
n gia n ho
.
n.
1) A =
_
3 1
0 3
_
=
_
3 0
0 3
_
+
_
0 1
0 0
_
= B +

B,
B
m
=
_
3 0
0 3
_
m
=
_
3
m
0
0 3
n
_
(xem b`ai 4. 3) du
.
o
.
i day),

B
m
=
_
0 0
0 0
_
m 2.
Tie

p theo do B

B =

BB nen ta co the

ap du
.
ng cong th u
.
c
(B +

B)
n
=
n

i=0
C
i
n
B
i

B
ni
(3.2)
(xem b`ai 5.3) du
.
o
.
i day). Theo (3.2) ta co
(B +

B)
n
= B
n
+C
1
n
B
n1

B +C
2
n
B
n2

B
2
+ +

B
n
= [do

B
m
= 0, m 2[
= B
n
+C
n
1
B
n1

B = B
n
+nB
n1

B
=
_
3
n
0
0 3
n
_
=
_
n3
n1
0
0 n3
n1
_ _
0 1
0 0
_
=
_
3
n
0
0 3
n
_
+
_
0 n3
n1
0 0
_
=
_
3
n
n3
n1
0 3
n
_
.
2) Tu
.
o
.
ng tu
.
.
nhu
.
tren ta co
A =
_
4 1
0 3
_
=
_
3 0
0 3
_
+
_
1 1
0 0
_
= B +

B.
B
m
=
_
3 0
0 3
_
m
=
_
3
m
0
0 3
m
_
, (3.3)

B
m
=
_
1 1
0 0
_
m
=
_
1 1
0 0
_
m 1 (3.4)
80 Chu
.
o
.
ng 3. Ma tra
.
n. D
-
i
.
nh th u
.
c
Tie

p theo do B

B =

BB nen ta co the

ap du
.
ng cong th u
.
c
A
n
= (B +

B)
n
= B
n
+C
1
n
B
n1

B +C
2
n
B
n2

B
2
+ +

B
n
. (3.5)
Ta tnh C
k
n
B
nk

B
k
. Theo (3.3) v`a (3.4) ta co
C
k
n
_
3
nk
0
0 3
nk
_ _
1 1
0 0
_
= C
k
n
_
3
nk
3
nk
0 0
_
=
_
C
k
n
3
nk
C
k
n
3
nk
0 0
_
.
(3.6)
T` u
.
(3.6), (3.3) v`a (3.5) ta thu du
.
o
.
.
c
A
n
=
_
3
n
0
0 3
n
_
+
n

k=1
_
C
k
n
3
nk
C
k
n
3
nk
0 0
_
=
_
_
3
n
+
n

k=1
C
k
n
3
nk
0 +
n

k=1
C
k
n
3
nk
0 3
n
_
_
.
V` 3
n
+
n

k=1
C
k
n
3
nk
= (3 + 1)
n
= 4
n
v`a 0 +
n

k=1
C
k
n
3
nk
=
n

k=0
C
k
n
3
nk

3
n
= 4
n
3
n
, do va
.
y
A
n
=
_
4
n
4
n
3
n
0 3
n
_
.
B
`
AI T

A
.
P
1. Tnh A+B, AB v`a BA ne

u
1) A =
_
1 2
3 4
_
, B =
_
4 4
0 i
_
;
2) A =
_

_
1 1 0
2 1 1
3 1 2
_

_, B =
_

_
2 1 2
0 4 5
2 3 7
_

_.
3.1. Ma tra
.
n 81
(DS. 1) A+B =
_
5 2
3 4 +i
_
, AB =
_
4 4 + 2i
12 12 + 4i
_
,
BA =
_
8 8
3 4i
_
;
2) A+B =
_

_
1 0 2
2 5 6
5 4 9
_

_, AB =
_

_
1 3 3
2 3 16
2 7 15
_

_,
BA =
_

_
6 1 5
23 1 14
17 12 11
_

_)
2. Tnh tch cac ma tra
.
n
1)
_

_
5 2 1
5 2 3
6 5 2
_

_
_

_
1 3 2
3 4 5
2 1 3
_

_
. (DS.
_

_
1 3 2
5 10 9
5 0 7
_

_
)
2)
_

_
3 4 9
2 1 6
5 3 5
_

_
_

_
5 6 4
8 9 7
4 5 3
_

_
. (DS.
_

_
11 9 13
22 27 17
29 32 26
_

_
)
3)
_

_
1 2 2
1 3 1
1 2 5
1 3 2
_

_
_

_
1 2 4
2 3 2
3 1 4
_

_. (DS.
_

_
1 2 0
4 6 6
12 3 20
1 5 2
_

_
)
4)
_

_
2 1 3
4 2 1
2 1 3
1 2 1
_

_
_

_
1
2
1
_

_. (DS.
_

_
1
7
3
9
_

_
)
82 Chu
.
o
.
ng 3. Ma tra
.
n. D
-
i
.
nh th u
.
c
5)
_
1 3 3 1
1 3 5 1
_
_

_
1 1
1 2
1 1
1 2
_

_
. (DS.
_
0 0
0 0
_
)
6)
_

_
1
2
3
_

_
_
3 2 1
_
. (DS.
_

_
3 2 1
6 4 2
9 6 3
_

_
)
3. Tnh cac tch AB v`a BA ne

u
1) A =
_

_
1 3 0
2 1 1
3 0 2
4 1 2
_

_
, B =
_
5 1 3 1
2 0 1 4
_
. (DS. Tch AB
khong to
`
n ta
.
i v` ma tra
.
n A khong tu
.
o
.
ng thch vo
.
i ma tra
.
n B; BA =
_
10 15 5
11 10 10
_
)
2) A =
_

_
2 0
1 4
3 1
0 1
_

_
, B =
_
5 1 0 3
_
. (DS. Tch AB khong
to
`
n ta
.
i v` A khong tu
.
o
.
ng thch vo
.
i B; BA =
_
11 1
_
)
3) A =
_
1 2 3 4
2 1 2 3
_
, B =
_

_
1 5 3
6 8 2
1 2 1
3 0 1
_

_
. (DS. AB =
_
28 27 8
15 14 13
_
, tch BA khong to
`
n ta
.
i)
4) A =
_
cos sin
cos cos
_
, B =
_
cos sin
sin cos
_
.
3.1. Ma tra
.
n 83
(DS. AB = BA =
_
cos( +) = sin( +)
sin( +) cos( +)
_
)
4. Tnh cac lu y th` u
.
a cu a ma tra
.
n A
n
ne

u:
1) A =
_
1 1
0 1
_
. (DS. A
n
=
_
1 n
0 1
_
)
Ch da

n. Su
.
du
.
ng phu
.
o
.
ng phap quy na
.
p toan ho
.
c
2) A =
_
cos sin
sin cos
_
. (DS. A
n
=
_
cos n sinn
sin n cos n
_
)
3) A =
_

_
d
1
d
2
_
.
.
.
_
.
.
.
d
n
_

_
. (DS. A
n
= diag
_
d
n
1
d
n
2
. . . d
n
n
_
)
4) A =
_

_
2 1 0
0 1 0
0 0 1
_

_. (DS.
_

_
2 2
n
1 0
0 1 0
0 0 2
_

_)
5. Ch u
.
ng minh ra
`
ng ne

u AB = BA th`
1) (A+B)
2
= A
2
+ 2AB +B
2
.
2) A
2
B
2
= (A+B)(AB).
3) (A+B)
n
= A
n
+C
1
n
A
n1
B +C
2
n
A
n2
B
2
+ +B
n
.
Ch da

n. Su
.
du
.
ng phu
.
o
.
ng phap quy na
.
p toan ho
.
c.
Gia su
.
cho da th u
.
c P(x) = a
0
+ a
1
x + + a + kx
k
. Khi do ma
tra
.
n vuong
P(A) = a
0
E +a
1
A+ +a
k
A
k
, x = A
du
.
o
.
.
c go
.
i l`a gia tri
.
cu a da th u
.
c P(x) ta
.
i x = A v`a bie

u th u
.
c
P(A) = a
0
E +a
A
+ +a
k
A
k
go
.
i l`a da th u
.
c cu a ma tra
.
n A.
6. Gia su
.
P(x) v` a Q(x) l`a hai da th u
.
c vo
.
i he
.
so T v`a A l`a ma tra
.
n
vuong cap n. Ch u
.
ng minh ra
`
ng
84 Chu
.
o
.
ng 3. Ma tra
.
n. D
-
i
.
nh th u
.
c
1) (x) = P(x) +Q(x) (A) = P(A) +Q(A).
2) (x) = P(x)Q(x) (A) = P(A)Q(A).
3) P(A)Q(A) = Q(A)P(A).
7. T`m gia tri
.
cu a da th u
.
c ma tra
.
n
1) P(x) = x
2
5x + 3, A =
_
2 1
3 3
_
. (DS.
_
0 0
0 0
_
)
2) P(x) = 3x
2
2x + 5, A =
_

_
1 2 3
2 4 1
3 5 2
_

_
. (DS.
_

_
21 23 15
13 34 10
9 22 25
_

_)
3) P(x) = 3x
5
4x
4
10x
3
+ 3x
2
7, A =
_

_
0 1 0
0 0 1
0 0 0
_

_
.
(DS.
_

_
7 0 3
0 7 0
0 0 7
_

_
)
4) Ch u
.
ng minh ra
`
ng ma tra
.
n
_

_
1 2 2
1 0 3
1 3 0
_

_
l`a nghie
.
m cu a da th u
.
c P(x) = x
3
x
2
9x + 9.
5) Ch u
.
ng minh ra
`
ng ma tra
.
n
A =
_

_
1 0 0
0 1 0
0 0 3
_

_
l`a nghie
.
m cu a da th u
.
c P(x) = x
3
5x
2
+ 7x 3.
3.2. D
-
i
.
nh th u
.
c 85
8. Ch u
.
ng minh ra
`
ng ne

u A l`a ma tra
.
n du
.
`o
.
ng cheo cap n vo
.
i c ac
pha
`
n tu
.
tren du
.
`o
.
ng cheo chnh l`a
1
,
2
, . . . ,
n
th` vo
.
i mo
.
i da th u
.
c
P(x) ma tra
.
n P(A) c ung l`a ma tra
.
n du
.
`o
.
ng cheo vo
.
i cac pha
`
n tu
.
tren
du
.
`o
.
ng cheo chnh l`a P(
1
), P(
2
), . . . , P(
n
). Hay xet tru
.
`o
.
ng ho
.
.
p
khi A l`a ma tra
.
n vuong cap 3.
9. Ch u
.
ng minh ra
`
ng (A
n
)
T
= (A
T
)
n
.
Ch da

n. Ch u
.
ng minh ba
`
ng phu
.
o
.
ng phap quy na
.
p v`a su
.
du
.
ng he
.
th u
.
c (AB)
T
= B
T
A
T
.
10. Ch u
.
ng minh ra
`
ng mo
.
i ma tra
.
n vuong A de
`
u co the

bie

u die

n du
.
o
.
i
da
.
ng to

ng mo
.
t ma tra
.
n doi x u
.
ng v`a mo
.
t ma tra
.
n pha n x u
.
ng.
Ch da

n. Da
.
t P =
1
2
(A+A
T
), Q =
1
2
(AA
T
), A = P +Q.
3.2 D
-
i
.
nh th u
.
c
3.2.1 Nghi
.
ch the

Mo
.
i cach sa

p xe

p th u
.
tu
.
.
n pha
`
n tu
.
cu a ta
.
p ho
.
.
p so J = 1, 2, . . . , n
du
.
o
.
.
c go
.
i l`a mo
.
t hoan vi
.
cu a n pha
`
n tu
.
do. So cac hoan vi
.
co the

co
cu a n pha
`
n tu
.
cu a J l`a n!. Hai so trong mo
.
t hoan vi
.
la
.
p th`anh mo
.
t
nghi
.
ch the

ne

u so lo
.
n ho
.
n d u
.
ng tru
.
o
.
c so be ho
.
n. So nghi
.
ch the

cu a
hoan vi
.
(
1
, . . . ,
n
) du
.
o
.
.
c k y hie
.
u l`a
inv(
1
,
2
, . . . ,
n
),
do chnh l`a so ca
.
p la
.
p th`anh nghi
.
ch the

trong hoan vi
.
.
Hoan vi
.

1
, . . . ,
n
du
.
o
.
.
c go
.
i l`a hoan vi
.
cha

n ne

u so nghi
.
ch the

cu a no l`a cha

n v`a go
.
i l`a hoan vi
.
le ne

u so nghi
.
ch the

l`a le.
3.2.2 D
-
i
.
nh th u
.
c
Mo

i ma tra
.
n vuong cap n (v`a ch co ma tra
.
n vuong !) de
`
u tu
.
o
.
ng u
.
ng
vo
.
i mo
.
t so - go
.
i l`a di
.
nh th u
.
c cu a no.
86 Chu
.
o
.
ng 3. Ma tra
.
n. D
-
i
.
nh th u
.
c
Gia su
.
cho ma tra
.
n vuong cap n tren tru
.
`o
.
ng T(R, C):
A =
_
_
a
ij
_
_
n
1
=
_

_
a
11
a
12
. . . a
1n
a
21
a
22
. . . a
2n
.
.
.
.
.
.
.
.
.
.
.
.
a
n1
a
n2
. . . a
nn
_

_
(3.7)
Di
.
nh th u
.
c cu a ma tra
.
n A l`a mo
.
t so thu du
.
o
.
.
c t` u
.
cac pha
`
n tu
.
cu a
ma tra
.
n theo quy ta

c sau day:
1) di
.
nh th u
.
c c ap n ba
`
ng to

ng da
.
i so cu a n! so ha
.
ng;
2) mo

i so ha
.
ng cu a di
.
nh th u
.
c l`a tch
a
i
1
j
1
a
i
2
j
2
a
injn
(3.8)
cu a n pha
`
n tu
.
cu a ma tra
.
n m`a c u
.
mo

i h`ang v`a mo

i co
.
t de
`
u co d ung
mo
.
t pha
`
n tu
.
trong tch n`ay;
3) so ha
.
ng a
i
1
j
1
a
i
2
j
2
a
injn
cu a di
.
nh th u
.
c co dau co
.
ng ne

u hoan
vi
.
la
.
p nen bo
.
i cac so hie
.
u h`ang i
1
, i
2
, . . . , i
n
v`a hoan vi
.
la
.
p nen bo
.
i
cac so hie
.
u c o
.
t j
1
, j
2
, . . . , j
n
l`a c` ung cha

n ho a
.
c c` ung le v`a co dau
tr` u
.
( ) trong tru
.
`o
.
ng ho
.
.
p ngu
.
o
.
.
c la
.
i.
K y hie
.
u: Di
.
nh th u
.
c cu a ma tra
.
n A du
.
o
.
.
c k y hie
.
u l`a
det A, [A[ hay

a
11
a
12
. . . a
1n
a
21
a
22
. . . a
2n
.
.
.
.
.
.
.
.
.
.
.
.
a
n1
a
n2
. . . a
nn

.
Nha
.
n xet. 1) Nhu
.
va
.
y, de

xac di
.
nh dau cu a so ha
.
ng di
.
nh th u
.
c ta
ca
`
n tnh
s = inv(i
1
, . . . , i
n
)
= inv(j
1
, . . . , j
n
)
v`a khi do dau cu a so ha
.
ng di
.
nh th u
.
c l`a dau cu a th` u
.
a so (1)
s+
.
3.2. D
-
i
.
nh th u
.
c 87
2) Ne

u ta vie

t c ac th` u
.
a so cu a tch (3.8) theo th u
.
tu
.
.
tang da
`
n cu a
so hie
.
u h`ang:
a
i
1
j
1
a
i
2
j
2
a
injn
= a
1
1
a
2
2
a
nn
th`
det A =

(
1
,...,n)
(1)
inv(
1
,...,n)
a
1
1
a
2
2
a
nn
. (3.9)
trong do to

ng lay theo mo
.
i hoan vi
.
(
1
,
2
, . . . ,
n
) cu a cac so
1, 2, . . . , n.
Trong ma tra
.
n vuong (3.7) ta c o di
.
nh k (k < n) h`ang v`a k co
.
t n`ao
do. Gia su
.
do l`a cac h`ang vo
.
i so hie
.
u i
1
< i
2
< < i
k
v`a cac co
.
t v o
.
i
so hie
.
u j
1
< j
2
< < j
k
. T` u
.
cac pha
`
n tu
.
na
`
m tren giao cu a h`ang
v`a cac co
.
t du
.
o
.
.
c cho
.
n ta c o the

la
.
p di
.
nh th u
.
c cap k

a
i
1
j
1
a
i
1
j
2
. . . a
i
1
j
k
a
i
2
j
1
a
i
2
j
2
. . . a
i
2
j
k
.
.
.
.
.
.
.
.
.
.
.
.
a
i
k
j
1
a
i
k
j
2
. . . a
i
k
j
k

.
Di
.
nh th u
.
c n`ay du
.
o
.
.
c go
.
i l`a di
.
nh th u
.
c con cap k cu a ma tra
.
n A. K y
hie
.
u M
i
1
i
2
...i
k
j
1
j
2
j
k
.
Ne

u ta bo di cac h`ang th u
.
i
1
, i
2
, . . . , i
k
v`a cac co
.
t th u
.
j
1
, j
2
, . . . , j
k
th` cac pha
`
n tu
.
c`on la
.
i cu a ma tra
.
n A se ta
.
o th`anh mo
.
t ma tra
.
n vuong
cap n k. Di
.
nh th u
.
c cu a ma tra
.
n vu ong n`ay l`a di
.
nh th u
.
c con cap
n k cu a ma tra
.
n A v`a du
.
o
.
.
c go
.
i l`a pha
`
n b` u (hay di
.
nh th u
.
c con b` u)
cu a di
.
nh th u
.
c con M
i
1
i
2
i
k
j
1
j
2
j
k
v`a du
.
o
.
.
c k y hie
.
u l`a M
i
1
i
2
i
k
j
1
j
2
j
k
.
Di
.
nh th u
.
c con b` u vo
.
i dau
(1)
(i
1
+i
2
++i
k
)+(j
1
+j
2
++j
k
)
du
.
o
.
.
c go
.
i l`a pha
`
n b` u da
.
i so cu a di
.
nh th u
.
c con M
i
1
i
k
j
1
j
k
.
Tru
.
`o
.
ng ho
.
.
p da
.
c bie
.
t: di
.
nh th u
.
c con b` u M
ij
cu a di
.
nh th u
.
c con cap
1 l`a |a
ij
| cu a A du
.
o
.
.
c go
.
i l`a pha
`
n b` u cu a pha
`
n tu
.
a
ij
cu a A v`a so
A
ij
= (1)
i+j
M
ij
go
.
i l`a pha
`
n b` u da
.
i so cu a pha
`
n tu
.
a
ij
.
88 Chu
.
o
.
ng 3. Ma tra
.
n. D
-
i
.
nh th u
.
c
3.2.3 Tnh cha

t cu a di
.
nh th u
.
c
Di
.
nh th u
.
c c o c ac tnh chat sau
I. Qua phep chuye

n vi
.
ma tra
.
n, di
.
nh th u
.
c cu a no khong do

i, t u
.
c
l`a det A = det A
T
.
T` u
.
tnh chat b`nh da

ng n`ay gi u
.
a c ac h`ang v`a c ac co
.
t cu a di
.
nh
th u
.
c suy ra ra
`
ng mo
.
t die
`
u kha

ng di
.
nh n`ao do da d ung vo
.
i h`ang th`
no c ung d ung vo
.
i co
.
t. Do do cac tnh chat tie

p theo day ch ca
`
n phat
bie

u cho h`ang.
II. Ne

u do

i cho

hai h`ang cho nhau th` di


.
nh th u
.
c do

i dau.
III. Th` u
.
a so chung cu a mo
.
i pha
`
n tu
.
cu a mo
.
t h`ang cu a di
.
nh th u
.
c
co the

du
.
a ra ngo`ai dau di
.
nh th u
.
c.
IV. Di
.
nh th u
.
c c o mo
.
t h`ang ba
`
ng 0 l`a ba
`
ng 0.
V. Di
.
nh th u
.
c co hai h`ang gi ong nhau l`a ba
`
ng 0.
VI. Ne

u di
.
nh th u
.
c co hai h`ang ty le
.
vo
.
i nhau th` no ba
`
ng 0.
VII. Ne

u c ac pha
`
n tu
.
cu a h`ang th u
.
i cu a di
.
nh th u
.
c D co da
.
ng
a
ij
= b
ij
+ c
iJ
, i = 1, n, j = 1, n th` di
.
nh th u
.
c D ba
`
ng to

ng hai di
.
nh
th u
.
c D
1
+ D
2
, trong do di
.
nh th u
.
c D
1
co h`ang th u
.
i l`a (b
i1
b
i2
b
in
)
v`a di
.
nh th u
.
c D
2
co h`ang th u
.
i l`a (c
i1
, c
i2
, . . . , c
in
) c` on cac h`ang khac
l`a cac h`ang tu
.
o
.
ng u
.
ng cu a D.
VIII. Ne

u di
.
nh th u
.
c co mo
.
t h`ang l`a to

ho
.
.
p tuye

n tnh cu a cac
h`ang khac th` di
.
nh th u
.
c ba
`
ng 0.
IX. Di
.
nh th u
.
c khong do

i ne

u them v`ao mo
.
t h`ang n`ao do mo
.
t to

ho
.
.
p tuye

n tnh cu a cac h`ang khac.


X. Di
.
nh th u
.
c ba
`
ng to

ng cac tch cu a cac pha


`
n tu
.
cu a mo
.
t h`ang
n`ao do vo
.
i pha
`
n b` u da
.
i so tu
.
o
.
ng u
.
ng.
det A = a
i1
A
i1
+a
i2
A
i2
+ +a
in
A
in
=
n

j=1
a
ij
A
ij
. (3.10)
Nha
.
n xet. Ngu
.
`o
.
i ta c ung d` ung tnh chat X n`ay de

l`am di
.
nh ngha
di
.
nh th u
.
c.
3.2. D
-
i
.
nh th u
.
c 89
XI. To

ng cac tch cu a cac pha


`
n tu
.
cu a mo
.
t h`ang n`ao do vo
.
i pha
`
n
b` u da
.
i so tu
.
o
.
ng u
.
ng cu a cac pha
`
n tu
.
cu a h`ang khac l`a ba
`
ng 0:
n

j=1
a
ij
A
kj
= 0, k ,= i; i, k = 1, n.
Nha
.
n xet. Cac tnh chat I-III l`a nh u
.
ng tnh chat co
.
ba n. C ac tnh
chat sau l`a nh u
.
ng he
.
qua cu a ba tnh chat ay.
3.2.4 Phu
.
o
.
ng phap tnh di
.
nh th u
.
c
I. Di
.
nh th u
.
c cap 1, cap 2 v`a cap 3 du
.
o
.
.
c tnh theo c ac cong th u
.
c
[a
11
[ = a
11
;

a
11
a
12
a
21
a
22

= a
11
a
22
a
12
a
21
; (3.11)

a
11
a
12
a
13
a
21
a
22
a
23
a
31
a
32
a
33

= a
11
a
22
a
33
+a
12
a
23
a
31
+a
13
a
21
a
32
a
13
a
22
a
31
a
11
a
23
a
32
a
12
a
21
a
33
.
Khi tnh di
.
nh th u
.
c cap 3 ta c o the

su
.
du
.
ng quy ta

c Surrus da
.
ng
tam giac hoa
.
c da
.
ng du
.
`o
.
ng song song sau day

(+) ()
a
11
a
12
a
13
a
11
a
12
a
21
a
22
a
23
a
21
a
22
a
31
a
32
a
33
a
31
a
32
90 Chu
.
o
.
ng 3. Ma tra
.
n. D
-
i
.
nh th u
.
c

II. Tnh di
.
nh th u
.
c ca

p n
1
+
Khai trie

n di
.
nh th u
.
c theo cac pha
`
n tu
.
cu a mo
.
t h`ang hoa
.
c mo
.
t
co
.
t (tnh chat XI, (3.10)).
2
+
Su
.
du
.
ng cac tnh chat cu a di
.
nh th u
.
c de

bie

n do

i di
.
nh th u
.
c da
cho th`anh di
.
nh th u
.
c mo
.
i sao cho ngoa
.
i tr` u
.
mo
.
t pha
`
n tu
.
a
i
0
j
0
,= 0, tat
ca cac pha
`
n tu
.
c`on la
.
i cu a h`ang th u
.
i
0
(hoa
.
c co
.
t j
0
) de
`
u ba
`
ng 0. Khi
do
det A = (1)
i
0
+j
0
a
i
0
j
0
M
i
0
j
0
.
Tie

p theo l`a la
.
p la
.
i qua tr`nh do doi vo
.
i M
i
0
j
0
l`a di
.
nh th u
.
c cap thap
ho
.
n mo
.
t do
.
n vi
.
.
3
+
Su
.
du
.
ng cac tnh chat cu a di
.
nh th u
.
c de

bie

n do

i di
.
nh th u
.
c da
cho th`anh di
.
nh th u
.
c tam giac (t u
.
c l`a di
.
nh th u
.
c m`a mo
.
i pha
`
n tu
.
o
.

mo
.
t pha cu a du
.
`o
.
ng cheo chnh de
`
u ba
`
ng 0). Khi do di
.
nh th u
.
c ba
`
ng
tch cac pha
`
n tu
.
tren du
.
`o
.
ng cheo chnh.
4
+
Phu
.
o
.
ng phap truy ho
`
i: bie

n do

i, khai trie

n di
.
nh th u
.
c theo h`ang
hoa
.
c theo co
.
t sao cho di
.
nh th u
.
c da cho c o the

bie

u die

n qua cac di
.
nh
th u
.
c c` ung da
.
ng nhu
.
ng cap thap ho
.
n.
5
+
Bie

u die

n di
.
nh th u
.
c da cho du
.
o
.
i da
.
ng to

ng cac di
.
nh th u
.
c c` ung
cap.
6
+
D` ung di
.
nh l y Laplace: Gia su
.
trong ma tra
.
n vuong A cap n ta
cho
.
n mo
.
t cach t` uy y m h`ang (hay m co
.
t) 1 m n1. Khi do di
.
nh
th u
.
c det A ba
`
ng to

ng c ac tch cu a mo
.
i di
.
nh th u
.
c con cap m na
`
m tren
cac h`ang du
.
o
.
.
c cho
.
n nhan vo
.
i pha
`
n b` u da
.
i so tu
.
o
.
ng u
.
ng cu a ch ung.
C

AC V

I DU
.
V du
.
1. 1) Tnh so nghi
.
ch the

trong hoan vi
.
_
5 3 1 6 4 2
_
.
2) Vo
.
i nh u
.
ng gia tri
.
n`ao cu a i v`a j th` so ha
.
ng a
51
a
1i
a
2j
a
43
a
32
cu a
di
.
nh th u
.
c cap 5 co dau tr` u
.
.
3.2. D
-
i
.
nh th u
.
c 91
Gia i. 1) De

tnh so nghi
.
ch the

tie
.
n lo
.
.
i ho
.
n ca l`a tie

n h`anh nhu
.
sau: (i) da
`
u tien, tnh c o bao nhieu so d u
.
ng tru
.
o
.
c so 1 (gia su
.
co k
1
so) ro
`
i ga
.
ch bo so 1 khoi hoan vi
.
; (ii) tie

p de

n tnh xem co bao nhieu


so d u
.
ng tru
.
o
.
c so 2 (gia su
.
k
2
) ro
`
i ga
.
ch bo so 2 khoi hoan vi
.
; v.v... Khi
do
inv(
1
,
2
, . . . ,
n
) = k
1
+k
2
+ +k
n
.
Ba
`
ng phu
.
o
.
ng phap v` u
.
a neu de

thay l`a
inv(531642) = 2 + 4 + 1 + 2 = 9.
2) Cac ch so i v`a j ch co the

nha
.
n cac gi a tri
.
sau day: (a) i = 4,
j = 5; hoa
.
c (b) i = 5 v`a j = 4 v` vo
.
i cac gi a tri
.
khac cu a i v`a j tch
da cho ch u
.
a t nhat hai pha
`
n tu
.
cu a c` ung mo
.
t c o
.
t. De

xac di
.
nh dau
cu a so ha
.
ng ta sa

p xe

p cac th` u
.
a so cu a tch theo th u
.
tu
.
.
tang cu a ch
so th u
.
nhat ro
`
i tnh so nghi
.
ch the

cu a hoan vi
.
cac ch so th u
.
hai. Ta
co
a
1i
a
2j
a
32
a
43
a
51
+) Gia su
.
i = 4, j = 5 inv(45231) = 8. Do va
.
y vo
.
i i = 4, j = 5
so ha
.
ng da cho c o dau (+).
+) Gia su
.
i = 5, j = 4 inv(54231) = 9. Do do so ha
.
ng da cho
co dau tr` u
.
. V a
.
y so ha
.
ng da cho ch co dau tr` u
.
khi i = 5, j = 4.
V du
.
2. Tnh cac di
.
nh th u
.
c sau day
1)
1
=

0 0 0 a
14
0 0 a
23
0
0 a
32
0 0
a
41
0 0 0

; 2)
2
=

1 4 2 4
2 3 3 6
3 2 1 2
4 1 1 2

92 Chu
.
o
.
ng 3. Ma tra
.
n. D
-
i
.
nh th u
.
c
Gia i. 1) Co the

tnh
1
ba
`
ng cach su
.
du
.
ng tnh chat X.

1
= (1)
1+4
a
14

0 0 a
23
0 a
32
0
a
41
0 0

= (1)
1+4
a
14
(1)
2+3
a
23

0 a
32
a
41
0

= a
14
a
23
a
32
a
41
.
Ke

t qua n`ay c ung co the

thu du
.
o
.
.
c nh`o
.
di
.
nh ngha di
.
nh th u
.
c. Theo
di
.
nh ngha
1
l`a to

ng da
.
i so cu a 4! = 24 so ha
.
ng, trong do ch co so
ha
.
ng
a
14
a
23
a
32
a
41
l`a khac 0. V` hoan vi
.
cu a cac ch so th u
.
hai cha

n nen so ha
.
ng co dau
co
.
ng. T` u
.
do ta thu du
.
o
.
.
c
1
= a
14
a
23
a
32
a
41
.
2)

Ap du
.
ng tnh chat XI ta co the

khai trie

n di
.
nh th u
.
c theo co
.
t
th u
.
nhat

2
= 1

3 3 6
2 1 2
1 1 2

4 2 4
2 1 2
2 1 2

+ 3

4 2 4
3 3 6
1 1 2

4 2 4
3 3 6
2 1 2

= 1 0 2 0 + 3 0 4 0 = 0.
O
.

day mo
.
i di
.
nh th u
.
c c ap 3 de
`
u co hai co
.
t ty le
.
vo
.
i nhau, nen ch ung
ba
`
ng 0.
V du
.
3. Tnh cac di
.
nh th u
.
c
1)
1
=

1 1 2 3
1 2 3 1
2 3 6 4
3 5 9 4

, 2)
2
=

2 0 1 3 1
1 1 2 2 3
1 4 0 1 5
2 1 3 1 2
1 2 1 3 1

.
3.2. D
-
i
.
nh th u
.
c 93
Gia i. Ta bie

n do

i c ac di
.
nh th u
.
c de

thu du
.
o
.
.
c cac so 0 trong mo
.
t
h`ang (co
.
t). Ta quy u
.
o
.
c cac k y hie
.
u: h
2
h
1
h

2
co ngha l`a lay
h`ang th u
.
hai tr` u
.
di h`ang th u
.
nhat de

thu du
.
o
.
.
c h`ang th u
.
hai mo
.
i.
Tu
.
o
.
ng tu
.
.
nhu
.
va
.
y ta k y hie
.
u cac phep bie

n do

i theo co
.
t.
1) Ta c o

1
=

1 1 2 3
1 2 3 1
2 3 6 4
3 5 9 4

h
2
h
1
h

2
h
3
2h
1
h

3
h
4
3h
1
h
4
=

1 1 2 3
0 1 1 2
0 1 2 2
0 2 3 5

= 1 (1)
1+1

1 1 2
1 2 2
2 3 5

1 1 2
1 2 2
2 3 5

h
2
h
1
h

2
=

1 1 2
0 1 0
2 3 5

= 1 (1)
2+2

1 2
2 5

= 1.
2) De

tnh
2
ta thu
.
.
c hie
.
n phep bie

n do

i: c
1
2c
3
c

1
; c
4
3c
3

c

4
; c
5
c
3
c

5
v`a thu du
.
o
.
.
c

2
=

0 0 1 0 0
5 1 2 4 1
1 4 0 1 5
4 1 3 1 5
4 1 3 8 1
3 2 1 6 2

= a
13
A
13
= 1 (1)
1+3

5 1 4 1
1 4 1 5
4 1 8 1
3 2 6 2

Doi vo
.
i di
.
nh th u
.
c c ap 4 v` u
.
a thu du
.
o
.
.
c ta c ung tie

n h`anh tu
.
o
.
ng tu
.
.
:
94 Chu
.
o
.
ng 3. Ma tra
.
n. D
-
i
.
nh th u
.
c
c
1
+ 5c
4
c

1
; c
2
c
4
c

2
; c
3
+ 4c
4
c

3
v`a thu du
.
o
.
.
c

2
=

0 0 0 1
26 1 19 5
9 2 12 1
13 0 14 2

= a
14
A
14
= 1 (1)
1+4

26 1 19
9 2 12
13 0 14

Nhu
.
va
.
y ta da du
.
a vie
.
c tnh di
.
nh th u
.
c c ap 5 ve
`
tnh di
.
nh th u
.
c c ap 3.
De

tnh di
.
nh th u
.
c cap 3 n`ay ta c o the

d` ung quy ta

c Sarrus hoa
.
c tie
.
n
ho
.
n ca l`a bie

n do

i no theo h`ang: h
2
+ 2h
1
h

2
v`a co

2
=

26 1 19
43 0 26
13 0 14

= a
12
A
12
= (1)(1)
1+2

43 26
13 14

= 264.
V du
.
4. Tnh cac di
.
nh th u
.
c
1)
1
=

1 2 1 5
1 5 6 3
1 2 3 5
2 4 2 8

, 2)
2
=

1 1 3 2 4
0 3 2 0 1
0 0 4 1 1
0 6 4 2 3
1 1 3 2 5

.
Gia i. Ta se tnh c ac di
.
nh th u
.
c da cho ba
`
ng phu
.
o
.
ng phap du
.
a ve
`
di
.
nh th u
.
c tam giac.
1) Ta c o

1
=

1 2 1 5
1 5 6 3
1 2 3 5
2 4 2 8

h
2
h
1
h

2
h
3
+h
1
h

3
h
4
2h
1
h

4
=

1 2 1 5
0 3 7 2
0 0 2 10
0 0 0 2

.
V` di
.
nh th u
.
c tam gi ac ba
`
ng tch c ac pha
`
n tu
.
tren du
.
`o
.
ng cheo chnh
nen

1
= 1 3 2 (2) = 12.
3.2. D
-
i
.
nh th u
.
c 95
2)

2
=

1 1 3 2 4
0 3 2 0 1
0 0 4 1 1
0 6 4 2 3
1 1 3 2 5

h
4
2h
2
h

4
h
5
h
1
h

5
=

1 1 3 2 4
0 3 2 0 1
0 0 4 1 1
0 0 0 2 1
0 0 0 0 1

= 1 3 4 2 1 = 24.
V du
.
5. Tnh cac di
.
nh th u
.
c
1)
n
=

a
0
1 0 0 . . . 0 0
a
1
x 1 0 . . . 0 0
a
2
0 x 1 . . . 0 0
.
.
.
.
.
.
.
.
.
.
.
.
.
.
.
.
.
.
.
.
.
a
n1
0 0 0 . . . 0 1
a
n
0 0 0 . . . 0 x

;
2)
n
=

7 4 0 0 . . . 0 0
3 7 4 0 . . . 0 0
0 3 7 4 . . . 0 0
.
.
.
.
.
.
.
.
.
.
.
.
.
.
.
.
.
.
.
.
.
0 0 0 0 . . . 3 7

3)
n
=

+ 0 . . . 0 0
1 + . . . 0 0
0 1 + . . . 0 0
.
.
.
.
.
.
.
.
.
.
.
.
.
.
.
.
.
.
0 0 0 . . . +
0 0 0 . . . 1 +

96 Chu
.
o
.
ng 3. Ma tra
.
n. D
-
i
.
nh th u
.
c
Gia i. 1) Khai trie

n
n+1
theo h`ang cuoi (h`ang th u
.
n + 1) ta c o

n+1
= (1)
n+1
a
n

1 0 . . . 0
x 1 . . . 0
.
.
.
.
.
.
.
.
.
.
.
.
0 0 . . . 1

+x

a
0
1 0 . . . 0
a
1
x 1 . . . 0
.
.
.
.
.
.
.
.
.
.
.
.
.
.
.
a
n1
0 0 . . . x

Di
.
nh th u
.
c th u
.
nhat o
.
ve

pha i l`a di
.
nh th u
.
c tam giac (= (1)
n
), di
.
nh
th u
.
c th u
.
hai l`a di
.
nh th u
.
c c` ung da
.
ng vo
.
i
1
nhu
.
ng cap n. Do v a
.
y
di
.
nh th u
.
c
n+1
co the

bie

u die

n bo
.
i he
.
th u
.
c truy ho
`
i sau day:

n+1
= a
n
(1)
n
(1)
n
+x
n
.
De

thu du
.
o
.
.
c bie

u th u
.
c to

ng quat cu a
n+1
ta xet
1
v`a
2
:

1
= a
0
;
2
=

a
0
1
a
1
x

= a
0
x a
1
.
Nhu
.
va
.
y
1
l`a da th u
.
c ba
.
c 0 v o
.
i he
.
so a
0
, c`on
2
l`a da th u
.
c ba
.
c nhat
vo
.
i he
.
so a
0
v`a a
1
.
Ta ch u
.
ng to ra
`
ng
n+1
co da
.
ng tu
.
o
.
ng tu
.
.
:

n+1
= a
0
x
n
+a
1
x
n1
+ +a
n
.
Gia su
.
da ch u
.
ng minh
n
= a
0
x
n1
+ +a
n1
. Khi do

n+1
= a
n
+x
n
= a
n
+x(a
0
x
n1
+ +a
n1
)
= a
0
x
n
+a
1
x
n1
+ +a
n1
x +a
n
.
2) Khai trie

n di
.
nh th u
.
c theo h`ang th u
.
nhat ta thu du
.
o
.
.
c he
.
th u
.
c
truy ho
`
i:

n
= 7
n1
12
n2

n
3
n1
= 4
n1
3 4
n2
= 4[
n1
3
n2
].
3.2. D
-
i
.
nh th u
.
c 97
T` u
.
do suy ra

n
3
n1
= 4
n2
(
2

1
)

1
= 7,
2
=

7 4
3 7

= 37
v`a do do

n
3
n1
= 4
n2
[37 21] = 4
n2
4
2
= 4
n
.
Ne

u t` u
.
he
.
th u
.
c truy ho
`
i ta bie

n do

i cach khac th` thu du


.
o
.
.
c

n
4
n1
= 3[
n1
4
n2
] = = 3
n2
(
2

1
)
= 3
n2
3
2
= 3
n
.
Nhu
.
va
.
y

n
3
n1
= 4
n

n
4
n1
= 3
n
_

n1
= 4
n
3
n
v`a do do

n
= 3
n1
+ 4
n
= 4
n+1
3
n+1
.
3) Ta bie

u die

n co
.
t th u
.
nhat du
.
o
.
i da
.
ng cac to

ng hai so ha
.
ng +,
98 Chu
.
o
.
ng 3. Ma tra
.
n. D
-
i
.
nh th u
.
c
1 +0, 0 +0, . . . , 0 +0 v`a vie

t di
.
nh th u
.
c du
.
o
.
i da
.
ng to

ng hai di
.
nh th u
.
c

n
=

0 . . . 0 0
1 + . . . 0 0
0 1 + . . . 0 0
.
.
.
.
.
.
.
.
.
.
.
.
.
.
.
.
.
.
0 0 0 . . . +
0 0 0 . . . 1 +

. .
D
1
+

0 . . . 0 0
0 + . . . 0 0
0 1 + . . . 0 0
.
.
.
.
.
.
.
.
.
.
.
.
.
.
.
.
.
.
0 0 0 . . . +
0 0 0 . . . 1 +

. .
D
2
= D
1
+D
2
.
Tnh D
1
. Lay co
.
t th u
.
hai tr` u
.
di co
.
t th u
.
nhat nhan vo
.
i , lay co
.
t
th u
.
ba tr` u
.
di co
.
t th u
.
hai v` u
.
a thu du
.
o
.
.
c nhan vo
.
i , v.v... Ke

t qua ta
thu du
.
o
.
.
c di
.
nh th u
.
c tam giac
D
1
=

0 0 . . . 0 0
1 0 . . . 0 0
0 1 . . . 0 0
.
.
.
.
.
.
.
.
.
.
.
.
.
.
.
.
.
.
0 0 0 . . . 0
0 0 0 . . . 1

=
n
.
3.2. D
-
i
.
nh th u
.
c 99
Tnh D
2
. Khai trie

n D
2
theo co
.
t th u
.
nhat ta thu du
.
o
.
.
c:
D
2
=

+ . . . 0 0
1 + . . . 0 0
.
.
.
.
.
.
.
.
.
.
.
.
.
.
.
0 0 . . . +
0 0 . . . 1 +

=
n1
.
Nhu
.
va
.
y ta thu du
.
o
.
.
c cong th u
.
c truy ho
`
i
n
=
n
+
n1
.
Ta tnh mo
.
t v`ai di
.
nh th u
.
c da
`
u tien

1
= + =

2

2

;

2
=

+
1 +

=
2
+ +
2
=

3

3

,

3
=

+ 0
1 +
0 1 +

=
3
+
2
+
2
+
4
=

4

4

; ................
Ta se ch u
.
ng minh ra
`
ng he
.
th u
.
c

m
=

m+1

m+1

(*)
d ung vo
.
i m N bat k` y. Ta ap du
.
ng phu
.
o
.
ng phap quy na
.
p toan ho
.
c.
Gia su
.
() d ung vo
.
i m = n1. Ta ch u
.
ng minh no d ung vo
.
i m = n.
Khi m = n 1 ta c o

n1
=

n

n
=
n
+

n

=

n+1

n
+
n

n+1

=

n+1

n+1


100 Chu
.
o
.
ng 3. Ma tra
.
n. D
-
i
.
nh th u
.
c
Nhu
.
va
.
y he
.
th u
.
c () d ung m N. Do do

n
=

n+1

n1


B
`
AI T

A
.
P
1. Xac di
.
nh so nghi
.
ch the

trong c ac hoan vi
.
.
1) (1 3 5 7 9 2 4 6 8). (DS. 10)
2) (9 8 7 6 5 4 3 2 1). (DS. 36)
3) (2 5 8 1 4 7 3 6 9). (DS. 12)
4) (7 5 4 6 1 2 3 9 8). (DS. 17)
2. Cho
.
n k v`a sao cho hoan vi
.
1) (7 4 3 k 8 5 2) l` a ho an vi
.
le. (DS. k = 6, = 1)
2) (k 3 4 7 2 6 5) l` a ho an vi
.
cha

n. (DS. k = 8, = 1)
3) (4 8 k 2 5 1 7) l`a ho an vi
.
cha

n. (DS. k = 6, = 3)
4) (6 3 4 k 7 2 1) l`a ho an vi
.
le. (DS. k = 5, = 8)
3. Xac di
.
nh so nghi
.
ch the

trong c ac hoan vi
.
.
1) n n 1 n 2 . . . 2 1. (DS.
n(n 1)
2
)
2) 1 3 5 7 . . . 2n 1 2 4 6 . . . 2n. (DS.
n(n 1)
2
)
3) 2 4 6 . . . 2n 1 3 5 . . . 2n 1. (DS.
n(n + 1)
2
)
4) 2n 1 2n 3 . . . 5 3 1 2n 2n 2 . . . 6 4 2. (DS.
3n(n 1)
2
)
4. Trong c ac tch sau day, tch n`ao l`a so ha
.
ng cu a di
.
nh th u
.
c cap 7;
xac di
.
nh dau cu a so ha
.
ng do.
1) a
43
a
53
a
63
a
15
a
23
a
34
a
71
. (DS. Khong pha i)
2) a
23
a
67
a
54
a
16
a
35
a
41
a
72
. (DS. So ha
.
ng co dau co
.
ng)
3) a
15
a
28
a
74
a
36
a
61
a
43
. (DS. Khong pha i)
4) a
72
a
16
a
33
a
55
a
27
a
61
a
44
. (DS. So ha
.
ng co dau co
.
ng)
3.2. D
-
i
.
nh th u
.
c 101
5. Trong cac tch sau day, tch n`ao l`a so ha
.
ng cu a di
.
nh th u
.
c c ap tu
.
o
.
ng
u
.
ng xac di
.
nh dau cu a so ha
.
ng d o.
1) a
43
a
61
a
52
a
13
a
25
a
34
. (DS. Khong pha i)
2) a
27
a
63
a
14
a
56
a
35
a
41
a
72
. (DS. L`a so ha
.
ng cu a di
.
nh th u
.
c cap 7
vo
.
i dau +)
3) a
15
a
28
a
75
a
36
a
81
a
43
. (DS. Khong pha i)
4) a
n1
a
n1 2
. . . a
1n
.
(DS. L`a so ha
.
ng cu a di
.
nh th u
.
c c ap n vo
.
i dau (1)
n(n1)
2
)
5) a
12
a
23
. . . a
k,k+1
. . . a
n1,n
a
n1
.
(DS. L`a so ha
.
ng cu a di
.
nh th u
.
c c ap n vo
.
i dau (1)
n1
)
6) a
13
a
24
a
35
. . . a
n2,n
a
n1,1
a
n2
.
(DS. So ha
.
ng cu a di
.
nh th u
.
c cap n vo
.
i dau +)
6. Xac di
.
nh cac so k v`a sao cho trong di
.
nh th u
.
c cap 6:
1
+
Cac tch sau l`a so ha
.
ng cu a no vo
.
i dau :
1) a
62
a
35
a
k3
a
44
a
6
a
21
. (DS. k = 5, = 1)
2) a
1k
a
25
a
44
a
6
a
52
a
31
. (DS. k = 6, = 3)
2
+
Cac tch sau l`a so ha
.
ng co dau +:
3) a
63
a
16
a
5
a
45
a
2k
a
31
. (DS. k = 2, = 4)
4) a
k5
a
21
a
34
a
13
a
6
a
62
. (DS. k = 5, = 4)
7. Trong di
.
nh th u
.
c c ap n
1) tch c ac pha
`
n tu
.
cu a du
.
`o
.
ng cheo chnh l`a so co dau g` ?
(DS. +)
2) tch c ac pha
`
n tu
.
cu a du
.
`o
.
ng cheo phu
.
co dau g` ?
(DS. Co dau + ne

u n = 4k hoa
.
c n = 4k + 1; v`a co dau
ne

u n = 4k + 2 hoa
.
c n = 4k + 3)
8. Tnh cac di
.
nh th u
.
c c ap hai:
1)

a
2
ab
ab b
2

2)

a
2
+ab +b
2
a
2
ab +b
2
a +b a b

3)

cos sin
sin cos

4)

sin cos
sin cos

102 Chu
.
o
.
ng 3. Ma tra
.
n. D
-
i
.
nh th u
.
c
5)

1 log
b
a
log
a
b 1

6)

a +bi c +di
c +di a bi

; i
2
1.
7)

(1 t)
2
1 +t
2
2t
1 +t
2
2t
1 +t
2

(1 +t)
2
1 +t
2

8)

, = cos
2
3
+i sin
2
3
.
(DS. 1) 0; 2) 2b
3
; 3) 1; 4) sin( ); 5) 0; 6) a
2
+ b
2
+ c
2
+ d
2
;
7) 1; 8) 1)
9. Tnh cac di
.
nh th u
.
c cap ba
1)

3 2 1
2 5 3
3 4 3

2)

a b c
b c a
c a b

3)

cos sin cos sinsin


sin cos cos cos sin
0 sin cos

.
4)

1 i 1 +i
i 1 0
1 i 0 1

; i
2
= 1, 5)

a
2
+ 1 ab ac
ab b
2
+ 1 bc
ac bc c
2
+ 1

6)

sin cos 1
sin cos 1
sin cos 1

7)

1 1
1 1
2

, = cos
2
3
+i sin
2
3
8)

a +b c 1
b +c a 1
c +a b 1

(DS. 1) 8; 2) 3abc a
3
b
3
c
3
; 3) 1; 4) 2; 5) 1 +a
2
+ b
2
+c
2
;
6) sin( ) + sin( ) + sin( ); 7) 3; 8) 0)
3.2. D
-
i
.
nh th u
.
c 103
10. Tnh di
.
nh th u
.
c Vandermonde
1

1 1 1 1
a b c d
a
2
b
2
c
2
d
2
a
3
b
3
c
3
d
3

(DS. (b a)(c a)(d a)(c b)(d b)(d c))


Ch da

n. Lay cac c o
.
t tr` u
.
di co
.
t th u
.
nhat ro
`
i khai trie

n di
.
nh th u
.
c
thu du
.
o
.
.
c theo h`ang th u
.
nhat v`a tie

p tu
.
c nhu
.
va
.
y doi vo
.
i di
.
nh th u
.
c
cap ba.
11. Tnh di
.
nh th u
.
c

1 1 1 0 0
1 2 3 0 0
0 1 1 1 1
0 x
1
x
2
x
3
x
4
0 x
2
1
x
2
2
x
2
3
x
2
4

(DS. (x
3
x
2
)(x
4
x
2
)(x
4
x
3
) 2(x
3
x
1
)(x
4
x
1
)(x
4
x
3
))
Ch da

n. D` ung di
.
nh l y Laplace cho h`ang th u
.
nhat v`a th u
.
hai v`a
ch da

n cho b`ai 10.


12. Tnh di
.
nh th u
.
c ba
`
ng cach khai trie

n (theo c ac pha
`
n tu
.
cu a h`ang
hoa
.
c c o
.
t):
1)

a 3 0 5
0 b 0 2
1 2 c 3
0 0 0 d

. (DS. abcd)
2)

1 1 1 a
2 2 1 b
3 2 1 c
1 2 3 d

theo cac pha


`
n tu
.
co
.
t th u
.
tu
.
.
1
A. T. Vandermonde (1735-1796) l`a nh`a toan ho
.
c Phap.
104 Chu
.
o
.
ng 3. Ma tra
.
n. D
-
i
.
nh th u
.
c
(DS. 4a c d)
3)

a 1 1 1
b 0 1 1
c 1 0 1
d 1 1 0

theo cac pha


`
n tu
.
cu a co
.
t th u
.
nhat.
(DS. 2a +b c +d)
4)

1 2 1 2
2 1 2 1
a b c d
2 1 1 2

theo cac pha


`
n tu
.
cu a h`ang th u
.
ba.
(DS. 5a 5b 5c 5d)
5)

2 3 5 4
3 5 4 2
4 2 3 5
5 4 2 3

theo cac pha


`
n tu
.
h`ang th u
.
hai.
(DS. 2858)
6)

5 1 4 1
1 4 1 5
4 1 8 1
3 2 6 2

theo cac pha


`
n tu
.
h`ang th u
.
nhat
(DS. 264)
13. D` ung di
.
nh ngha de

tnh c ac di
.
nh th u
.
c sau
1)

1 0 0
2 2 1
3 3 2

. (DS. 1)
2)

log
b
a 1 0
0 2 0
2 1 log
a
b

. (DS. 1)
3.2. D
-
i
.
nh th u
.
c 105
3)

1 0 0 2
3 0 0 4
0 5 6 0
0 7 8 0

. (DS. 4)
4)

0 0 3 4
0 0 4 3
1 2 0 0
2 1 0 0

. (DS. 21)
5)

a
1
0 0 . . . 0
a
1
a
1
0 . . . 0
.
.
.
.
.
.
.
.
.
.
.
.
.
.
.
a
n
a
n1
a
n2
. . . a
1

. (DS. a
n
1
)
6)

0 . . . 0 0 1
0 . . . 0 2 0
0 . . . 3 0 0
.
.
.
.
.
.
.
.
.
.
.
.
.
.
.
n . . . 0 0 0

. (DS. (1)
n(n+1)
2
n!)
7)

1 a a . . . a
0 2 a . . . a
0 0 3 . . . a
.
.
.
.
.
.
.
.
.
.
.
.
.
.
.
0 0 0 . . . n

. (DS. n!)
8)

0 . . . 0 0 a
1
0 . . . 0 a
2
a
1
0 . . . a
3
a
2
a
1
.
.
.
.
.
.
.
.
.
.
.
.
.
.
.
a
n
. . . a
3
a
2
a
1

. (DS. (1)
n(n1)
2
a
1
a
2
. . . a
n
)
106 Chu
.
o
.
ng 3. Ma tra
.
n. D
-
i
.
nh th u
.
c
9)

2 1 0 4
1 2 0 4
2 3 0 5
3 4 0 6

. (DS. 0)
10)

1 2 1 2 1
1 1 1 1 1
2 3 0 0 0
3 2 0 0 0
1 2 0 0 0

. (DS. 0)
14. Gia i cac phu
.
o
.
ng tr`nh
1)

1 1 4 4
1 3 x
2
3 3
7 7 5 5
7 7 6 x
2
3

= 0. (DS. x
1,2
= 3; x
3,4
= 3)
2)

1 2 3 4
2 2 x 1 7
3 6 4 +x 12
4 x 14 2 3

= 0. (DS. x
1
= 6; x
2
= 5)
3)

1 x x
2
x
3
1 2 4 8
1 3 9 27
1 4 16 64

= 0. (DS. x
1
= 2, x
2
= 3, x
3
= 4)
15. Tnh cac di
.
nh th u
.
c cap n
1)

2 2 3 . . . n
1 0 3 . . . n
1 2 0 . . . n
.
.
.
.
.
.
.
.
.
.
.
.
.
.
.
1 2 3 . . . 0

. (DS. n!)
Ch da

n. Them h`ang th u
.
nhat v`ao mo
.
i h`ang cu a di
.
nh th u
.
c ba

t
3.2. D
-
i
.
nh th u
.
c 107
da
`
u t` u
.
h`ang th u
.
hai.
2)

1 2 2 . . . 2
2 2 2 . . . 2
2 2 3 . . . 2
.
.
.
.
.
.
.
.
.
.
.
.
.
.
.
2 2 2 . . . n

. (DS. 2(n 2)!)


Ch da

n. Lay mo
.
i h`ang (ke

t` u
.
h`ang th u
.
ba) tr` u
.
di h`ang th u
.
hai,
sau do lay h`ang th u
.
hai tr` u
.
di h`ang th u
.
nhat nhan vo
.
i 2.
3)

x a
1
a
2
. . . a
n1
1
a
1
x a
2
. . . a
n1
1
a
1
a
2
x . . . a
n1
1
.
.
.
.
.
.
.
.
.
.
.
.
.
.
.
a
1
a
2
a
3
. . . x 1
a
1
a
2
a
3
. . . a
n
1

. (DS. (xa
1
)(xa
2
) (xa
n
))
Ch da

n. Lay tat ca cac c o


.
t cu a di
.
nh th u
.
c tr` u
.
di c o
.
t cuoi c` ung
nhan tu
.
o
.
ng u
.
ng vo
.
i a
1
, a
2
, . . . , a
n
.
4)

0 1 1 . . . 1
1 0 1 . . . 1
1 1 0 . . . 1
.
.
.
.
.
.
.
.
.
.
.
.
.
.
.
1 1 1 . . . 0

nn
. (DS. (1)
n1
(n 1))
Ch da

n. Them cho co
.
t th u
.
nhat tat ca cac co
.
t c` on la
.
i; sau do lay
mo
.
i h`ang ke

t` u
.
h`ang th u
.
hai tr` u
.
di h`ang th u
.
nhat.
5)

1 n n . . . n
n 2 n . . . n
n n 3 . . . n
.
.
.
.
.
.
.
.
.
.
.
.
.
.
.
n n n . . . n

. (DS. (1)
n
n!)
Ch da

n. Lay cac h`ang th u


.
nhat, th u
.
hai, ... th u
.
n1 tr` u
.
di h`ang
th u
.
n.
108 Chu
.
o
.
ng 3. Ma tra
.
n. D
-
i
.
nh th u
.
c
6)

1 x
1
x
2
. . . x
n1
x
n
1 x x
2
. . . x
n1
x
n
1 x
1
x . . . x
n1
x
n
.
.
.
.
.
.
.
.
.
.
.
.
1 x
1
x
2
. . . x x
n
1 x
1
x
2
. . . x
n1
x

. (DS. (xx
1
)(xx
2
) (xx
n
))
Ch da

n. Nhan h`ang th u
.
nhat vo
.
i (1) ro
`
i co
.
ng vo
.
i tat ca cac
h`ang c`on la
.
i.
7)

1 2 3 . . . n 1 n
1 3 3 . . . n 1 n
1 2 5 . . . n 1 n
.
.
.
.
.
.
.
.
.
.
.
.
.
.
.
1 2 3 . . . 2n 3 n
1 2 3 . . . n 1 2n 1

. (DS. (n 1)!)
Ch da

n. Nhan h`ang th u
.
nhat vo
.
i (1) ro
`
i co
.
ng vo
.
i tat ca cac
h`ang c`on la
.
i.
8)

a b 0 . . . 0 0
0 a b . . . 0 0
.
.
.
.
.
.
.
.
.
.
.
.
.
.
.
0 0 0 . . . a b
b 0 0 . . . 0 a

. (DS. a
n
+ (1)
n+1
b
n
)
9)

a
0
a
1
a
2
. . . a
n1
a
n
y
1
x
1
0 . . . 0 0
0 y
2
x
2
. . . 0 0
.
.
.
.
.
.
.
.
.
.
.
.
.
.
.
0 0 0 . . . y
n
x
n

.
(DS. a
0
x
1
x
2
x
n
+ a
1
y
1
x
2
x
n
+ a
2
y
1
y
2
x
3
x
n
+ +
a
n
y
1
y
2
y
n
)
Ch da

n. Khai trie

n di
.
nh th u
.
c theo co
.
t cu oi de

thu du
.
o
.
.
c he
.
th u
.
c
truy ho
`
i.
3.3. Ha
.
ng cu a ma tra
.
n 109
10)

1 2 3 4 . . . n 1 n
1 x 0 0 . . . 0 0
.
.
.
.
.
.
.
.
.
.
.
.
.
.
.
.
.
.
.
.
.
0 0 0 0 . . . x 0
0 0 0 0 . . . 1 x

(DS.
n
= x
n1
+n,
n
= x
n1
+ 2x
n2
+ + (n 1)x +n)
Ch da

n. Khai trie

n di
.
nh th u
.
c theo co
.
t cu oi.
3.3 Ha
.
ng cu

a ma tr a
.
n
3.3.1 D
-
i
.
nh ngha
So nguyen r > 0 du
.
o
.
.
c go
.
i l`a ha
.
ng cu a ma tra
.
n A ne

u no thoa man
hai die
`
u kie
.
n sau day:
(i) Ma tra
.
n A co t nhat mo
.
t di
.
nh th u
.
c con khac 0 cap r.
(ii) Mo
.
i di
.
nh th u
.
c con c ap r + 1 v`a c ap cao ho
.
n (ne

u co) cu a ma
tra
.
n A de
`
u ba
`
ng 0.
Ha
.
ng cu a ma tra
.
n Athu
.
`o
.
ng du
.
o
.
.
c k y hie
.
u l`a r(A), r
A
hoa
.
c rank(A).
T` u
.
di
.
nh ngha suy ra:
a) Doi vo
.
i (mn)-ma tra
.
n A ta co: 0 r(A) min(m; n).
b) r = r(A) = 0 khi v`a ch khi mo
.
i pha
`
n tu
.
cu a ma tra
.
n de
`
u ba
`
ng
0.
c) Doi vo
.
i ma tra
.
n vuong cap n ta co r(A) = n detA ,= 0.
3.3.2 Phu
.
o
.
ng phap t`m ha
.
ng cu a ma tra
.
n
Phu
.
o
.
ng phap I (phu
.
o
.
ng phap di
.
nh th u
.
c bao) du
.
.
a tren di
.
nh
ngha ha
.
ng cu a ma tra
.
n, go
`
m cac bu
.
o
.
c sau day
(i) T`m mo
.
t di
.
nh th u
.
c con n`ao do khac 0; gia su
.
do l`a di
.
nh th u
.
c

r
,= 0.
110 Chu
.
o
.
ng 3. Ma tra
.
n. D
-
i
.
nh th u
.
c
(ii) Tnh tie

p cac di
.
nh th u
.
c con
r+1
cap r + 1 bao di
.
nh th u
.
c
r
(t u
.
c l`a di
.
nh th u
.
c con
r+1
ch u
.
a di
.
nh th u
.
c con
r
) ne

u ch ung to
`
n ta
.
i.
+) Ne

u tat ca cac di
.
nh th u
.
c con cap r +1 de
`
u ba
`
ng 0 th` ke

t lua
.
n
r(A) = r.
+ Ne

u co mo
.
t di
.
nh th u
.
c con c ap r +1 khac 0 (
r+1
,= 0) th` tnh
tie

p cac di
.
nh th u
.
c con cap r + 2 bao di
.
nh th u
.
c
r+1
do (ne

u ch ung
to
`
n ta
.
i). Ne

u mo
.
i di
.
nh th u
.
c cap r + 2 de
`
u ba
`
ng 0 th` r(A) = r + 1,
c`on ne

u co mo
.
t di
.
nh th u
.
c con c ap r + 2 khac 0 th` quy tr`nh la
.
i tie

p
tu
.
c.
Phu
.
o
.
ng phap II du
.
.
a tren cac phep bie

n do

i so
.
cap thu
.
.
c hie
.
n
tren ma tra
.
n da cho.
D
-
i
.
nh ngha. Cac phep bie

n do

i sau day tren ma tra


.
n du
.
o
.
.
c go
.
i l`a cac
phep bie

n do

i so
.
cap:
1
+
Do

i cho

hai h`ang (hoa


.
c hai co
.
t) cho nhau.
2
+
Nhan tat ca cac pha
`
n tu
.
cu a mo
.
t h`ang (hoa
.
c co
.
t) vo
.
i mo
.
t so
khac 0.
3
+
Co
.
ng v`ao mo
.
t h`ang cu a ma tra
.
n mo
.
t h`ang khac sau khi nhan
vo
.
i mo
.
t so t` uy y ,= 0.
D
-
i
.
nh l y. Ha
.
ng cu a ma tr a
.
n l`a bat bie

n qua cac phep bie

n d o

i so
.
cap.
Khi thu
.
.
c hie
.
n cac phep bie

n do

i so
.
cap tren ma tra
.
n ta luon quy
u
.
o
.
c ra
`
ng dau A B co ngha l`a mo
.
t ma tra
.
n thu du
.
o
.
.
c t` u
.
ma tra
.
n
kia bo
.
i cac phep bie

n d o

i so
.
cap v`a r(A) = r(B).
C

AC V

I DU
.
V du
.
1. T`m ha
.
ng r(A) ne

u
A =
_

_
1 0 0 1
0 1 1 2
1 1 1 1
4 2 3 1
3 1 2 0
_

_
3.3. Ha
.
ng cu a ma tra
.
n 111
Gia i. Ta t`m ha
.
ng cu a ma tra
.
n da cho theo phu
.
o
.
ng phap I. Hie

n
nhien ma tra
.
n A co di
.
nh th u
.
c con

2
=

1 0
0 1

= 1 ,= 0.
Ta tnh cac di
.
nh th u
.
c con
3
bao
2
. Ta co

(1)
3
=

1 0 0
0 1 1
1 1 1

= (1)

1 1
1 1

= 0;

(2)
3
=

1 0 0
0 1 1
4 2 3

= 1 ,= 0.
Nhu
.
va
.
y co mo
.
t di
.
nh th u
.
c bao
(2)
3
,= 0. Ta tnh di
.
nh th u
.
c bao cu a

(2)
3
. Ta co

(1)
4
=

1 0 0 1
0 1 1 2
1 1 1 1
4 2 3 1

= 0
(ta
.
i sao ?). T` u
.
do suy ra r(A) = 3.
V du
.
2. T`m ha
.
ng r(A) ne

u
A =
_

_
1 3 2 5
2 4 3 1
0 2 7 11
7 15 7 2
1 1 5 6
_

_
Gia i. Ta gia i theo phu
.
o
.
ng phap I. Hie

n nhien ma tra
.
n A co di
.
nh
th u
.
c con

2
=

1 3
2 4

= 2 ,= 0.
112 Chu
.
o
.
ng 3. Ma tra
.
n. D
-
i
.
nh th u
.
c
Tat ca cac di
.
nh th u
.
c con bao
2
:

1 3 2
2 4 3
0 2 7

1 3 5
2 4 1
0 2 11

1 3 2
2 4 3
7 15 7

1 3 5
2 4 1
7 15 2

1 3 2
2 4 3
1 1 5

1 3 5
2 4 3
1 1 6

de
`
u ba
`
ng 0. Do do r(A) = 2.
V du
.
3. Ba
`
ng cac phep bie

n do

i so
.
cap, tnh ha
.
ng cu a cac ma tra
.
n
1) A =
_

_
1 2 3 5
3 1 4 2
5 3 10 8
_

_
; 2) B =
_

_
1 0 0 1
0 1 1 2
1 1 1 1
4 2 3 1
3 1 2 0
_

_
.
Gia i. 1) Ta thu
.
.
c hie
.
n phep bie

n do

i so
.
cap theo h`ang v`a thu du
.
o
.
.
c
A =
_

_
1 2 3 5
3 1 4 2
5 3 10 8
_

_h
2
3h
1
h

2
h
3
5h
1
h

_
1 2 3 5
0 7 5 7
0 7 5 17
_

_
h
3
h
2
h

_
1 2 3 5
0 7 5 17
0 0 0 0
_

_
.
Do l`a ma tra
.
n h`nh thang v`a hie

n nhien no co ha
.
ng ba
`
ng 2. Do do
r(A) = 2.
3.3. Ha
.
ng cu a ma tra
.
n 113
2) Ta c o
B =
_

_
1 0 0 1
0 1 1 2
1 1 1 1
4 2 3 1
3 2 1 0
_

_
h
3
+h
1
h

3
h
5
+ 4h
1
h

4
h
5
+ 3h
1
h

_
1 0 0 1
0 1 1 2
0 1 1 2
0 2 3 5
0 1 2 3
_

_
h
3
h
2
h

3
h
4
2h
2
h

4
h
5
h
2
h

_
1 0 0 1
0 1 1 2
0 0 0 0
0 0 1 1
0 0 1 1
_

_
1 0 0 1
0 1 1 2
0 0 1 1
0 0 0 0
0 0 1 1
_

_
h
5
h
3
h

_
1 0 0 1
0 1 1 2
0 0 1 1
0 0 0 0
0 0 0 0
_

_
.
T` u
.
do thu du
.
o
.
.
c r(B) = 3.
V du
.
4. Tnh ha
.
ng cu a cac ma tra
.
n
1) A =
_

_
1 2 4 5 2
2 3 1 1 3
0 1 7 9 1
1 3 11 14 3
_

_
; 2) B =
_

_
1 3 2 0 5
2 6 9 7 12
2 5 2 4 5
1 4 8 4 20
_

_
.
Gia i. 1) Ta thu
.
.
c hie
.
n cac phep bie

n do

i sau:
A =
_

_
1 2 4 5 2
2 3 1 1 3
0 1 7 9 1
1 3 11 14 3
_

_
1 2 4 5 2
0 1 7 9 1
0 1 7 9 1
0 1 7 9 1
_

_
1 2 4 5 2
0 1 7 9 1
0 1 7 9 1
0 1 7 9 1
_

_
1 2 4 5 2
0 1 7 9 1
0 0 0 0 0
0 0 0 0 0
_

_
1 2 4 5 2
0 1 7 9 1
_
T` u
.
do suy ra
`
ng r(A) = 2.
114 Chu
.
o
.
ng 3. Ma tra
.
n. D
-
i
.
nh th u
.
c
2) Ta thu
.
.
c hie
.
n cac phep bie

n do

i
B =
_

_
1 3 2 0 5
2 6 9 7 12
2 5 2 4 5
1 4 8 4 20
_

_
1 3 2 0 5
0 0 5 7 2
0 1 6 4 15
0 1 6 4 15
_

_
1 3 2 0 5
0 1 6 4 15
0 0 5 7 2
0 1 6 4 15
_

_
1 3 2 0 5
0 1 6 4 15
0 0 5 7 2
0 0 0 0 0
_

_
T` u
.
do suy ra
`
ng r(B) = 3.
B
`
AI T

A
.
P
T`m ha
.
ng cu a cac ma tra
.
n:
1. A =
_
1 2
3 1
_
. (DS. r(A) = 2)
2. A =
_
1 3
2 6
_
. (DS. r(A) = 1)
3. A =
_
1 2
3 6
_
. (DS. r(A) = 1)
4. A =
_

_
1 2
3 4
5 6
_

_
. (DS. r(A) = 2)
5. A =
_
1 2 1
1 4 3
_
. (DS. r(A) = 2)
3.3. Ha
.
ng cu a ma tra
.
n 115
6. A =
_

_
0 1 3
0 3 1
0 2 0
_

_. (DS. r(A) = 2)
7. A =
_

_
1 2 3
2 4 6
5 1 4
_

_. (DS. r(A) = 2)
8. A =
_

_
1 3 2
2 6 4
1 3 2
_

_. (DS. r(A) = 1)
9. A =
_

_
1 2 4 0
1 3 5 1
2 1 4 0
_

_
. (DS. r(A) = 3)
Su
.
du
.
ng cac phep bie

n do

i so
.
cap de

t`m ha
.
ng cu a ma tra
.
n:
10. A =
_

_
1 0 3 2
2 3 1 3
3 6 1 8
_

_. (DS. r(A) = 2)
11. A =
_

_
2 2 1
3 1 1
5 4 1
1 0 0
_

_
. (DS. r(A) = 3)
12. A =
_

_
4 9 0 7 2
1 1 6 0 3
0 1 2 1 3
4 3 1 9 6
_

_
(DS. r(A) = 4)
13. A =
_

_
1 3 2 1 3
4 1 2 4 1
6 9 1 2 6
4 6 1 12 4
_

_
. (DS. r(A) = 3)
116 Chu
.
o
.
ng 3. Ma tra
.
n. D
-
i
.
nh th u
.
c
14. A =
_

_
2 9 5 2 9 5
4 4 3 7 4 4
2 3 1 3 3 3
2 2 1 2 6 2
1 1 3 1 1 1
_

_
. (DS. r(A) = 4)
T`m ha
.
ng cu a ma tra
.
n ba
`
ng phu
.
o
.
ng phap di
.
nh th u
.
c bao:
15. A =
_

_
1 1 0 0 0
2 3 0 0 0
0 0 5 0 0
0 0 0 6 0
0 0 0 0 8
_

_
. (DS. r(A) = 5)
16. A =
_

_
1 2 3 4
1 3 0 1
2 4 1 8
1 7 6 9
0 10 1 10
_

_
. (DS. r(A) = 3)
17. A =
_

_
1 1 3 3 2
2 2 1 1 4
1 1 3 3 2
_

_
(DS. r(A) = 2)
18. A =
_

_
2 4 2 3 3 1
1 2 1 1 7 2
1 2 1 4 10 3
_

_. (DS. r(A) = 2)
19. A =
_

_
1 1 2 3 1
0 2 1 2 2
0 0 3 3 3
0 0 0 4 0
1 3 6 12 2
1 3 3 5 1
_

_
. (DS. r(A) = 4)
3.3. Ha
.
ng cu a ma tra
.
n 117
20. Vo
.
i gia tri
.
n`ao cu a th` ma tra
.
n
A =
_
1
1 2
_
co ha
.
ng ba
`
ng 1 ? (DS. =
1
2
)
21. Vo
.
i gia tri
.
n`ao cu a th` ha
.
ng r(A) = 2, ne

u
A =
_

_
0 1
3 4 1
1 1 2
_

_
? (DS. =
7
9
)
22. Vo
.
i gia tri
.
n`ao cu a th` ha
.
ng r(A) = 3 ne

u
A =
_

_
1 0 1
2 2 3
1 0 4
_

_? (DS. ,= 2)
23. Vo
.
i gia tri
.
n`ao cu a th` ha
.
ng r(A) = 3 ne

u
A =
_

_
1 0
2 3 4
0 0 8
_

_
? (DS. R)
24. V o
.
i gia tri
.
n`ao cu a th` ha
.
ng: 1) r(A) = 1; 2) r(A) = 2;
3) r(A) = 3 ne

u:
A =
_

_
1 2
2 1 4
4 2 8
_

_?
(DS. 1) =
1
2
; 2) ,=
1
2
; 3) Khong to
`
n ta
.
i)
118 Chu
.
o
.
ng 3. Ma tra
.
n. D
-
i
.
nh th u
.
c
3.4 Ma tra
.
n nghi
.
ch da

o
3.4.1 D
-
i
.
nh ngha
Ne

u A l`a ma tra
.
n vuong cap n th` ma tra
.
n vuong B cap n thoa man
die
`
u kie
.
n
AB = BA = E
n
trong do E
n
l`a ma tra
.
n do
.
n vi
.
cap n du
.
o
.
.
c go
.
i l`a ma tr a
.
n nghi
.
ch da o
doi vo
.
i ma tra
.
n A v`a du
.
o
.
.
c k y hie
.
u l`a B = A
1
.
Nhu
.
va
.
y theo di
.
nh ngha
AA
1
= A
1
A = E
n
.
D
-
i
.
nh l y. Ma tra
.
n vuong A c o ma tr a
.
n nghi
.
ch da o khi v` a ch khi ma
tra
.
n A khong suy bie

n (t u
.
c l` a khi detA ,= 0) v` a khi d o
A
1
=
1
detA
P
A
, (3.12)
P
A
=
_

_
A
11
A
21
. . . A
n1
A
12
A
22
. . . A
n2
.
.
.
.
.
.
.
.
.
.
.
.
A
1n
A
2n
. . . A
nn
_

_
trong do A
ij
l`a pha
`
n b` u da
.
i s o cu a pha
`
n tu
.
a
ij
(i, j = 1, n) cu a ma
tra
.
n A. Ma tra
.
n P
A
du
.
o
.
.
c go
.
i l`a ma tr a
.
n phu
.
ho
.
.
p cu a ma tr a
.
n A.
Tnh cha

t
1
+
Ne

u ma tra
.
n A co ma tra
.
n nghi
.
ch da o v`a m ,= 0 th` ma tra
.
n
mA c ung co ma tra
.
n nghi
.
ch da o v`a
(mA)
1
=
1
m
A
1
.
3.4. Ma tra
.
n nghi
.
ch da o 119
2
+
Ne

u A v`a B l`a hai ma tra


.
n vuong c` ung cap v`a de
`
u co ma tra
.
n
nghi
.
ch da o th`
(AB)
1
= B
1
A
1
.
3
+
Ne

u A co ma tra
.
n nghi
.
ch da o A
1
th` A
1
c ung co ma tra
.
n
nghi
.
ch da o v`a
_
A
1
_
1
= A.
3.4.2 Phu
.
o
.
ng phap t`m ma tra
.
n nghi
.
ch da o
Phu
.
o
.
ng phap I go
`
m cac bu
.
o
.
c sau
Bu
.
o
.
c 1. Tnh detA
+ Ne

u detA = 0 th` A khong co ma tra


.
n nghi
.
ch da o.
+ Ne

u detA ,= 0 th` chuye

n sang bu
.
o
.
c 2.
Bu
.
o
.
c 2. T`m ma tra
.
n phu
.
ho
.
.
p P
A
. T` u
.
do ap du
.
ng cong th u
.
c
(3.12) ta thu du
.
o
.
.
c ma tra
.
n A
1
.
Phu
.
o
.
ng phap II (phu
.
o
.
ng phap Gauss-Jordan)
Da
`
u tien ta vie

t ma tra
.
n do
.
n vi
.
c` ung cap vo
.
i ma tra
.
n A v`ao ben
pha i ma tra
.
n A v`a thu du
.
o
.
.
c ma tra
.
n
M =
_
A[E
n
_
. (3.13)
Tie

p theo thu
.
.
c hie
.
n c ac phep bie

n do

i so
.
cap tren cac h`ang cu a
ma tra
.
n M de

du
.
a khoi ma tra
.
n A ve
`
ma tra
.
n do
.
n vi
.
E
n
c`on khoi E
n
trong (3.13) th`anh ma tra
.
n B:
_
A[E
n
_

_
E
n
[B
_
.
Khi do B = A
1
.
C

AC V

I DU
.
120 Chu
.
o
.
ng 3. Ma tra
.
n. D
-
i
.
nh th u
.
c
V du
.
1. T`m ma tra
.
n nghi
.
ch da o doi vo
.
i cac ma tra
.
n sau:
1) A =
_

_
3 5 2
1 3 2
6 7 3
_

_; 2) A =
_

_
1 0 2 1 7
5 4 2 0 3
1 6 7 8 9
2 6 5 9 2
3 2 1 0 1
_

_
Gia i. 1) Ta c o detA = 10 ,= 0. Do do ma tra
.
n A trong 1) co
ma tra
.
n nghi
.
ch da o. Pha
`
n b` u da
.
i so cu a cac pha
`
n tu
.
cu a no ba
`
ng:
A
11
= 5; A
12
= 15; A
13
= 25; A
21
= 1; A
22
= 3; A
23
= 9; A
31
= 4;
A
32
= 8; A
33
= 14.
T` u
.
do theo cong th u
.
c (3.12) ta co
A
1
=
1
10
_

_
5 1 4
15 3 8
25 9 14
_

_ =
_

1
2
1
10
2
5
3
2
3
10

4
5
5
2
9
10

7
5
_

_
.
2) Ta tnh detA. Lay h`ang th u
.
ba co
.
ng v`ao h`ang th u
.
nhat ta co
detA =

2 6 5 9 2
5 4 2 0 3
1 6 7 8 9
2 6 5 9 2
3 2 1 0 1

= 0
v` trong ma tra
.
n thu du
.
o
.
.
c co h`ang th u
.
nhat v`a th u
.
tu
.
giong nhau.
Nhu
.
va
.
y ma tra
.
n A trong 2) l`a ma tra
.
n suy bie

n, do do no khong co
ma tra
.
n nghi
.
ch da o.
V du
.
2. D` ung cac phep bie

n do

i so
.
cap t`m ma tra
.
n nghi
.
ch da o doi
3.4. Ma tra
.
n nghi
.
ch da o 121
vo
.
i ma tra
.
n
1) A =
_

_
2 0 4
1 1 2
1 2 3
_

_
; 2) A =
_

_
2 3 4
2 6 8
2 6 12
_

_
.
Gia i. 1) Ta la
.
p ma tra
.
n
M =
_

_
2 0 4

1 0 0
1 1 2

0 1 0
1 2 3

0 0 1
_

_
.
Nhan h` ang th u
.
nhat vo
.
i
1
2
ta thu du
.
o
.
.
c
M
_

_
1 0 2

1
2
0 0
1 1 2

0 1 0
1 2 3

0 0 1
_

_
h
2
h
1
h

2
h
3
+h
1
h

_
1 0 2

1
2
0 0
0 1 4


1
2
1 0
0 2 5

1
2
0 1
_

_
h
2
(1)h

_
1 0 2

1
2
0 0
0 1 4

1
2
1 0
0 2 5

1
2
0 1
_

_
h
3
2h
2
h

_
1 0 2

1
2
0 0
0 1 4

1
2
1 0
0 0 3


1
2
2 1
_

_
h
3
(
1
3
) h

122 Chu
.
o
.
ng 3. Ma tra
.
n. D
-
i
.
nh th u
.
c
_

_
1 0 2

1
2
0 0
0 1 4

1
2
1 0
0 0 1

1
6

2
3

1
3
_

_
h
1
2h
3
h

h
2
4h
3
h

2
_

_
1 0 0

1
6
4
3
2
3
0 1 0


1
6
5
3
4
4
0 0 1

1
6

2
3

1
3
_

_
T` u
.
do suy ra
`
ng
A
1
=
_

_
1
6
4
3
2
3

1
6
5
3
4
3
1
6

2
3

1
3
_

_
2) Ta la
.
p ma tra
.
n
M =
_

_
2 3 4

1 0 0
2 6 8

0 1 0
2 6 12

0 0 1
_

_
.
Thu
.
.
c hie
.
n cac phep bie

n do

i so
.
cap
M
h
2
h
1
h

h
3
h
1
h

3
_

_
2 3 4

1 0 0
0 3 4

1 1 0
0 3 8

1 0 1
_

h
3
h
2
h

3
_

_
2 3 4

1 0 0
0 3 4

1 1 0
0 0 4

0 1 1
_

_
h
1
h
3
h

1
h
2
h
3
h

_
2 3 0

1 1 1
0 3 0

1 2 1
0 0 4

0 1 1
_

_
3.4. Ma tra
.
n nghi
.
ch da o 123
h
1
h
2
h

_
2 0 0

2 1 0
0 3 0

1 2 1
0 0 4

0 1 1
_

_
h
1
(
1
2
) h

h
2
(
1
3
) h

2
h
3
(
1
4
) h

4
_

_
1 0 0

1
1
2
0
0 1 0


1
3
2
3

1
3
0 0 1

0
1
4
1
4
_

_
T` u
.
do suy ra
`
ng
A
1
=
_

_
1
1
2
0

1
3
2
3

1
3
0
1
4
1
4
_

_
V du
.
3. Ch u
.
ng minh c ac tnh chat sau day cu a di
.
nh th u
.
c
1) detA
1
= (detA)
1
.
2) Ne

u A v`a B khong suy bie

n th` tch AB c ung khong suy bie

n
v`a
(AB)
1
= B
1
A
1
.
3)
_
A
1
_
1
= A.
4)
_
A
T
_
1
=
_
A
1
_
T
.
Gia i. 1) Ta se ap du
.
ng cong th u
.
c tnh di
.
nh th u
.
c cu a tch hai ma
tra
.
n vuong c` ung cap A v`a B:
detAB = detA detB
Ta co
AA
1
= E det(AA
1
) = detE = 1
detA detA
1
= 1 (detA)
1
= det(A
1
).
124 Chu
.
o
.
ng 3. Ma tra
.
n. D
-
i
.
nh th u
.
c
2) Ta c o
(B
1
A
1
)(AB) = B
1
(A
1
A)B = B
1
B = E
v`a t` u
.
do suy ra B
1
A
1
= (AB)
1
. Tu
.
o
.
ng tu
.
.
B
1
A
1
(AB) = E v`a
do do ma tra
.
n B
1
A
1
l`a ma tra
.
n nghi
.
ch da o cu a ma tra
.
n AB.
3) Ta thay
_
A
1
_
1
l`a ma tra
.
n duy nhat m`a tch cu a no nhan vo
.
i
A
1
ba
`
ng E. Nhu
.
ng ma tra
.
n A c ung co tnh chat do. Nhu
.
va
.
y 3)
du
.
o
.
.
c ch u
.
ng minh.
4) De

ch u
.
ng minh
_
A
T
_
1
=
_
A
1
_
T
ta xet da

ng th u
.
c AA
1
= E.
T` u
.
do ap du
.
ng tnh chat cu a ma tra
.
n chuye

n vi
.
ta co
(AA
1
)
T
= E (A
1
)
T
A
T
= E (A
1
)
T
= (A
T
)
1
theo di
.
nh ngha ma tra
.
n nghi
.
ch da o.
V du
.
4. 1) Ch u
.
ng minh ra
`
ng ne

u A l`a ma tra
.
n vuong thoa man
die
`
u kie
.
n A
2
3A+E = O th` A
1
= 3E A.
2) Ch u
.
ng minh ra
`
ng (E A)
1
= E +A+A
2
ne

u A
3
= O.
Gia i. 1) T` u
.
die
`
u kie
.
n da cho ta co
E = 3AA
2
= A(3E A).
Do va
.
y
detA det(3E A) = detE = 1
v`a do do detA ,= 0, t u
.
c l`a A co ma tra
.
n nghi
.
ch da o. Do
E = A(3E A) A
1
E = A
1
A(3E A)
A
1
= 3E A.
2) Ta co the

nhan ma tra
.
n E A vo
.
i E +A+ A
2
. Ne

u ch ung l`a
ma tra
.
n nghi
.
ch da o nhau th` ke

t qua l`a ma tra


.
n do
.
n vi
.
. Ta co
(E A)(E +A+A
2
) = E A+AA
2
+A
2
A
3
= E A
3
= E
3.4. Ma tra
.
n nghi
.
ch da o 125
v` theo gia thie

t A
3
= O. T` u
.
do suy ra die
`
u pha i ch u
.
ng minh.
V du
.
5. T`m ma tra
.
n nghi
.
ch da o doi vo
.
i ma tra
.
n
A =
_


_
.
Gia i. De

to
`
n ta
.
i ma tra
.
n nghi
.
ch da o ta c a
`
n gia thie

t ra
`
ng detA =
,= 0. Vo
.
i gia thie

t do ta t`m c ac pha
`
n b` u da
.
i so: A
11
= ;
A
12
= ; A
21
= ; A
22
= . Do do
A
1
=
1

_


_
.
T` u
.
v du
.
n`ay ta r ut ra quy ta

c t`m ma tra
.
n nghi
.
ch da o vo
.
i ma
tra
.
n c ap 2:
Ma tra
.
n nghi
.
ch da o cu a ma tra
.
n cap hai ba
`
ng tch cu a mo
.
t so
l`a nghi
.
ch da o cu a di
.
nh th u
.
c cu a no nhan vo
.
i ma tra
.
n m`a du
.
`o
.
ng
cheo chnh l`a hoan vi
.
cu a hai pha
`
n tu
.
cu a du
.
`o
.
ng cheo chnh cu a
no v`a c ac pha
`
n tu
.
cu a du
.
`o
.
ng cheo th u
.
hai c ung chnh l`a cac
pha
`
n tu
.
cu a du
.
`o
.
ng cheo th u
.
hai cu a ma tra
.
n da cho nhu
.
ng vo
.
i
dau ngu
.
o
.
.
c la
.
i.
Cha

ng ha
.
n, ne

u A =
_
6 5
2 2
_
th`
A
1
=
1
2
_
2 5
2 6
_
.
V du
.
6. 1) Gia su
.
A l`a ma tra
.
n khong suy bie

n. H ay gia i cac phu


.
o
.
ng
tr`nh ma tra
.
n:
AX = B, Y A = B.
2) Gia i cac phu
.
o
.
ng tr`nh trong 1) ne

u
A =
_
7 3
2 1
_
, B =
_
1 2
0 1
_
.
126 Chu
.
o
.
ng 3. Ma tra
.
n. D
-
i
.
nh th u
.
c
Gia i. Nhan ben trai hai ve

cu a phu
.
o
.
ng tr`nh AX = B vo
.
i A
1
v`a
thu
.
.
c hie
.
n cac phep tnh da
.
i so tu
.
o
.
ng u
.
ng ta c o
A
1
AX = A
1
B EX = A
1
B X = A
1
B.
Tu
.
o
.
ng tu
.
.
Y AA
1
= BA
1
Y E = BA
1
Y = BA
1
.
Ro r`ang l`a ne

u A
1
v`a B khong giao ho an th` X ,= Y .
2) Vo
.
i
A =
_
7 3
2 1
_
A
1
=
1
detA
_
1 3
2 7
_
=
_
1 3
2 7
_
T` u
.
do
X = A
1
B =
_
1 3
2 7
_ _
1 2
0 1
_
=
_
1 5
2 11
_
,
Y = BA
1
=
_
1 2
0 1
_ _
1 3
2 7
_
=
_
3 11
2 7
_
.
B
`
AI T

A
.
P
T`m ma tra
.
n nghi
.
ch da o cu a ma tra
.
n da cho (ne

u ch ung to
`
n ta
.
i)
1.
_
2 1
3 5
_
. (DS.
1
13
_
5 1
3 2
_
)
2.
_
0 1
3 2
_
. (DS.
1
3
_
2 1
3 0
_
)
3.
_

_
1 1 3
5 1 2
1 4 1
_

_. (DS.
1
41
_

_
9 11 5
7 4 13
19 5 6
_

_)
3.4. Ma tra
.
n nghi
.
ch da o 127
4.
_

_
1 2 3
3 1 5
5 3 1
_

_. (DS. Khong to
`
n ta
.
i)
5.
_

_
2 3 0
1 1 4
3 2 5
_

_
. (DS.
1
25
_

_
13 15 12
17 10 8
1 5 1
_

_
)
6.
_

_
1 0 1
0 0 2
1 3 1
_

_. (DS.
1
6
_

_
6 3 0
2 2 2
0 3 0
_

_)
7.
_

_
1 3 4
0 1 2
0 1 5
_

_. (DS.
1
3
_

_
3 11 2
0 5 +2
0 1 1
_

_)
8.
_

_
3 2 1
1 1 2
2 2 5
_

_
. (DS.
_

_
1 12 5
1 17 7
0 2 1
_

_
)
9.
_

_
1 0 0
0
1

2
1

2
0
1

2
1

2
_

_
. (DS.
_

_
1 0 0
0
1

2
0
1

2
1

2
_

_
)
10.
_

_
1 2 2
2 1 2
2 2 1
_

_
. (DS.
1
9
_

_
1 2 2
2 1 2
2 2 1
_

_
)
11.
_

_
2 1 1
3 1 2
3 1 0
_

_. (DS.
_

_
1
1
2
1
2
3
3
2

1
2
0
1
2
1
2
_

_
)
128 Chu
.
o
.
ng 3. Ma tra
.
n. D
-
i
.
nh th u
.
c
12.
_

_
1 5 2
1 4 1
1 2 1
_

_. (DS.
_

1
3
1
6
1
2
0
1
2

1
2
1
3

7
6
3
2
_

_
)
13.
_

_
1 4 1
1 1 4
1 2 2
_

_
. (DS.
_

_
2
9
2
9

5
9
2
9

1
9
1
9

1
9
2
9
1
9
_

_
)
14.
_

_
1 3 1
1 4 1
1 9 2
_

_. (DS.
_

_
17 15 1
1 1 0
13 12 1
_

_)
15.
_

_
1 2 3 4
1 1 0 0
3 5 1 3
2 1 0 3
_

_
. (DS. Khong to
`
n ta
.
i)
16.
_

_
1 1 1 1
0 1 1 1
0 0 1 1
0 0 0 1
_

_
. (DS.
_

_
1 1 0 0
0 1 1 0
0 0 1 1
0 0 0 1
_

_
)
17.
_

_
1 1 0 0
1 0 1 1
0 0 1 1
1 1 1 0
_

_
. (DS.
_

_
0 1 1 0
1 1 1 0
1 0 0 1
1 0 1 1
_

_
)
18.
_

_
1 2 1 2
2 5 3 5
0 0 5 4
0 0 6 5
_

_
. (DS.
_

_
5 2 5 4
2 1 11 9
0 0 5 4
0 0 6 5
_

_
)
3.4. Ma tra
.
n nghi
.
ch da o 129
19.
_

_
1 3 5 7
0 1 2 3
0 0 1 2
0 0 0 1
_

_
. (DS.
_

_
1 3 11 38
0 1 2 7
0 0 1 2
0 0 0 1
_

_
)
20.
_

_
a
11
0 . . . 0
0 a
22
. . . 0
.
.
.
.
.
.
.
.
.
.
.
.
0 0
.
.
. a
nn
_

_
, a
11
a
12
a
nn
,= 0.
(DS.
_

_
1
a
11
0 . . . 0
0
1
a
22
. . . 0
.
.
.
.
.
.
.
.
.
.
.
.
0 0 . . .
1
a
nn
_

_
)
21.
_

_
1 1 1 . . . 1
0 1 1 . . . 1
0 0 1 . . . 1
.
.
.
.
.
.
.
.
.
.
.
.
.
.
.
0 0 0
.
.
. 1
_

_
. (DS.
_

_
1 1 0 . . . 0 0
0 1 1 . . . 0 0
0 0 1 . . . 0 0
.
.
.
.
.
.
.
.
.
.
.
.
.
.
.
.
.
.
0 0 0 . . . 1 1
0 0 0
.
.
. 0 1
_

_
)
22.
_

_
1 a a
2
. . . a
n
0 1 a . . . a
n1
0 0 1 . . . a
n2
.
.
.
.
.
.
.
.
.
.
.
.
.
.
.
0 0 0 . . . 1
_

_
. (DS.
_

_
1 a 0 . . . 0 0
0 1 a . . . 0 0
0 0 1 . . . 0 0
.
.
.
.
.
.
.
.
.
.
.
. 1 a
0 0 0 . . . 0 1
_

_
)
23. Vo
.
i gia tri
.
n`ao cu a th` c ac ma tra
.
n sau day co ma tra
.
n nghi
.
ch
da o:
130 Chu
.
o
.
ng 3. Ma tra
.
n. D
-
i
.
nh th u
.
c
1)
_

_
1 2 2
3 0
2 1 1
_

_; 2)
_

_
2 0
2 1
0 1
_

_.
(DS. 1) ,=
9
4
; 2) ,= 0, =

5)
24. T`m ma tra
.
n X thoa man cac phu
.
o
.
ng tr`nh
1)
_
2 1
3 1
_
X =
_
1 1
0 1
_
. (DS.
1
5
_
1 0
3 5
_
)
2) X
_
3 1
2 1
_
=
_
2 1
3 1
_
. (DS.
_
5 2
13 5
_
)
3)
_
2 1
1 0
_
X
_
3 1
1 2
_
=
_
1 1
3 0
_
. (DS.
_
6 3
11 2
_
)
4) AX +B = 2C, trong do
A =
_

_
1 1 1
0 1 1
0 0 1
_

_, B =
_

_
1 1 2
0 3 4
2 0 1
_

_, C =
_

_
2 3 0
4 3 5
1 1 0
_

_.
(DS.
_

_
5 16 8
4 7 5
4 2 1
_

_
)
5) XA2B = E, trong do
A =
_

_
1 1 3
2 5 7
1 1 2
_

_, B =
_

_
1 3 2
1 2 0
3 1 4
_

_.
(DS.
1
15
_

_
21 45 156
21 15 21
51 20 79
_

_)
25. Gia su
.
A l`a ma tra
.
n cap n v`a (E + A)
k
= O vo
.
i so tu
.
.
nhien k
n`ao do. Ch u
.
ng minh ra
`
ng ma tra
.
n A khong suy bie

n.
3.4. Ma tra
.
n nghi
.
ch da o 131
26. Ch u
.
ng minh ra
`
ng cac ma tra
.
n A+ E v`a A E khong suy bie

n
v`a nghi
.
ch da o nhau ne

u A
2
= O.
27. Ch u
.
ng minh ra
`
ng ma tra
.
n A+E v`a A
2
+E A khong suy bie

n
v`a nghi
.
ch da o nhau ne

u A
3
= O.
28. Ch u
.
ng minh ra
`
ng ne

u A, B, C l`a nh u
.
ng ma tra
.
n khong suy bie

n
th` ABC v`a C
1
B
1
A
1
l`a nghi
.
ch da o nhau.
29. Ma tra
.
n vuong A cap n du
.
o
.
.
c go
.
i l`a d o
`
ng da
.
ng vo
.
i ma tr a
.
n vu ong
c` ung cap B ne

u to
`
n ta
.
i ma tra
.
n kha nghi
.
ch T sao cho B = T
1
AT.
Ch u
.
ng minh cac tnh chat sau cu a ma tra
.
n do
`
ng da
.
ng:
1
+
Mo
.
i ma tra
.
n de
`
u do
`
ng da
.
ng vo
.
i chnh no.
2
+
Ne

u A do
`
ng da
.
ng vo
.
i B th` B do
`
ng da
.
ng vo
.
i A.
3
+
Ne

u A do
`
ng da
.
ng vo
.
i B, c`on B do
`
ng da
.
ng vo
.
i C th` A do
`
ng
da
.
ng vo
.
i C.
Ch da

n. 1
+

Ap du
.
ng he
.
th u
.
c E
1
= E. 2
+
Nhan ben pha i he
.
th u
.
c B = T
1
AT vo
.
i T
1
v`a nhan ben trai vo
.
i T. 3
+

Ap du
.
ng di
.
nh
ngha.
30. Ma tra
.
n vuong du
.
o
.
.
c go
.
i l`a ma tr a
.
n tru
.
.
c giao ne

u AA
T
= A
T
A =
E, ngha l`a ma tr a
.
n chuye

n vi
.
A
T
b a
`
ng ma tr a
.
n nghi
.
ch da o A
1
cu a
A. Ch u
.
ng minh c ac tnh chat sau cu a ma tra
.
n tru
.
.
c giao:
1
+
Ne

u A tru
.
.
c giao th` A
1
tru
.
.
c giao.
2
+
Tch c ac ma tra
.
n tru
.
.
c giao c` ung cap l`a ma tra
.
n tru
.
.
c giao.
3
+
Ne

u A l`a ma tra
.
n tru
.
.
c giao th` A
T
c ung l`a ma tra
.
n tru
.
.
c giao.
4
+
Di
.
nh th u
.
c cu a ma tra
.
n tru
.
.
c giao l`a ba
`
ng 1.
Ch da

n 4
+
. Xuat phat t` u
.
AA
T
= E v`a ap du
.
ng di
.
nh l y det(AB) =
detAdetB.
Chu
.
o
.
ng 4
He
.
phu
.
o
.
ng tr`nh tuye

n tnh
4.1 He
.
n phu
.
o
.
ng tr`nh vo
.
i n a

n co di
.
nh th u
.
c
khac 0 . . . . . . . . . . . . . . . . . . . . . . 132
4.1.1 Phu
.
o
.
ng phap ma tra
.
n . . . . . . . . . . . . 133
4.1.2 Phu
.
o
.
ng phap Cramer . . . . . . . . . . . . 134
4.1.3 Phu
.
o
.
ng phap Gauss . . . . . . . . . . . . . 134
4.2 He
.
t` uy y cac phu
.
o
.
ng tr`nh tuye

n tnh . . . 143
4.3 He
.
phu
.
o
.
ng tr`nh tuye

n tnh thua
`
n nhat . . 165
4.1 He
.
n phu
.
o
.
ng tr`nh vo
.
i n a

n c o di
.
nh
th u
.
c khac 0
He
.
phu
.
o
.
ng tr`nh tuye

n tnh tren tru


.
`o
.
ng so T du
.
o
.
.
c go
.
i l`a he
.
Cramer
1
ne

u so phu
.
o
.
ng tr`nh ba
`
ng so a

n v`a di
.
nh th u
.
c cu a ma tra
.
n co
.
ba n (ma
tra
.
n he
.
so) cu a he
.
l`a khac khong.
1
G. Cramer (1704-1752) l`a nh`a toan ho
.
c Thu
.
y S.
4.1. He
.
n phu
.
o
.
ng tr`nh vo
.
i n a

n co di
.
nh th u
.
c khac 0 133
He
.
Cramer co da
.
ng
a
11
x
1
+a
12
x
2
+ +a
1n
x
n
= h
1
,
a
21
x
1
+a
22
x
2
+ +a
2n
x
n
= h
2
,
. . . . . . . . . . . . . . . . . .
a
n1
x
1
+a
n2
x
2
+ +a
nn
x
n
= h
n
_

_
(4.1)
hay du
.
o
.
i da
.
ng ma tra
.
n
AX = H (4.2)
trong do
A =
_

_
a
11
a
12
. . . a
1n
a
21
a
22
. . . a
2n

.
.
.
.
.
.
.
.
.
a
n1
a
n2
. . . a
nn
_

_
, X =
_

_
x
1
x
2
.
.
.
x
n
_

_
, H =
_

_
h
1
h
2
.
.
.
h
n
_

_
hoa
.
c
_

_
a
11
a
21
.
.
.
a
n1
_

_
x
1
+
_

_
a
12
a
22
.
.
.
a
n2
_

_
x
2
+ +
_

_
a
1n
a
2n
.
.
.
a
nn
_

_
x
n
=
_

_
h
1
h
2
.
.
.
h
n
_

_
.
4.1.1 Phu
.
o
.
ng phap ma tra
.
n
V` detA ,= 0 nen to
`
n ta
.
i ma tra
.
n nghi
.
ch da o A
1
. Khi do t` u
.
(4.2) ta
thu du
.
o
.
.
c
A
1
AX = A
1
H EX = X = A
1
H.
Va
.
y he
.
nghie
.
m duy nhat l`a
X = A
1
H. (4.3)
Tuy nhien vie
.
c t`m ma tra
.
n nghi
.
ch da o noi chung l`a rat ph u
.
c ta
.
p ne

u
cap cu a ma tra
.
n A lo
.
n.
134 Chu
.
o
.
ng 4. He
.
phu
.
o
.
ng tr`nh tuye

n tnh
4.1.2 Phu
.
o
.
ng phap Cramer
Nghie
.
m duy nhat cu a he
.
Cramer du
.
o
.
.
c xac di
.
nh theo cong th u
.
c
Cramer:
x
j
=
det(A
j
)
detA
, j = 1, n (4.4)
trong do A
j
l`a ma tra
.
n thu du
.
o
.
.
c t` u
.
ma tra
.
n A ba
`
ng cach thay co
.
t
th u
.
j bo
.
i co
.
t c ac he
.
so tu
.
.
do H, v`a cac c o
.
t khac gi u
.
nguyen.
4.1.3 Phu
.
o
.
ng phap Gauss
No
.
i dung chu ye

u cu a phu
.
o
.
ng phap Gauss (hay thua
.
t toan Gauss) l`a
khu
.
lien tie

p cac a

n cu a he
.
. Thua
.
t toan Gauss du
.
.
a tren cac phep bie

n
do

i so
.
cap he
.
phu
.
o
.
ng tr`nh. Do l`a cac phep bie

n do

i:
1
+
Nhan mo
.
t phu
.
o
.
ng tr`nh n`ao do cu a he
.
vo
.
i mo
.
t so khac 0.
2
+
Them v`ao mo
.
t phu
.
o
.
ng tr`nh n`ao do cu a he
.
mo
.
t phu
.
o
.
ng tr`nh
khac nhan vo
.
i mo
.
t so t` uy y.
3
+
Do

i cho

hai phu
.
o
.
ng tr`nh cu a he
.
.
D
-
i
.
nh l y. Mo
.
i phep bie

n do

i so
.
cap thu
.
.
c hie
.
n tren he
.
phu
.
o
.
ng tr`nh
(4.1) de
`
u du
.
a de

n mo
.
t he
.
phu
.
o
.
ng tr`nh m o
.
i tu
.
o
.
ng du
.
o
.
ng.
Vie
.
c thu
.
.
c hie
.
n c ac phep bie

n do

i so
.
cap tren he
.
phu
.
o
.
ng tr`nnh
(4.1) thu
.
.
c chat l`a thu
.
.
c hie
.
n cac phep bie

n do

i so
.
cap tren cac h`ang
cu a ma tra
.
n mo
.
ro
.
ng cu a he
.
.
Do do sau mo
.
t so bu
.
o
.
c bie

n do

i ta thu du
.
o
.
.
c he
.
(4.1) tu
.
o
.
ng du
.
o
.
ng
vo
.
i he
.
tam giac
b
11
x
1
+b
12
x
2
+ +b
1n
x
n
= h
1
b
22
x
2
+ +b
2n
x
n
= h
2
. . . . . . . . .
b
nn
x
n
= h
n
_

_
T` u
.
do r ut ra x
n
, x
n1
, . . . , x
2
, x
1
.
4.1. He
.
n phu
.
o
.
ng tr`nh vo
.
i n a

n co di
.
nh th u
.
c khac 0 135
C

AC V

I DU
.
V du
.
1. Gia i cac he
.
phu
.
o
.
ng tr`nh sau ba
`
ng phu
.
o
.
ng phap ma tra
.
n
1)
x
1
+x
2
+x
3
= 4,
x
1
+ 2x
2
+ 4x
3
= 4,
x
1
+ 3x
2
+ 9x
3
= 2.
_

_
(4.5)
2)
3x
1
+ 2x
2
x
3
= 1,
x
1
+x
2
+ 2x
3
= 2,
2x
1
+ 2x
2
+ 5x
3
= 3.
_

_
(4.6)
Gia i. 1) Ta k y hie
.
u
A =
_

_
1 1 1
1 2 4
1 3 9
_

_
, X =
_

_
x
1
x
2
x
3
_

_
, H =
_

_
4
4
2
_

_
.
Khi do phu
.
o
.
ng tr`nh (4.5) co da
.
ng
AX = H.
V` detA = 2 ,= 0 nen A co ma tra
.
n nghi
.
ch da o v`a do va
.
y he
.
(4.5) c o
nghie
.
m duy nhat:
X = A
1
H.
De

d`ang thay ra
`
ng
A
1
=
_

_
3 3 1

5
2
4
3
2
1
2
1
1
2
_

_
v`a do do
_

_
x
1
x
2
x
3
_

_
=
_

_
3 3 1

5
2
4
3
2
1
2
1
1
2
_

_
_

_
4
4
2
_

_
.
136 Chu
.
o
.
ng 4. He
.
phu
.
o
.
ng tr`nh tuye

n tnh
Thu
.
.
c hie
.
n phep nhan ma tra
.
n o
.
ve

pha i ta thu du
.
o
.
.
c
x
1
= 3 4 3 4 + 1 2 = 2,
x
2
=
5
2
4 + 4 4
3
2
2 = 3,
x
3
=
1
2
4 1 4 +
1
2
2 = 1.
2) Vie

t ma tra
.
n A cu a he
.
v`a t`m A
1
:
A =
_

_
3 2 1
1 1 2
2 2 5
_

_ A
1
=
_

_
1 12 5
1 17 7
0 2 1
_

_.
T` u
.
do suy ra
`
ng
_

_
x
1
x
2
x
3
_

_ =
_

_
1 12 5
1 17 7
0 2 1
_

_
_

_
1
2
3
_

_ =
_

_
8
12
1
_

_
t u
.
c l`a
x
1
= 8, x
2
= 12, x
3
= 1.
V du
.
2.

Ap du
.
ng quy ta

c Cramer, gia i cac he


.
phu
.
o
.
ng tr`nh
1)
x
1
+ 2x
2
+ 3x
3
= 6,
2x
1
x
2
+x
3
= 2,
3x
1
x
2
2x
3
= 2.
_

_
(4.7)
2)
x
1
2x
2
+ 3x
3
x
4
= 6,
2x
1
+ 3x
2
4x
3
+ 4x
4
= 7,
3x
1
+x
2
2x
3
2x
4
= 9,
x
1
3x
2
+ 7x
3
+ 6x
4
= 7.
_

_
(4.8)
Gia i. 1)

Ap du
.
ng cong th u
.
c (4.4)
x
j
=
det(A
j
)
detA
, j = 1, 3
4.1. He
.
n phu
.
o
.
ng tr`nh vo
.
i n a

n co di
.
nh th u
.
c khac 0 137
trong do
detA =

1 2 3
3 1 1
3 1 2

= 30 ,= 0; detA
1
=

6 2 3
2 1 1
2 1 2

= 30;
detA
2
=

1 6 3
2 2 1
3 2 2

= 30; detA
3
=

1 2 6
2 1 2
3 1 2

= 30.
T` u
.
do suy ra
x
1
= 1, x
2
= 1, x
3
= 1.
2) Tnh di
.
nh th u
.
c cu a he
.
:
detA =

1 2 3 1
2 3 4 4
3 1 2 2
1 3 7 6

= 35.
V` detA ,= 0 nen he
.
co nghie
.
m duy nhat v`a nghie
.
m du
.
o
.
.
c t`m theo
cong th u
.
c (4.4). Ta tnh cac di
.
nh th u
.
c
det(A
1
) =

6 2 3 1
7 3 4 4
9 1 2 2
7 3 7 6

= 70,
138 Chu
.
o
.
ng 4. He
.
phu
.
o
.
ng tr`nh tuye

n tnh
det(A
2
) =

1 6 3 1
2 7 4 4
3 9 2 2
1 7 7 6

= 35,
det(A
3
) =

1 2 6 1
2 3 7 4
3 1 9 2
1 3 7 6

= 0,
det(A
4
) =

1 2 3 6
2 3 4 7
3 1 2 9
1 3 7 7

= 70.
Do do
x
1
=
det(A
1
)
detA
= 2, x
2
=
det(A
2
)
detA
= 1,
x
3
=
det(A
3
)
detA
= 0, x
4
=
det(A
4
)
detA
= 2.
V du
.
3.

Ap du
.
ng phu
.
o
.
ng phap Gauss gia i cac he
.
phu
.
o
.
ng tr`nh
1)
x
1
2x
3
= 3,
2x
1
+x
2
+ 6x
3
= 11,
x
1
+ 5x
2
4x
3
= 4.
2)
2x
1
x
2
+ 3x
3
x
4
= 9,
x
1
+x
2
2x
3
+ 4x
4
= 1,
3x
1
+ 2x
2
x
3
+ 3x
4
= 0,
5x
1
2x
2
+x
3
2x
4
= 9.
4.1. He
.
n phu
.
o
.
ng tr`nh vo
.
i n a

n co di
.
nh th u
.
c khac 0 139
Gia i. 1) La
.
p ma tra
.
n mo
.
ro
.
ng v`a thu
.
.
c hie
.
n cac phep bie

n do

i:

A =
_

_
1 0 2

3
2 1 6

11
1 5 4

4
_

_h
2
+ 2h
1
h

2
h
3
+h
1
h

_
1 0 2

3
0 1 2

5
0 5 6

7
_

h
3
5h
2
h

3
_

_
1 0 2

3
0 1 2

5
0 0 16

32
_

_.
T` u
.
do suy ra
x
1
2x
3
= 3
x
2
+ 2x
3
= 5
16x
3
= 32
_

_
x
1
= 1, x
2
= 1, x
3
= 2.
2) La
.
p ma tra
.
n mo
.
ro
.
ng v`a thu
.
.
c hie
.
n cac phep bie

n do

i so
.
cap:
_

_
2 1 3 1

9
1 1 2 4

1
3 2 1 3

0
5 2 1 2

9
_

_
h
1
h

2
h
2
h

_
1 1 2 4

1
2 1 3 1

9
3 2 1 3

0
5 2 1 2

9
_

h
2
2h
1
h

2
h
3
3h
1
h

3
h
4
5h
1
h

4
_

_
1 1 2 4

1
0 3 7 9

11
0 1 5 9

3
0 7 11 22

14
_

_
h
2
h

3
h
3
h

140 Chu
.
o
.
ng 4. He
.
phu
.
o
.
ng tr`nh tuye

n tnh

_
1 1 2 4

1
0 1 5 9

3
0 3 7 9

11
0 7 11 22

14
_

_h
3
3h
2
h

3
h
4
7h
2
h

_
1 1 2 4

1
0 1 5 9

3
0 0 8 18

2
0 0 24 41

7
_

h
4
3h
3
h

4
_

_
1 1 2 4

1
0 1 5 9

3
0 0 8 18

2
0 0 0 13

13
_

_
T` u
.
do suy ra ra
`
ng x
1
= 1, x
2
= 2, x
3
= 2, x
4
= 1.
B
`
AI T

A
.
P
Gia i cac he
.
phu
.
o
.
ng tr`nh tuye

n tnh sau
1.
x
1
x
2
+ 2x
3
= 11,
x
1
+ 2x
2
x
3
= 11,
4x
1
3x
2
3x
3
= 24.
_

_
. (DS. x
1
= 9, x
2
= 2, x
3
= 2)
2.
x
1
3x
2
4x
3
= 4,
2x
1
+x
2
3x
3
= 1,
3x
1
2x
2
+x
3
= 11.
_

_
. (DS. x
1
= 2, x
2
= 2, x
3
= 1)
3.
2x
1
+ 3x
2
x
3
= 4,
x
1
+ 2x
2
+ 2x
3
= 5,
3x
1
+ 4x
2
5x
3
= 2.
_

_
. (DS. x
1
= x
2
= x
3
= 1)
4.1. He
.
n phu
.
o
.
ng tr`nh vo
.
i n a

n co di
.
nh th u
.
c khac 0 141
4.
x
1
+ 2x
2
+x
3
= 8,
2x
1
+ 3x
2
3x
3
= 5,
3x
1
4x
2
+ 5x
3
= 10.
_

_
. (DS. x
1
= 1, x
2
= 2, x
3
= 3)
5.
2x
1
+x
2
x
3
= 0,
3x
2
+ 4x
3
= 6,
x
1
+x
3
= 1.
_

_
. (DS. x
1
= 1, x
2
= 2, x
3
= 0)
6.
2x
1
3x
2
x
3
+ 6 = 0,
3x
1
+ 4x
2
+ 3x
3
+ 5 = 0,
x
1
+x
2
+x
3
+ 2 = 0.
_

_
. (DS. x
1
= 2, x
2
= 1, x
3
= 1)
7.
x
2
+ 3x
3
+ 6 = 0,
x
1
2x
2
x
3
= 5,
3x
1
+ 4x
2
2x = 13.
_

_
. (DS. x
1
= 3, x
2
= 0, x
3
= 2)
8.
2x
1
x
2
+x
3
+ 2x
4
= 5,
x
1
+ 3x
2
x
3
+ 5x
4
= 4,
5x
1
+ 4x
2
+ 3x
3
= 2,
3x
1
3x
2
x
3
6x
4
= 6.
_

_
.
(DS. x
1
=
1
3
, x
2
=
2
3
, x
3
= 1, x
4
=
4
3
)
9.
x
1
2x
2
+ 3x
3
x
4
= 8,
2x
1
+ 3x
2
x
3
+ 5x
4
= 19,
4x
1
x
2
+x
3
+x
4
= 1,
3x
1
+ 2x
2
x
3
2x
4
= 2.
_

_
.
(DS. x
1
=
1
2
, x
2
=
3
2
, x
3
=
1
2
, x
4
= 3)
10.
x
1
x
3
+x
4
= 3,
2x
1
+ 3x
2
x
3
x
4
= 2,
5x
1
3x
4
= 6
x
1
+x
2
+x
3
+x
4
= 2.
_

_
.
(DS. x
1
= 0, x
2
= 1, x
3
= 1, x
4
= 2)
142 Chu
.
o
.
ng 4. He
.
phu
.
o
.
ng tr`nh tuye

n tnh
11.
2x
1
+ 3x
2
+ 8x
4
= 0,
x
2
x
3
+ 3x
4
= 0,
x
3
+ 2x
4
= 1,
x
1
+x
4
= 24
_

_
.
(DS. x
1
= 19, x
2
= 26, x
3
= 11, x
4
= 5)
12.
3x
1
+x
2
x
3
+x
4
= 0,
2x
1
+ 3x
2
x
4
= 0,
x
1
+ 5x
2
3x
3
= 7,
3x
2
+ 2x
3
+x
4
= 2,
_

_
.
(DS. x
1
= 1, x
2
= 1, x
3
= 1, x
4
= 1)
13.
x
1
2x
2
+x
3
4x
4
x
5
= 13,
x
1
+ 2x
2
+ 3x
3
5x
4
= 15,
x
2
2x
3
+x
4
+ 3x
5
= 7,
x
1
7x
3
+ 8x
4
x
5
= 30,
3x
1
x
2
5x
5
= 4.
_

_
.
(DS. x
1
= 1, x
2
= 1, x
3
= 2, x
4
= 2, x
5
= 0)
14.
x
1
+x
2
+ 4x
3
+x
4
x
5
= 2,
x
1
2x
2
2x
3
+ 3x
5
= 0,
4x
2
+ 3x
3
2x
4
+ 2x
5
= 2,
2x
1
x
3
+ 3x
4
2x
5
= 2,
3x
1
+ 2x
2
5x
4
+ 3x
5
= 3.
_

_
.
(DS. x
1
=
2
5
, x
2
=
3
5
, x
3
=
4
5
, x
4
= 0, x
5
= 0)
4.2. He
.
t` uy y cac phu
.
o
.
ng tr`nh tuye

n tnh 143
4.2 He
.
t` uy y cac phu
.
o
.
ng tr`nh tuye

n tnh
Ta xet he
.
t` uy y cac phu
.
o
.
ng tr`nh tuye

n tnh g o
`
m m phu
.
o
.
ng tr`nh vo
.
i
n a

n
a
11
x
1
+a
12
x
2
+ +a
1n
x
n
= b
1
,
a
21
x
1
+a
22
x
2
+ +a
2n
x
n
= b
2
,
. . . . . . . . . . . . . . .
a
m1
x
1
+a
m2
x
2
+ +a
mn
x
n
= b
m
,
_

_
(4.9)
vo
.
i ma tra
.
n co
.
ba n
A =
_

_
a
11
a
12
. . . a
1n
. . . . . . . . . . . .
a
m1
a
m2
. . . a
mn
_

_
v`a ma tra
.
n mo
.
ro
.
ng

A =
_

_
a
11
a
12
. . . a
1n

b
1
. . . . . . . . . . . .

. . .
a
m1
a
m2
. . . a
mn

b
m
_

_
Hie

n nhien ra
`
ng r(A) r(

A) v` mo

i di
.
nh th u
.
c con cu a A de
`
u l`a di
.
nh
th u
.
c con cu a

A nhu
.
ng khong c o die
`
u ngu
.
o
.
.
c la
.
i. Ta luon luon gia thie

t
ra
`
ng cac pha
`
n tu
.
cu a ma tra
.
n A khong do
`
ng th`o
.
i ba
`
ng 0 tat ca .
Ngu
.
`o
.
i ta quy u
.
o
.
c go
.
i di
.
nh th u
.
c con khac 0 cu a mo
.
t ma tra
.
n m`a
cap cu a no ba
`
ng ha
.
ng cu a ma tra
.
n do l`a di
.
nh th u
.
c con co
.
so
.
cu a no.
Gia su
.
doi vo
.
i ma tra
.
n da cho ta da cho
.
n mo
.
t di
.
nh th u
.
c con co
.
so
.
.
Khi do cac h`ang v`a cac c o
.
t m`a giao cu a ch ung la
.
p th`anh di
.
nh th u
.
c
con co
.
so
.
do du
.
o
.
.
c go
.
i l`a h`ang, co
.
t co
.
so
.
.
D
-
i
.
nh ngha. 1
+
Bo
.
co th u
.
tu
.
.
n so (
1
,
2
, . . . ,
n
) du
.
o
.
.
c go
.
i l`a nghie
.
m
cu a he
.
(4.9) ne

u khi thay x =
1
, x =
2
, . . . , x =
n
v`ao c ac phu
.
o
.
ng
tr`nh cu a (4.9) th` hai ve

cu a mo

i phu
.
o
.
ng tr`nh cu a (4.9) tro
.
th`anh
do
`
ng nhat.
144 Chu
.
o
.
ng 4. He
.
phu
.
o
.
ng tr`nh tuye

n tnh
2+ He
.
(4.9) du
.
o
.
.
c go
.
i l`a tu
.
o
.
ng thch ne

u co t nhat mo
.
t nghie
.
m v`a
go
.
i l`a khong tu
.
o
.
ng thch ne

u no vo nghie
.
m.
3
+
He
.
tu
.
o
.
ng thch du
.
o
.
.
c go
.
i l`a he
.
xac di
.
nh ne

u no co nghie
.
m duy
nhat v`a go
.
i l`a he
.
vo di
.
nh ne

u no co nhie
`
u ho
.
n mo
.
t nghie
.
m.
D
-
i
.
nh l y Kronecker-Capelli.
2
He
.
phu
.
o
.
ng tr`nh tuye

n tnh (4.9)
tu
.
o
.
ng thch khi v` a ch khi ha
.
ng cu a ma tr a
.
n co
.
ba n b a
`
ng ha
.
ng cu a
ma tra
.
n mo
.
ro
.
ng cu a he
.
, t u
.
c l`a r(A) = r(

A).
Doi vo
.
i he
.
tu
.
o
.
ng thch ngu
.
`o
.
i ta go
.
i cac a

n m`a he
.
so cu a ch ung la
.
p
nen di
.
nh th u
.
c con co
.
so
.
cu a ma tra
.
n co
.
ba n l`a a

n co
.
so
.
, cac a

n c`on
la
.
i du
.
o
.
.
c go
.
i l`a a

n tu
.
.
do.
Phu
.
o
.
ng phap chu ye

u de

gia i he
.
to

ng quat l`a:
1.

Ap du
.
ng quy ta

c Kronecker-Capelli.
2. Phu
.
o
.
ng phap khu
.
da
`
n cac a

n (phu
.
o
.
ng phap Gauss).
Quy ta

c Kronecker-Capelli g o
`
m cac bu
.
o
.
c sau.
1
+
Kha o sat tnh tu
.
o
.
ng thch cu a he
.
. Tnh ha
.
ng r(

A) v`a r(A)
a) Ne

u r(

A) > r(A) th` he


.
khong tu
.
o
.
ng thch.
b) Ne

u r(

A) = r(A) = r th` he
.
tu
.
o
.
ng thch. T`m di
.
nh th u
.
c con
co
.
so
.
cap r n`ao do (v`a do va
.
y r a

n co
.
so
.
tu
.
o
.
ng u
.
ng xem nhu
.
du
.
o
.
.
c
cho
.
n) v`a thu du
.
o
.
.
c he
.
phu
.
o
.
ng tr`nh tu
.
o
.
ng du
.
o
.
ng go
`
m r phu
.
o
.
ng tr`nh
vo
.
i n a

n m`a (r n)-ma tra


.
n he
.
so cu a no ch u
.
a cac pha
`
n tu
.
cu a di
.
nh
th u
.
c con co
.
so
.
da cho
.
n. C ac phu
.
o
.
ng tr`nh c`on la
.
i co the

bo qua.
2
+
T`m nghie
.
m cu a he
.
tu
.
o
.
ng du
.
o
.
ng thu du
.
o
.
.
c
a) Ne

u r = n, ngha l`a so a

n co
.
so
.
ba
`
ng so a

n cu a he
.
th` he
.
co
nghie
.
m duy nhat v`a c o the

t`m theo c ong th u


.
c Cramer.
b) Ne

u r < n, ngha l`a so a

n co
.
so
.
be ho
.
n so a

n cu a he
.
th` ta
chuye

n n r so ha
.
ng co ch u
.
a a

n tu
.
.
do cu a cac phu
.
o
.
ng tr`nh sang
ve

pha i de

thu du
.
o
.
.
c he
.
Cramer doi vo
.
i cac a

n co
.
so
.
. Gia i he
.
n`ay ta
thu du
.
o
.
.
c c ac bie

u th u
.
c cu a cac a

n co
.
so
.
bie

u die

n qua cac a

n tu
.
.
do.
2
L. Kronecker (1823-1891) l`a nh`a toan ho
.
c D u
.
c,
A. Capelli (1855-1910) l`a nh`a toan ho
.
c Italia.
4.2. He
.
t` uy y cac phu
.
o
.
ng tr`nh tuye

n tnh 145
Do l`a nghie
.
m to

ng quat cu a he
.
. Cho n r a

n tu
.
.
do nh u
.
ng gia tri
.
cu
.
the

t` uy y ta t`m du
.
o
.
.
c c ac gia tri
.
tu
.
o
.
ng u
.
ng cu a a

n co
.
so
.
. T` u
.
do thu
du
.
o
.
.
c nghie
.
m rieng cu a he
.
.
Tie

p theo ta tr`nh b`ay no


.
i dung cu a phu
.
o
.
ng phap Gauss.
Khong gia m to

ng quat, co the

cho ra
`
ng a
11
,= 0. No
.
i dung cu a
phu
.
o
.
ng phap Gauss l`a nhu
.
sau.
1
+
Thu
.
.
c hie
.
n cac phep bie

n do

i so
.
cap tren cac phu
.
o
.
ng tr`nh cu a
he
.
de

thu du
.
o
.
.
c he
.
tu
.
o
.
ng du
.
o
.
ng m`a ba

t da
`
u t` u
.
phu
.
o
.
ng tr`nh th u
.
hai
mo
.
i phu
.
o
.
ng tr`nh de
`
u khong ch u
.
a a

n x
1
. K y hie
.
u he
.
n`ay l`a S
(1)
.
2
+
C ung khong mat to

ng quat, c o the

cho ra
`
ng a

22
,= 0. La
.
i thu
.
.
c
hie
.
n cac phep bie

n do

i so
.
cap tren cac phu
.
o
.
ng tr`nh cu a he
.
S
(1)
(tr` u
.
ra phu
.
o
.
ng tr`nh th u
.
nhat du
.
o
.
.
c gi u
.
nguyen!) nhu
.
da l`am trong bu
.
o
.
c
1
+
ta thu du
.
o
.
.
c he
.
tu
.
o
.
ng du
.
o
.
ng m`a ba

t da
`
u t` u
.
phu
.
o
.
ng tr`nh th u
.
ba
mo
.
i phu
.
o
.
ng tr`nh de
`
u khong ch u
.
a a

n x
2
,...
3
+
Sau mo
.
t so bu
.
o
.
c ta co the

ga
.
p mo
.
t trong cac tru
.
`o
.
ng ho
.
.
p sau
day.
a) Thay ngay du
.
o
.
.
c he
.
khong tu
.
o
.
ng thch.
b) Thu du
.
o
.
.
c mo
.
t he
.
tam gi ac. He
.
n`ay co nghie
.
m duy nhat.
c) Thu du
.
o
.
.
c mo
.
t he
.
h`nh thang da
.
ng
a
11
x
1
+a
12
x
2
+ . . . +a
1n
x
n
= h
1
,
b
22
x
2
+ . . . +b
2n
x
n
= h
2
,
. . . . . . . . . . . .
b
rr
x
r
+ +b
rn
x
n
= h
r
,
0 = h
r+1
,
. . . . . .
0 = h
m
.
_

_
Ne

u cac so
h
r+1
, . . . , h
m
khac 0 th` he
.
vo nghie
.
m. Ne

u h
r+1
=
= h
m
= 0 th` he
.
co nghie
.
m. Cho x
r+1
= , . . . , x
m
= th`
thu du
.
o
.
.
c he
.
Cramer vo
.
i a

n l` a x
1
, . . . , x
r
. Gia i he
.
do ta thu du
.
o
.
.
c
146 Chu
.
o
.
ng 4. He
.
phu
.
o
.
ng tr`nh tuye

n tnh
nghie
.
m x
1
= x
1
; x
2
= x
2
, . . . , x
r
= x
r
v`a nghie
.
m cu a he
.
da cho l`a
(x
1
, x
2
, . . . , x
r
, , . . . , ).
Lu
.
u y ra
`
ng vie
.
c gia i he
.
phu
.
o
.
ng tr`nh tuye

n tnh ba
`
ng phu
.
o
.
ng
phap Gauss thu
.
.
c chat l` a thu
.
.
c hie
.
n cac phep bie

n do

i so
.
cap tren cac
h`ang cu a ma tra
.
n mo
.
r o
.
ng cu a he
.
du
.
a no ve
`
da
.
ng tam giac hay da
.
ng
h`nh thang.
C

AC V

I DU
.
V du
.
1. Gia i he
.
phu
.
o
.
ng tr`nh
3x
1
x
2
+x
3
= 6,
x
1
5x
2
+x
3
= 12,
2x
1
+ 4x
2
= 6,
2x
1
+x
2
+ 3x
3
= 3,
5x
1
+ 4x
3
= 9.
_

_
Gia i. 1. T`m ha
.
ng cu a cac ma tra
.
n
A =
_

_
3 1 1
1 5 1
2 4 0
2 1 3
5 0 4
_

_
,

A =
_

_
3 1 1

6
1 5 1

12
2 4 0

6
2 1 3

3
5 0 4

9
_

_
Ta thu du
.
o
.
.
c r(

A) = r(A) = 3. Do do he
.
tu
.
o
.
ng thch.
Ta cho
.
n di
.
nh th u
.
c con co
.
so
.
l`a
=

1 5 1
2 4 0
2 1 3

v` = 36 ,= 0 v`a r(A) = 3 v`a cac a

n co
.
so
.
l`a x
1
, x
2
, x
3
.
4.2. He
.
t` uy y cac phu
.
o
.
ng tr`nh tuye

n tnh 147
2. He
.
phu
.
o
.
ng tr`nh da cho tu
.
o
.
ng du
.
o
.
ng vo
.
i he
.
x
1
5x
2
+x
3
= 12,
2x
1
+ 4x
2
= 6,
2x
1
+x
2
+ 3x
3
= 3.
_

_
So a

n co
.
so
.
ba
`
ng so a

n cu a he
.
nen he
.
co nghie
.
m duy nhat l`a x
1
= 1,
x
2
= 2, x
4
= 1.
V du
.
2. Gia i he
.
phu
.
o
.
ng tr`nh
x
1
+ 2x
2
3x
3
+ 4x
4
= 7,
2x
1
+ 4x
2
+ 5x
3
x
4
= 2,
5x
1
+ 10x
2
+ 7x
3
+ 2x
4
= 11.
_

_
Gia i. T`m ha
.
ng cu a cac ma tra
.
n
A =
_

_
1 2 3 4
2 4 5 1
5 10 7 2
_

_
,

A =
_

_
1 2 3 4

7
2 4 5 1

2
5 10 7 2

11
_

_
Ta thu du
.
o
.
.
c r(

A) = r(A) = 2. Do do he
.
tu
.
o
.
ng thch.
Ta co the

lay di
.
nh th u
.
c con co
.
so
.
l`a
=

2 3
4 5

v` = 22 ,= 0 v` a c ap cu a di
.
nh th u
.
c = r(A) = 2. Khi cho
.
n l`am
di
.
nh th u
.
c con, ta co x
2
v`a x
3
l`a a

n co
.
so
.
.
He
.
da cho tu
.
o
.
ng du
.
o
.
ng vo
.
i he
.
x
1
+ 2x
2
3x
3
+ 4x
4
= 7,
2x
1
+ 4x
2
+ 5x
3
x
4
= 2
hay
2x
2
3x
3
= 7 x
1
4x
4
,
4x
2
+ 5x
3
= 2 2x
1
+x
4
.
148 Chu
.
o
.
ng 4. He
.
phu
.
o
.
ng tr`nh tuye

n tnh
2. Ta co the

gia i he
.
theo quy ta

c Cramer. Da
.
t x
1
= , x
4
= ta
co
2x
2
3x
3
= 7 4,
4x
2
+ 5x
3
= 2 2 +.
Theo cong th u
.
c Cramer ta t`m du
.
o
.
.
c
x
2
=

7 4 3
2 2 + 5

22
=
41 11 17
22
,
x
3
=

2 7 4
4 2 2 +

22
=
24 + 18
22

Do do ta
.
p ho
.
.
p cac nghie
.
m cu a he
.
co da
.
ng
_
;
41 11 17
22
;
9 12
11
;

, R
_

V du
.
3. Ba
`
ng phu
.
o
.
ng phap Gauss hay gia i he
.
phu
.
o
.
ng tr`nh
4x
1
+ 2x
2
+x
3
= 7,
x
1
x
2
+x
3
= 2,
2x
1
+ 3x
2
3x
3
= 11,
4x
1
+x
2
x
3
= 7.
_

_
Gia i. Trong he
.
da cho ta co a
11
= 4 ,= 0 nen de

cho tie
.
n ta do

i cho

hai phu
.
o
.
ng tr`nh da
`
u v`a thu du
.
o
.
.
c he
.
tu
.
o
.
ng du
.
o
.
ng
x
1
x
2
+x
3
= 2,
4x
1
+ 2x
2
+x
3
= 7,
2x
1
+ 3x
2
3x
3
= 11,
4x
1
+x
2
x
3
= 7.
_

_
4.2. He
.
t` uy y cac phu
.
o
.
ng tr`nh tuye

n tnh 149
Tie

p theo ta bie

n do

i ma tra
.
n mo
.
ro
.
ng

A =
_

_
1 1 1

2
4 2 1

7
2 3 3

11
4 1 1

7
_

_
h
2
4h
1
h

2
h
3
2h
1
h

3
h
4
4h
1
h

_
1 1 1

2
0 6 3

15
0 5 5

15
0 5 5

15
_

_
h
4
h
3
h

_
1 1 1

2
0 6 3

15
0 5 5

15
0 0 0

0
_

_
h
2
5 h

2
h
3
6 h

_
1 1 1

2
0 30 15

75
0 30 30

90
0 0 0

0
_

h
3
h
2
h

3
_

_
1 1 1

2
0 30 15

75
0 0 15

15
0 0 0

0
_

_
.
T` u
.
do thu du
.
o
.
.
c he
.
tu
.
o
.
ng du
.
o
.
ng
x
1
x
2
+x
3
= 2
30x
2
15x
3
= 75
15x
3
= 15
_

_
v`a do do thu du
.
o
.
.
c nghie
.
m x
1
= 1, x
2
= 2, x
3
= 1.
V du
.
4. Gia i he
.
phu
.
o
.
ng tr`nh
x
1
+x
2
+x
3
+x
4
+x
5
= 1,
2x
1
+ 2x
2
+ 3x
4
+x
5
= 1,
2x
3
+ 2x
4
x
5
= 1,
2x
3
+ 4x
4
3x
5
= 7,
6x
3
+ 3x
4
x
5
= 1.
_

_
150 Chu
.
o
.
ng 4. He
.
phu
.
o
.
ng tr`nh tuye

n tnh
Gia i. 1) B a
`
ng cac phep bie

n do

i so
.
cap (ch thu
.
.
c hie
.
n tren c ac
h`ang !) ma tra
.
n mo
.
ro
.
ng

A du
.
o
.
.
c du
.
a ve
`
ma tra
.
n ba
.
c thang
A
_

_
1 1 1 1 1

1
0 0 2 1 1

3
0 0 0 3 2

4
0 0 0 0 0

0
0 0 0 0 0

0
_

_
.
2) Ma tra
.
n n`ay tu
.
o
.
ng u
.
ng vo
.
i he
.
phu
.
o
.
ng tr`nh
x
1
+x
2
+x
3
+x
4
+x
5
= 1,
2x
3
+x
4
x
5
= 3,
3x
4
2x
5
= 4.
_

_
he
.
n`ay tu
.
o
.
ng du
.
o
.
ng vo
.
i he
.
da cho v`a co x
1
, x
3
, x
4
l`a a

n co
.
so
.
, c` on
x
2
, x
5
l`a a

n tu
.
.
do.
3) Chuye

n cac so ha
.
ng ch u
.
a a

n tu
.
.
do sang ve

pha i ta c o
x
1
+x
3
+x
4
= 1 x
2
x
5
,
2x
3
+x
4
= 3 +x
5
,
3x
4
= 4 + 2x
5
.
_

_
4) Gia i he
.
n`ay (t` u
.
du
.
o
.
i len) ta thu du
.
o
.
.
c nghie
.
m to

ng quat
x
1
=
3 3x
2
x
5
2
,
x
3
=
5 x
5
6
, x
4
=
4 + 2x
5
3

V du
.
5. Gia i he
.
phu
.
o
.
ng tr`nh
x
1
+ 3x
2
+ 5x
3
+ 7x
4
+ 9x
5
= 1,
x
1
2x
2
+ 3x
3
4x
4
+ 5x
5
= 2,
2x
1
+ 11x
2
+ 12x
3
+ 25x
4
+ 22x
5
= 4.
_

_
4.2. He
.
t` uy y cac phu
.
o
.
ng tr`nh tuye

n tnh 151
Gia i. Ta thu
.
.
c hie
.
n cac phep bie

n do

i so
.
cap tren c ac h`ang cu a ma
tra
.
n mo
.
ro
.
ng:

A =
_

_
1 3 5 7 9

1
1 2 3 4 5

2
2 11 12 25 22

4
_

_ h
2
h
1
h

2
h
3
2h
1
h

_
1 3 5 7 9

1
0 5 2 11 4

1
0 5 2 11 4

2
_

_
h
3
+h
2
h

_
1 3 5 7 9

1
0 5 2 11 4

1
0 0 0 0 0

3
_

_
T` u
.
do suy ra
`
ng r(

A) = 3; r(A) = 2 v`a do va
.
y r(

A) > r(A) v`a he


.
da cho khong tu
.
o
.
ng thch. .
V du
.
6. Gia i v`a bie
.
n lua
.
n he
.
phu
.
o
.
ng tr`nh theo tham so :
x
1
+x
2
+x
3
= 1,
x
1
+x
2
+x
3
= 1,
z
1
+x
2
+x
3
= 1.
_

_
Gia i. Ta co
A =
_

_
1 1
1 1
1 1
_

_ detA = ( + 2)( 1)
2
= D,
tie

p theo de

d`ang thu du
.
o
.
.
c
D
x
1
= D
x
2
= D
x
3
= ( 1)
2
.
1
+
Ne

u D ,= 0, t u
.
c l`a ne

u ( +2)( 1)
2
,= 0 ,= 2 v`a ,= 1
th` he
.
da cho co nghie
.
m duy nhat v` a theo cac c ong th u
.
c Cramer ta co
x
1
= x
2
= x
3
=
1
+ 2

152 Chu
.
o
.
ng 4. He
.
phu
.
o
.
ng tr`nh tuye

n tnh
2
+
Ne

u = 2 th` D = 0 v`a ta co
A =
_

_
2 1 1
1 2 1
1 1 2
_

_
r(A) = 2
_

2 1
1 2

,= 0
_
,

A =
_

_
2 1 1

1
1 2 1

1
1 1 2

1
_

_
.
Ba
`
ng cach thu
.
.
c hie
.
n cac phep bie

n do

i so
.
cap tren cac ma tra
.
n

A ta
thu du
.
o
.
.
c r(

A) = 3.
Do do vo
.
i = 2 th` r(

A) > r(A) v`a he


.
vo nghie
.
m.
3
+
Ne

u = 1 th` detA = 0 v`a de

thay ra
`
ng r(

A) = r(A) = 1 < 3
(so a

n cu a he
.
l`a 3). T` u
.
do suy ra he
.
co vo so nghie
.
m phu
.
thuo
.
c hai
tham so: x
1
+x
2
+x
3
= 1.
V du
.
7. Gia i v`a bie
.
n lua
.
n he
.
phu
.
o
.
ng tr`nh theo tham so
x
1
+x
2
+x
3
= 1,
x
1
+x
2
+x
3
= ,
x
1
+x
2
+x
3
=
2
.
_

_
Gia i. Di
.
nh th u
.
c cu a he
.
ba
`
ng
D =

1 1
1 1
1 1

= ( 1)
2
( + 2).
Ne

u D ,= 0
1
,= 1,
2
,= 2 th` he
.
co nghie
.
m duy nhat. Ta tnh
4.2. He
.
t` uy y cac phu
.
o
.
ng tr`nh tuye

n tnh 153
D
x
1
, D
x
2
, D
x
3
:
D
x
1
=

1 1 1
1

2
1

= ( 1)
2
( + 1),
D
x
2
=

1 1
1 1
1
2

= ( 1)
2
,
D
x
3
=

1 1
1
1 1
2

= ( 1)
2
( + 1)
2
.
T` u
.
do theo cong th u
.
c Cramer ta thu du
.
o
.
.
c
x
1
=
+ 1
+ 2
, x
2
=
1
+ 2
, x
3
=
( + 1)
2
+ 2

Ta c`on xet gia tri
.
= 1 v`a = 2.
Khi = 1 he
.
da cho tro
.
th`anh
x
1
+x
2
+x
3
= 1,
x
1
+x
2
+x
3
= 1,
x
1
+x
2
+x
3
= 1.
_

_
He
.
n`ay co vo so nghie
.
m phu
.
thuo
.
c hai tham so. Ne

u da
.
t x
2
= ,
x
3
= th`
x
1
=1 ,
, R,
v`a nhu
.
va
.
y ta
.
p ho
.
.
p nghie
.
m co the

vie

t du
.
o
.
i da
.
ng (1
; ; ; , R).
Khi = 2 th` he
.
da cho tro
.
th`anh
2x
1
+x
2
+x
2
= 2,
x
1
2x
2
+x
3
= 2,
x
1
+x
2
2x
3
= 4.
_

_
154 Chu
.
o
.
ng 4. He
.
phu
.
o
.
ng tr`nh tuye

n tnh
Ba
`
ng cach co
.
ng ba phu
.
o
.
ng tr`nh la
.
i vo
.
i nhau ta thay ngay he
.
da cho
vo nghie
.
m.
V du
.
8. Xet he
.
phu
.
o
.
ng tr`nh
x
1
+ 2x
2
+x
3
= 3,
3x
1
x
2
x
3
= 2,
2x
1
+x
2
+ 3x
3
= .
_

_
Vo
.
i gia tri
.
n`ao cu a cac tham so v`a th`
1) he
.
co nghie
.
m duy nhat ?
2) he
.
vo nghie
.
m ?
3) he
.
co vo so nghie
.
m ?
Gia i. Ta vie

t c ac ma tra
.
n
A =
_

_
1 2
3 1
2 1 3
_

_;

A =
_

_
1 2

3
3 1

2
2 1 3

_
Ta co
D = detA =

1 2
3 1
2 1 3

= 2 21.
T` u
.
do
1
+
He
.
da cho c o nghie
.
m duy nhat khi v`a ch khi
detA ,= 0 ,=
21
2
, t` uy y.
2
+
De

he
.
vo nghie
.
m da
`
u tien no pha i thoa man
detA = 0 =
21
2

Khi =
21
2
th` detA = 0 v`a do va
.
y
r(A) < 3.
4.2. He
.
t` uy y cac phu
.
o
.
ng tr`nh tuye

n tnh 155
V` di
.
nh th u
.
c

1 2
3 1

= 7 ,= 0 nen:
r(A) = 2 khi =
21
2

Theo di
.
nh l y Kronecker-Capelli he
.
da cho vo nghie
.
m khi v`a ch khi
r(

A) > r(A) = 2.
Ta t`m die
`
u kie
.
n de

he
.
th u
.
c n`ay thoa man. Cu
.
the

l`a t`m r(

A) khi
=
21
2
. Ta co

A =
_

_
1 2
21
2

3
3 1
21
2

2
2 1 3

_
h
1
2 h

1
h
2
2 h

_
2 4 21

6
6 2 21

4
2 1 3

_h
2
3h
1
h

2
h
3
h
1
h

_
2 4 21

6
0 14 84

14
0 3 18

6
_

_h
2

_
1
14
_
h

_
2 4 21

6
0 1 6

1
0 3 18

6
_

_
h
3
+ 3h
1
h

_
2 4 21

6
0 1 6

1
0 0 0

3
_

_
T` u
.
ke

t qua bie

n do

i ta thu du
.
o
.
.
c
r(

A) =
_
_
_
2 ne

u = 3,
3 ne

u ,= 3,
156 Chu
.
o
.
ng 4. He
.
phu
.
o
.
ng tr`nh tuye

n tnh
V` r(A) = 2 nen he
.
da cho vo nghie
.
m ne

u
=
21
2
v`a ,= 3.
3
+
He
.
da cho c o vo so nghie
.
m khi v`a ch khi
r(

A) = r(A) = r < 3
t u
.
c l`a khi ha
.
ng cu a A v`a

A ba
`
ng nhau nhu
.
ng be ho
.
n so a

n cu a he
.
l`a
3. T` u
.
la
.
p lua
.
n tren suy ra
`
ng he
.
co vo so nghie
.
m ne

u
r(

A) = r(A) = 2
_
_
_
=
21
2
,
= 3.
Khi do he
.
da cho tu
.
o
.
ng du
.
o
.
ng vo
.
i he
.
2x
1
+ 4x
2
= 6 21,
6x
1
2x
2
= 4 + 21.
_
= x
3
,
v`a nghie
.
m cu a no l`a
_
1 +
3
2
, 1 6,

R
_
.
B
`
AI T

A
.
P
Gia i cac he
.
phu
.
o
.
ng tr`nh tuye

n tnh
1.
6x
1
+ 3x
2
+ 4x
3
= 3;
3x
1
x
2
+ 2x
3
= 5.
_
(DS. x
2
=
7
5
, x
3
=
18 15x
1
10
, x
1
t` uy y)
2.
x
1
x
2
+x
3
= 1,
2x
1
+x
2
x
3
= 5.
_
(DS. x
1
=
4 + 2x
3
3
, x
2
=
7 x
3
3
, x
3
t` uy y)
4.2. He
.
t` uy y cac phu
.
o
.
ng tr`nh tuye

n tnh 157
3.
x
1
+x
2
+ 2x
3
+x
4
= 1,
x
1
2x
2
x
4
= 2.
_
(DS. x
3
=
1
2
(2x
1
+x
2
1), x
4
= x
1
2x
2
+ 2,
x
1
, x
2
t` uy y)
4.
x
1
+ 5x
2
+ 4x
3
+ 3x
4
= 1,
2x
1
x
2
+ 2x
3
x
4
= 0,
5x
1
+ 3x
2
+ 8x
3
+x
4
= 1.
_

_
(DS. x
1
=
14
11
x
3
+
2
11
x
4
+
1
11
, x
2
=
6
11
x
3

7
11
x
4
+
2
11
,
x
3
, x
4
t` uy y)
5.
3x
1
+ 5x
2
+ 2x
3
+ 4x
4
= 3,
2x
1
+ 3x
2
+ 4x
3
+ 5x
4
= 1,
5x
1
+ 9x
2
2x
3
+ 2x
4
= 9.
_

_
(DS. He
.
vo nghie
.
m)
6.
x
1
+ 2x
2
+ 3x
3
= 14,
3x
1
+ 2x
2
+x
3
= 10,
x
1
+x
2
+x
3
= 6,
2x
1
+ 3x
2
x
3
= 5,
x
1
+x
2
= 3.
_

_
(DS. x
1
= 1, x
2
= 2, x
3
= 3)
7.
x
1
+ 3x
2
2x
3
+x
4
+x
5
= 1,
x
1
+ 3x
2
x
3
+ 3x
4
+ 2x
5
= 3,
x
1
+ 3x
2
3x
3
x
4
= 2.
_

_
(DS. He
.
vo nghie
.
m)
8.
5x
1
+x
2
3x
3
= 6,
2x
1
5x
2
+ 7x
3
= 9,
4x
1
+ 2x
2
4x
3
= 7,
5x
1
2x
2
+ 2x
3
= 1.
_

_
(DS. x
1
=
1
3
, x
2
=
1
6
, x
3
=
3
2
)
158 Chu
.
o
.
ng 4. He
.
phu
.
o
.
ng tr`nh tuye

n tnh
9.
x
1
+x
2
+x
3
+x
4
= 1,
x
1
+x
2
2x
3
x
4
= 0,
x
1
+x
2
4x
3
+ 3x
4
= 2,
x
1
+x
2
+ 7x
3
+ 5x
4
= 3.
_

_
(DS. x
1
=
2 3x
2
2x
4
3
, x
3
=
1 2x
4
3
, x
2
, x
4
t` uy y)
10.
x
1
+ 2x
2
+ 3x
3
+ 4x
4
= 5,
x
2
+ 2x
3
+ 3x
4
= 1,
x
1
+ 3x
3
+ 4x
4
= 2,
x
1
+x
2
+ 5x
3
+ 6x
4
= 1.
_

_
(DS. x
1
=
15
4
, x
2
=
3
2
, x
3
=
13
4
, x
4
= 2)
11.
x
1
+ 2x
2
+ 3x
3
+ 4x
4
= 30,
x
1
+ 2x
2
3x
3
+ 4x
4
= 10,
x
2
x
3
+x
4
= 3,
x
1
+x
2
+x
3
+x
4
= 10.
_

_
(DS. x
1
= 1, x
2
= 2, x
3
= 3, x
4
= 4)
12.
5x
1
+x
2
3x
3
= 6,
2x
1
5x
2
+ 7x
3
= 9,
4x
1
+ 2x
2
4x
3
= 7,
5x
1
2x
2
+ 2x
3
= 1.
_

_
(DS. x
1
=
1
3
, x
2
=
1
6
, x
3
=
3
2
)
13.
x
1
x
2
+x
3
x
4
= 4,
x
1
+x
2
+ 2x
3
+ 3x
4
= 8,
2x
1
+ 4x
2
+ 5x
3
+ 10x
4
= 20,
2x
1
4x
2
+x
3
6x
4
= 4.
_

_
(DS. x
1
= 6
3
2
x
3
x
4
, x
2
= 2
1
2
x
3
2x
4
, x
3
v`a x
4
t` uy y)
4.2. He
.
t` uy y cac phu
.
o
.
ng tr`nh tuye

n tnh 159
14.
x
1
2x
2
+ 3x
3
4x
4
= 2,
3x
1
+ 3x
2
5x
3
+x
4
= 3,
2x
1
+x
2
+ 2x
3
3x
4
= 5,
3x
1
+ 3x
3
10x
4
= 8.
_

_
(DS. He
.
vo nghie
.
m)
15.
x
1
+ 2x
2
+ 3x
3
2x
4
= 1,
2x
1
x
2
2x
3
3x
4
= 2,
3x
1
+ 2x
2
x
3
+ 2x
4
= 5,
2x
1
3x
2
+ 2x
3
+x
4
= 11.
_

_
(DS. x
1
=
2
3
, x
2
=
43
18
, x
3
=
13
9
, x
4
=
7
18
)
16.
x
1
+ 2x
2
3x
3
+ 5x
4
= 1,
x
1
+ 3x
2
13x
3
+ 22x
4
= 1,
3x
1
+ 5x
2
+x
3
2x
4
= 5,
2x
1
+ 3x
2
+ 4x
3
7x
4
= 4.
_

_
(DS. x
1
= 17x
3
+ 29x
4
+ 5, x
2
= 10x
3
17x
4
2, x
3
, x
4
t` uy y)
17.
x
1
5x
2
8x
3
+x
4
= 3,
3x
1
+x
2
3x
3
5x
4
= 1,
x
1
7x
3
+ 2x
4
= 5,
11x
2
+ 20x
3
9x
4
= 2.
_

_
(DS. He
.
vo nghie
.
m)
18.
_

_
x
2
3x
3
+ 4x
4
= 5,
x
1
2x
3
+ 3x
4
= 4,
3x
1
+ 2x
2
5x
4
= 12,
4x
1
+ 3x
2
5x
3
= 5.
(DS. x
1
= 1, x
2
= 2, x
3
= 1, x
4
= 1)
Kha o sat tnh tu
.
o
.
ng thch cu a cac he
.
phu
.
o
.
ng tr`nh sau day
19.
x
1
+x
2
+x
3
x
4
= 0,
x
1
x
2
x
3
+x
4
= 1,
x
1
+ 3x
2
+ 3x
3
3x
4
= 0.
_

_
160 Chu
.
o
.
ng 4. He
.
phu
.
o
.
ng tr`nh tuye

n tnh
(DS. He
.
khong tu
.
o
.
ng thch)
20.
x
1
+x
2
+x
3
+x
4
= 1,
x
1
+x
2
+ 2x
3
+x
4
= 0,
x
1
+x
2
x
3
+x
4
= 3.
_

_
(DS. He
.
tu
.
o
.
ng thch)
21.
x
1
2x
2
+x
3
+x
4
= 1,
x
1
2x
2
+x
3
x
4
= 1,
x
1
2x
2
+x
3
+ 5x
4
= 5.
_

_
(DS. He
.
tu
.
o
.
ng thch)
22.
x
1
+x
2
+x
3
+x
4
+x
5
= 7,
3x
1
+ 2x
2
+x
3
+x
4
3x
5
= 2,
x
2
+ 2x
3
+ 2x
4
+ 6x
5
= 23,
5x
1
+ 4x
2
+ 3x
3
+ 3x
4
x
5
= 12.
_

_
(DS. He
.
tu
.
o
.
ng thch)
23.
2x
1
+x
2
x
3
+x
4
= 1,
3x
1
2x
2
+ 2x
3
3x
4
= 2,
5x
1
+x
2
x
3
+ 2x
4
= 1,
2x
1
x
2
+x
3
3x
4
= 4.
_

_
(DS. He
.
khong tu
.
o
.
ng thch)
24.
3x
1
+x
2
2x
3
+x
4
x
5
= 1,
2x
1
x
2
+ 7x
3
3x
4
+ 5x
5
= 2,
x
1
+ 3x
2
2x
3
+ 5x
4
7x
5
= 3,
3x
1
2x
2
+ 7x
3
5x
4
+ 8x
5
= 3.
_

_
(DS. He
.
khong tu
.
o
.
ng thch)
25.
5x
1
+ 7x
2
+ 4x
3
+ 5x
4
8x
5
+ 3x
6
= 1,
2x
1
+ 3x
2
+ 3x
3
6x
4
+ 7x
5
9x
6
= 2,
7x
1
+ 9x
2
+ 3x
3
+ 7x
4
5x
5
8x
6
= 5.
_

_
(DS. He
.
tu
.
o
.
ng thch)
4.2. He
.
t` uy y cac phu
.
o
.
ng tr`nh tuye

n tnh 161
Kha o sat tnh tu
.
o
.
ng thch v`a gia i c ac he
.
phu
.
o
.
ng tr`nh (ne

u he
.
tu
.
o
.
ng thch)
26.
2x
1
x
2
+ 3x
3
= 3,
3x
1
+x
2
5x
3
= 0,
4x
1
x
2
+x
4
= 3,
x
1
+ 3x
2
13x
3
= 6.
_

_
(DS. x
1
= 1, x
2
= 2, x
3
= 1)
27.
2x
1
x
2
+x
3
x
4
= 1,
2x
1
x
2
3x
4
= 2,
3x
1
x
3
+x
4
= 3,
2x
1
+ 2x
2
2x
3
+ 5x
4
= 6.
_

_
(DS. x
1
= 0, x
2
= 2, x
3
=
5
3
, x
4
=
4
3
)
28.
2x
1
+x
2
+x
3
= 2,
x
1
+ 3x
2
+x
3
= 5,
x
1
+x
2
+ 5x
3
= 7,
2x
1
+ 3x
2
5x
3
= 14.
_

_
(DS. x
1
= 1, x
2
= 2, x
3
= 2)
29.
2x
1
+ 3x
2
+ 4x
3
+ 3x
4
= 0,
4x
1
+ 6x
2
+ 9x
3
+ 8x
4
= 3,
6x
1
+ 9x
2
+ 9x
3
+ 4x
4
= 8.
_

_
(DS. x
1
=
7
2

3x
2
2
, x
3
= 1, x
4
= 1, x
2
t` uy y)
30.
3x
1
+ 3x
2
6x
3
2x
4
= 1,
6x
1
+x
2
2x
4
= 2,
6x
1
7x
2
+ 21x
3
+ 4x
4
= 3,
9x
1
+ 4x
2
+ 2x
4
= 3,
12x
1
6x
2
+ 21x
3
+ 2x
4
= 1.
_

_
(DS. x
1
=
7
5
, x
2
= 4, x
3
=
11
5
, x
4
=
16
5
)
162 Chu
.
o
.
ng 4. He
.
phu
.
o
.
ng tr`nh tuye

n tnh
31.
x
1
+x
2
+ 2x
3
+ 3x
4
= 1,
3x
1
x
2
x
3
2x
4
= 4,
2x
1
+ 3x
2
x
3
x
4
= 6,
x
1
+ 2x
2
+ 3x
3
x
4
= 4.
_

_
(DS. x
1
= x
2
= 1, x
3
= 0, x
4
= 1)
32.
x
1
+ 2x
2
+ 3x
3
2x
4
= 6,
2x
1
x
2
2x
3
3x
4
= 8,
3x
1
+ 2x
2
x
3
+ 2x
4
= 4,
2x
1
3x
2
+ 2x
3
+x
4
= 8.
_

_
(DS. x
1
= 1, x
2
= 2, x
3
= 1, x
4
= 2)
33.
x
2
3x
3
+ 4x
4
= 5,
x
1
2x
3
+ 3x
4
= 4,
3x
1
+ 2x
2
5x
4
= 12,
4x
1
+ 3x
2
5x
3
= 5.
_

_
(DS. x
1
= 1, x
2
= 2, x
3
= 1, x
4
= 1)
34.
x
1
+x
2
x
3
+x
4
= 4,
2x
1
x
2
+ 3x
3
2x
4
= 1,
x
1
x
3
+ 2x
4
= 6,
3x
1
x
2
+x
3
x
4
= 0.
_

_
(DS. x
1
= 1, x
2
= 2, x
3
= 3, x
4
= 4)
35.
x
1
+x
2
+x
3
+x
4
= 0,
x
2
+x
3
+x
4
+x
5
= 0,
x
1
+ 2x
2
+ 3x
4
= 2,
x
2
+ 2x
3
+ 3x
4
= 2,
x
3
+ 2x
4
+ 3x
5
= 2.
_

_
(DS. x
1
= 1, x
2
= 1, x
3
= 1, x
4
= 1, x
5
= 1)
4.2. He
.
t` uy y cac phu
.
o
.
ng tr`nh tuye

n tnh 163
36.
3x
1
x
2
+x
3
+ 2x
5
= 18,
2x
1
5x
2
+x
4
+x
5
= 7,
x
1
x
4
+ 2x
5
= 8,
2x
2
+x
3
+x
4
x
5
= 10,
x
1
+x
2
3x
3
+x
4
= 1.
_

_
(DS. x
1
= 5, x
2
= 4, x
3
= 3, x
4
= 1, x
5
= 2)
164 Chu
.
o
.
ng 4. He
.
phu
.
o
.
ng tr`nh tuye

n tnh
Gia i v`a bie
.
n lua
.
n he
.
phu
.
o
.
ng tr`nh tuye

n tnh theo tham so


37.
x
1
+ 2x
2
+ 3x
3
= 1,
2x
1
+ 2x
2
+ 2x
3
= 3,
5x
1
+ 6x
2
+ 7x
3
= .
_

_
(DS. a) Ne

u = 4 nghie
.
m to

ng quat l`a x
1
= 5 +x
3
,
x
2
=
7 4x
3
2
, x
3
t` uy y;
b) Ne

u ,= 4 he
.
khong tu
.
o
.
ng thch)
38.
2x
1
x
2
+x
3
= 0,
x
1
2x
2
2x
3
= 3,
x
1
+x
2
+ 3x
3
= 1.
_

_
(DS. a) Ne

u ,= 1, he
.
co nghie
.
m duy nhat
x
1
=
5 11
3( 1)
, x
2
=
2 22
3( 1)
, x
3
=
4
1
;
b) Ne

u = 1 he
.
khong tu
.
o
.
ng thch)
39.
x
1
+x
2
+x
3
= 1,
x
1
+x
2
+x
3
= 1,
x
1
+x
2
+x
3
= 1.
_

_
(DS. a) Ne

u ,= 2, 1 he
.
co nghie
.
m duy nhat x
1
= x
2
= x
3
=
1
+ 2
b) Ne

u = 2 he
.
khong tu
.
o
.
ng thch;
c) Ne

u = 1 he
.
co vo so nghie
.
m phu
.
thuo
.
c hai tham so v`a x
1
+
x
2
+x
3
= 1)
40.
x
1
+x
2
+ 2x
3
= 3,
3x
1
+ 2x
2
+ 4x
3
= a,
5x
1
+ 3x
2
+ 6x
3
= a
2
.
_

_
(DS. a) Ne

u a = 1 hoa
.
c a = 3 he
.
tu
.
o
.
ng thch v`a x
1
= 5,
x
2
= 8 2x
3
, x
3
t` uy y;
b) Ne

u a ,= 1, a ,= 3 th` he
.
khong tu
.
o
.
ng thch)
4.3. He
.
phu
.
o
.
ng tr`nh tuye

n tnh thua
`
n nhat 165
41.
(1 +)x
1
+x
2
+x
3
= 1,
x
1
+ (1 +)x
2
+x
3
= ,
x
1
+x
2
+ (1 +)x
3
=
2
.
_

_
(DS. a) Ne

u ( + 3) ,= 0 he
.
co nghie
.
m duy nhat
x
1
=
2
2
( + 3)
, x
2
=
2 1
( + 3)
, x
3
=

3
+ 2
2
1
( + 3)

b) Ne

u = 0 hoa
.
c = 3 he
.
khong tu
.
o
.
ng thch)
42.
x
1
+x
2
+x
3
+x
4
= 1,
x
1
+x
2
+x
3
+x
4
= 1,
x
1
+x
2
+x
3
+x
4
= 0,
x
1
+x
2
+x
3
+x
4
= 0.
_

_
(DS. a) Khi ,= 3 v`a ,= 1 he
.
co nghie
.
m duy nhat;
x
1
= 0, x
2
= 0, x
3
=
1
1
, x
4
=
1
1

b) Khi = 3 nghie
.
m to

ng quat l`a
x
1
=
1
4
+x
4
; x
2
=
1
4
+x
4
, x
3
=
1
2
+x
4
; x
3
t` uy y;
c) Khi = 1 he
.
khong tu
.
o
.
ng thch)
4.3 He
.
phu
.
o
.
ng tr`nh tuye

n tnh thua
`
n
nha

t
He
.
phu
.
o
.
ng tr`nh tuye

n tnh du
.
o
.
.
c go
.
i l`a he
.
thua
`
n nhat ne

u so ha
.
ng tu
.
.
do cu a mo

i phu
.
o
.
ng tr`nh de
`
u ba
`
ng 0.
He
.
thua
`
n nhat c o da
.
ng
a
11
x
1
+a
12
x
2
+ +a
1n
x
n
= 0,
. . . . . . . . . . . . . . .
a
m1
x
1
+a
m2
x
2
+ +a
mn
x
n
= 0.
_

_
(4.10)
166 Chu
.
o
.
ng 4. He
.
phu
.
o
.
ng tr`nh tuye

n tnh
He
.
phu
.
o
.
ng tr`nh thua
`
n nhat luon luon tu
.
o
.
ng thch v` no c o t nhat
l`a nghie
.
m-khong. Nghie
.
m n`ay du
.
o
.
.
c go
.
i l`a nghie
.
m ta
`
m thu
.
`o
.
ng.
D
-
i
.
nh l y. 1
+
He
.
(4.10) co nghie
.
m khong t a
`
m thu
.
`o
.
ng khi v` a ch khi
ha
.
ng cu a ma tra
.
n cu a he
.
be ho
.
n so a

n cu a he
.
d o.
2
+
He
.
thua
`
n nhat n phu
.
o
.
ng tr`nh vo
.
i n a

n c o nghie
.
m khong t a
`
m
thu
.
`o
.
ng khi v`a ch khi di
.
nh th u
.
c D cu a he
.
ba
`
ng 0.
Gia su
.
x
1
=
1
, x
2
=
2
, . . . , x
n
=
n
l`a nghie
.
m khong ta
`
m thu
.
`o
.
ng
n`ao do cu a he
.
(4.10). Nghie
.
m n`ay c o the

xem nhu
.
mo
.
t h`ang go
`
m n
pha
`
n tu
.

e
1
= (
1
,
2
, . . . ,
n
).
Khi do theo di
.
nh ngha, h`ang e
1
= (
1
, . . . ,
n
) c ung l`a nghie
.
m
cu a (4.10). Gia su
.
h`ang
e
2
= (
1
,
2
, . . . ,
n
)
l`a mo
.
t nghie
.
m khac cu a (4.10). Khi do h`ang to

ho
.
.
p tuye

n tnh
e
1
+e
2
def
=
_

1
+
1
,
1

2
+
2
, . . . ,
n
+
n
)
c ung l`a nghie
.
m cu a (4.10). T` u
.
do: mo
.
i to

ho
.
.
p tuye

n tnh cac nghie


.
m
cu a he
.
thua
`
n nhat (4.10) c ung l`a nghie
.
m cu a no.
D
-
i
.
nh ngha 1. 1
+
Cac h`ang e
1
, e
2
, . . . , e
m
du
.
o
.
.
c go
.
i l`a phu
.
thuo
.
c tuye

n
tnh ne

u co the

t`m du
.
o
.
.
c c ac so
1
,
2
, . . . ,
m
khong d o
`
ng th`o
.
i ba
`
ng 0
sao cho

1
e
1
+
2
e
2
+ +
m
e
m
= 0. (4.11)
2
+
Ne

u c ac so
i
, i = 1, m nhu
.
va
.
y kh ong to
`
n ta
.
i (t u
.
c l`a da

ng
th u
.
c (4.11) ch thoa man khi
1
=
2
= =
m
= 0) th` ngu
.
`o
.
i ta
noi ra
`
ng e
1
, e
2
, . . . , e
m
do
.
c la
.
p tuye

n tnh.
D
-
i
.
nh ngha 2. He
.
do
.
c la
.
p tuye

n tnh cac nghie


.
m
e
1
, e
2
, . . . , e
m
4.3. He
.
phu
.
o
.
ng tr`nh tuye

n tnh thua
`
n nhat 167
cu a he
.
phu
.
o
.
ng tr`nh (4.10) du
.
o
.
.
c go
.
i l`a he
.
nghie
.
m co
.
ba n cu a no ne

u
mo

i nghie
.
m cu a he
.
(4.10) de
`
u l` a to

ho
.
.
p tuye

n tnh cu a cac nghie


.
m
e
1
, e
2
, . . . , e
m
.
D
-
i
.
nh l y (ve
`
su
.
.
to
`
n ta
.
i he
.
nghie
.
m co
.
ba n). Ne

u ha
.
ng cu a ma tr a
.
n
cu a he
.
(4.10) be ho
.
n so a

n th` he
.
(4.10) co he
.
nghie
.
m co
.
ba n.
168 Chu
.
o
.
ng 4. He
.
phu
.
o
.
ng tr`nh tuye

n tnh
Phu
.
o
.
ng phap t`m he
.
nghie
.
m co
.
ba n
1) Da
`
u tien ca
`
n tach ra he
.
a

n co
.
so
.
(gia su
.
do l`a x
1
, . . . , x
r
) v`a thu
du
.
o
.
.
c he
.
a
11
x
1
+ +a
1r
x
r
= a
1r+1
x
r+1
a
1n
x
n
,
. . . . . . . . . . . . . . . . . . . . .
a
r1
x
1
+ +a
rr
x
r
= a
rr+1
x
r+1
a
rn
x
n
.
_

_
(4.12)
2) Gia su
.
he
.
(4.12) c o nghie
.
m l`a
x
i
=
_

(i)
1
,
(i)
2
, . . . ,
(i)
r
; x
r+1
, . . . , x
n
)
_
; i = 1, r.
Cho c ac a

n tu
.
.
do cac gia tri
.
x
r+1
= 1, x
r+2
= 0, . . . , x
n
= 0
ta thu du
.
o
.
.
c
e
1
=
_

(1)
1
,
(1)
2
, . . . ,
(1)
r
; 1, 0, . . . , 0
_
Tu
.
o
.
ng tu
.
.
, vo
.
i x
r+1
= 0, x
r+2
= 1, x
r+3
= 0, . . . , x
n
= 0 ta co
e
2
=
_

(2)
1
, . . . ,
(2)
r
; 0, 1, 0, . . . , 0
_
, . . .
v`a sau c` ung vo
.
i x
r+1
= 0, . . . , x
n1
= 0, x
n
= 1 ta thu du
.
o
.
.
c
e
k
= (
(k)
1
, . . . ,
(k)
r
, 0, . . . , 1), k = n r.
He
.
cac nghie
.
m e
1
, e
2
, . . . , e
k
v` u
.
a thu du
.
o
.
.
c l`a he
.
nghie
.
m co
.
ba n.
C

AC V

I DU
.
V du
.
1. T`m nghie
.
m to

ng quat v`a he
.
nghie
.
m co
.
ba n cu a he
.
phu
.
o
.
ng
tr`nh
2x
1
+x
2
x
3
+x
4
= 0,
4x
1
+ 2x
2
+x
3
3x
4
= 0.
_
4.3. He
.
phu
.
o
.
ng tr`nh tuye

n tnh thua
`
n nhat 169
Gia i. 1) V` so phu
.
o
.
ng tr`nh be ho
.
n so a

n nen ta
.
p ho
.
.
p nghie
.
m cu a
he
.
l`a v o ha
.
n.
Hie

n nhien ha
.
ng cu a ma tra
.
n cu a he
.
ba
`
ng 2 v` trong c ac di
.
nh th u
.
c
con c ap 2 co di
.
nh th u
.
c con

2 1
4 1

,= 0.
Do va
.
y he
.
da cho tu
.
o
.
ng du
.
o
.
ng vo
.
i he
.
2x
1
x
3
= x
1
x
4
,
4x
1
+x
3
= 2x
2
+ 3x
4
.
T` u
.
do suy ra
x
1
=
3x
2
+ 2x
4
6
, x
3
=
5
3
x
4
. (4.13)
Do do ta
.
p ho
.
.
p nghie
.
m cu a he
.
c o da
.
ng
_
3 + 2
6
; ;
5
3
;

, R
_
(*)
2) Ne

u trong (4.13) ta cho c ac a

n tu
.
.
do bo
.
i c ac gia tri
.
la
`
n lu
.
o
.
.
t
ba
`
ng cac pha
`
n tu
.
cu a cac co
.
t di
.
nh th u
.
c

1 0
0 1

(,= 0)
th` thu du
.
o
.
.
c cac nghie
.
m
e
1
=
_

1
2
; 1; 0; 0
_
v`a e
2
=
_
1
3
; 0;
5
3
; 1
_
.
Do l`a he
.
nghie
.
m co
.
ba n cu a he
.
phu
.
o
.
ng tr`nh da cho v`a nghie
.
m to

ng
quat cu a he
.
da cho c o the

bie

u die

n du
.
o
.
i da
.
ng
X = e
1
+e
2
=
_

1
2
; 1; 0; 0
_
+
_
1
3
; 0;
5
3
; 1
_
170 Chu
.
o
.
ng 4. He
.
phu
.
o
.
ng tr`nh tuye

n tnh
trong do v`a l`a cac ha
`
ng so t` uy y:
X =
_
3 + 2
6
; ;
5
3
;

, R
_
.
Khi cho v`a cac gi a tri
.
so khac nhau ta se thu du
.
o
.
.
c cac nghie
.
m
rieng khac nhau.
V du
.
2. Gia i he
.
x
1
+ 2x
2
x
3
= 0,
3x
1
6x
2
+ 3x
3
= 0,
7x
1
+ 14x
2
7x
3
= 0.
_

_
Gia i. He
.
da cho tu
.
o
.
ng du
.
o
.
ng vo
.
i phu
.
o
.
ng tr`nh
x
1
+ 2x
2
x
3
= 0.
T` u
.
do suy ra nghie
.
m cu a he
.
l`a:
x
1
= 2x
2
+x
3
,
x
2
= x
2
,
x
3
= x
3
; x
2
v`a x
3
t` uy y,
hay du
.
o
.
i da
.
ng khac
e = (2x
2
+x
3
; x
2
; x
3
).
Cho x
2
= 1, x
3
= 0 ta co
e
1
= (2; 1; 0),
la
.
i cho x
2
= 0, x
3
= 1 ta thu du
.
o
.
.
c
e
2
= (1, 0, 1).
Hai h`ang e
1
v`a e
2
l`a do
.
c la
.
p tuye

n tnh v`a mo
.
i nghie
.
m cu a he
.
de
`
u co
da
.
ng
X = e
1
+e
2
= (2 +; ; )
4.3. He
.
phu
.
o
.
ng tr`nh tuye

n tnh thua
`
n nhat 171
trong do v`a l`a cac so t` uy y.
V du
.
3. T`m nghie
.
m to

ng quat v`a he
.
nghie
.
m co
.
ba n cu a he
.
phu
.
o
.
ng
tr`nh
x
1
+ 3x
2
+ 3x
3
+ 2x
4
+ 4x
5
= 0,
x
1
+ 4x
2
+ 5x
3
+ 3x
4
+ 7x
5
= 0,
2x
1
+ 5x
2
+ 4x
3
+x
4
+ 5x
5
= 0,
x
1
+ 5x
2
+ 7x
3
+ 6x
4
+ 10x
5
= 0.
_

_
Gia i. Ba
`
ng cac phep bie

n do

i so
.
cap, de

d`ang thay ra
`
ng he
.
da cho
co the

du
.
a ve
`
he
.
ba
.
c thang sau day
x
1
+ 3x
2
+ 3x
3
+ 2x
4
+ 4x
5
= 0,
x
2
+ 2x
3
+x
4
+ 3x
5
= 0,
x
4
= 0.
_

_
Ta se cho
.
n x
1
, x
2
v`a x
4
l`am a

n co
.
so
.
; c`on x
3
v`a x
5
l`am a

n tu
.
.
do. Ta
co he
.
x
1
+ 3x
2
+ 2x
4
= 3x
3
4x
5
,
x
2
+x
4
= 2x
3
3x
5
,
x
4
= 0.
_

_
Gia i he
.
n`ay ta thu du
.
o
.
.
c nghie
.
m to

ng quat l`a
x
1
= 3x
3
+ 5x
5
,
x
2
= 2x
3
3x
5
,
x
4
= 0.
Cho cac a

n tu
.
.
do la
`
n lu
.
o
.
.
t cac gia tri
.
ba
`
ng x
3
= 1, x
5
= 0 (khi do
x
1
= 3, x
2
= 2, x
3
= 1, x
4
= 0, x
5
= 0) v`a cho x
3
= 0, x
5
= 1 (khi do
x
1
= 5, x
2
= 3, x
3
= 0, x
4
= 0, x
5
= 1) ta thu du
.
o
.
.
c he
.
nghie
.
m co
.
ba n
e
1
= (3; 2; 1; 0; 0),
e
2
= (5; 3; 0; 0; 1).
172 Chu
.
o
.
ng 4. He
.
phu
.
o
.
ng tr`nh tuye

n tnh
T` u
.
do nghie
.
m to

ng quat c o the

vie

t du
.
o
.
i da
.
ng
X = (3; 2; 1; 0; 0) +(5; 3; 0; 0; 1)
= (3 + 5; 2 3; ; 0; ); , R.
Ba
`
ng cach cho v`a nh u
.
ng gia tri
.
so khac nhau ta thu du
.
o
.
.
c cac
nghie
.
m rieng khac nhau. Do
`
ng th`o
.
i, mo
.
i nghie
.
m rieng co the

thu
du
.
o
.
.
c t` u
.
do ba
`
ng cach cho
.
n cac he
.
so v`a thch ho
.
.
p.
4.3. He
.
phu
.
o
.
ng tr`nh tuye

n tnh thua
`
n nhat 173
B
`
AI T

A
.
P
Gia i cac he
.
phu
.
o
.
ng tr`nh thua
`
n nhat
1.
x
1
+ 2x
2
+ 3x
3
= 0,
2x
1
+ 3x
2
+ 4x
3
= 0,
3x
1
+ 4x
2
+ 5x
3
= 0.
_

_
.
(DS. x
1
= , x
2
= 2, x
3
= , R)
2.
x
1
+x
2
+x
3
= 0,
3x
1
x
2
x
3
= 0,
2x
1
+ 3x
2
+x
3
= 0.
_

_
. (DS. x
1
= x
2
= x
3
= 0)
3.
3x
1
4x
2
+x
3
x
4
= 0,
6x
1
8x
2
+ 2x
3
+ 3x
4
= 0.
_
(DS. x
1
=
4
3
, x
2
= , x
3
= , x
4
= 0; , R t` uy y)
4.
3x
1
+ 2x
2
8x
3
+ 6x
4
= 0,
x
1
x
2
+ 4x
3
3x
4
= 0.
_
(DS. x
1
= 0, x
2
= , x
3
= , x
4
=
+ 4
3
; , R t` uy y)
5.
x
1
2x
2
+ 3x
3
x
4
= 0,
x
1
+x
2
x
3
+ 2x
4
= 0,
4x
1
5x
2
+ 8x
3
+x
4
= 0.
_

_
(DS. x
1
=
1
4
, x
2
= , x
3
=
3
4
, x
4
= 0; R t` uy y)
6.
3x
1
x
2
+ 2x
3
+x
4
= 0,
x
1
+x
2
x
3
x
4
= 0,
5x
1
+x
2
x
3
= 0.
_

_
(DS. x
1
=

4
, x
2
=
5
4
+, x
3
= , x
4
= ; , R t` uy y)
7.
2x
1
+x
2
+x
3
= 0,
3x
1
+ 2x
2
3x
3
= 0,
x
1
+ 3x
2
4x
3
= 0,
5x
1
+x
2
2x
3
= 0.
_

_
174 Chu
.
o
.
ng 4. He
.
phu
.
o
.
ng tr`nh tuye

n tnh
(DS. x
1
=

7
, x
2
=
9
7
, x
3
= ; R t` uy y)
4.3. He
.
phu
.
o
.
ng tr`nh tuye

n tnh thua
`
n nhat 175
T`m nghie
.
m to

ng quat v`a he
.
nghie
.
m co
.
ba n cu a cac he
.
phu
.
o
.
ng
tr`nh
8.
9x
1
+ 21x
2
15x
3
+ 5x
4
= 0,
12x
1
+ 28x
2
20x
3
+ 7x
4
= 0.
_
(DS. Nghie
.
m to

ng quat: x
1
=
7
3
x
2
+
5
3
x
3
, x
4
= 0.
He
.
nghie
.
m co
.
ba n e
1
= (7, 3, 0, 0), e
2
= (5, 0, 3, 0))
9.
14x
1
+ 35x
2
7x
3
63x
4
= 0,
10x
1
25x
2
+ 5x
3
+ 45x
4
= 0,
26x
1
+ 65x
2
13x
3
117x
4
= 0.
_

_
(DS. Nghie
.
m to

ng quat: x
3
= 2x
1
+ 5x
2
9x
3
.
He
.
nghie
.
m co
.
ba n: e
1
= (1, 0, 2, 0); e
2
= (0, 1, 5, 0); e
3
=
(0, 0, 9, 1))
10.
x
1
+ 4x
2
+ 2x
3
3x
5
= 0,
2x
1
+ 9x
2
+ 5x
3
+ 2x
4
+x
5
= 0,
x
1
+ 3x
2
+x
3
2x
4
9x
5
= 0.
_

_
(DS. Nghie
.
m to

ng quat: x
1
= 2x
3
+ 8x
4
, x
2
= x
2
2x
4
; x
5
= 0.
He
.
nghie
.
m co
.
ba n: e
1
= (2, 1, 1, 0, 0); e
2
= (8, 2, 0, 1, 0)
11.
x
1
+ 2x
2
+ 4x
3
3x
4
= 0,
3x
1
+ 5x
2
+ 6x
3
4x
4
= 0,
4x
1
+ 5x
2
2x
3
+ 3x
4
= 0,
3x
1
+ 8x
2
+ 24x
3
19x
4
= 0.
_

_
(DS. Nghie
.
m to

ng quat: x
1
= 8x
3
7x
4
, x
2
= 6x
3
+ 5x
4
.
He
.
nghie
.
m co
.
ba n: e
1
= (8, 6, 1, 0), e
2
= (7, 5, 0, 1))
12.
x
1
+ 2x
2
2x
3
+x
4
= 0,
2x
1
+ 4x
2
+ 2x
3
x
4
= 0,
x
1
+ 2x
2
+ 4x
3
2x
4
= 0,
4x
1
+ 8x
2
2x
3
+x
4
= 0.
_

_
(DS. Nghie
.
m to

ng quat x
1
= 2x
2
, x
4
= 2x
3
.
He
.
nghie
.
m co
.
ba n: e
1
= (2, 1, 0, 0), e
2
= (0, 0, 1, 2))
176 Chu
.
o
.
ng 4. He
.
phu
.
o
.
ng tr`nh tuye

n tnh
13.
x
1
+ 2x
2
+ 3x
3
+ 4x
4
+ 5x
5
= 0,
2x
1
+ 3x
2
+ 4x
3
+ 5x
4
+x
5
= 0,
3x
1
+ 4x
2
+ 5x
3
+x
4
+ 2x
5
= 0,
x
1
+ 3x
2
+ 5x
3
+ 12x
4
+ 9x
5
= 0,
4x
1
+ 5x
2
+ 6x
3
3x
4
+ 3x
5
= 0.
_

_
(DS. Nghie
.
m to

ng quat x
1
= x
3
+15x
5
, x
2
= 2x
3
12x
5
, x
4
= x
5
.
He
.
nghie
.
m co
.
ba n: e
1
= (1, 2, 1, 0, 0), e
2
= (15, 12, 0, 1, 1))
Chu
.
o
.
ng 5
Khong gian Euclide R
n
5.1 D
-
i
.
nh ngha khong gian n-chie `u v`a mo
.
t so
khai nie
.
m co
.
ba n ve ` vecto
.
. . . . . . . . . . 177
5.2 Co
.
so
.
. D
-
o

i co
.
so
.
. . . . . . . . . . . . . . . . 188
5.3 Khong gian vecto
.
Euclid. Co
.
so
.
tru
.
.
c chua

n201
5.4 Phep bie

n do

i tuye

n tnh . . . . . . . . . . . 213
5.4.1 D
-
i
.
nh ngha . . . . . . . . . . . . . . . . . . 213
5.4.2 Ma tra
.
n cu a phep bdtt . . . . . . . . . . . 213
5.4.3 Cac phep toan . . . . . . . . . . . . . . . . 215
5.4.4 Vecto
.
rieng v`a gia tri
.
rieng . . . . . . . . . 216
5.1 D
-
i
.
nh ngha khong gian n-chie
`
u v` a
mo
.
t so

khai nie
.
m co
.
ba

n ve
`
vecto
.
1

. Gia su
.
n N. Ta
.
p ho
.
.
p mo
.
i bo
.
co the

co (x
1
, x
2
, . . . , x
n
) g o
`
m n
so thu
.
.
c (ph u
.
c) du
.
o
.
.
c go
.
i l`a khong gian thu
.
.
c (ph u
.
c) n-chie
`
u v`a du
.
o
.
.
c
178 Chu
.
o
.
ng 5. Khong gian Euclide R
n
k y hie
.
u l`a R
n
(C
n
). Mo

i bo
.
so do du
.
o
.
.
c ch bo
.
i
x = (x
1
, x
2
, . . . , x
n
)
v`a du
.
o
.
.
c go
.
i l`a die

m hay vecto
.
cu a R
n
(C
n
). C ac so x
1
, . . . , x
n
du
.
o
.
.
c
go
.
i l`a to
.
a do
.
cu a die

m (cu a vecto
.
) x hay c ac th`anh pha
`
n cu a vecto
.
x.
Hai vecto
.
x = (x
1
, . . . , x
n
) v` a y = (y
1
, . . . , y
n
) cu a R
n
du
.
o
.
.
c xem l`a
ba
`
ng nhau ne

u cac to
.
a do
.
tu
.
o
.
ng u
.
ng cu a ch ung ba
`
ng nhau
x
i
= y
i
i = 1, n.
Cac vecto
.
x = (x
1
, . . . , x
n
), y = (y
1
, . . . , y
n
) c o the

co
.
ng vo
.
i nhau
v`a co the

nhan vo
.
i cac so , , . . . l`a so thu
.
.
c ne

u khong gian du
.
o
.
.
c xet
l`a khong gian thu
.
.
c v`a l` a so ph u
.
c ne

u khong gian du
.
o
.
.
c xet l`a khong
gian ph u
.
c.
Theo di
.
nh ngha: 1
+
to

ng cu a vecto
.
x v`a y l`a vecto
.
x +y
def
= (x
1
+y
1
, x
2
+y
2
, . . . , x
n
+y
n
). (5.1)
2
+
tch cu a vecto
.
x vo
.
i so hay tch so vo
.
i vecto
.
x l`a vecto
.
x = x
def
= (x
1
, x
2
, . . . , x
n
). (5.2)
Hai phep toan 1
+
v` a 2
+
thoa man cac tnh chat (tien de
`
) sau day
I. x +y = y +x, x, y R
n
(C
n
),
II. (x +y) +z = x + (y +z) x, y, z = R
n
(C
n
),
III. To
`
n ta
.
i vecto
.
- khong = (0, 0, . . . , 0
. .
n
) R
n
sao cho
x + = +x = x,
IV. To
`
n ta
.
i vecto
.
d oi x = (1)x = (x
1
, x
2
, . . . , x
n
) sao cho
x + (x) = ,
V. 1 x = x,
5.1. D
-
i
.
nh ngha khong gian n-chie
`
u v`a mo
.
t so khai nie
.
m co
.
ba n ve
`
vecto
.
179
VI. (x) = ()x, , R (C),
VII. ( +)x = x +x,
VIII. (x +y) = x +y
trong do v`a l`a cac so, c`on x, y R
n
(C
n
).
D
-
i
.
nh ngha 5.1.1. 1
+
Gia su
.
1 l`a ta
.
p ho
.
.
p khong ro

ng t` uy y vo
.
i cac
pha
`
n tu
.
du
.
o
.
.
c k y hie
.
u l`a x, y, z, . . . Ta
.
p ho
.
.
p 1 du
.
o
.
.
c go
.
i l`a khong gian
tuye

n tnh (hay khong gian vecto


.
) ne

u x, y 1 xac di
.
nh du
.
o
.
.
c pha
`
n
tu
.
x + y 1 (go
.
i l`a to

ng cu a x v`a y) v`a R (C) v`a x 1 xac


di
.
nh du
.
o
.
.
c pha
`
n tu
.
x 1 (go
.
i l`a tch cu a so vo
.
i pha
`
n tu
.
x) sao
cho c ac tien de
`
I-VIII du
.
o
.
.
c thoa man.
Khong gian tuye

n tnh vo
.
i phep nhan cac pha
`
n tu
.
cu a no vo
.
i cac
so thu
.
.
c (ph u
.
c) du
.
o
.
.
c go
.
i l`a khong gian tuye

n tnh thu
.
.
c (tu
.
o
.
ng u
.
ng:
ph u
.
c).
Khong gian R
n
co the

xem nhu
.
mo
.
t v du
.
ve
`
khong gian tuye

n
tnh, cac v du
.
khac se du
.
o
.
.
c xet ve
`
sau. V`a trong giao tr`nh n`ay ta
luon gia thie

t ra
`
ng cac khong gian du
.
o
.
.
c xet l`a nh u
.
ng kh ong gian thu
.
.
c.
2

. Cho he
.
go
`
m m vecto
.
n-chie
`
u
x
1
, x
2
, . . . , x
m
. (5.3)
Khi do vecto
.
da
.
ng
y =
1
x
1
+
2
x
2
+ +
m
x
m
;
1
,
2
, . . . ,
m
R.
du
.
o
.
.
c go
.
i l`a to

ho
.
.
p tuye

n tnh cu a cac vecto


.
da cho hay vecto
.
y bie

u
die

n tuye

n tnh du
.
o
.
.
c qua cac vecto
.
(5.3).
D
-
i
.
nh ngha 5.1.2. 1
+
He
.
vecto
.
(5.3) du
.
o
.
.
c go
.
i l`a he
.
d o
.
c la
.
p tuye

n
tnh (dltt) ne

u t` u
.
da

ng th u
.
c vecto
.

1
x
1
+
2
x
2
+ +
m
x
m
= (5.4)
keo theo
1
=
2
= =
m
= 0.
180 Chu
.
o
.
ng 5. Khong gian Euclide R
n
2
+
He
.
(5.3) go
.
i l`a he
.
phu
.
thuo
.
c tuye

n tnh (pttt) ne

u to
`
n ta
.
i cac so

1
,
2
, . . . ,
m
khong do
`
ng th`o
.
i ba
`
ng 0 sao cho da

ng th u
.
c (5.4) du
.
o
.
.
c
thoa man.
So nguyen du
.
o
.
ng r du
.
o
.
.
c go
.
i l`a ha
.
ng cu a he
.
vecto
.
(5.3) ne

u
a) Co mo
.
t ta
.
p ho
.
.
p con g o
`
m r vecto
.
cu a he
.
(5.3) la
.
p th`anh he
.
dltt.
b) Mo
.
i ta
.
p con go
`
m nhie
`
u ho
.
n r vecto
.
cu a he
.
(5.3) de
`
u phu
.
thuo
.
c
tuye

n tnh.
De

t`m ha
.
ng cu a he
.
vecto
.
ta la
.
p ma tra
.
n cac to
.
a do
.
cu a no
A =
_

_
a
11
a
12
. . . a
1n
a
21
a
22
. . . a
2n
.
.
.
.
.
.
.
.
.
.
.
.
a
m1
a
m2
. . . a
mn
_

_
D
-
i
.
nh l y. Ha
.
ng cu a he
.
vecto
.
(5.3) ba
`
ng ha
.
ng cu a ma tr a
.
n A c ac to
.
a
do
.
cu a no.
T` u
.
do, de

ke

t lua
.
n he
.
vecto
.
(5.3) dltt hay pttt ta c a
`
n la
.
p ma tra
.
n
to
.
a do
.
A cu a ch ung v`a tnh r(A):
1) Ne

u r(A) = m th` he
.
(5.3) do
.
c la
.
p tuye

n tnh.
2) Ne

u r(A) = s < m th` he


.
(5.3) phu
.
thuo
.
c tuye

n tnh.
C

AC V

I DU
.
V du
.
1. Ch u
.
ng minh ra
`
ng he
.
vecto
.
a
1
, a
2
, . . . , a
m
(m > 1) phu
.
thuo
.
c
tuye

n tnh khi v`a ch khi t nhat mo


.
t trong cac vecto
.
cu a he
.
l`a to

ho
.
.
p
tuye

n tnh cu a cac vecto


.
c`on la
.
i.
Gia i. 1
+
Gia su
.
he
.
a
1
, a
2
, . . . , a
m
phu
.
thuo
.
c tuye

n tnh. Khi do
to
`
n ta
.
i cac so
1
,
2
, . . . ,
m
khong do
`
ng th`o
.
i ba
`
ng 0 sao cho

1
a
1
+
2
a
2
+ +
m
a
m
= .
Gia su
.

m
,= 0. Khi do
a
m
=
1
a
1
+
2
a
2
+ +
m1
a
m1
,
i
=

i

m
5.1. D
-
i
.
nh ngha khong gian n-chie
`
u v`a mo
.
t so khai nie
.
m co
.
ba n ve
`
vecto
.
181
t u
.
c l` a a
m
bie

u die

n tuye

n tnh qua cac vecto


.
c`on la
.
i.
2
+
Ngu
.
o
.
.
c la
.
i, cha

ng ha
.
n ne

u vecto
.
a
m
bie

u die

n tuye

n tnh qua
a
1
, a
2
, . . . , a
m1
a
m
=
1
a
1
+
2
a
2
+ +
m1
a
m1
th` ta co

1
a
1
+
2
a
2
+ +
m1
a
m1
+ (1)a
m
= .
Do do he
.
da cho phu
.
thuo
.
c tuye

n tnh v` trong da

ng th u
.
c tren c o he
.
so cu a a
m
l`a khac 0 (cu
.
the

l`a = 1).
V du
.
2. Ch u
.
ng minh ra
`
ng mo
.
i he
.
vecto
.
co ch u
.
a vecto
.
-khong l`a he
.
phu
.
thuo
.
c tuye

n tnh.
Gia i. Vecto
.
- khong luon luon bie

u die

n du
.
o
.
.
c du
.
o
.
i da
.
ng to

ho
.
.
p
tuye

n tnh cu a cac vecto


.
a
1
, a
2
, . . . , a
m
:
= 0 a
1
+ 0 a
2
+ + 0 a
m
Do do theo di
.
nh ngha he
.
, a
1
, . . . , a
m
phu
.
thuo
.
c tuye

n tnh (xem v
du
.
1).
V du
.
3. Ch u
.
ng minh ra
`
ng mo
.
i he
.
vecto
.
co ch u
.
a hai vecto
.
ba
`
ng
nhau l`a he
.
phu
.
thuo
.
c tuye

n tnh.
Gia i. Gia su
.
trong he
.
a
1
, a
2
, . . . , a
n
co hai vecto
.
a
1
= a
2
. Khi do
ta co the

vie

t
a
1
= 1 a
2
+ 0 a
3
+ + 0 a
m
t u
.
c l`a vecto
.
a
1
cu a he
.
co the

bie

u die

n du
.
o
.
i da
.
ng to

ho
.
.
p tuye

n tnh
cu a cac vecto
.
c`on la
.
i. Do do he
.
phu
.
thuo
.
c tuye

n tnh (v du
.
1).
V du
.
4. Ch u
.
ng minh ra
`
ng ne

u he
.
m vecto
.
a
1
, a
2
, . . . , a
m
do
.
c la
.
p
tuye

n tnh th` mo
.
i he
.
con cu a he
.
do c ung do
.
c la
.
p tuye

n tnh.
Gia i. De

cho xac di
.
nh ta xet he
.
con a
1
, a
2
, . . . , a
k
, k < m v`a ch u
.
ng
minh ra
`
ng he
.
con n`ay do
.
c la
.
p tuye

n tnh.
182 Chu
.
o
.
ng 5. Khong gian Euclide R
n
Gia su
.
ngu
.
o
.
.
c la
.
i: he
.
con a
1
, a
2
, . . . , a
k
phu
.
thuo
.
c tuye

n tnh. Khi
do ta c o cac da

ng th u
.
c vecto
.

1
a
1
+
2
a
2
+ +
k
a
k
=
trong do co t nhat mo
.
t trong cac he
.
so
1
,
2
, . . . ,
k
khac 0. Ta vie

t
da

ng th u
.
c do du
.
o
.
i da
.
ng

1
a
1
+
2
A
2
+ +
k
a
k
+
k+1
a
k+1
+ +
m
a
m
=
trong do ta gia thie

t
k+1
= 0, . . . ,
m
= 0. Da

ng th u
.
c sau c` ung n`ay
ch u
.
ng to he
.
a
1
, a
2
, . . . , a
m
phu
.
thuo
.
c tuye

n tnh. Mau thua

n.
V du
.
5. Ch u
.
ng minh ra
`
ng he
.
vecto
.
cu a khong gian R
n
e
1
= (1, 0, . . . , 0),
e
2
= (0, 1, . . . , 0),
. . . . . . . . . . . .
e
n
= (0, . . . , 0, 1)
l`a do
.
c la
.
p tuye

n tnh.
Gia i. T` u
.
da

ng th u
.
c vecto
.

1
e
1
+
2
e
2
+ +
n
e
n
=
suy ra ra
`
ng
(
1
,
2
, . . . ,
n
) = (0, 0, . . . , 0)
1
=
2
= =
n
= 0.
v`a do do he
.
e
1
, e
2
, . . . , e
n
do
.
c la
.
p tuye

n tnh.
V du
.
6. Ch u
.
ng minh ra
`
ng mo
.
i he
.
go
`
m n+1 vecto
.
cu a R
n
l`a he
.
phu
.
thuo
.
c tuye

n tnh.
Gia i. Gia su
.
n + 1 vecto
.
cu a he
.
l`a:
a
1
= (a
11
, a
21
, . . . , a
n1
)
a
2
= (a
12
, a
22
, . . . , a
n2
)
. . . . . . . . . . . .
a
n+1
= (a
1,n+1
, a
2,n+1
, . . . , a
n,n+1
).
5.1. D
-
i
.
nh ngha khong gian n-chie
`
u v`a mo
.
t so khai nie
.
m co
.
ba n ve
`
vecto
.
183
Khi do t` u
.
da

ng th u
.
c vecto
.
x
1
a
1
+x
2
a
2
+ +x
n
a
n
+x
n+1
a
n+1
=
suy ra
a
11
x
1
+a
12
x
2
+ +a
1n+1
x
n+1
= 0,
. . . . . . . . . . . . . . . . . .
a
n1
x
1
+a
n2
x
2
+ +a
nn+1
x
n+1
= 0.
_

_
Do l` a he
.
thua
`
n nhat n phu
.
o
.
ng tr`nh vo
.
i (n +1) a

n nen he
.
co nghie
.
m
khong ta
`
m thu
.
`o
.
ng v`a
(x
1
, x
2
, . . . , x
n
, x
n+1
) ,= (0, 0, . . . , 0).
Do do theo di
.
nh ngha he
.
da xet l`a phu
.
thuo
.
c tuye

n tnh.
V du
.
7. T`m ha
.
ng cu a he
.
vecto
.
trong R
4
a
1
= (1, 1, 1, 1); a
2
= (1, 2, 3, 4);
a
3
= (2, 3, 2, 3); a
4
= (2, 4, 5, 6).
Gia i. Ta la
.
p ma tra
.
n cac to
.
a do
.
v`a t`m ha
.
ng cu a no. Ta co
A =
_

_
1 1 1 1
1 2 3 4
2 3 2 3
3 4 5 6
_

_
h
2
h
1
h

2
h
3
2h
1
h

3
h
4
3h
1
h

_
1 1 1 1
0 1 2 3
0 1 0 1
0 1 2 3
_

_h
3
h
2
h

3
h
4
h
2
h

_
1 1 1 1
0 1 2 3
0 0 2 3
0 0 0 0
_

_
.
T` u
.
do suy ra
`
ng r(A) = 3. Theo di
.
nh l y da neu ha
.
ng cu a he
.
vecto
.
ba
`
ng 3.
184 Chu
.
o
.
ng 5. Khong gian Euclide R
n
V du
.
8. Kha o sat su
.
.
phu
.
thuo
.
c tuye

n tnh gi u
.
a cac vecto
.
cu a R
4
:
a
1
= (1, 4, 1, 1); a
2
= (2, 3, 1, 1);
a
3
= (1, 9, 4, 2); a
4
= (1, 6, 5, 1).
Gia i. La
.
p ma tra
.
n m`a cac h`ang cu a no l`a cac vecto
.
da cho v` a t`m
ha
.
ng cu a no
S =
_

_
1 4 1 1
2 3 1 1
1 9 4 2
1 6 5 1
_

_
r(A) = 2.
Do do ha
.
ng cu a he
.
vecto
.
ba
`
ng 2. V` cac pha
`
n tu
.
cu a di
.
nh th u
.
c con
=

1 4
2 3

= 5 ,= 0
na
`
m o
.
hai h`ang da
`
u nen a
1
v`a a
2
do
.
c la
.
p tuye

n tnh, c` on a
3
v`a a
4
bie

u
die

n tuye

n tnh qua a
1
v`a a
2
. [Lu
.
u y ra
`
ng mo
.
i ca
.
p vecto
.
cu a he
.
de
`
u
do
.
c la
.
p tuye

n tnh v` ta c o cac di
.
nh th u
.
c con c ap hai sau day ,= 0:

1 4
1 9

1 4
1 6

2 3
1 9

2 3
1 6

1 9
1 6

.]
Ta t`m cac bie

u th u
.
c bie

u die

n a
3
v`a a
4
qua a
1
v`a a
2
.
Ta vie

t
a
3
=
1
a
1
+
2
a
2
hay l`a
(1, 9, 4, 2) =
1
(1, 4, 1, 1) +
2
(2, 3, 1, 1)
(1, 9, 4, 2) = (
1
+ 2
2
, 4
1
+ 3
2
,
1

2
,
1
+
2
)
5.1. D
-
i
.
nh ngha khong gian n-chie
`
u v`a mo
.
t so khai nie
.
m co
.
ba n ve
`
vecto
.
185
v`a thu du
.
o
.
.
c he
.
phu
.
o
.
ng tr`nh

1
+ 2
2
= 1,
4
1
+ 3
2
= 9,

2
= 4,

1
+
2
= 2.
_

_
Ta ha
.
n che

hai phu
.
o
.
ng tr`nh da
`
u. Di
.
nh th u
.
c cu a cac he
.
so cu a hai
phu
.
o
.
ng tr`nh n`ay chnh l`a di
.
nh th u
.
c chuye

n vi
.
. V` ,= 0 nen he
.
hai phu
.
o
.
ng tr`nh

1
+ 2
2
= 1
4
1
+ 3
2
= 9
co nghie
.
m duy nhat l`a
1
= 3,
2
= 1. Do do
a
3
= 3a
1
a
2
.
Tu
.
o
.
ng tu
.
.
ta c o
a
4
= 2a
2
3a
1
.
B
`
AI T

A
.
P
1. Ch u
.
ng minh ra
`
ng trong khong gian R
3
:
1) Vecto
.
(x, y, z) l`a to

ho
.
.
p tuye

n tnh cu a cac vecto


.
e
1
= (1, 0, 0),
e
2
= (0, 1, 0), e
3
= (0, 0, 1).
2) Vecto
.
x = (7, 2, 6) l`a to

ho
.
.
p tuye

n tnh cu a cac vecto


.
a
1
=
(3, 1, 2), a
2
= (5, 2, 3), a
3
= (1, 1, 1).
2. Hay xac di
.
nh so de

vecto
.
x R
3
l`a to

ho
.
.
p tuye

n tnh cu a cac
vecto
.
a
1
, a
2
, a
3
R
3
ne

u:
1) x = (1, 3, 5); a
1
= (3, 2, 5); a
2
= (2, 4, 7); a
3
= (5, 6, ).
186 Chu
.
o
.
ng 5. Khong gian Euclide R
n
(DS. ,= 12)
2) x = (7, 2, ); a
1
= (2, 3, 5); a
2
= (3, 7, 8); a
3
= (1, 6, 1).
(DS. = 15)
3) x = (5, 9, ); a
1
= (4, 4, 3); a
2
= (7, 2, 1); a
3
= (4, 1, 6).
(DS. R)
3. Ch u
.
ng minh ra
`
ng trong khong gian R
3
:
1) He
.
ba vecto
.
e
1
= (1, 0, 0), e
2
= (0, 1, 0), e
3
= (0, 0, 1) l`a he
.
dltt.
2) Ne

u them vecto
.
x R
3
bat k` y v`ao he
.
th` he
.
e
1
, e
2
, e
3
, x
l`a phu
.
thuo
.
c tuye

n tnh.
3) He
.
go
`
m bon vecto
.
bat k` y cu a R
3
l`a pttt.
4. Cac he
.
vecto
.
sau day trong khong gian R
3
l`a dltt hay pttt:
1) a
1
= (1, 2, 1); a
2
= (0, 1, 2); a
3
= (0, 0, 2). (DS. Dltt)
2) a
1
= (1, 1, 0); a
2
= (1, 0, 1); a
3
= (1, 2, 0). (DS. Dltt)
3) a
1
= (1, 3, 3); a
2
= (1, 1, 1); a
3
= (2, 4, 4). (DS. Pttt)
4) a
1
= 1, 3, 0); a
2
= (3, 3, 1); a
3
= (2, 0, 1). (DS. Pttt)
5) a
1
= (2, 3, 1); a
2
= (1, 1, 1); a
3
= (1, 2, 0). (DS. Pttt)
5. Gia su
.
v
1
, v
2
v`a v
3
l`a he
.
do
.
c la
.
p tuye

n tnh. Ch u
.
ng minh ra
`
ng he
.
sau day c ung l`a dltt:
1) a
1
= v
1
+v
2
; a
2
= v
1
+v
3
; a
3
= v
1
2v
2
.
2) a
1
= v
1
+v
3
; a
2
= v
3
v
1
; a
3
= v
1
+v
2
v
3
.
6. Ch u
.
ng minh ra
`
ng cac he
.
vecto
.
sau day l`a phu
.
thuo
.
c tuye

n tnh.
Doi vo
.
i he
.
vecto
.
n`ao th` vecto
.
b l`a to

ho
.
.
p tuye

n tnh cu a cac vecto


.
c`on la
.
i ?
1) a
1
= (2, 0, 1), a
2
= (3, 0, 2), a
3
= (1, 0, 1), b = (1, 2, 0).
(DS. b khong l`a to

ho
.
.
p tuye

n tnh)
2) a
1
= (2, 0, 1), a
2
= (1, 1, 0), a
3
= (0, 1, 2); b = (2, 3, 6).
(DS. b l`a to

ho
.
.
p tuye

n tnh)
5.1. D
-
i
.
nh ngha khong gian n-chie
`
u v`a mo
.
t so khai nie
.
m co
.
ba n ve
`
vecto
.
187
7. T`m so cu
.
.
c da
.
i cac vecto
.
dltt trong c ac he
.
vecto
.
sau day
1) a
1
= (2, 3, 1, 4); a
2
= (1, 1, 2, 0); a
3
= (0, 0, 1, 1);
a
4
= (1, 4, 1, 4); a
5
= (2, 3, 0, 5). (DS. = 3)
2) a
1
= (1, 0, 0, 0); a
2
= (0, 1, 0, 0); a
3
= (0, 0, 1, 0)
a
4
= (0, 0, 0, 1); a
5
= (1, 2, 3, 4). (DS. = 4)
3) a
1
= (1, 1, 1, 1); a
2
= (1, 1, 1, 0); a
3
= (1, 1, 0, 0);
a
4
= (1, 0, 0, 0); a
5
= (1, 2, 3, 4). (DS. = 4)
Ch da

n. La
.
p ma tra
.
n cac to
.
a do
.
m`a mo

i co
.
t cu a no l`a to
.
a do
.
cu a
vecto
.
cu a he
.
ro
`
i tnh ha
.
ng cu a ma tra
.
n.
8. Cac he
.
vecto
.
sau day trong khong gian R
4
l`a dltt hay pttt
1) a
1
= (1, 2, 3, 4), a
2
= (1, 2, 3, 4). (DS. Pttt)
2) a
1
= (1, 2, 3, 4), a
2
= (1, 2, 3, 4). (DS. Pttt)
3) a
1
= (1, 2, 3, 4), a
2
= (3, 6, 9, 12). (DS. Pttt)
4) a
1
= (1, 2, 3, 4), (a
2
= (1, 2, 3, 5). (DS. Dltt)
5) a
1
= (1, 0, 0, 0), a
2
= (0, 1, 0, 0), a
3
= (0, 0, 1, 0), a
4
= (0, 0, 0, 1)
v`a a l`a vecto
.
t` uy y cu a R
4
. (DS. Pttt)
6) a
1
= (1, 1, 1, 1), a
2
= (0, 1, 1, 1), a
3
= (0, 0, 1, 1), a
4
= (0, 0, 0, 1).
(DS. Dltt)
7) a
1
= (1, 2, 3, 4), a
2
= (3, 6, 9, 12), a
3
= (1, 2, 3, 6). (DS. Pttt)
9. Cac he
.
vecto
.
sau day dltt hay pttt. Trong tru
.
`o
.
ng ho
.
.
p pttt hay ch
ra mo
.
t su
.
.
pttt. Hay ch ra mo
.
t he
.
con cu
.
.
c da
.
i n`ao do l`a dltt.
1) a
1
= (2, 1, 2, 1), a
2
= (9, 5, 6, 21), a
3
= (2, 5, 1, 3),
a
4
= (1, 1, 1, 5), a
5
= (1, 2, 3, 4).
(DS. a
1
+a
2
+a
3
3a
4
2a
5
= ; a
1
, a
2
, a
3
, a
4
)
2) a
1
= (1, 1, 1, 1), a
2
= (2, 0, 1, 1), a
3
= (3, 4, 0, 1),
a
4
= (13, 10, 3, 2). (DS. 2a
1
+a
2
+ 3a
3
a
4
= ; a
1
, a
2
, a
3
)
3) a
1
= (1, 1, 1, 1), a
2
= (2, 0, 1, 1), a
3
= (3, 1, 1, 1),
a
4
= (4, 2, 1, 2). (DS. He
.
do
.
c la
.
p tuye

n tnh)
4) a
1
= (1, 2, 2, 1), a
2
= (1, 0, 2, 1), a
3
= (0, 1, 0, 1),
a
4
= (3, 6, 0, 4). (DS. He
.
do
.
c la
.
p tuye

n tnh)
188 Chu
.
o
.
ng 5. Khong gian Euclide R
n
10. Tnh ha
.
ng r cu a he
.
vecto
.
v`a ch ro he
.
da cho l`a pttt hay dltt:
1) a
1
= (1, 2, 2, 8, 2), a
2
= (1, 2, 1, 5, 3), a
3
= (1, 2, 4, 7, 0).
(DS. r = 3, he
.
do
.
c la
.
p tuye

n tnh)
2) a
1
= (2, 3, 1, 1), a
2
= (3, 1, 4, 2), a
3
= (1, 2, 3, 1),
a
4
= (1, 4, 7, 5). (DS. r = 3, he
.
pttt)
3) a
1
= (2, 1, 3, 2, 6), a
2
= (1, 5, 2, 3, 4), a
3
= (3, 4, 1, 5, 7),
a
4
= (3, 7, 4, 1, 7), a
5
= (0, 11, 5, 4, 4). (DS. r = 3 he
.
pttt)
4) a
1
= (2, 1, 4, 4, 17), a
2
= (0, 0, 5, 7, 9), a
3
=
(2, 1, 6, 10, 11),
a
4
= (8, 4, 1, 5, 11), a
5
= (2, 2, 9, 11, 10). (DS. r = 5, he
.
dltt)
5.2 Co
.
so
.

. D
-
o

i co
.
so
.

D
-
i
.
nh ngha 5.2.1. He
.
vecto
.
E
1
, E
2
, . . . , E
n
go
`
m n vecto
.
cu a khong
gian vecto
.
R
n
du
.
o
.
.
c go
.
i l`a mo
.
t co
.
so
.
cu a no ne

u
1) he
.
E
1
, E
2
, . . . , E
n
l`a he
.
dltt;
2) mo
.
i vecto
.
x R
n
de
`
u bie

u die

n tuye

n tnh du
.
o
.
.
c qua c ac vecto
.
cu a he
.
E
1
, . . . , E
n
.
Ch u y ra
`
ng co
.
so
.
cu a R
n
l`a mo
.
t he
.
c o th u
.
tu
.
.
bat k` y go
`
m n vecto
.
do
.
c la
.
p tuye

n tnh cu a no.
Die
`
u kie
.
n 2) c o ngha ra
`
ng x R
n
, (x
1
, x
2
, . . . , x
n
) sao cho
x = x
1
E
1
+x
2
E
2
+ +x
n
E
n
, (5.5)
trong do x
1
, x
2
, . . . , x
n
l`a to
.
a do
.
cu a vecto
.
x trong co
.
so
.
E
1
, E
2
, . . . , E
n
v`a (5.5) go
.
i l`a khai trie

n vecto
.
x theo co
.
so
.
E
1
, E
2
, . . . , E
n
.

Y ngha co
.
ba n cu a khai nie
.
m co
.
so
.
l`a: cac phep toan tuye

n tnh
tren c ac vecto
.
trong co
.
so
.
cho tru
.
o
.
c chuye

n th`anh cac phep toan tren


cac so l`a to
.
a do
.
cu a ch ung.
D
-
i
.
nh l y 5.2.1. Trong kh ong gian R
n
:
1) To
.
a do
.
cu a mo
.
t vecto
.
d oi vo
.
i mo
.
t co
.
so
.
l` a duy nhat.
5.2. Co
.
so
.
. D
-
o

i co
.
so
.
189
2) Mo
.
i he
.
dltt go
`
m n vecto
.
de
`
u l a
.
p th`anh co
.
so
.
cu a khong gian
R
n
.
Ta xet v an de
`
: Khi co
.
so
.
thay do

i th` to
.
a do
.
cu a mo
.
t vecto
.
trong
khong gian R
n
thay do

i the

n`ao ?
Gia su
.
trong khong gian R
n
co hai co
.
so
.

c :c
1
, c
2
, . . . , c
n
- co
.
so
.
c u (5.6)
E :E
1
, E
2
, . . . , E
n
- co
.
so
.
mo
.
i (5.7)
V` E
1
, E
2
, . . . , E
n
R
n
nen
E
1
= t
11

1
+t
21

2
+ +t
n1

n
,
E
2
= t
12

1
+t
22

2
+ +t
n2

n
,
. . . . . . . . . . . . . . .
E
n
= t
1n

1
+t
2n

2
+ +t
nn

n
.
_

_
(5.8)
Co the

noi ra
`
ng co
.
so
.
E
1
, . . . , E
n
thu du
.
o
.
.
c t` u
.
co
.
so
.
c
1
, c
2
, . . . , c
n
nh`o
.
ma tra
.
n
T
EE
=
_

_
t
11
t
12
. . . t
1n
t
21
t
22
. . . t
2n
.
.
.
.
.
.
.
.
.
.
.
.
t
n1
t
n2
. . . t
nn
_

_
(5.9)
trong do co
.
t th u
.
i cu a ma tra
.
n (5.9) chnh l`a cac to
.
a do
.
cu a vecto
.
E
i
trong co
.
so
.
(5.6).
Ma tra
.
n T = T
EE
trong (5.9) du
.
o
.
.
c go
.
i l`a ma tr a
.
n chuye

n t` u
.
co
.
so
.
(5.6) de

n co
.
so
.
(5.7). Di
.
nh th u
.
c cu a ma tra
.
n chuye

n detT ,= 0
v` trong tru
.
`o
.
ng ho
.
.
p ngu
.
o
.
.
c la
.
i th` cac vecto
.
co
.
t (v`a do do cac vecto
.
E
1
, . . . , E
n
) l`a phu
.
thuo
.
c tuye

n tnh.
Nhu
.
va
.
y de

t`m ma tra
.
n chuye

n t` u
.
co
.
so
.
c u sang co
.
so
.
mo
.
i da
`
u
tien ta c a
`
n khai trie

n cac vecto
.
cu a co
.
so
.
mo
.
i theo co
.
so
.
c u. Tie

p do
ta la
.
p ma tra
.
n m`a co
.
t th u
.
i cu a no l`a cac to
.
a do
.
cu a vecto
.
th u
.
i cu a
co
.
so
.
mo
.
i trong co
.
so
.
c u. Do chnh l`a ma tra
.
n chuye

n.
190 Chu
.
o
.
ng 5. Khong gian Euclide R
n
Gia su
.
vecto
.
a R
n
v`a
a = x
1

1
+x
2

2
+ +x
n

n
,
a = y
1
E
1
+y
2
E
2
+ +y
n
E
n
.
Khi do quan he
.
gi u
.
a cac to
.
a do
.
cu a c` ung mo
.
t vecto
.
doi vo
.
i hai co
.
so
.

khac nhau (5.6) v` a (5.7) du


.
o
.
.
c mo ta nhu
.
sau
x
1
= t
11
y
1
+t
12
y
2
+ +t
1n
y
n
,
x
2
= t
21
y
1
+t
22
y
2
+ +t
2n
y
n
,
. . . . . . . . . . . . . . .
x
n
= t
n1
y
1
+t
n2
y
2
+ +t
nn
y
n
.
_

_
(5.10)
hay l`a
X = T
EE
Y, (5.11)
X =
_

_
x
1
x
2
.
.
.
x
n
_

_
, Y =
_

_
y
1
y
2
.
.
.
y
n
_

_
T` u
.
do c ung suy ra
Y = T
1
EE
X. (5.11*)
C

AC V

I DU
.
V du
.
1. Trong khong gian R
3
he
.
cac vecto
.
c
1
(1, 0, 0), c
2
(0, 2, 0),
c
3
(0, 0, 3) l`a co
.
so
.
cu a no.
Gia i. 1) He
.
vecto
.
c
1
, c
2
, c
3
l`a he
.
do
.
c la
.
p tuye

n tnh. Th a
.
t va
.
y,
da

ng th u
.
c vecto
.

1
c
1
+
2
c
2
+
3
c
3
= (0, 0, 0)

1
(1, 0, 0) +
2
(0, 2, 0) +
3
(0, 0, 3) = (0, 0, 0)
(
1
, 2
2
, 3
3
) = (0, 0, 0)

1
=
2
=
3
= 0.
5.2. Co
.
so
.
. D
-
o

i co
.
so
.
191
2) Gia su
.
x R
3
, x = (
1
,
2
,
3
). Khi do
x =
1
(1, 0, 0) +

2
2
(0, 2, 0) +

3
3
(0, 0, 3)
=
1
c
1
+

2
2
c
2
+

3
3
c
3
t u
.
c l` a x l`a to

ho
.
.
p tuye

n tnh cu a c
1
, c
2
, c
3
.
V du
.
2. Ch u
.
ng minh ra
`
ng trong khong gian R
3
cac vecto
.
c
1
=
(2, 1, 1), c
2
= (1, 3, 1), c
3
= (2, 1, 3) la
.
p th`anh mo
.
t co
.
so
.
. T`m to
.
a
do
.
cu a vecto
.
x = (2, 4, 2) theo co
.
so
.
do.
Gia i. 1) He
.
c
1
, c
2
, c
3
l`a dltt. Tha
.
t va
.
y gia su
.

1
c
1
+
2
c
2
+
3
c
3
=

2
1
+
2
2
3
= 0,

1
+ 3
2
+
3
= 0,

1
+
2
+ 3
3
= 0.
_

_
He
.
n`ay co detA ,= 0 v`a l`a he
.
thua
`
n nhat nen no ch co nghie
.
m ta
`
m
thu
.
`o
.
ng
1
=
2
=
3
= 0 v`a do do c
1
, c
2
, c
3
do
.
c la
.
p tuye

n tnh. Theo
di
.
nh l y 1 (pha
`
n 2) c ac vecto
.
n`ay la
.
p th`anh co
.
so
.
cu a R
3
.
2) De

khai trie

n vecto
.
x = (2, 4, 2) theo co
.
so
.
c
1
, c
2
, c
3
ta da
.
t
x =
1
c
1
+
2
c
2
+
3
c
3
v`a t` u
.
do
2
1
+
2
2
3
= 2,

1
+ 3
2
+
3
= 4,

1
+
2
+ 3
3
= 2.
_

_
He
.
n`ay co nghie
.
m l`a
1
= 1,
2
= 2,
3
= 1. Va
.
y trong co
.
so
.

c
1
, c
2
, c
3
vecto
.
x co to
.
a do
.
l`a (1, 2, 1).
V du
.
3. Ch u
.
ng minh ra
`
ng ba vecto
.
c
1
= (1, 0, 2), c
2
= (4, 1, 5),
c
3
= (1, 3, 4) la
.
p th`anh co
.
so
.
cu a R
3
.
192 Chu
.
o
.
ng 5. Khong gian Euclide R
n
Gia i. Ta co the

t`m ha
.
ng cu a he
.
ba vecto
.
da cho. Ta co
_

_
1 0 2
4 1 5
1 3 4
_

_
1 0 2
0 1 13
0 3 6
_

_
1 0 2
0 1 13
0 0 45
_

_
.
T` u
.
do suy ra ra
`
ng ha
.
ng cu a he
.
vecto
.
da cho ba
`
ng 3 v`a do va
.
y he
.
do
l`a do
.
c la
.
p tuye

n tnh. Theo di
.
nh l y 1 no la
.
p th`anh mo
.
t co
.
so
.
.
V du
.
4. Gia su
.
trong co
.
so
.
c
1
, c
2
vecto
.
x co to
.
a do
.
l`a 1; 2. T`m
to
.
a do
.
cu a vecto
.
do trong co
.
so
.
E
1
= c
1
, E
2
= c
1
+c
2
.
Gia i. Da
`
u tien ta vie

t ma tra
.
n chuye

n t` u
.
co
.
so
.
c
1
, c
2
de

n E
1
, E
2
.
Ta co
E
1
= 1 e
1
+ 0 e
2
,
E
2
= 1 e
1
+ 1 e
2
.
Do do
T =
_
1 1
0 21
_
T
1
=
_
1 1
0 1
_
.

Ap du
.
ng cong th u
.
c (11*) ta co
_
y
1
y
2
_
= T
1
_
x
1
x
2
_
=
_
1 1
0 1
__
1
2
_
=
_
3
2
_
.
Do do y
1
= 3, y
2
= 2.
V du
.
5 (phep quay tru
.
c to
.
a do
.
). Hay da

n ra c ong th u
.
c bie

n do

i cac
to
.
a do
.
cu a vecto
.
trong R
2
trong mo
.
t co
.
so
.
thu du
.
o
.
.
c t` u
.
co
.
so
.
chnh
ta

c e
1
= (1, 0), e
2
= (0, 1) sau phep quay tru
.
c to
.
a do
.
goc .
5.2. Co
.
so
.
. D
-
o

i co
.
so
.
193
H`nh 5.1
Gia i. T` u
.
h`nh ve suy ra ra
`
ng vecto
.
e

1
la
.
p vo
.
i cac vecto
.
e
1
v`a e
2
cac goc tu
.
o
.
ng u
.
ng ba
`
ng v`a

2
. Do do to
.
a do
.
cu a e

1
trong co
.
so
.

e
1
, e
2
l`a cos v`a cos
_


2
_
= sin:
e

1
= cos e
1
+ sin e
2
Vecto
.
e

2
la
.
p vo
.
i e
1
v`a e
2
cac goc tu
.
o
.
ng u
.
ng ba
`
ng

2
+ v`a . Do do
to
.
a do
.
cu a no trong co
.
so
.
e
1
, e
2
l`a cos
_

2
+
_
= sin v`a cos :
e

2
= sin e
1
+ cos e
2
.
Nhu
.
va
.
y
e

1
= cos e
1
+ sin e
2
,
e

2
= sin e
1
+ cos e
2
.
v`a t` u
.
do
T
ee
=
_
cos sin
sin cos
_
T
1
ee
=
_
cos sin
sin cos
_
.
Do va
.
y cac to
.
a do
.
cu a vecto
.
trong co
.
so
.
c u v`a mo
.
i lien he
.
bo
.
i cac he
.
th u
.
c
x = x

cos y

sin,
y = x

sin +y

cos .
_
x

= xcos +y sin,
y

= xsin +y cos .
_

194 Chu
.
o
.
ng 5. Khong gian Euclide R
n
V du
.
6. Gia su
.
x = (3, 1, 0) l`a vecto
.
cu a R
3
vo
.
i co
.
so
.
c
1
, c
2
, c
3
.
T`m to
.
a do
.
cu a x doi vo
.
i co
.
so
.

E
1
= 2c
1
c
2
+ 3c
3
,
E
2
= c
1
+c
3
,
E
3
= c
2
+ 2c
3
.
Gia i. T` u
.
cac khai trie

n E
1
, E
2
v`a E
3
theo co
.
so
.
c
1
, c
2
, c
3
ta co ma
tra
.
n chuye

n
T =
_

_
2 1 0
1 0 1
3 1 2
_

_
t` u
.
co
.
so
.
c
1
, c
2
, c
3
sang co
.
so
.
E
1
, E
2
, E
3
.
Ta k y hie
.
u x
1
, x
2
, x
3
l`a to
.
a do
.
cu a x trong co
.
so
.
E
1
, E
2
, E
3
. Ta co
_

_
x
1
x
2
x
3
_

_
= T
1
_

_
3
1
0
_

_
V` T
1
=
_

_
1 2 1
1 4 2
1 1 1
_

_ nen
_

_
x
1
x
2
x
3
_

_ =
_

_
1 2 1
1 4 2
1 1 1
_

_
_

_
3
1
0
_

_ =
_

_
5
7
4
_

_.
Va
.
y trong co
.
so
.
mo
.
i E
1
, E
2
, E
3
ta co
x = (5, 7, 4).
V du
.
7. Trong khong gian R
2
cho co
.
so
.
c
1
, c
2
v`a cac vecto
.
E
1
=
e
1
2e
2
, E
2
= 2e
1
+e
2
, x = 3e
1
4e
2
.
5.2. Co
.
so
.
. D
-
o

i co
.
so
.
195
1
+
Ch u
.
ng minh ra
`
ng E
1
, E
2
la
.
p th`anh co
.
so
.
cu a R
2
.
2
+
T`m to
.
a do
.
vecto
.
x trong co
.
so
.
E
1
, E
2
.
3
+
T`m to
.
a do
.
cu a vecto
.
x trong co
.
so
.
E
2
, E
1
.
Gia i. 1
+
Ta la
.
p ma tra
.
n cac to
.
a do
.
cu a E
1
v`a E
2
:
A =
_
1 2
2 1
_
detA = 5 ,= 0.
Do do he
.
hai vecto
.
E
1
, E
2
l`a dltt trong khong gian 2-chie
`
u R
2
nen no
la
.
p th`anh co
.
so
.
.
2
+
Trong co
.
so
.
da cho vecto
.
x co to
.
a do
.
l`a (3, 4). Gia su
.
trong
co
.
so
.
E
1
, E
2
vecto
.
x co to
.
a do
.
(x
1
, x
2
). Ta la
.
p ma tra
.
n chuye

n t` u
.
co
.
so
.
c
1
, c
2
de

n co
.
so
.
E
1
, E
2
:
T =
_
1 2
2 1
_
T
1
=
1
5
_
1 2
2 1
_
Khi do
_
x
1
x
2
_
= T
1
_
3
4
_

_
x
1
x
2
_
=
1
5
_
1 2
2 1
__
3
4
_
=
1
5
_
11
2
_
=
_

_
11
5
2
5
_

_.
Va
.
y x
1
=
11
5
, x
2
=
+2
5
.
3
+
V` E
1
, E
2
l`a co
.
so
.
cu a R
2
nen E
2
, E
1
c ung l`a co
.
so
.
cu a R
2
. Ma
tra
.
n chuye

n t` u
.
co
.
so
.
c
1
, c
2
de

n co
.
so
.
E
2
, E
1
co da
.
ng
A

=
_
2 1
1 2
_
, A
1
=
1
5
_
2 1
1 2
__
3
4
_
=
1
5
_
2
11
_
=
_

_
2
5
11
5
_

_
Do do x
1
=
2
5
, x
2
=
11
5
trong co
.
so
.
E
2
, E
1
.
V du
.
8. Trong khong gian R
3
cho co
.
so
.
c
1
, c
2
, c
3
n`ao do v`a trong
co
.
so
.
do cac vecto
.
E
1
, E
2
, E
3
v`a x co to
.
a do
.
l`a E
1
= (1, 1, 1); E
2
=
(1, 2, 2), E
3
= (1, 1, 3) v`a x = (6, 9, 14).
196 Chu
.
o
.
ng 5. Khong gian Euclide R
n
1
+
Ch u
.
ng minh ra
`
ng E
1
, E
2
, E
3
c ung la
.
p th`anh co
.
so
.
trong R
3
.
2
+
T`m to
.
a do
.
cu a x trong co
.
so
.
E
1
, E
2
, E
3
.
Gia i. 1
+
tu
.
o
.
ng tu
.
.
nhu
.
trong v du
.
7, ha
.
ng cu a he
.
ba vecto
.
E
1
, E
2
, E
3
ba
`
ng 3 nen he
.
vecto
.
do do
.
c la
.
p tuye

n tnh trong khong


gian 3-chie
`
u nen no la
.
p th`anh co
.
so
.
cu a R
3
.
2+ De

t`m to
.
a do
.
cu a x trong co
.
so
.
E
1
, E
2
, E
3
ta co the

tie

n h`anh
theo hai phu
.
o
.
ng phap sau.
(I) V` E
1
, E
2
, E
3
la
.
p th`anh co
.
so
.
cu a R
3
nen
x = x
1
E
1
+x
2
E
2
+x
3
E
3
(6, 9, 14) = x
1
(1, 1, 1) +x
2
(1, 2, 2) +x
3
(1, 1, 3)
v`a do do x
1
, x
2
, x
3
l`a nghie
.
m cu a he
.
x
1
+x
2
+x
3
= 6,
x
1
+ 2x +x
3
= 9,
x
1
+ 2x
2
+ 3x
3
= 14.
_

_
x
1
=
1
2
, x
2
= 3, x
3
=
5
2

(II) La
.
p ma tra
.
n chuye

n t` u
.
co
.
so
.
c
1
, c
2
, c
3
sang co
.
so
.
E
1
, E
2
, E
3
:
T
EE
=
_

_
1 1 1
1 2 1
1 2 3
_

_ T
1
EE
=
1
2
_

_
4 1 1
2 2 0
0 1 1
_

_.
Do do
_

_
x
1
x
2
x
3
_

_ = T
1
EE
_

_
6
9
14
_

_ =
1
2
_

_
1
6
5
_

_ =
_

_
1
2
3
5
2
_

_
v`a thu du
.
o
.
.
c ke

t qua nhu
.
tronng (I).
B
`
AI T

A
.
P
5.2. Co
.
so
.
. D
-
o

i co
.
so
.
197
1. Ch u
.
ng minh ra
`
ng cac he
.
vecto
.
sau day l`a nh u
.
ng co
.
so
.
trong kh ong
gian R
4
:
1) e
1
= (1, 0, 0, 0); e
2
= (0, 1, 0, 0); e
3
= (0, 0, 1, 0); e
4
= (0, 0, 0, 1).
2) c
1
= (1, 1, 1, 1); c
2
= (0, 1, 1, 1); c
3
= (0, 0, 1, 1); c
4
= (0, 0, 0, 1).
2. Ch u
.
ng minh ra
`
ng he
.
vecto
.
do
.
n vi
.
:
e
1
= (1, 0, . . . , 0
. .
n1
); e
2
= (0, 1, 0, . . . , 0), . . . , e
n
= (0, 0, . . . , 0
. .
n1
, 1)
la
.
p th`anh co
.
so
.
trong R
n
. Co
.
so
.
n`ay du
.
o
.
.
c go
.
i l`a co
.
so
.
chnh ta

c.
3. Ch u
.
ng minh ra
`
ng he
.
vecto
.
c
1
= (1, 0, . . . , 0),
c
2
= (1, 1, . . . , 0),
. . . . . . . . .
c
n
= (1, 1, . . . , 1)
l`a mo
.
t co
.
so
.
trong R
n
.
4. Ch u
.
ng minh ra
`
ng he
.
vecto
.
c
1
= (1, 2, 3, . . . , n 1, n),
c
2
= (1, 2, 3, . . . , n 1, 0),
. . . . . . . . . . . . . . .
c
n
= (1, 0, 0, . . . , 0, 0)
la
.
p th`anh co
.
so
.
trong khong gian R
n
.
5. Hay kie

m tra xem mo

i he
.
vecto
.
sau day co la
.
p th`anh co
.
so
.
trong
khong gian R
4
khong v`a t`m cac to
.
a do
.
cu a vecto
.
x = (1, 2, 3, 4) trong
mo

i co
.
so
.
do.
1) a
1
= (0, 1, 0, 1); a
2
= (0, 1, 0, 1); a
3
= (1, 0, 1, 0);
a
4
= (1, 0, 1, 0). (DS. 3, 1, 2, 1)
2) a
1
= (1, 2, 3, 0); a
2
= (1, 2, 0, 3); a
3
= (1, 0, 2, 3);
198 Chu
.
o
.
ng 5. Khong gian Euclide R
n
a
4
= (0, 1, 2, 3). (DS.
2
3
,
1
6
,
1
2
, 1)
3) a
1
= (1, 1, 1, 1); a
2
= (1, 1, 1, 1); a
3
= (1, 1, 1, 1);
a
4
= (1, 1, 1, 1). (DS.
3
2
,
1
2
, 1, 1)
4) a
1
= (1, 2, 3, 4); a
2
= (4, 1, 2, 3); a
3
= (3, 4, 1, 2);
a
4
= (2, 3, 4, 1). (DS.
13
10
,
7
10
,
13
10
,
17
10
)
Nha
.
n xet. Ta nha

c la
.
i ra
`
ng cac k y hie
.
u e
1
, e
2
, . . . , e
n
du
.
o
.
.
c d` ung de

ch cac vecto
.
do
.
n vi
.
cu a tru
.
c x
i
(i = 1, 2, . . . , n):
e
i
= (1, 0, . . . , 0
. .
n1
), e
2
= (0, 1, 0, . . . , 0), . . . , e
n
= (0, . . . , 0
. .
n1
, 1)
6. T`m ma tra
.
n chuye

n t` u
.
co
.
so
.
e
1
, e
2
, e
3
de

n co
.
so
.
e
2
, e
3
, e
1
.
(DS.
_

_
0 0 1
1 0 0
0 1 0
_

_
)
7. T`m ma tra
.
n chuye

n t` u
.
co
.
so
.
e
1
, e
2
, e
3
, e
4
de

n co
.
so
.
e
3
, e
4
, e
2
, e
1
.
(DS.
_

_
0 0 0 1
0 0 1 0
1 0 0 0
0 1 0 0
_

_
)
8. Cho ma tra
.
n
_
1 1
2 0
_
l`a ma tra
.
n chuye

n t` u
.
co
.
so
.
e
1
, e
2
de

n co
.
so
.

c
1
, c
2
. T`m to
.
a do
.
cu a vecto
.
c
1
, c
2
.
(DS. c
1
= (1, 2); c
2
= (1, 0))
9. Gia su
.

_
1 2 1
3 1 0
2 0 1
_

_
5.2. Co
.
so
.
. D
-
o

i co
.
so
.
199
l`a ma tra
.
n chuye

n t` u
.
co
.
so
.
e
1
, e
2
, e
3
de

n co
.
so
.
c
1
, c
2
, c
3
. T`m to
.
a do
.
cu a vecto
.
c
2
trong co
.
so
.
e
1
, e
2
, e
3
. (DS. c
2
= (2, 1, 0))
10. T`m ma tra
.
n chuye

n t` u
.
co
.
so
.
e
1
, e
2
, e
3
de

n co
.
so
.

c
1
= 2e
1
e
3
+e
2
; c
2
= 3e
1
e
2
+e
3
; c
3
= e
3
.
(DS.
_

_
2 3 0
1 1 0
1 1 1
_

_
)
11. T`m ma tra
.
n chuye

n t` u
.
co
.
so
.
e
1
, e
2
, e
3
de

n co
.
so
.

c
1
= e
2
+e
3
; c
2
= e
1
+ 2e
3
; c
3
= e
1
+e
2
.
(DS.
_

_
0 1 1
1 0 1
1 2 0
_

_
)
12. T`m ma tra
.
n chuye

n t` u
.
co
.
so
.
e
1
, e
2
, e
3
, e
4
de

n co
.
so
.

c
1
= 2e
2
+ 3e
3
+e
4
; c
2
= e
1
2e
2
+ 3e
3
e
4
; c
3
= e
1
+e
4
;
c
4
= 2e
1
+e
2
e
3
+e
4
.
(DS.
_

_
0 1 1 2
2 2 0 1
3 3 0 1
1 1 1 1
_

_
)
13. Cho
_
2 1
1 2
_
l`a ma tra
.
n chuye

n t` u
.
co
.
so
.
e
1
, e
2
de

n co
.
so
.
c
1
, c
2
. T`m to
.
a do
.
cu a cac
vecto
.
e
1
, e
2
trong co
.
so
.
c
1
, c
2
.
(DS. e
1
=
_
2
5
,
1
5
_
. e
2
=
_

1
5
,
2
5
_
)
Ch da

n. T` u
.
ma tra
.
n da cho t`m khai trie

n c
1
, c
2
theo co
.
so
.
e
1
, e
2
.
T` u
.
do t`m khai trie

n e
1
, e
2
theo co
.
so
.
c
1
, c
2
.
200 Chu
.
o
.
ng 5. Khong gian Euclide R
n
14. Cho ma tra
.
n
_

_
1 1 3
5 1 2
1 4 1
_

_
l`a ma tra
.
n chuye

n t` u
.
co
.
so
.
e
1
, e
2
, e
3
de

n co
.
so
.
c
1
, c
2
, c
3
. T`m to
.
a do
.
vecto
.
e
2
trong co
.
so
.
c
1
, c
2
, c
3
.
(DS. e
2
=
_
11
41
,
4
41
,
5
41
_
)
15. Cho ma tra
.
n
_

_
1 0 1
0 0 2
1 3 1
_

_
l`a ma tra
.
n chuye

n t` u
.
co
.
so
.
e
1
, e
2
, e
3
de

n co
.
so
.
c
1
, c
2
, c
3
. T`m to
.
a do
.
cac vecto
.
e
1
, e
2
, e
3
trong co
.
so
.
c
1
, c
2
, c
3
.
(DS. e
1
=
_
1,
1
3
, 0
_
, e
2
=
_

1
2
,
1
3
,
1
2
_
, e
3
=
_
0,
1
3
, 0
_
)
16. Trong co
.
so
.
e
1
, e
2
vecto
.
x co to
.
a do
.
l`a (1; 2). T`m to
.
a do
.
cu a
vecto
.
do trong co
.
so
.
c
1
= e
1
+ 2e
2
; c
2
= e
1
+e
2
.
(DS. x =
_

1
3
,
4
3
_
)
17. Trong co
.
so
.
e
1
, e
2
vecto
.
x co to
.
a do
.
l`a (3; 1). T`m to
.
a do
.
cu a
vecto
.
do trong co
.
so
.
c
1
= 2e
1
+e
2
; c
2
= e
2
.
(DS. x =
_
3
2
,
1
2
_
)
18. Trong co
.
so
.
e
1
, e
2
, e
3
vecto
.
x co to
.
a do
.
l`a (1; 2; 0). T`m to
.
a do
.
cu a vecto
.
do trong co
.
so
.
c
1
= 2e
1
e
2
+ 3e
3
, c
2
= 3e
1
+ e
2
2e
3
;
c
3
= 4e
2
+ 5e
3
. (DS. (0, 68; 0, 12; 0, 36))
19. Trong co
.
so
.
e
1
, e
2
, e
3
vecto
.
x co to
.
a do
.
l`a (1, 1, 0). T`m to
.
a do
.
cu a vecto
.
do trong co
.
so
.
: c
1
= 3e
1
+ e
2
+ 6e
3
, c
2
= 5e
1
3e
2
+ 7e
3
,
c
3
= 2e
1
+ 2e
2
3e
3
.
5.3. Khong gian vecto
.
Euclid. Co
.
so
.
tru
.
.
c chua

n 201
(DS. x = (0, 6; 1, 2; 1, 6))
20. Trong co
.
so
.
e
1
, e
2
, e
3
vecto
.
x co to
.
a do
.
l`a (4, 0, 12). T`m to
.
a
do
.
cu a vecto
.
do trong co
.
so
.
c
1
= e
1
+ 2e
2
+e
3
, c
2
= 2e
1
+ 3e
2
+ 4e
3
,
c
3
= 3e
1
+ 4e
2
+ 3e
3
.
(DS. x = (4, 8, 8))
21. Trong khong gian vo
.
i mo
.
t co
.
so
.
l`a e
1
, e
2
, e
3
cho c ac vecto
.
c
1
=
e
1
+e
2
, c
2
= 2e
1
e
2
+e
3
, c
3
= e
2
e
3
.
1) Ch u
.
ng minh ra
`
ng c
1
, c
2
, c
3
la
.
p th`anh co
.
so
.
.
2) T`m to
.
a do
.
cu a vecto
.
x = e
1
+ 8e
2
5e
3
trong co
.
so
.
c
1
, c
2
, c
3
.
(DS. x = (3, 1, 4))
22. Trong co
.
so
.
e
1
, e
2
, e
3
cho cac vecto
.
a = (1, 2, 3), b = (0, 3, 1),
c = (0, 0, 2), d = (4, 3, 1). Ch u
.
ng minh ra
`
ng cac vecto
.
a, b, c la
.
p th`anh
co
.
so
.
v`a t`m to
.
a do
.
cu a vecto
.
d trong co
.
so
.
do.
(DS. d
_
4,
5
3
,
14
3
_
)
5.3 Khong gian vecto
.
Euclid. Co
.
so
.

tru
.
.
c
chua

n
Khong gian tuye

n tnh thu
.
.
c 1 du
.
o
.
.
c go
.
i l`a khong gian Euclid ne

u trong
1 du
.
o
.
.
c trang bi
.
mo
.
t tch vo hu
.
o
.
ng, t u
.
c l`a ne

u vo
.
i mo

i ca
.
p pha
`
n tu
.

x, y 1 de
`
u tu
.
o
.
ng u
.
ng vo
.
i m o
.
t so thu
.
.
c (k y hie
.
u l`a x, y)) sao cho
x, y, z 1 v`a so R phep tu
.
o
.
ng u
.
ng do thoa man cac tien de
`
sau
day
(I) x, y) = y, x);
(II) x +y, z) = x, z) +y, z);
(III) x, y) = x, y);
(IV) x, x) > 0 ne

u x ,= .
Trong khong gian vecto
.
R
n
doi vo
.
i ca
.
p vecto
.
a = (a
1
, a
2
, . . . , a
n
),
202 Chu
.
o
.
ng 5. Khong gian Euclide R
n
b = (b
1
, b
2
, . . . , b
n
) th` quy ta

c tu
.
o
.
ng u
.
ng
a, b) =
n

i=1
a
i
b
i
= a
1
b
1
+a
2
b
2
+ +a
n
b
n
(5.12)
se xac di
.
nh mo
.
t tch vo hu
.
o
.
ng cu a hai vecto
.
a v`a b.
Nhu
.
va
.
y khong gian R
n
vo
.
i tch vo hu
.
o
.
ng x ac di
.
nh theo cong
th u
.
c (5.12) tro
.
th`anh khong gian Euclid. Do do khi noi ve
`
khong gian
Euclid R
n
ta luon luon hie

u l`a tch vo hu
.
o
.
ng trong do xac di
.
nh theo
(5.12).
Gia su
.
x R
n
. Khi do so
_
x, x) du
.
o
.
.
c go
.
i l`a do
.
d`ai (hay chu a

n)
cu a vecto
.
x v`a du
.
o
.
.
c k y hie
.
u l`a |x|. Nhu
.
va
.
y
|x|
def
=
_
x, x) (5.13)
Vecto
.
x vo
.
i do
.
d`ai = 1 du
.
o
.
.
c go
.
i l`a du
.
o
.
.
c chua

n hoa hay vecto


.
do
.
n
vi
.
. De

chua

n hoa mo
.
t vecto
.
khac bat k` y ta ch ca
`
n nhan no v o
.
i so
=
1
|x|
.
Do
.
d`ai co c ac tnh chat
1
+
|x| = 0 x = .
2
+
|x| = [[ |x|, R.
3
+
[x, y)[ |x| |y| (bat da

ng th u
.
c Cauchy-Bunhiakovski)
4
+
|x + y| |x| + |y| (bat da

ng th u
.
c tam giac hay bat da

ng
th u
.
c Minkovski).
T` u
.
bat da

ng th u
.
c 3
+
suy ra
`
ng vo
.
i hai vecto
.
khac bat k` y x, y R
n
ta de
`
u co
[x, y)[
|x| | cos y|
1 1
x, y)
|x| |y|
1.
So
x, y)
|x| |y|
co the

xem nhu
.
cosin cu a goc n`ao do. Goc m`a
cos =
x, y)
|x| |y|
, 0 (5.14)
5.3. Khong gian vecto
.
Euclid. Co
.
so
.
tru
.
.
c chua

n 203
du
.
o
.
.
c go
.
i l`a goc gi u
.
a hai vecto
.
x v` a y.
Hai vecto
.
x, y R
n
du
.
o
.
.
c go
.
i l`a vu ong g oc hay tru
.
.
c giao ne

u tch
vo hu
.
o
.
ng cu a ch ung ba
`
ng 0: x, y) = 0.
He
.
vecto
.
a
1
, a
2
, . . . , a
m
R
n
du
.
o
.
.
c go
.
i l`a he
.
tru
.
.
c giao ne

u ch ung
tru
.
.
c giao t` u
.
ng doi mo
.
t, t u
.
c l`a ne

u a
i
, a
j
) = 0 i ,= j.
He
.
vecto
.
a
1
, a
2
, . . . , a
m
R
n
du
.
o
.
.
c go
.
i l`a he
.
tru
.
.
c giao v`a chu a

n
hoa (hay he
.
tru
.
.
c chu a

n) ne

u
a
i
, a
i
) =
ij
=
_
_
_
0 ne

u i ,= j
1 ne

u i = j
D
-
i
.
nh l y 5.3.1. Mo
.
i he
.
tru
.
.
c giao c ac vecto
.
khac khong de
`
u l`a he
.
do
.
c
la
.
p tuye

n tnh.
He
.
go
`
m n vecto
.
c
1
, c
2
, . . . , c
n
R
n
du
.
o
.
.
c go
.
i l`a co
.
so
.
tru
.
.
c giao
ne

u no l`a mo
.
t co
.
so
.
go
`
m cac vecto
.
tru
.
.
c giao t` u
.
ng doi mo
.
t.
Trong khong gian R
n
to
`
n ta
.
i nh u
.
ng co
.
so
.
da
.
c bie
.
t tie
.
n lo
.
.
i du
.
o
.
.
c
go
.
i l`a nh u
.
ng co
.
so
.
tru
.
.
c chua

n (vai tr`o nhu


.
co
.
so
.
Dec ac vuong goc
trong h`nh ho
.
c gia i tch).
He
.
go
`
m n vecto
.
c
1
, c
2
, . . . , c
n
R
n
du
.
o
.
.
c go
.
i l`a mo
.
t co
.
so
.
tru
.
.
c
chu a

n cu a R
n
ne

u cac vecto
.
n`ay t` u
.
ng doi mo
.
t tru
.
.
c giao v`a do
.
d`ai cu a
mo

i vecto
.
cu a he
.
de
`
u ba
`
ng 1, t u
.
c l`a
(c
i
, c
k
) =
_
_
_
0 ne

u i ,= k,
1 ne

u i = k.
D
-
i
.
nh l y 5.3.2. Trong mo
.
i khong gian Euclid n-chie
`
u de
`
u to
`
n ta
.
i co
.
so
.
tru
.
.
c chu a

n.
De

co die
`
u do ta co the

su
.
du
.
ng phep tru
.
.
c giao hoa Gram-Smidth
du
.
a m o
.
t co
.
so
.
ve
`
co
.
so
.
tru
.
.
c chua

n. N o
.
i dung cu a thua
.
t toan do nhu
.
sau
Gia su
.
c
1
= a
1
. Tie

p do phep du
.
.
ng du
.
o
.
.
c tie

n h`anh theo quy na


.
p.
204 Chu
.
o
.
ng 5. Khong gian Euclide R
n
Ne

u c
1
, c
2
, . . . , c
i
da du
.
o
.
.
c du
.
.
ng th` c
i+1
co the

lay
c
i+1
= a
i+1
+
i

j=1

j
a
j
,
trong do

j
=
a
i+1
, c
j
)
c
j
, c
j
)
, j = 1, i
du
.
o
.
.
c t`m t` u
.
die
`
u kie
.
n c
i+1
tru
.
.
c giao vo
.
i mo
.
i vecto
.
c
1
, c
2
, . . . , c
i
.
C

AC V

I DU
.
1. Trong c ac phep toan du
.
o
.
i day phep toan n`ao l`a tch vo hu
.
o
.
ng cu a
hai vecto
.
x = (x
1
, x
2
, x
3
), y = (y
1
, y
2
, y
3
) R
3
:
1) x, y) = x
2
1
y
2
1
+x
2
2
y
2
2
+x
2
3
y
2
3
;
2) x, y) = x
1
y
1
+ 2x
2
y
2
+ 3x
3
y
3
;
3) x, y) = x
1
y
1
+x
2
y
2
x
3
y
3
.
Gia i. 1) Phep toan n`ay khong l`a tch vo hu
.
o
.
ng v` no khong thoa
man tien de
`
III cu a tch vo hu
.
o
.
ng:
x, y) =
2
x
2
1
y
2
1
+
2
x
2
2
y
2
2
+
2
x
2
3
y
2
3
,= (x
2
1
y
2
1
+x
2
2
y
2
2
+x
2
3
y
2
3
)
2) Phep toan n`ay l`a tch vo hu
.
o
.
ng. Tha
.
t va
.
y, hie

n nhien cac tien


de
`
I v`a II thoa man. Ta kie

m tra cac tien de


`
III v`a IV.
Gia su
.
x

= (x

1
, x

2
, x

3
), x

= (x

1
, x

2
, x

3
) R
3
. Khi do
x

+x

, y) = (x

1
+x

1
)y
1
+ 2(x

2
+x

2
)y
2
+ 3(x

3
+x

3
)y
3
= (x

1
y
1
+ 2x

2
y
2
+ 3x

3
y
3
) + (x

1
y
1
+ 2x

2
y
2
+ 3x

3
y
3
)
= x

, y) +x

, y).
Tie

p theo ta xet
x, x) = x
2
1
+ 2x
2
2
+ 3x
2
3
0 v`a
x, x) = 0 x
2
1
+ 2x
2
2
+ 3x
2
3
= 0 x
1
= x
2
= x
3
= 0 x = .
5.3. Khong gian vecto
.
Euclid. Co
.
so
.
tru
.
.
c chua

n 205
V du
.
2. T`m do
.
d`ai cac ca
.
nh v`a g oc trong ta
.
i A cu a tam giac v o
.
i
d nh A(2, 1, 2, 3), B(2, 1, 2, 1) v` a C(6, 5, 2, 1).
Gia i. Ta t`m to
.
a do
.
cu a c ac vecto
.

AB,

AC v`a

BC. Ta co

AB(0, 2, 4, 4),

AC(4, 4, 0, 2),

BC(4, 6, 4, 2).

Ap du
.
ng di
.
nh ngha do
.
d`ai vecto
.
trong co
.
so
.
tru
.
.
c chua

n ta c o
|

AB| =
_
0
2
+ (2)
2
+ 4
2
+ 4
2
=

36 = 6
v`a tu
.
o
.
ng tu
.
.
|

AC| = 6, |

BC| = 6

2. Theo c ong th u
.
c (5.14) ta co
cos A =

AB,

AC)
|AB| |AC|
=
0 4 + (2) 4 + 4 0 + 4 2
6 6
= 0.
Do do

A =

2
.
V du
.
3. Ch u
.
ng minh ra
`
ng trong bat da

ng th u
.
c Cauchy-Bunhiakovski
[a, b)[ |a| |b| dau ba
`
ng = da
.
t du
.
o
.
.
c khi v`a ch khi a v`a b phu
.
thuo
.
c tuye

n tnh.
Gia i. 1) Ne

u a = b th`
[a, b)[ = [b, b) = [[ |b|
2
= |b| |b| = |a| |b|.
Ngu
.
o
.
.
c la
.
i, ne

u [a, b)[ = |a| |b| th`


_
a
a, b)
|b|
2
b, a
a, b)
|b|
2
b
_
= |a|
2
2
a, b)
|b|
2
a, b) +
a, b)
2
|b|
4
|b|
2
=
= |a|
2
2
|a|
2
|b|
2
|b|
2
+
|a|
2
|b|
2
|b|
2
|b|
4
= 0.
Nhu
.
ng tch vo hu
.
o
.
ng x, x) = 0 x = . T` u
.
do suy ra ra
`
ng a =
a, b)
|b|
2
b, t u
.
c l`a a, b phu
.
thuo
.
c tuye

n tnh.
V du
.
4. He
.
cac vecto
.
do
.
n vi
.
trong R
n
vo
.
i tch vo hu
.
o
.
ng (5.12)
e
1
= (1, 0, 0, . . . , 0)
e
2
= (0, 1, 0, . . . , 0)
. . . . . . . . . . . .
e
n
= (0, 0, 0, . . . , 1)
206 Chu
.
o
.
ng 5. Khong gian Euclide R
n
l`a mo
.
t v du
.
ve
`
co
.
so
.
tru
.
.
c chua

n trong R
n
. Co
.
so
.
n`ay go
.
i l`a co
.
so
.

chnh ta

c trong R
n
.
Gia i. Hie

n nhien e
i
, e
j
) = 0 i ,= j, |e
j
| = 1 j = 1, n. T` u
.
do
thu du
.
o
.
.
c die
`
u ca
`
n ch u
.
ng minh.
V du
.
5. To
.
a do
.
cu a vecto
.
a R
n
bat k` y doi vo
.
i co
.
so
.
tru
.
.
c chua

n
ba
`
ng tch vo hu
.
o
.
ng cu a vecto
.
do vo
.
i vecto
.
co
.
so
.
tu
.
o
.
ng u
.
ng.
Gia i. Gia su
.
a R
n
v`a c
1
, c
2
, . . . , c
n
l`a mo
.
t co
.
so
.
tru
.
.
c chua

n cu a
R
n
. Khi do
a =
n

i=1

i
c
i
.
Nhan vo hu
.
o
.
ng da

ng th u
.
c n`ay vo
.
i c
k
, k = 1, 2, . . . , n ta thu du
.
o
.
.
c
a, c
k
) =
k
, k = 1, 2, . . . , n.
Do do
a =
n

i=1
a, c
i
)c
i
a R
n
.
So
k
= a, c
k
) k = 1, 2, . . . , n chnh l`a to
.
a do
.
cu a vecto
.
a R
n
theo
co
.
so
.
tru
.
.
c chua

n da cho.
V du
.
6. 1) Trong khong gian R
3
vo
.
i tch vo hu
.
o
.
ng (5.12) cho co
.
so
.
c
1
= (1, 2, 1); c
2
= (1, 1, 0); c
3
= (2, 0, 0). Hay d` ung phu
.
o
.
ng phap
tru
.
.
c giao hoa de

t`m co
.
so
.
tru
.
.
c giao trong R
3
t` u
.
co
.
so
.
da cho.
2) Trong khong gian R
3
vo
.
i tch vo hu
.
o
.
ng (5.12) cho co
.
so
.
c
1
=
(1, 1, 1), c
2
= (2, 3, 4), c
3
= (2, 2, 6). Hay du
.
.
ng co
.
so
.
tru
.
.
c chua

n
trong R
3
theo co
.
so
.
da cho.
Gia i. 1) Tru
.
o
.
c he

t ta cho
.
n E
1
= c
1
= (1, 2, 1). Tie

p theo da
.
t
E
2
= c
2
+E
1
sao cho E
2
, E
1
) = 0, t u
.
c l`a
E
2
, E
1
) = E
1
, c
2
) +E
1
, E
1
) = 0.
5.3. Khong gian vecto
.
Euclid. Co
.
so
.
tru
.
.
c chua

n 207
Nhu
.
ng E
1
, E
1
) ,= 0 (cu
.
the

l`a > 0) v` E
1
= c
1
,= . Do do
=
E
1
, c
2
)
E
1
, E
1
)
=
(1, 2, 1), (1, 1, 0))
1
2
+ 2
2
+ 1
2
=
1
2

Do do
E
2
= (1, 1, 0)
1
2
(1, 2, 1) =
_
1
2
, 0,
1
2
_
.
Tie

p theo da
.
t
E
3
= c
3
+E
1
+E
2
sao cho E
3
, E
1
) = E
3
, E
2
) = 0. Tu
.
o
.
ng tu
.
.
nhu
.
tren, t` u
.
die
`
u kie
.
n
E
3
, E
1
) = 0 ta co =
1
3
v`a t` u
.
die
`
u kie
.
n E
3
, E
2
) = 0 ta co = 2.
Do do
E
3
= c
3

1
3
E
1
2E
2
=
_
2
3
,
2
3
,
2
3
_
v`a thu du
.
o
.
.
c co
.
so
.
tru
.
.
c giao
E
1
= (1, 2, 1), E
2
=
_
1
2
, 0,
1
2
_
, E
3
=
_
2
3
,
2
3
,
2
3
_
.
2) Tu
.
o
.
ng tu
.
.
nhu
.
pha
`
n 1), d a
`
u tien ta da
.
t
E
1
= c
1
= (1, 1, 1)
E
2
= c
2
+E
1
sao cho E
2
, E
1
) = 0. T` u
.
do thu du
.
o
.
.
c
=
E
1
, c
2
)
E
1
, E
1
)
=
2 + 3 + 4
3
= 3,
v`a do do
E
2
= (1, 0, 1).
208 Chu
.
o
.
ng 5. Khong gian Euclide R
n
Tie

p theo ta t`m
E
3
= c
3
+E
1
+E
2
sao cho E
3
, E
1
) = 0, E
3
, E
2
) = 0 v`a t` u
.
do thu du
.
o
.
.
c
=
E
1
, c
3
)
E
1
, E
1
)
= 2; =
E
2
, c
3
)
E
2
, E
2
)
= 2.
Nhu
.
va
.
y
E
3
= (2, 4, 2).
Sau c` ung ta chua

n hoa c ac vecto
.
E
1
, E
2
, E
3
v`a thu du
.
o
.
.
c co
.
so
.
tru
.
.
c
chua

n
e
1
=
_
1

3
,
1

3
,
1

3
_
, e
2
=
_

2
, 0,
1

2
_
,
e
3
=
_
1

6
,
2

6
,
1

6
_
.
V du
.
7. Hay bo

sung cho he
.
tru
.
.
c giao g o
`
m ba vecto
.
trong R
4
:
b
1
= (1, 1, 1, 1), b
2
= (2, 2, 2, 2), b
3
=
_

1
2
,
1
2
,
7
2
,
7
2
_
de

thu du
.
o
.
.
c co
.
so
.
tru
.
.
c giao trong khong gian do.
Gia i. Ta co the

bo

sung ba
`
ng hai cach
1
+
V` so vecto
.
cu a he
.
da cho nho ho
.
n 4 (l`a so chie
`
u cu a khong
gian R
4
) nen trong khong gian R
4
ta co the

cho
.
n vecto
.
a
4
sao cho he
.
vecto
.
b
1
, b
2
, b
3
, a
4
do
.
c la
.
p tuye

n tnh v`a sau do ap du


.
ng phep tru
.
.
c giao
hoa Gram-Smidth.
2
+
Ta co the

cho
.
n vecto
.
x = (x
1
, x
2
, x
3
, x
4
) do
`
ng th`o
.
i tru
.
.
c giao vo
.
i
cac vecto
.
b
1
, b
2
, b
3
, t u
.
c l`a thu du
.
o
.
.
c he
.
phu
.
o
.
ng tr`nh
x
1
+x
2
+x
3
+x
4
= 0,
2x
1
+ 2x
2
2x
3
2x
4
= 0,

1
2
x
1
+
1
2
x
2

7
2
x
3
+
7
2
x
4
= 0.
5.3. Khong gian vecto
.
Euclid. Co
.
so
.
tru
.
.
c chua

n 209
Cha

ng ha
.
n, t` u
.
he
.
do ta c o x = (7, 7, 1, 1).
V du
.
8. 1
+
Ch u
.
ng to ra
`
ng c ac vecto
.
x
1
= (1, 1, 1, 2) v`a x
2
=
(1, 2, 3, 3) l`a tru
.
.
c giao vo
.
i nhau.
2
+
Hay bo

sung cho he
.
hai vecto
.
do de

thu du
.
o
.
.
c co
.
so
.
tru
.
.
c giao
cu a R
4
.
Gia i. 1
+
Ta co
x
1
, x
2
) = 1 1 + 1 2 + 1 3 2 3 = 0.
Do do ch ung tru
.
.
c giao.
2
+
Gia su
.
x
3
= (, , , 0), trong do , , du
.
o
.
.
c xac di
.
nh t` u
.
cac
die
`
u kie
.
n x
3
, x
1
) = 0, x
3
, x
2
) = 0 t u
.
c l`a
+ + = 0
+ 2 + 3 = 0.
T` u
.
do x
3
= (1, 2, 1, 0).
Bay gi`o
.
ta se bo

sung them cho he


.
vecto
.
x
1
, x
2
, x
3
mo
.
t vecto
.
n u
.
a.
Gia su
.
x
4
= (, , , ), trong do cac to
.
a do
.
, , , du
.
o
.
.
c xac di
.
nh
t` u
.
cac da

ng th u
.
c:
x
4
, x
1
) = 0, x
4
, x
2
) = 0, x
4
, x
2
) = 0.
T` u
.
do
+ + + 2 = 0,
+ + 3 3 = 0,
2 + = 0.
T` u
.
do thu du
.
o
.
.
c x
4
= (25, 4, 17, 6). Nhu
.
va
.
y ta da bo

sung them
hai vecto
.
x
3
, x
4
v`a thu du
.
o
.
.
c he
.
vecto
.
tru
.
.
c giao x
1
, x
2
, x
3
, x
4
trong
khong gian 4-chie
`
u. Do l`a co
.
so
.
tru
.
.
c giao.
B
`
AI T

A
.
P
210 Chu
.
o
.
ng 5. Khong gian Euclide R
n
1. Gia su
.
a = (a
1
, a
2
), b = (b
1
, b
2
) l`a nh u
.
ng vecto
.
t` uy y cu a R
2
. Trong
cac quy ta

c sau day, quy ta

c n`ao xac di
.
nh tch v o hu
.
o
.
ng tren R
2
:
1) a, b) = a
1
b
1
+a
2
b
2
.
2) a, b) = ka
1
b
1
+a
2
b
2
, k, ,= 0.
3) a, b) = a
1
b
1
+a
1
b
2
+a
2
b
1
.
4) a, b) = 2a
1
b
1
+a
1
b
2
+a
2
b
1
+a
2
b
2
.
5) a, b) = 3a
1
b
1
+a
1
b
2
+a
2
b
1
a
2
b
2
.
(DS. 1), 2) v`a 4) xac di
.
nh tch v o hu
.
o
.
ng
3) v`a 5) khong xac di
.
nh tch v o hu
.
o
.
ng).
2. Trong khong gian Euclide R
4
, xac di
.
nh goc gi u
.
a cac vecto
.
:
1) a = (1, 1, 1, 1), b = (3, 5, 1, 1). (DS. arccos
5
6
)
2) a = (1, 1, 1, 1), b = (3, 5, 1, 1). (DS.

2
)
3) a = (1, 1, 1, 1), b = (3, 3, 3, 3). (DS. )
3. Trong khong gian Euclid R
4
, t`m do
.
d`ai cu a cac ca
.
nh v`a c ac goc
cu a tam gi ac la
.
p bo
.
i cac vecto
.
a, b, a +b ne

u
1) a v`a b nhu
.
trong 2.1)
2) a v`a b nhu
.
trong 2.2)
3) a = (2, 1, 2, 4), b = (2, 1, 2, 4).
(DS. 1) |a| = 2, |b| = 6, |a + b| = 2

15, cos(

a, b) =
5
6
,
cos(

a, a +b) =
7
2

15
; cos(

b, a +b) =
13
6

15
; 2) |a| = 2, |b| = 6,
|a + b| = 2

10, cos(

a, b) = 0, cos(

a, a +b) =
1

10
, cos(

b, a +b) =
3

10
; 3) |a| = 5, |b| = 5, |a + b| = 6, cos(

a, b) =
7
25
,
cos(

a, a +b) =
4
5
, cos(

b, a +b) =
4
15
)
4. Ch u
.
ng minh ra
`
ng trong khong gian Euclide
1) a a a = .
2) Ne

u vecto
.
a b
i
i = 1, s th` a tru
.
.
c giao vo
.
i mo
.
i to

ho
.
.
p tuye

n
tnh cu a b
1
, . . . , b
s
.
5.3. Khong gian vecto
.
Euclid. Co
.
so
.
tru
.
.
c chua

n 211
3) He
.
cac vecto
.
khac khong v`a tru
.
.
c giao vo
.
i nhau t` u
.
ng doi mo
.
t l`a
he
.
do
.
c la
.
p tuye

n tnh.
5. Gia su
.
mo
.
t tam giac trong khong gian Euclide du
.
o
.
.
c la
.
p nen bo
.
i
cac vecto
.
a, b, a +b. Ch u
.
ng minh:
1) di
.
nh l y Pithago: Ne

u a b |a + b|
2
= |a|
2
+|b|
2
.
2) di
.
nh l y da o cu a di
.
nh l y Pithago: Ne

u |a +b|
2
= |a|
2
+|b|
2

a b.
3) di
.
nh l y h`am cosin:
|a +b|
2
= |a|
2
+|b|
2
+ 2|a| |b| cos(

a, b).
4) bat da

ng th u
.
c tam giac
|a| |b| |a +b| |a| +|b|.
Ch da

n. Su
.
du
.
ng bat da

ng th u
.
c Cauchy-Bunhiakovski.
6. Ch u
.
ng minh ra
`
ng trong h`nh b`nh h`anh du
.
.
ng tren hai vecto
.
a v`a b
to

ng cac b`nh phu


.
o
.
ng do
.
d`ai cu a cac du
.
`o
.
ng cheo ba
`
ng to

ng cac b`nh
phu
.
o
.
ng do
.
d`ai cac ca
.
nh
|a +b|
2
+|a b|
2
= 2|a|
2
+ 2|b|
2
.
7. Ch u
.
ng minh ra
`
ng ne

u c ac vecto
.
a
1
, a
2
, . . . , a
m
cu a kh ong gian
Euclide l`a t` u
.
ng doi mo
.
t tru
.
.
c giao th`
|a
1
+a
2
+ +a
m
|
2
= |a
1
|
2
+|a
2
|
2
+ +|a
m
|
2
.
Ch da

n. Xet tch vo hu
.
o
.
ng
a
1
+a
2
+ +a
m
, a
1
+a
2
+ +a
m
)
8.

Ap du
.
ng qua tr`nh tru
.
.
c giao hoa doi vo
.
i cac he
.
vecto
.
sau day cu a
R
n
:
1) a
1
= (1, 2, 2), a
2
= (1, 0, 1), a
3
= (5, 3, 7).
212 Chu
.
o
.
ng 5. Khong gian Euclide R
n
(DS. c
1
= a
1
= (1, 2, 2); c
2
=
_

2
3
,
2
3
,
1
3
_
; c
3
= (6, 3, 6))
2) a
1
= (1, 1, 1, 1), a
2
= (3, 3, 1, 1), a
3
= (2, 0, 6, 8).
(DS. c
1
= a
1
= (1, 1, 1, 1); c
2
= (2, 2, 2, 2), c
3
= (1, 1, 1, 1))
3) a
1
= (1, 1, 1, 1); a
2
= (3, 3, 1, 1); a
3
= (1, 0, 3, 4).
(DS. c
1
= a
1
= (1, 1, 1, 1), c
2
= (2, 2, 2, 2), c
3
=
_

1
2
,
1
2
,
7
2
,
7
2
_
)
9. Tru
.
.
c chua

n hoa cac he
.
vecto
.
sau day cu a khong gian R
4
:
1) a
1
= (1, 1, 1, 1), a
2
= (1, 1, 3, 3), a
3
= (4, 3, 0, 1).
(DS. c
1
=
_
1
2
,
1
2
,
1
2
,
1
2
_
, c
2
=
_
1
2
,
1
2
,
1
2
,
1
2
_
, c
3
=
_
1
2
,
1
2
,
1
2
,
1
2
_
)
2) a
1
= (1, 2, 2, 0), a
2
= (1, 1, 3, 5), a
3
= (1, 0, 1, 0).
(DS. c
1
=
_
1
3
,
2
3
,
2
3
, 0
_
, c
2
=
_
0,
1
3

3
,
1
3

3
,
5
3

3
_
,
c
3
=
_
6

78
,
17
3

78
,
8
3

78
,
5
3

78
_
)
10. Ch u
.
ng to ra
`
ng cac he
.
vecto
.
sau day trong R
4
l`a tru
.
.
c giao v`a bo

sung cho cac he


.
do de

tro
.
th`anh co
.
so
.
tru
.
.
c giao:
1) a
1
= (1, 2, 1, 3), a
2
= (2, 1, 3, 1)
(DS. Cha

ng ha
.
n, cac vecto
.
a
3
= (1, 1, 1, 0), a
4
= (1, 1, 0, 1))
2) a
1
= (1, 1, 1, 3), a
2
= (4, 1, 5, 0).
(DS. Cha

ng ha
.
n, cac vecto
.
a
3
= (2, 3, 1, 0) v` a a
4
= (1, 1, 1, 1))
11. Ch u
.
ng to ra
`
ng cac vecto
.
sau day trong R
4
l`a tru
.
.
c giao v`a bo

sung
cho c ac he
.
do de

tro
.
th`anh co
.
so
.
tru
.
.
c giao v`a chua

n hoa cac co
.
so
.
do
1) a
1
= (1, 1, 1, 1), a
2
= (1, 1, 1, 1).
(DS. c
1
=
_
1
2
,
1
2
,
1
2
,
1
2
_
, c
2
=
_
1
2
,
1
2
,
1
2
,
1
2
_
, c
3
=
_

1

2
, 0,
1

2
, 0
_
,
5.4. Phep bie

n do

i tuye

n tnh 213
c
4
=
_
0,
1

2
, 0,
1

2
_
)
2) a
1
= (1, 1, 1, 3), a
2
= (1, 1, 3, 1)
(DS. c
1
=
_
1
2

3
,
1
2

3
,
1
2

3
,

3
2
_
, c
2
=
_
1
2

3
,
1
2

3
,

3
2
,
1
2

3
_
,
c
3
=
_
2

6
,
1

6
,
1

6
, 0
_
, c
4
=
_

6
,
2

6
, 0,
1

6
_
)
5.4 Phep bie

n do

i tuye

n tnh
5.4.1 D
-
i
.
nh ngha

Anh xa
.
L : R
n
R
n
bie

n khong gian R
n
th`anh chnh no du
.
o
.
.
c go
.
i l`a
mo
.
t phep bie

n do

i tuye

n tnh (bdtt) cu a khong gian R


n
ne

u no thoa
man hai die
`
u kie
.
n sau day
(i) Vo
.
i hai vecto
.
a v`a b R
n
bat k` y
L(a +b) = L(a) +L(b). (5.15)
(ii) Vo
.
i vecto
.
a R
n
bat k` y v`a R ta co
L(a) = L(a). (5.16)
Hai die
`
u kie
.
n (5.15) v`a (5.16) tu
.
o
.
ng du
.
o
.
ng vo
.
i die
`
u kie
.
n:
L(
1
a +
2
b) =
1
L(a) +
2
L(b).
T` u
.
di
.
nh ngha suy ra: ne

u he
.
vecto
.
a
1
, a
2
, . . . , a
m
R
n
l`a pttt th`
he
.
cac vecto
.
a nh f(a
1
), . . . , f(a
m
) c ung l`a pttt.
5.4.2 Ma tra
.
n cu a phep bdtt
Gia su
.
trong khong gian R
n
ta co di
.
nh mo
.
t co
.
so
.
(c) n`ao do:
c = c
1
, c
2
, . . . , c
n
. (5.17)
214 Chu
.
o
.
ng 5. Khong gian Euclide R
n
Khi do x R
n
ta co khai trie

n
x = x
1
c
1
+x
2
|
2
+ +x
n
c
n
.
Mo
.
i ma tra
.
n vuong A =
_
_
a
ij
_
_
nn
de
`
u xac di
.
nh phep bdtt L theo
cong th u
.
c
_

_
y
1
y
2

y
n
_

_
=
_

_
a
11
a
12
. . . a
1n
a
21
a
22
. . . a
2n
.
.
.
.
.
.
.
.
.
.
.
.
a
n1
a
n2
. . . a
nn
_

_
_

_
x
1
x
2
.
.
.
x
n
_

_
(5.18)
Noi c ach khac: de

thu du
.
o
.
.
c to
.
a do
.
a nh y = L(x) ta ca
`
n nhan ma tra
.
n
A vo
.
i co
.
t to
.
a do
.
cu a x. Vie

t ra chi tie

t ta co
y
1
= a
11
x
1
+ a
12
x
2
+ +a
1n
x
n
,
. . . . . . . . . . . . . . . . . . (5.19)
y
n
= a
n1
x
1
+a
n2
x
2
+ +a
nn
x
n
.
Ngu
.
o
.
.
c la
.
i, trong co
.
so
.
da cho
.
n (5.17) mo

i phep bdtt L de
`
u tu
.
o
.
ng
u
.
ng vo
.
i mo
.
t ma tra
.
n A = |a
ij
| cap n v`a su
.
.
tac do
.
ng cu a phep bdtt
du
.
o
.
.
c thu
.
.
c hie
.
n theo cong th u
.
c (5.18) hay (5.19).
Vie
.
c t`m ma tra
.
n cu a phep bdtt du
.
o
.
.
c tie

n h`anh nhu
.
sau
1
+
Tac do
.
ng L len cac vecto
.
co
.
so
.
cu a (5.17) v`a thu du
.
o
.
.
c a nh
L(c
i
), i = 1, n.
2
+
Khai trie

n cac a nh L(c
i
) theo co
.
so
.
(5.17):
L(c
1
) = a
11
c
1
+a
21
c
2
+ +a
n1
c
n
,
L(c
2
) = a
12
c
1
+a
22
c
2
+ +a
n2
c
n
,
. . . . . . . . . . . . . . .
L(c
\
) = a
1n
c
1
+a
2n
c
2
+ +a
nn
c
n
.
_

_
(5.20)
T` u
.
cac to
.
a do
.
trong (5.20) ta la
.
p ma tra
.
n A sao cho to
.
a do
.
cu a vecto
.
5.4. Phep bie

n do

i tuye

n tnh 215
L(c
i
), i = 1, n l`a co
.
t th u
.
i cu a A, t u
.
c l`a
A =
_

_
a
11
a
12
. . . a
1n
a
21
a
22
. . . a
2n
.
.
.
.
.
.
.
.
.
.
.
.
a
n1
a
n2
. . . a
nn
_

_
Do l`a ma tra
.
n cu a phep bdtt.
Ta lu
.
u y ra
`
ng khi thay do

i co
.
so
.
th` ma tra
.
n cu a phep bie

n do

i
tuye

n tnh se thay do

i. Gia su
.
ma tra
.
n chuye

n t` u
.
co
.
so
.
(c) de

n co
.
so
.

(c

) du
.
o
.
.
c k y hie
.
u l`a T
EE
, trong do
c

= c

1
, c

2
, . . . , c

v`a A l`a ma tra


.
n phep bie

n do

i tuye

n tnh L theo co
.
so
.
(5.17). Khi do,
ma tra
.
n B cu a L theo co
.
so
.
(c

) lien he
.
vo
.
i ma tra
.
n A cu a no theo
co
.
so
.
(5.17) bo
.
i cong th u
.
c
B = T
1
EE

AT
EE
(5.21)
hay l`a
A = T
EE
BT
1
EE

(5.22)
5.4.3 Cac phep toan
Gia su
.
/ v`a B l`a hai phep bdtt cu a khong gian R
n
vo
.
i ma tra
.
n tu
.
o
.
ng
u
.
ng l`a A = |a
ij
| v`a B = |b
ij
| t` uy y.
1
+
To

ng /+B l`a phep bie

n do

i ( sao cho
((x) = /(x) +B(x) x R
n
vo
.
i ma tra
.
n tu
.
o
.
ng u
.
ng l`a C = A+B = |a
ij
| +|b
ij
|.
216 Chu
.
o
.
ng 5. Khong gian Euclide R
n
2
+
Tch c ac phep bie

n do

i tuye

n tnh / vo
.
i so thu
.
.
c R l`a phep
bie

n do

i / sao cho
(/)(x) = /(x)
vo
.
i ma tra
.
n l`a |a
ij
|.
3
+
Tch /B l`a phep bie

n do

i
((x) = /(B(x))
vo
.
i ma tra
.
n l`a C = A B.
5.4.4 Vecto
.
rieng v`a gia tri
.
rieng
Vecto
.
khac khong x R
n
du
.
o
.
.
c go
.
i l`a vecto
.
rieng cu a phep bie

n do

i
tuye

n tnh L ne

u t`m du
.
o
.
.
c so sao cho da

ng th u
.
c sau thoa man
L(x) = x. (5.23)
So du
.
o
.
.
c go
.
i l`a gia tri
.
rieng cu a phep bdtt L tu
.
o
.
ng u
.
ng vo
.
i vecto
.
rieng x.
Cac tnh chat cu a vecto
.
rieng
1
+
Mo

i vecto
.
rieng ch co mo
.
t gi a tri
.
rieng.
2
+
Ne

u x
1
v`a x
2
l`a cac vecto
.
rieng cu a phep bdtt L vo
.
i c` ung mo
.
t
gia tri
.
rieng th` to

ng x
1
+ x
2
c ung l`a vecto
.
rieng cu a L vo
.
i gia tri
.
rieng .
3
+
Ne

u x l`a vecto
.
rieng cu a L vo
.
i gia tri
.
rieng th` mo
.
i vecto
.
x
( ,= 0) c ung l`a vecto
.
rieng cu a L vo
.
i gia tri
.
rieng .
Ne

u trong khong gian R


n
da cho
.
n mo
.
t co
.
so
.
xac di
.
nh th` (5.23) co
the

vie

t du
.
o
.
i da
.
ng ma tra
.
n
AX = X (5.24)
v`a khi do: mo
.
i co
.
t khac khong thoa man (5.24) du
.
o
.
.
c go
.
i l`a vecto
.
rieng
cu a ma tra
.
n A tu
.
o
.
ng u
.
ng vo
.
i gia tri
.
rieng .
5.4. Phep bie

n do

i tuye

n tnh 217
V` X = EX, trong do E l`a ma tra
.
n do
.
n vi
.
nen (5.24) co the

vie

t du
.
o
.
i da
.
ng
(AE)X = 0
v`a du
.
o
.
i da
.
ng to
.
a do
.
ta thu du
.
o
.
.
c
(a
11
)x
1
+a
12
x
2
+ +a
1n
x
n
= 0,
a
21
x
1
+ (a
22
)x
2
+ +a
2n
x
n
= 0,
. . . . . . . . . . . . . . .
a
n1
x
1
+a
n2
x
2
+ + (a
nn
)x
n
= 0.
_

_
(5.25)
De

t`m cac vecto


.
rieng, tru
.
o
.
c he

t c a
`
n t`m nghie
.
m khac 0 cu a he
.
(5.25). Nghie
.
m khac 0 cu a he
.
(5.25) to
`
n ta
.
i khi v` a ch khi di
.
nh th u
.
c
cu a no ba
`
ng 0, t u
.
c l`a
[AE[ =

a
11
a
12
. . . a
1n
a
21
a
22
. . . a
2n
.
.
.
.
.
.
.
.
.
.
.
.
a
n1
a
n2
. . . a
nn

= 0. (5.26)
Phu
.
o
.
ng tr`nh (5.26) du
.
o
.
.
c go
.
i l`a phu
.
o
.
ng tr`nh da
.
c tru
.
ng cu a ma
tra
.
n A, c`on cac nghie
.
m cu a no go
.
i l`a c ac so da
.
c tru
.
ng hay gia tri
.
rieng
cu a ma tra
.
n A. Sau khi t`m du
.
o
.
.
c cac so da
.
c tru
.
ng
1
,
2
, . . . ,
n
ta
ca
`
n thay gia tri
.

i
v`ao (5.25) de

t`m cac to
.
a do
.
x
1
, . . . , x
n
cu a vecto
.
rieng tu
.
o
.
ng u
.
ng.
C

AC V

I DU
.
V du
.
1. Cho L : R
2
R
2
(a
1
, a
2
) L(a
1
, a
2
) = (a
1
+a
2
, 2a
1
).
1
+
Ch u
.
ng minh ra
`
ng L l`a phep bie

n do

i tuye

n tnh.
2
+
T`m ma tra
.
n cu a L theo co
.
so
.
chnh ta

c e = e
1
, e
2
.
218 Chu
.
o
.
ng 5. Khong gian Euclide R
n
Gia i. 1
+
Gia su
.
x = (x
1
, x
2
), y = (y
1
, y
2
). Khi do
x +y = (x
1
, x
2
) +(y
1
, y
2
) = (x
1
+y
1
, x
2
+y
2
)
v`a do do
L(x +y) = L(x
1
+y
1
, x
2
+y
2
)
=
_
x
1
+y
1
+x
2
+y
2
, 2(x
1
+y
1
)

=
_
(x
1
+x
2
) +(y
1
+y
2
), 2x
1
+2y
1

=
_
(x
1
+x
2
), 2x
1
_
+
_
(y
1
+y
2
), 2y
1
_
= (x
1
+x
2
, 2x
1
) +(y
1
+y
2
, 2y
1
)
= L(x
1
, x
2
) +L(y
1
, y
2
)
= L(x) +L(y).
Nhu
.
va
.
y L l`a phep bdtt.
2
+
De

t`m ma tra
.
n cu a phep bie

n do

i tuye

n tnh L ta khai trie

n
a nh L(e
1
) v`a L(e
2
) theo co
.
so
.
chnh ta

c. Ta c o
L(e
1
) = L(1, 0) = (1, 2 1) = L(1, 2) = 1 e
1
+ 2 e
2
,
L(e
2
) = L(0, 1) = (1, 2 0) = L(1, 0) = 1 e
1
+ 0 e
2
.
T` u
.
do thu du
.
o
.
.
c
A =
_
1 1
2 0
_
.
V du
.
2. Xet khong gian R
3
vo
.
i co
.
so
.
c: c
1
= (1, 1, 1), c
2
= (0, 1, 1),
c
3
= (0, 0, 1) v`a phep bie

n do

i L : R
3
R
3
xac di
.
nh bo
.
i da

ng th u
.
c
L[(u
1
, u
2
, u
3
)] = (u
1
, u
2
u
1
, u
3
u
1
) u = (u
1
, u
2
, u
3
) R
3
.
1
+
Ch u
.
ng minh ra
`
ng L l`a phep bdtt.
2
+
T`m ma tra
.
n cu a L trong co
.
so
.
da cho
.
n.
5.4. Phep bie

n do

i tuye

n tnh 219
Gia i. 1
+
Gia su
.
x = (x
1
, x
2
, x
3
), y = (y
1
, y
2
, y
3
) R
3
v`a , R.
Ta co
L(x +y) = L
_
(x
1
, x
2
, x
3
) +(y
1
, y
2
, y
3
)

= L
_
(x
1
+ y
1
, x
2
+y
2
, x
3
+y
3
)

=
_
x
1
+y
1
, x
2
+y
2
x
1
y
1
, x
3
+y
3
x
1
y
1
_
=
_
x
1
, (x
2
x
1
), (x
3
x
1
)
_
+
_
y
1
, (y
2
y
1
), (y
3
y
2
)
_
= (x
1
, x
2
x
1
, x
3
x
1
) +(y
1
, y
2
y
1
, y
3
y
1
)
= L(x) +L(y).
Va
.
y L l`a phep bie

n do

i phan tuye

n tnh.
2
+
De

t`m ma tra
.
n cu a L doi vo
.
i co
.
so
.
c
1
, c
2
, c
3
ta co
L(c
1
) = L(1, 1, 1) = (1, 1 1, 1 1) = (1, 0, 0) = c
1
+ 0 c
2
+ 0 c
3
,
L(c
2
) = L(0, 1, 1) = (0, 1, 1) = 0 c
1
+ 1 c
2
+ 1 c
3
,
L(c
3
) = L(0, 0, 1) = (0, 0, 1) = 0 c
1
+ 0 c
2
+ 1 c
3
.
T` u
.
do suy ra
`
ng ma tra
.
n cu a L doi vo
.
i co
.
so
.
da cho l`a
A =
_

_
1 0 0
0 1 0
0 1 1
_

_
.
V du
.
3. Trong khong gian R
3
cho co
.
so
.
chnh ta

c e = e
1
, e
2
, e
3
v`a
c = c
1
, c
2
, c
3
, c
1
= 2e
1
e
2
+ 3e
3
, c
2
= e
1
+ e
3
, c
3
= e
2
+ 2e
3
l`a
mo
.
t co
.
so
.
khac v`a gia su
.
L : R
3
R
3
l`a anh xa
.
du
.
o
.
.
c xac di
.
nh theo
co
.
so
.
e
1
, e
2
, e
3
nhu
.
sau
x = (x
1
, x
2
, x
3
) f(x) = (x, x
1
+x
2
, x
1
+x
2
+x
3
)
1
+
T`m to
.
a do
.
cu a vecto
.
x = 3e
1
e
2
= (3, 1, 0) doi vo
.
i co
.
so
.

(c
1
, c
2
, c
3
).
2
+
Ch u
.
ng minh ra
`
ng L l`a phep bdtt.
220 Chu
.
o
.
ng 5. Khong gian Euclide R
n
3
+
T`m ma tra
.
n cu a L theo co
.
so
.
e
1
, e
2
, e
3
v`a c
1
, c
2
, c
3
.
Gia i. 1
+
Ma tra
.
n chuye

n t` u
.
co
.
so
.
e
1
, e
2
, e
3
de

n co
.
so
.
(c
1
, c
2
, c
3

l`a:
T
eE
=
_

_
2 1 0
1 0 1
3 1 2
_

_ T
1
eE
_

_
1 2 1
1 4 2
1 1 1
_

_.
Gia su
.
(x

1
, x

2
, x

3
) l`a to
.
a do
.
cu a x doi vo
.
i co
.
so
.
c
1
, c
2
, c
3
. Khi
do
_

_
x

1
x

2
x

3
_

_
= T
1
eE
_

_
x
1
x
2
x
3
_

_
=
_

_
1 2 1
1 4 2
1 1 1
_

_
_

_
3
1
0
_

_
=
_

_
5
7
4
_

_
.
Va
.
y to
.
a do
.
cu a x doi vo
.
i co
.
so
.
c
1
, c
2
, c
3
l`a (5, 7, 4) v`a do do x =
5c
1
7c
2
4c
3
.
2
+
Vie
.
c ch u
.
ng minh L l`a phep bdtt du
.
o
.
.
c tie

n h`anh tu
.
o
.
ng tu
.
.
nhu
.
v du
.
2.
3
+
Gia su
.
f(x) = (y
1
, y
2
, y
3
). Khi do ap du
.
ng (5.18) ta co
f(x
1
, x
2
, x
2
) = (x
1
, x
1
+x
2
, x
1
+x
2
+x
3
) = (y
1
, y
2
, y
3
).
Do va
.
y trong co
.
so
.
chnh ta

c ta co
_

_
y
1
y
2
y
3
_

_
=
_

_
x
1
x
1
+x
2
x
1
+x
2
+x
3
_

_
=
_

_
1 0 0
1 1 0
1 1 1
_

_
_

_
x
1
x
2
x
3
_

_
v`a do do
A =
_

_
1 0 0
1 1 0
1 1 1
_

_
.
5.4. Phep bie

n do

i tuye

n tnh 221
Gia su
.
B l`a ma tra
.
n cu a L theo co
.
so
.
c
1
, c
2
, c
3
. Khi do
B = T
1
eE
AT
eE
=
_

_
1 2 1
1 4 2
1 1 1
_

_
_

_
1 0 0
1 1 0
1 1 1
_

_
_

_
2 1 0
1 0 1
3 1 2
_

_
=
_

_
4 3 1
10 7 2
3 2 0
_

_
.
V du
.
4. Gia su
.
L : R
3
R
3
v`a L

: R
3
R
3
l`a hai phep bie

n do

i
tuye

n tnh cu a R
3
, trong do
L(x
1
, x
2
, x
3
) = (x
2
+x
3
, 2x
1
, x
1
+x
2
),
L

(x
1
, x
2
, x
3
) = (c
1
x
2
, 2x
3
, 2x
2
+x
3
).
T`m L +L

, L L

, L

L v`a ma tra
.
n cu a ch ung.
Gia i. Ta co:
1) (L +L

)(x
1
, x
2
, x
2
) = L(x
1
, x
2
, x
3
) +L

(x
1
, x
2
, x
3
)
= (x
2
+x
3
, 2x
1
, x
1
+x
3
) + (x
1
x
2
, 2x
3
, 2x
2
+x
3
)
= (x
1
+x
3
, 2(x
1
x
3
), x
1
+ 3x
2
+x
3
).
T` u
.
do phep bie

n do

i L +L

co the

du
.
o
.
.
c cho bo
.
i cong th u
.
c
y
1
= x
1
+x
3
,
y
2
= 2x
1
+ 2x
3
,
y
3
= x
1
+ 3x
2
+x
3
v`a ma tra
.
n A
L+L
cu a L +L

co da
.
ng
A
L+L
=
_

_
1 0 1
2 0 2
1 3 1
_

_
.
222 Chu
.
o
.
ng 5. Khong gian Euclide R
n
Ta lu
.
u y ra
`
ng t` u
.
cac cong th u
.
c cho L v`a L

ta co
A
L
=
_

_
0 1 1
2 0 0
1 1 0
_

_, A
L
=
_

_
1 1 0
0 0 2
0 2 1
_

_
v`a thu du
.
o
.
.
c A
L+L
ba
`
ng phep co
.
ng A
L
vo
.
i A
L
.
2) Ta c o
(L L

)(x) = L[L

(x)] = L
_
x
1
x
2
, 2x
3
, 2x
2
+x
3
)
=
_
2x
3
..
+2x
2
+x
3
. .
, 2(x
1
x
2
), x
1
x
2
. .
+ 2x
3
..
_
= (2x
2
+ 3x
3
, 2x
1
+ 2x
2
, x
1
x
2
+ 2x
3
)
v`a tu
.
o
.
ng tu
.
.
nhu
.
tren ta co
A
LL
=
_

_
0 2 3
2 2 0
1 1 2
_

_
(Lu
.
u y ra
`
ng A
LL
= A
L
A
L
).
3) Tru
.
`o
.
ng ho
.
.
p L

L du
.
o
.
.
c gia i tu
.
o
.
ng tu
.
.
2).
V du
.
5. 1) Ch u
.
ng minh ra
`
ng trong co
.
so
.
e = e
1
, e
2
cu a khong
gian R
2
vecto
.
x = e
1
3e
2
l`a vecto
.
rieng cu a phep bdtt L co ma tra
.
n
trong co
.
so
.
e l`a
A =
_
2 1
3 0
_
v`a vo
.
i gia tri
.
rieng = 1.
2) T`m gia tri
.
rieng v`a vecto
.
rieng cu a phep bdtt xac di
.
nh bo
.
i cac
phu
.
o
.
ng tr`nh y
1
= 5x
1
+ 4x
2
, y
2
= 8x
1
+ 9x
2
.
Gia i. 1) Tru
.
o
.
c he

t ta lu
.
u y ra
`
ng vecto
.
x ,= .
Ta ca
`
n ch u
.
ng to L(x) = x. Th a
.
t va
.
y, ta co
L(x) =
_
2 1
3 0
__
1
3
_
=
_
1
3
_
=
_
1
3
_
= x.
5.4. Phep bie

n do

i tuye

n tnh 223
Nhu
.
va
.
y L(x) = x v`a do do x l`a vecto
.
rieng u
.
ng vo
.
i gia tri
.
rieng
= 1.
2) 1
+
Ta co ma tra
.
n cu a phep bie

n do

i l`a
_
5 4
8 9
_
.
Phu
.
o
.
ng tr`nh da
.
c tru
.
ng co da
.
ng

5 4
8 9

= 0
2
14 + 13 = 0

1
= 1,

2
= 13.
2
+
Ca hai gia tri
.
= 1 v`a = 13 de
`
u l`a cac gi a tri
.
rieng.
3
+
De

t`m to
.
a do
.
cu a cac vecto
.
rieng ta co hai he
.
phu
.
o
.
ng tr`nh
tuye

n tnh
(I)
_
(5
1
)
1
+ 4
2
= 0,
8
1
+ (9
1
)
2
= 0.
(II)
_
(5
2
)
1
+ 4
2
= 0,
8
1
+ (9
2
)
2
= 0.
i) V`
1
= 1 nen he
.
(I) co da
.
ng
4
1
+ 4
2
= 0,
8
1
+ 8
2
= 0.
T` u
.
do suy ra
2
=
1
, do do nghie
.
m cu a he
.
n`ay co da
.
ng
1
=
1
,

2
=
1
, trong do
1
l`a da
.
i lu
.
o
.
.
ng t` uy y. V` vecto
.
rieng khac khong
nen cac vecto
.
u
.
ng vo
.
i gia tri
.
rieng
1
= 1 l`a cac vecto
.
u(
1
,
1
),
trong do
1
,= 0 l`a t` uy y.
ii) Tu
.
o
.
ng tu
.
.
khi
2
= 13 he
.
(II) tro
.
th`anh
8
1
+ 4
2
= 0,
8
1
4
2
= 0,
224 Chu
.
o
.
ng 5. Khong gian Euclide R
n
t u
.
c l`a
2
= 2
1
. Da
.
t
1
=
2
= 2. Va
.
y he
.
(II) co nghie
.
m l`a

1
= ,
2
= 2. V` vecto
.
rieng khac khong nen cac vecto
.
rieng u
.
ng
vo
.
i gia tri
.

2
= 13 l`a c ac vecto
.
v(, 2).
V du
.
6. T`m gi a tri
.
rieng v`a vecto
.
rieng cu a phep bie

n do

i tuye

n
tnh L vo
.
i ma tra
.
n
A =
_
1 2
5 4
_
.
Gia i. Da th u
.
c da
.
c tru
.
ng cu a phep bie

n do

i L
P() =

1 2
5 4

=
2
5 6.
No co hai nghie
.
m thu
.
.
c
1
= 6,
2
= 1. Cac vecto
.
da
.
c tru
.
ng du
.
o
.
.
c
t`m t` u
.
hai he
.
phu
.
o
.
ng tr`nh
_
(1
i
)
1
+ 2
2
= 0,
5
1
+ (4
i
)
2
= 0,
i = 1, 2.
V` di
.
nh th u
.
c cu a he
.
= 0 nen mo

i he
.
ch thu ve
`
mo
.
t phu
.
o
.
ng tr`nh.
1
+
Vo
.
i
1
= 6 ta c o 5
1
2
2
= 0

1

2
=
2
5
v`a do do ta co the

lay vecto
.
rieng tu
.
o
.
ng u
.
ng l`a u = (2, 5) (hoa
.
c mo
.
i vecto
.
u, R,
,= 0)
2
+
Vo
.
i
2
= 1 ta co
1
+
2
= 0

1

2
= 1 v`a vecto
.
rieng tu
.
o
.
ng
u
.
ng l`a v = (1, 1) (hay mo
.
i vecto
.
da
.
ng v, ,= 0).
V du
.
7. T`m cac gia tri
.
rieng v`a vecto
.
rieng cu a phep bie

n do

i tuye

n
tnh L tren R
3
vo
.
i ma tra
.
n theo co
.
so
.
chnh ta

c l`a
A =
_

_
1 1 4
2 0 4
1 1 5
_

_
5.4. Phep bie

n do

i tuye

n tnh 225
Gia i. Ta co da th u
.
c da
.
c tru
.
ng cu a ma tra
.
n A l`a
det(AE) =

1 1 4
2 4
1 1 5

=
3
+ 6
2
11 + 6
v`a
det(AE) = 0
_

1
= 1,

2
= 2,

3
= 3.
Gia su
.
x = (
1
,
2
,
3
) ,= 0 l`a vecto
.
rieng u
.
ng vo
.
i gia tri
.
rieng .
Khi do x l`a nghie
.
m cu a he
.
thua
`
n nhat
(1 )
1
+
2
+ 4
3
= 0,
2
1

2
4
3
= 0,

1
+
2
+ (5 )
3
= 0.
_

_
(*)
1
+
Khi = 1 ta co
()

2
+ 4
3
= 0,
2
1

2
4
3
= 0,

1
+
2
+ 4
3
= 0.
_

_
nghie
.
m to

ng quat l`a (0, 4, ), ,= 0 t` uy y.


Va
.
y vo
.
i gia tri
.
rieng
1
= 1 ta co cac vecto
.
rieng u
.
ng vo
.
i no l`a
(0, 4, ), R, ,= 0.
2
+
Khi = 2 ta co
()

1
+
2
+ 4
3
= 0,
2
1
2
2
4
3
= 0,

1
+
2
+ 3
3
= 0
_

_
he
.
co nghie
.
m to

ng quat l`a (, , 0), ,= 0 v`a do do vecto


.
rieng u
.
ng
vo
.
i = 2 l`a (, , 0), ,= 0.
226 Chu
.
o
.
ng 5. Khong gian Euclide R
n
3
+
Khi = 3, thu
.
.
c hie
.
n tu
.
o
.
ng tu
.
.
nhu
.
o
.
1
+
v`a 2
+
ta thu du
.
o
.
.
c
vecto
.
rieng tu
.
o
.
ng u
.
ng (2, 0, ), ,= 0 t` uy y.
V du
.
8. T`m gia tri
.
rieng v`a vecto
.
rieng cu a phep bdtt vo
.
i ma tra
.
n
A =
_

_
7 12 6
10 19 10
12 24 13
_

_.
Gia i. Phu
.
o
.
ng tr`nh da
.
c tru
.
ng
P() =

7 12 6
10 19 10
12 24 13

= 0
co nghie
.
m
1
=
2
= 1,
1
= 1. Cac vecto
.
da
.
c tru
.
ng du
.
o
.
.
c xac di
.
nh
t` u
.
hai he
.
phu
.
o
.
ng tr`nh
(7
i
) 12 + 6 = 0,
10 (19 +
i
) + 10 = 0,
12 24 + (13
i
) = 0; i = 1, 2.
1
+
Khi = 1 ta co
6 12 + 6 = 0,
10 20 + 10 = 0,
12 24 + 12 = 0.
Ha
.
ng cu a ma tra
.
n (go
.
i l`a ma tra
.
n da
.
c tru
.
ng) (A
1
E) cu a he
.
n`ay
l`a ba
`
ng r = 1. Do do he
.
tu
.
o
.
ng du
.
o
.
ng vo
.
i mo
.
t phu
.
o
.
ng tr`nh
2 + = 0.
T` u
.
do suy ra
`
ng he
.
co hai nghie
.
m do
.
c la
.
p tuye

n tnh, cha

ng ha
.
n
u = (4, 5, 6),
v = (3, 5, 7).
5.4. Phep bie

n do

i tuye

n tnh 227
2
+
Khi
2
= 1 ta c o
8 12 + 6 = 0,
10 18 + 10 = 0,
12 24 + 14 = 0.
Ha
.
ng cu a ma tra
.
n (A
3
E) cu a he
.
ba
`
ng r = 2. Do do he
.
tu
.
o
.
ng du
.
o
.
ng
vo
.
i he
.
hai phu
.
o
.
ng tr`nh. Nghie
.
m rieng cu a no co da
.
ng w = (3, 5, 6).
Nhu
.
va
.
y u, v, w l`a cac vecto
.
rieng cu a phep bdtt da cho.
V du
.
9. Cho ma tra
.
n
A =
_
0 1
1 0
_
.
T`m phep bie

n do

i tuye

n tnh L tu
.
o
.
ng u
.
ng vo
.
i ma tra
.
n do.
Gia i. Gia su
.
x = ae
1
+be
2
l`a vecto
.
t` uy y cu a ma
.
t pha

ng. De

t`m
phep bie

n do

i tuye

n tnh ta ca
`
n ch ro a nh y = Ax. Ta co
y =
_
0 1
1 0
__
a
b
_
=
_
b
a
_
= be
1
+ae
2
.
Nhu
.
va
.
y phep bie

n do

i L co tnh chat l`a: thay do

i vai tr`o cu a cac to


.
a
do
.
cu a mo

i vecto
.
x R
2
. T` u
.
d o suy ra
`
ng L l`a phep pha n xa
.
gu
.
o
.
ng
doi vo
.
i du
.
`o
.
ng phan giac th u
.
nh at.
B
`
AI T

A
.
P
Trong c ac b`ai toan (1 - 11) hay ch u
.
ng to phep bie

n do

i da cho l`a
phep bdtt v`a t`m ma tra
.
n cu a ch ung theo co
.
so
.
chnh ta

c.
1. Phep bie

n do

i L l`a phep quay mo


.
i vecto
.
cu a ma
.
t pha

ng xOy xung
quanh goc to
.
a do
.
mo
.
t g oc ngu
.
o
.
.
c chie
`
u kim do
`
ng ho
`
.
(DS. A
L
=
_
cos sin
sin cos
_
)
228 Chu
.
o
.
ng 5. Khong gian Euclide R
n
2. Phep bie

n do

i L l`a phep quay khong gian thu


.
.
c ba chie
`
u mo
.
t g oc
xung quanh tru
.
c Oz.
(DS.
_

_
cos sin 0
sin cos 0
0 0 1
_

_
)
3. Phep bie

n do

i L l`a phep chie

u vuong g oc vecto
.
a R
3
len ma
.
t
pha

ng xOy.
(DS.
_

_
1 0 0
0 1 0
0 0 0
_

_
)
4. Phep bie

n do

i L l`a tch vecto


.
y = [a, x], trong do a = a
1
x
1
+a
2
x
2
+
a
3
x
3
l`a vecto
.
co di
.
nh cu a R
3
.
(DS.
_

_
0 a
3
a
2
a
3
0 a
1
a
2
a
1
0
_

_
)
Ch da

n. Su
.
du
.
ng phep bie

u die

n tch vecto
.
du
.
o
.
i da
.
ng di
.
nh th u
.
c.
5. Phep bie

n do

i L l` a phep bie

n do

i do
`
ng nhat trong khong gian
n-chie
`
u R
n
trong mo
.
i co
.
so
.
.
(DS. E =
_

_
1 0 . . . 0
0 1 . . . 0
.
.
.
.
.
.
.
.
.
.
.
.
0 0 . . . 1
_

_
)
6. L l`a phep bie

n do

i d o
`
ng da
.
ng L(x) = x trong khong gian n-chie
`
u.
(DS.
_

_
0 . . . 0
0 . . . 0
.
.
.
.
.
.
.
.
.
.
.
.
0 0 . . .
_

_
)
5.4. Phep bie

n do

i tuye

n tnh 229
7. Phep bie

n do

i L co da
.
ng L(x) = x
2
e
1
+x
3
e
2
+x
4
e
3
+x
1
e
4
trong do
x = x
1
e
1
+x
2
e
2
+x
3
e
3
+x
4
e
4
.
(DS.
_

_
0 1 0 0
0 0 1 0
0 0 0 1
1 0 0 0
_

_
)
8. Phep bie

n do

i L l`a phep chie

u vuong goc khong gian 3-chie


`
u len
tru
.
c la
.
p vo
.
i cac tru
.
c to
.
a do
.
nh u
.
ng goc ba
`
ng nhau, t u
.
c l`a (

Ox, ) =
(

Oy, ) = (

Oz, ) = .
(DS.
_

_
1
3
1
3
1
3
1
3
1
3
1
3
1
3
1
3
1
3
_

_
)
Ch da

n. Su
.
du
.
ng tnh chat cu a cosin ch phu
.
o
.
ng cu a goc bat k` y
cos
2
+ cos
2
+ cos
2
= 1.
9. Phep bie

n do

i L l`a phep chie

u R
3
theo phu
.
o
.
ng song song vo
.
i ma
.
t
pha

ng vecto
.
e
2
, e
3
len tru
.
c to
.
a do
.
cu a vecto
.
e
1
(DS.
_

_
1 0 0
0 0 0
0 0 0
_

_
)
10. Phep bie

n do

i L l`a phep quay R


3
mo
.
t goc =
2
3
xung quanh
du
.
`o
.
ng tha

ng cho trong R
3
bo
.
i phu
.
o
.
ng tr`nh x
1
= x
2
= x
3
.
(DS. a)
_

_
0 0 1
1 0 0
0 1 0
_

_ ne

u quay t` u
.
e
1
de

n e
2
,
b)
_

_
0 1 0
0 0 1
1 0 0
_

_
ne

u quay t` u
.
e
2
de

n e
1
)
230 Chu
.
o
.
ng 5. Khong gian Euclide R
n
Trong cac b`ai toan (12-22) cho hai co
.
so
.
(e) : e
1
, e
2
, . . . , e
n
v`a
(c) : c
1
, c
2
, . . . , c
n
cu a khong gian R
n
v`a ma tra
.
n A
L
cu a phep bie

n
do

i tuye

n tnh L trong co
.
so
.
(e). T`m ma tra
.
n cu a L trong co
.
so
.
(c).
Phu
.
o
.
ng phap chung l`a: (i) t`m ma tra
.
n chuye

n T t` u
.
co
.
so
.
(e) de

n co
.
so
.
(c); (ii) T`m ma tra
.
n T
1
; (iii) T`m B
L
= T
1
AT.
11. A
L
=
_
17 6
6 8
_
, c
1
= e
1
2e
2
, c
2
= 2e
1
+e
2
). (DS.
_
5 0
0 20
_
)
12. A
L
=
_
3 1
2 1
_
, c
1
= e
2
, c
2
= e
1
+e
2
. (DS.
_
2 3
1 2
_
)
13. A
L
=
_
2 4
3 3
_
, c
1
= e
2
2e
1
, c
2
= 2e
1
4e
2
. (DS.
_
3 14
3 8
_
)
14. A
L
=
_
1 0
2 4
_
, c
1
= 3e
1
+2e
2
, c
2
= 2e
1
+2e
2
. (DS.
_
5 6
6 8
_
)
15. A
L
=
_

_
0 2 1
3 1 0
2 1 1
_

_
, c
1
= 3e
1
+e
2
+ 2e
3
, c
2
= 2e
1
+e
2
+ 2e
3
,
c
3
= e
1
+ 2e
2
+ 5e
3
. (DS.
_

_
85 59 18
121 84 25
13 9 3
_

_)
16. A
L
=
_

_
15 11 5
20 15 8
8 7 6
_

_, c
1
= 2e
1
+ 3e
2
+e
3
, c
2
= 3e
1
+ 4e
2
+e
3
,
c
3
= e
1
+ 2e
2
+ 2e
3
. (DS.
_

_
1 0 0
0 2 0
0 0 3
_

_
)
17. A
L
=
_

_
2 1 0
0 1 1
0 0 1
_

_
. c
1
= 2e
1
+e
2
e
3
, c
2
= 2e
1
e
2
+ 2e
3
,
5.4. Phep bie

n do

i tuye

n tnh 231
c
3
= 3e
1
+e
3
. (DS.
_

_
2 11 7
4 14 8
5 15 8
_

_)
18. A
L
=
_
2 1
0 3
_
, e
1
= 3c
1
c
2
, e
2
= c
1
+c
2
. (DS.
_
3 3
0 2
_
)
19. A
L
=
_
1 4
5 0
_
, e
1
= c
1
+c
2
, e
2
= 2c
1
. (DS.
_
2 7
2 3
_
)
20. A
L
=
_

_
1 2 3
0 3 1
1 2 5
_

_
, e
1
= c
1
, e
2
= 3c
1
+c
2
, e
3
= 2c
1
+c
2
+ 2c
3
.
(DS.
_

_
1 18 3
1 8 0
2 10 2
_

_)
21. A
L
=
_

_
2 1 0
0 1 1
0 0 1
_

_
, e
1
= 2c
1
+c
2
c
3
, e
2
= 2c
1
c
2
+ 2c
3
,
e
3
= 3c
1
+c
2
. (DS.
_

_
3 10 8
1 8 5
2 13 7
_

_
)
22. Trong c ac phep bie

n do

i sau day t` u
.
R
3
R
3
phep bie

n do

i n`ao
l`a tuye

n tnh (gia thie

t x = (x
1
, x
2
, x
3
) R
3
)
1) L(x
1
, x
2
, x
3
) = (x
1
+2x
2
+3x
3
, 4x
1
+5x
2
+6x
3
, 7x
1
+8x
2
+9x
3
);
2) L(x
1
, x
2
, x
3
) = (x
1
+3x
2
+4, 5x
3
; 6x
1
+7x
2
+9x
3
; 10, 5x
1
+12x
2
+
13x
3
)
3) L(x
1
, x
2
, x
3
) = (x
2
+x
3
, x
1
+x
3
, x
1
+x
2
).
4) L(x
1
, x
2
, x
3
) = (x
1
, x
2
+ 1, x
3
+ 2).
5) L(x
1
, x
2
, x
3
) = (x
2
+x
3
, 2x
1
+x
3
, 3x
1
x
2
+x
3
).
6) L(x
1
, x
2
, x
3
) = (2x
1
+x
2
, x
1
+x
3
, x
2
3
).
7) L(x
1
, x
2
, x
3
) = (x
1
x
2
x
3
, x
3
, x
2
).
232 Chu
.
o
.
ng 5. Khong gian Euclide R
n
(DS. 1), 2), 3), 5), 7) l`a phep bdtt; 4), 6) - khong)
23. T`m phu
.
o
.
ng tr`nh d a
.
c tru
.
ng v`a s o d a
.
c tru
.
ng cu a phep bdtt L
ne

u
1) L(e
1
) = 2e
1
; L(e
2
) = 5e
1
+ 3e
2
; L(e
3
) = 3e
1
+ 4e
2
6e
3
, trong
do e
1
, e
2
, e
3
l`a co
.
so
.
cu a khong gian. (DS. ( + 6)( 2)( 3) = 0)
2) L(e
1
) = e
1
, L(e
2
) = 2e
1
+ 5e
2
, L(e
3
) = 2e
1
e
2
+ 3e
3
+ 5e
4
,
L(e
4
) = e
1
+ 7e
2
+ 4e
3
+ 6e
4
, trong do e
1
, e
2
, e
3
, e
4
l`a co
.
so
.
cu a khong
gian.
(DS. ( + 1)( 5)(
2
9 2) = 0)
3) L(e
1
) = 2e
1
+ 2e
3
, L(e
2
) = 2e
1
+ 2e
2
, L(e
3
) = 2e
2
+ 2e
3
;
e
1
, e
2
, e
3
l`a co
.
so
.
cu a khong gian. (DS.
3
6
2
+ 12 = 0)
24. Gia su
.
trong co
.
so
.
e = e
1
, e
2
phep bdtt L co ma tra
.
n l`a
A
L
=
_
3 5
1 4
_
c`on trong co
.
so
.
c = c
1
, c
2
, c
1
= e
1
e
2
, c
2
= e
1
+ 2e
2
phep bdtt
L

co ma tra
.
n
A
L
=
_
0 2
1 1
_
.
T`m ma tra
.
n cu a cac phep bie

n do

i:
1) L +L

trong co
.
so
.
e
1
, e
2
;
2) L +L

trong co
.
so
.
c
1
, c
2
.
(DS. 1)
1
3
_
10 13
5 14
_
; 2)
1
3
_
1 13
3 23
_
)
25. Gia su
.
trong co
.
so
.
e = e
1
, e
2
, e
3
phep bdtt L co ma tra
.
n
A
L
=
_

_
2 0 2
1 1 0
3 0 1
_

_
5.4. Phep bie

n do

i tuye

n tnh 233
c`on trong co
.
so
.
c = c
1
, c
2
, c
3
, c
1
= e
1
+2e
2
, c
2
= e
1
e
3
, c
3
= e
2
+e
3
phep bdtt L

co ma tra
.
n
A
L
=
_

_
0 3 0
0 1 2
1 2 0
_

_.
T`m ma tra
.
n cu a phep bie

n do

i:
1) L +L

trong co
.
so
.
e. (DS.
_

_
6 2 2
16 6 7
8 2 3
_

_)
2) L +L

trong co
.
so
.
c. (DS.
_

_
2 4 4
4 12 8
8 17 7
_

_
)
26. Gia su
.
trong co
.
so
.
e = e
1
, e
2
phep bdtt L co ma tra
.
n
A
L
=
_
1 2
0 1
_
c`on trong co
.
so
.
c = c
1
, c
2
, c
1
= 2e
1
+ e
2
, c
2
= e
1
e
2
phep bdtt
L

co ma tra
.
n
A
L
=
_
3 2
1 0
_
.
T`m ma tra
.
n cu a cac phep bie

n do

i
1) L L

trong co
.
so
.
e. (DS.
_
1 1
0 2
_
)
2) L

L trong co
.
so
.
e. (DS.
_
1 7
0 2
_
)
3) L L

trong co
.
so
.
c. (DS.
1
3
_
1 2
7 4
_
)
234 Chu
.
o
.
ng 5. Khong gian Euclide R
n
4) L

L trong co
.
so
.
c. (DS.
1
3
_
7 10
1 4
_
)
27. Gia su
.
phep bdtt L co ma tra
.
n
_
2 1
5 3
_
trong co
.
so
.
c =
c
1
, c
2
, c
1
= (3, 7), c
2
= (1, 2) v`a trong co
.
so
.
c

= c

1
, c

2
,
c

1
= (6, 7), c

2
= (5, 6) phep bdtt L

co ma tra
.
n l`a
_
1 3
2 7
_
. T`m
ma tra
.
n cu a L L

trong co
.
so
.
m`a cac vecto
.
tren du
.
o
.
.
c cho.
(DS.
_
109 93
34 29
_
)
Ch da

n. T`m cac ma tra


.
n chuye

n co
.
so
.
T
ea
, T
eb
v`a ap du
.
ng
cong th u
.
c (5.22) de

t`m ma tra
.
n A
L
v`a A
L
trong co
.
so
.
e. T` u
.
do
A
LL
= A
L
A
L
.
Trong cac b`ai toan (28-31) hay xac di
.
nh trong c ac vecto
.
da cho
vecto
.
n`ao l`a vecto
.
rieng cu a phep bdtt vo
.
i ma tra
.
n da cho (trong co
.
so
.
n`ao do).
28. A =
_
1 0
2 1
_
; x
1
=
_
1
2
_
, x
2
=
_
0
3
_
, x
3
=
_
0
1
_
. (DS. x
2
v`a x
3
)
29. A =
_
1 1
6 2
_
; x
1
=
_
1
3
_
, x
2
=
_
2
4
_
, x
3
=
_
1
2
_
.
(DS. x
1
v`a x
3
)
30. A =
_

_
0 0 2
2 0 0
0 2 0
_

_
; x
1
=
_

_
1
1
3
_

_
, x
2
=
_

_
1
0
5
_

_
, x
3
=
_

_
2
2
2
_

_
. (DS. x
3
)
31. A =
_

_
0 1 0
6 3 2
3 0 1
_

_
; x
1
=
_

_
1
2
0
_

_
, x
2
=
_

_
1
0
3
_

_
, x
3
=
_

_
4
0
1
_

_
. (DS. x
2
)
Trong c ac b`ai toan (32-35) hay t`m c ac vecto
.
rieng cu a phep bie

n
do

i tuye

n tnh du
.
o
.
.
c cho trong mo
.
t co
.
so
.
n`ao do bo
.
i ma tra
.
n A.
5.4. Phep bie

n do

i tuye

n tnh 235
32. A =
_
2 4
1 3
_
. (DS.
_
4
1
_
,
_
1
1
_
vo
.
i ,= 0, ,= 0 bat k` y)
33. A =
_
3 4
2 1
_
. (DS.
_
2
1
_
,
_
1
1
_
vo
.
i ,= 0, ,= 0 bat k` y)
34. A =
_

_
1 2 2
1 0 3
1 3 0
_

_. (DS.
_

_
2
1
1
_

_,
_

_
0
1
1
_

_,
_

_
6
7
5
_

_,
,= 0, ,= 0, ,= 0 bat k` y)
35. A =
_

_
1 0 2
0 3 0
0 0 0
_

_
. (DS.
_

_
2
0
1
_

_
,
_

_
0
1
0
_

_
; ,= 0, ,= 0 bat k` y)
36. Cho phep bie

n do

i tuye

n tnh L : R
2
R
2
nhu
.
sau
L : (x
1
, x
2
) (5x
1
+ 4x
2
, 8x
1
+ 9x
2
).
T`m gi a tri
.
rieng v`a vecto
.
rieng cu a L.
(DS.
1
= 1, u = (, ), ,= 0;
2
= 13, v = (, 2), ,= 0)
37. T`m gia tri
.
rieng v`a vecto
.
rieng cu a ma tra
.
n
A =
_

_
2 1 1
1 2 1
0 0 1
_

_
(DS.
1
=
2
= 1, u = (, ), ,= 0;
3
= 3, v = (, ), ,= 0) .
Chu
.
o
.
ng 6
Da
.
ng to`an phu
.
o
.
ng v`a u
.
ng
du
.
ng de

nha
.
n da
.
ng du
.
`o
.
ng v`a
ma
.
t ba
.
c hai
6.1 Da
.
ng to`an phu
.
o
.
ng
Da th u
.
c da

ng cap ba
.
c hai cu a cac bie

n x
1
, x
2
, . . . , x
n
du
.
o
.
.
c go
.
i l`a da
.
ng
to`an phu
.
o
.
ng cu a n bie

n do:
(x
1
, . . . , x
n
) =
n

i=1
n

j=1
a
ij
x
i
x
j
=
n

i,j=1
a
ij
x
i
x
j
. (6.1)
Do l`a phep tu
.
o
.
ng u
.
ng da
.
t tu
.
o
.
ng u
.
ng mo

i vecto
.
x = (x
1
, x
2
, . . . , x
n
)
R
n
vo
.
i so (x
1
, . . . , x
n
).
Ne

u da
.
t
X =
_

_
x
1
x
2
.
.
.
x
n
_

_
, A =
_

_
a
11
a
12
. . . a
1n
a
21
a
22
. . . a
nn
.
.
.
.
.
.
.
.
.
.
.
.
a
n1
a
n2
. . . a
nn
_

_
6.1. Da
.
ng to`an phu
.
o
.
ng 237
th` thu du
.
o
.
.
c
(x
1
, x
2
, . . . , x
n
) = X
T
AX. (6.2)
D
-
i
.
nh l y. Ne

u C l`a ma tra
.
n cu a phep bdtt thu
.
.
c hie
.
n tren c ac bie

n
cu a da
.
ng to`an phu
.
o
.
ng (6.1) vo
.
i ma tr a
.
n A th` da
.
ng to`an phu
.
o
.
ng m o
.
i
thu du
.
o
.
.
c co ma tra
.
n l`a C
T
AC.
Da
.
ng to`an phu
.
o
.
ng da
.
ng

1
x
2
1
+
2
x
2
2
+ +
n
x
2
n
(6.3)
khong ch u
.
a cac so ha
.
ng vo
.
i tch cu a cac bie

n khac nhau (v`a do do no


co ma tra
.
n du
.
`o
.
ng cheo) du
.
o
.
.
c go
.
i l`a da
.
ng to`an phu
.
o
.
ng cheo hay da
.
ng
chnh ta

c.
Tie

p theo ta tr`nh b`ay no


.
i dung cu a cac phu
.
o
.
ng phap du
.
a da
.
ng
to`an phu
.
o
.
ng ve
`
da
.
ng chnh ta

c.
6.1.1 Phu
.
o
.
ng phap Lagrange
D
-
i
.
nh l y Lagrange. Ba
`
ng phep bie

n d o

i tuye

n tnh kh ong suy bie

n
doi v o
.
i c ac bie

n x
1
, . . . , x
n
mo
.
i da
.
ng to`an phu
.
o
.
ng de
`
u du
.
a du
.
o
.
.
c ve
`
da
.
ng chnh ta

c.
Tinh tha
`
n co
.
ba n cu a phu
.
o
.
ng phap Lagrange l`a nhu
.
sau.
1
+

It nhat mo
.
t trong cac he
.
so a
ii
khac khong.
Khong gia m to

ng quat, co the

cho ra
`
ng a
11
,= 0 (ne

u khong th`
danh so la
.
i). Khi do ba
`
ng phep trch mo
.
t b`nh phu
.
o
.
ng du t` u
.
cu
.
m tat
ca cac so ha
.
ng ch u
.
a x
1
ta co
() = y
2
1
+
2
(x
2
, x
3
, . . . , x
n
)
y
1
=
1
x
1
+
2
x
2
+ +
n
x
n
trong do
1
,
2
, . . . ,
n
l`a c ac ha
`
ng so,
2
(x
2
, . . . , x
n
) l`a da
.
ng to`an
phu
.
o
.
ng ch c`on n 1 bie

n (khong c` on x
1
). Doi vo
.
i
2
(x
2
, . . . , x
n
) ta
la
.
i thu
.
.
c hie
.
n thua
.
t toan nhu
.
v` u
.
a tr`nh b`ay,...
238 Chu
.
o
.
ng 6. Da
.
ng to`an phu
.
o
.
ng v` a u
.
ng du
.
ng
2
+
Tru
.
`o
.
ng ho
.
.
p a
ii
= 0 i = 1, n nh u
.
ng a
ij
,= 0 (i ,= j) du
.
o
.
.
c du
.
a
ve
`
tru
.
`o
.
ng ho
.
.
p tren ba
`
ng phep bie

n do

i tuye

n tnh khong suy bie

n
x
j
= y
j
+y
i
x
k
= y
k
, k ,= j
V du
.
1. Du
.
a da
.
ng to`an phu
.
o
.
ng
(x
1
, x
2
, x
3
) = x
2
1
+x
2
2
+x
2
3
+ 4x
1
x
2
+ 4x
1
x
3
+ 4x
2
x
3
ve
`
da
.
ng chnh ta

c.
Gia i. Nhom cac so ha
.
ng co ch u
.
a x
1
th`anh mo
.
t cu
.
m v` a trch t` u
.
cu
.
m do mo
.
t b`nh phu
.
o
.
ng du ta c o
() = (x
2
1
+ 4x
1
x
2
+ 4x
1
x
3
) +x
2
2
+x
2
3
+ 4x
2
x
3
= (x
1
+x
2
+ 2x
3
)
2
(2x
2
+ 2x
3
)
2
+x
2
2
+x
2
3
+ 4x
2
x
3
= (x
1
+ 2x
2
+ 2x
3
)
2
3x
2
2
3x
2
3
4x
2
x
3
.
Nhom cac so ha
.
ng co ch u
.
a x
2
ro
`
i trch b`nh phu
.
o
.
ng ta c o
() = (x
1
+ 2x
2
+ 2x
3
)
2
3(x
2
+
2
3
x
3
)
2

5
3
x
2
3
.
D` ung phep bie

n do

i tuye

n tnh khong suy bie

n
y
1
= x
1
+ 2x
2
+ 2x
3
y
2
= x
2
+
2
3
x
3
y
3
= x
3
_

x
1
= y
1
2y
2

2
3
y
3
x
2
= y
2

2
3
y
3
x
3
= y
3
ta thu du
.
o
.
.
c
() = y
2
1
3y
2
2

5
3
y
2
3
.
V du
.
2. Du
.
a da
.
ng to`an phu
.
o
.
ng
(x
1
, x
2
, x
3
) = x
1
x
2
+ 2x
1
x
3
+ 4x
2
x
3
6.1. Da
.
ng to`an phu
.
o
.
ng 239
ve
`
da
.
ng chnh ta

c.
Gia i. V` a
11
= a
22
= a
33
= 0 nen da
`
u tien thu
.
.
c hie
.
n phep bie

n do

i
so
.
bo
.
khong suy bie

n thu du
.
o
.
.
c so ha
.
ng co b`nh phu
.
o
.
ng:
x
1
= y
1
x
2
= y
1
+y
2
x
3
= y
3
_

_
(6.4)
v`a thu du
.
o
.
.
c
() = y
1
(y
1
+y
2
) + 2y
1
y
3
+ 4(y
1
+y
2
)y
3
= y
2
1
+y
1
y
2
+ 6y
1
y
3
+ 4y
2
y
3
.
Xuat phat t` u
.
da
.
ng to`an phu
.
o
.
ng mo
.
i thu du
.
o
.
.
c, tu
.
o
.
ng tu
.
.
nhu
.
trong
v du
.
1 ta co
() =
_
y
1
+
1
2
y
2
+ 3y
3
_
2

_
1
2
y
2
+ 3y
3
_
2
+ 4y
2
y
3
=
_
y
1
+
1
2
y
2
+ 3y
3
_
2

1
4
y
2
+y
2
y
3
9y
3
.
Thu
.
.
c hie
.
n phep bie

n do

i khong suy bie

n
z
1
= y
1
+
1
2
y
2
+ 3y
3
,
z
2
= y
2
,
z
3
= y
3
vo
.
i phep bie

n do

i ngu
.
o
.
.
c
y
1
= z
1

1
2
z
2
3z
3
,
y
2
= z
2
,
y
3
= z
3
_

_
(6.5)
ta thu du
.
o
.
.
c
() = z
2
1

1
4
z
2
2
+z
2
z
3
9z
2
3
.
240 Chu
.
o
.
ng 6. Da
.
ng to`an phu
.
o
.
ng v` a u
.
ng du
.
ng
Nhom cac so ha
.
ng co ch u
.
a z
2
ta co
() = z
2
1

1
4
(z
2
2z
3
)
2
8z
2
3
.
Thu
.
.
c hie
.
n phep bie

n do

i khong suy bie

n
u
1
= z
1
,
u
2
= z
2
2z
3
,
u
3
= z
3
_

z
1
= u
1
,
z
2
= u
2
+ 2u
3
,
z
3
= u
3
_

_
(6.6)
Sau ba phep bie

n do

i lien tie

p (6.4)-(6.6) da
.
ng da cho c o da
.
ng du
.
`o
.
ng
cheo
() = u
2
1

1
4
u
2
2
8u
2
3
.
De

t`m ma tra
.
n cu a phep bie

n do

i ho
.
.
p ta ca
`
n nhan c ac ma tra
.
n cu a
(6.4), (6.5) v` a (6.6). Ta co
_

_
1 0 0
1 1 0
0 0 1
_

_
_

_
1
1
2
3
0 1 0
0 0 1
_

_
_

_
1 0 0
0 1 2
0 0 1
_

_ =
_

_
1
1
2
4
1
1
2
2
0 0 1
_

_
= C.
Do phep bie

n do

i khong suy bie

n du
.
a da
.
ng ve
`
da
.
ng chnh ta

c l`a
x
1
= u
1

1
2
u
2
4u
3
,
x
2
= u
1
+
1
2
u
2
2u
3
,
x
3
= u
3
.
_

_
De

kie

m tra ta tnh tch C


T
AC. Ta co
C
T
AC =
_

_
1 1 0

1
2
1
2
0
4 2 1
_

_
_

_
0
1
2
1
1
2
0 2
1 2 0
_

_
_

_
1
1
2
4
1
1
2
2
0 0 1
_

_
=
_

_
1 0 0
0
1
4
0
0 0 8
_

_
Do l`a ma tra
.
n cu a da
.
ng chnh ta

c thu du
.
o
.
.
c.
6.1. Da
.
ng to`an phu
.
o
.
ng 241
6.1.2 Phu
.
o
.
ng phap Jacobi
Phu
.
o
.
ng phap n`ay ch ap du
.
ng du
.
o
.
.
c khi mo
.
i di
.
nh th u
.
c con chnh cu a
ma tra
.
n A cu a da
.
ng to`an phu
.
o
.
ng de
`
u khac 0, t u
.
c l`a khi

1
= a
11
,= 0,
2
=

a
11
a
12
a
21
a
22

,= 0, . . . ,
n
=

a
11
a
12
. . . a
1n
a
21
a
22
. . . a
2n
.
.
.
.
.
.
.
.
.
.
.
.
a
n1
a
n2
. . . a
nn

,= 0.
(6.7)
Cu
.
the

ta co
D
-
i
.
nh l y. Ne

u da
.
ng to`an phu
.
o
.
ng
(x
1
, . . . , x
n
) =
n

i,j=1
a
ij
x
i
x
j
thoa man die
`
u kie
.
n v` u
.
a neu:
i
,= 0 i = 1, n th` to
`
n ta
.
i phep bie

n
do

i tuye

n tnh khong suy bie

n t` u
.
cac bie

n x
1
, . . . , x
n
de

n cac bie

n
y
1
, . . . , y
n
sao cho
() =

1

0
y
2
1
+

2

1
y
2
2
+ +

n

n1
y
2
n
,
0
1.
Phep bie

n do

i neu trong di
.
nh l y Jacobi c o da
.
ng
x
1
= y
1
+
21
y
2
+
31
y
3
+ +
n1
y
n
,
x
2
= y
2
+
32
y
3
+ +
n2
y
n
,
. . . . . . . . . . . . . . .
x
n
= y
n
_

_
(6.8)
trong do c ac he
.
so
ji
cu a phep bdtt (6.8) du
.
o
.
.
c xac di
.
nh theo cac cong
th u
.
c

ij
= (1)
j+i
D
j1,i

j1
(6.9)
242 Chu
.
o
.
ng 6. Da
.
ng to`an phu
.
o
.
ng v` a u
.
ng du
.
ng
o
.
day
j1
l`a di
.
nh th u
.
c con chnh trong (6.7), c`on D
j1,i
l`a di
.
nh th u
.
c
con cu a ma tra
.
n A la
.
p nen bo
.
i c ac pha
`
n tu
.
na
`
m tren giao cu a cac
h`ang th u
.
1, 2, . . . , j 1 v`a cac co
.
t th u
.
1, 2, . . . , i 1, i + 1, . . . , j
V du
.
3. Du
.
a da
.
ng to`an phuwo
.
ng
(x
1
, x
2
, x
3
) = 2x
2
1
+ 3x
2
2
+x
2
3
4x
1
x
2
+ 2x
1
x
3
2x
2
x
3
ve
`
da
.
ng chnh ta

c.
Gia i. Ma tra
.
n cu a da
.
ng da cho c o da
.
ng
A =
_

_
2 2 1
2 3 1
1 1 1
_

_
vo
.
i cac di
.
nh th u
.
c con chnh

1
= 2,
2
= 2,
3
= 1.
Khi do da
.
ng to`an phu
.
o
.
ng da cho du
.
a du
.
o
.
.
c ve
`
da
.
ng chnh ta

c
() = 2y
2
1
+y
2
2
+
1
2
y
2
3
. (6.10)
Ta t`m phep bdtt du
.
a da
.
ng to`an phu
.
o
.
ng da cho ve
`
da
.
ng (6.10).
No c o da
.
ng
x
1
= y
1
+
21
y
2
+
31
y
3
,
x
2
= y
2
+
32
y
3
,
x
3
= y
3
.
_

_
(6.11)
Ta t`m cac he
.
so cu a (6.11) theo c ong th u
.
c (6.9). Ta co

21
= (1)
3
D
1,1

1
=
2
2
= 1,

31
= (1)
4
D
2,1

2
=

2 1
3 1

2
=
1
2
,

32
= (1)
5
D
2,2

2
=

2 1
2 1

2
= 0.
6.1. Da
.
ng to`an phu
.
o
.
ng 243
Nhu
.
va
.
y
x
1
= y
1
+y
2

1
2
y
3
,
x
2
= y
2
,
x
3
= y
3
.
_

_
V du
.
4. Du
.
a da
.
ng to`an phu
.
o
.
ng
(x
1
, x
2
, x
3
) = 2x
2
1
+ 3x
1
x
2
+ 4x
1
x
3
+x
2
2
+x
2
3
ve
`
da
.
ng chnh ta

c.
Gia i. Ta co ma tra
.
n cu a l`a
A =
_

_
2
3
2
2
3
2
1 0
2 0 1
_

_
vo
.
i cac di
.
nh th u
.
c con chnh ba
`
ng

1
= 2,
2
=
1
4
,
3
=
17
4

Khi do theo di
.
nh l y Jacobi ta thu du
.
o
.
.
c da
.
ng chnh ta

c l`a
() = 2y
2
1

1
8
y
2
2
+ 17y
3
nh`o
.
phep bie

n do

i
x
1
= y
1
+
21
y
2
+
31
y
3
,
x
2
= y
2
+
32
y
3
,
x
3
= y
3
_

_
244 Chu
.
o
.
ng 6. Da
.
ng to`an phu
.
o
.
ng v` a u
.
ng du
.
ng
vo
.
i cac he
.
so du
.
o
.
.
c xac di
.
nh theo (6.9).

Ap du
.
ng (6.9) ta thu du
.
o
.
.
c

21
= (1)
3
D
1,1

1
=
3
2
2
=
3
4
,

31
= (1)
4
D
2,1

2
=

3
2
2
1 0

1
4
= 8,

32
= (1)
4
D
2,2

2
=

2 2
3
2
0

1
4
= 12.
Va
.
y phep bie

n do

i l`a
x
1
= y
1

3
4
y
2
+ 8y
3
,
x
2
= y
2
12y
3
,
x
3
= y
3
_

_
.
6.1.3 Phu
.
o
.
ng phap bie

n do

i tru
.
.
c giao
V` ma tra
.
n A cu a da
.
ng to`an phu
.
o
.
ng l`a ma tra
.
n doi x u
.
ng, thu
.
.
c nen
b`ai toan du
.
a da
.
ng to`an phu
.
o
.
ng ve
`
da
.
ng chnh ta

c co the

quy ve
`
b`ai
toan du
.
a ma tra
.
n doi x u
.
ng A ve
`
da
.
ng du
.
`o
.
ng cheo. Cac nghie
.
m cu a
phu
.
o
.
ng tr`nh da
.
c tru
.
ng [A E[ = 0 l`a cac s o d a
.
c tru
.
ng, c` on c ac
vecto
.
rieng tu
.
o
.
ng u
.
ng vo
.
i cac so da
.
c tru
.
ng do l`a c ac hu
.
o
.
ng chnh cu a
da
.
ng to`an phu
.
o
.
ng (Lu
.
u y ra
`
ng hai vecto
.
rieng tu
.
o
.
ng u
.
ng vo
.
i cac gia
tri
.
rieng khac nhau cu a ma tra
.
n doi x u
.
ng l`a tru
.
.
c giao vo
.
i nhau). Ma
.
t
khac v` A l`a ma tra
.
n d oi x u
.
ng thu
.
.
c nen no c o n so da
.
c tru
.
ng thu
.
.
c
(ne

u mo

i so du
.
o
.
.
c tnh mo
.
t so la
`
n ba
`
ng bo
.
i cu a no).
T` u
.
do t`m du
.
o
.
.
c du n vecto
.
rieng do
.
c la
.
p tuye

n tnh. Ba
`
ng phep
tru
.
.
c chua

n hoa ta thu du
.
o
.
.
c mo
.
t co
.
so
.
go
`
m t` u
.
cac vecto
.
rieng cu a A.
6.1. Da
.
ng to`an phu
.
o
.
ng 245
Ma tra
.
n T chuye

n t` u
.
co
.
so
.
tru
.
.
c chua

n (e) de

n co
.
so
.
tru
.
.
c chua

n (c)
la
.
p nen t` u
.
cac vecto
.
rieng cu a phep bie

n do

i doi x u
.
ng (vo
.
i ma tra
.
n
A) l`a ma tra
.
n tru
.
.
c giao v` ca hai co
.
so
.
de
`
u tru
.
.
c chua

n.
Nhu
.
va
.
y doi vo
.
i mo
.
i ma tra
.
n doi x u
.
ng thu
.
.
c A co the

t`m mo
.
t
ma tra
.
n tru
.
.
c giao T c` ung cap sao cho B = T
1
AT l`a ma tra
.
n cheo.
Do c ung chnh l`a ma tra
.
n cu a da
.
ng to`an phu
.
o
.
ng da cho trong co
.
so
.

(c). T` u
.
do ta c o quy ta

c t`m phep bie

n do

i tru
.
.
c giao du
.
a da
.
ng to`an
phu
.
o
.
ng ve
`
da
.
ng chnh ta

c.
1) Vie

t ma tra
.
n A cu a da
.
ng to`an phu
.
o
.
ng v`a t`m c ac so da
.
c tru
.
ng
cu a no.
2) T`m he
.
vecto
.
rieng tru
.
.
c chua

n cu a A.
3) La
.
p phep bie

n do

i tru
.
.
c giao.
V du
.
5. Du
.
a da
.
ng to`an phu
.
o
.
ng
(x
1
, x
2
) = 27x
2
1
10x
1
x
2
+ 3x
2
3
ve
`
da
.
ng chnh ta

c.
Gia i. 1
+
Vie

t ma tra
.
n A cu a da
.
ng
A =
_
27 5
5 3
_
.
La
.
p phu
.
o
.
ng tr`nh da
.
c tru
.
ng
[AE[ =

27 5
5 3

= 0
2
30 + 56 = 0.
Gia i phu
.
o
.
ng tr`nh da
.
c tru
.
ng ta c o
1
= 2,
2
= 28.
2
+
T`m c ac vecto
.
rieng chua

n ta

c. De

t`m to
.
a do
.
cu a cac vecto
.
rieng ta la
`
n lu
.
o
.
.
t gia i cac he
.
phu
.
o
.
ng tr`nh
(27
i
)
1
5
2
= 0,
5
1
+ (3
i
)
2
= 0
246 Chu
.
o
.
ng 6. Da
.
ng to`an phu
.
o
.
ng v` a u
.
ng du
.
ng
khi
1
= 2 v`a
2
= 28.
a) Ne

u
1
= 2 th` ta co he
.
25
1
5
2
= 0,
5
1
+
2
= 0.
Do do
2
= 5
1
. Da
.
t
1
= . Khi do
2
= 5 v`a do do vecto
.
rieng co
da
.
ng
u = e
1
+ 5e
2
.
b) Ne

u
2
= 28 th` ta gia i he
.

1
5
2
= 0,
5
1
25
2
= 0
v`a thu du
.
o
.
.
c
1
= 5
2
. Da
.
t
2
= th`
1
= 5 v`a thu du
.
o
.
.
c vecto
.
rieng
v = 5e
1
+e
2
.
T` u
.
do thu du
.
o
.
.
c c ac vecto
.
rieng chua

n hoa
c
1
=
1

26
e
1
+
5

26
e
2
, c
2
=
5

26
e
1
+
1

26
e
2
.
3
+
La
.
p phep bie

n do

i tru
.
.
c giao.
Tru
.
o
.
c he

t ta la
.
p ma tra
.
n chuye

n T t` u
.
co
.
so
.
(e) sang co
.
so
.
(c)
_

_
1

26

26
5

26
1

26
_

_
V` (e) v`a (c) de
`
u l`a nh u
.
ng co
.
so
.
tru
.
.
c chua

n nen T l`a ma tra


.
n tru
.
.
c
giao. No tu
.
o
.
ng u
.
ng vo
.
i phep bie

n do

i tru
.
.
c giao cu a cac bie

n x
1
v`a x
2
:
x
1
=
1

26
x

26
x

2
,
x
2
=
5

26
x

1
+
1

26
X

2
.
6.1. Da
.
ng to`an phu
.
o
.
ng 247
T` u
.
do ta co
() = 27
_
1

26
x

26
x

2
_
2
10
_
1

26
x

26
x

2
__
5

26
x

1
+
1

26
x

2
_
+ 3
_
5

26
x

1
+
1

26
x

2
_
2
= 2x

1
2
+ 28x

2
2
.
Nha
.
n xet. He
.
th u
.
c cuoi c` ung co the

thu du
.
o
.
.
c ba
`
ng cach t`m ma
tra
.
n B cu a da
.
ng to`an phu
.
o
.
ng trong co
.
so
.
tru
.
.
c chua

n (c). Ta c o
B = T
1
AT = T
T
AT =
_
2 0
0 28
_
v`a do do
() = 2x

1
2
+ 28x

2
2
.
V du
.
6. Du
.
a da
.
ng to`an phu
.
o
.
ng
(x
1
, x
2
, x
3
) = 3x
2
1
+ 2x
2
2
+x
2
3
+ 4x
1
x
2
+ 4x
2
x
3
ve
`
da
.
ng chnh ta

c.
Gia i. 1
+
La
.
p v`a gia i phu
.
o
.
ng tr`nh da
.
c tru
.
ng

3 2 0
2 2 2
0 2 1

= 0
1
= 2,
2
= 1,
3
= 5.
2
+
De

t`m to
.
a do
.
cac vecto
.
rieng ta la
`
n lu
.
o
.
.
t gia i cac he
.
phu
.
o
.
ng
tr`nh
(3
i
)
1
+ 2
2
+ 0
3
= 0,
2
1
+ (2
i
)
2
+ 2
3
= 0,
0
1
+ 2
2
+ (1
i
)
3
= 0
_

_
(6.12)
vo
.
i
1
= 2,
2
= 1 v`a
3
= 5.
248 Chu
.
o
.
ng 6. Da
.
ng to`an phu
.
o
.
ng v` a u
.
ng du
.
ng
a) Gia su
.

1
= 2. Ha
.
ng cu a ma tra
.
n
A2E =
_

_
1 2 0
2 0 2
0 2 1
_

_
cu a he
.
phu
.
o
.
ng tr`nh thua
`
n nhat

1
+ 2
2
= 0,
2
1
+ 2
3
= 0,
2
2

3
= 0
_

_
(6.13)
ba
`
ng 2 nen he
.
nghie
.
m co
.
ba n cu a he
.
(6.13) ch go
`
m mo
.
t nghie
.
m. T` u
.
(6.13) suy ra
`
ng
1
= 2,
2
= ,
3
= 2. Do do vecto
.
rieng u
.
ng
vo
.
i
1
= 2 l`a
u
1
(2, , 2)
v`a sau khi chua

n hoa ta thu du
.
o
.
.
c
c =
2
3
e
1

1
3
e
2

2
3
e
3
trong do e
1
, e
2
, e
3
l`a co
.
so
.
m`a da
.
ng to`an phu
.
o
.
ng co ma tra
.
n l`a A.
b) Gia su
.
= 1. Ha
.
ng cu a ma tra
.
n
A+E =
_

_
4 2 0
2 3 2
0 2 2
_

_
cu a he
.
phu
.
o
.
ng tr`nh thua
`
n nhat
4
1
+ 2
2
= 0,
2
1
+ 3
2
+ 2
3
= 0,
2
2
+ 2
3
= 0
_

_
(6.14)
6.1. Da
.
ng to`an phu
.
o
.
ng 249
ba
`
ng 2 nen he
.
nghie
.
m co
.
ba n cu a no ch go
`
m mo
.
t nghie
.
m. T` u
.
(6.14)
suy ra
`
ng
1
= ,
2
= 2,
3
= 2, R. Do do vecto
.
rieng tu
.
o
.
ng
u
.
ng vo
.
i
2
= 1 se l`a
u
2
(, 2, 2)
v`a sau khi chua

n hoa ta thu du
.
o
.
.
c
c
2
=
1
3
e
1

2
3
e
2
+
2
3
e
3
.
c) Gia su
.

3
= 5. Tu
.
o
.
ng tu
.
.
nhu
.
tren, t` u
.
he
.
phu
.
o
.
ng tr`nh
2
1
+ 2
2
= 0,
2
1
3
2
+ 2
3
= 0,
2
2
4
3
= 0
_

_
ta co
1
= 2,
2
= 2,
3
= v` a vecto
.
rieng tu
.
o
.
ng u
.
ng co da
.
ng
u
3
(2, 2, ), R
v`a sau khi chua

n hoa ta c o
c
3
=
2
3
e
1
+
2
3
e
2
+
1
3
e
3
.
T` u
.
cac khai trie

n cu a c
1
, c
2
, c
3
suy ra
`
ng ch ung la
.
p th`anh mo
.
t co
.
so
.
tru
.
.
c chua

n cu a khong gian R
3
.
Ma tra
.
n cu a phep bie

n do

i tru
.
.
c giao c o da
.
ng
T =
_

_
2
3
1
3
2
3

1
3

2
3
2
3

1
3
2
3
1
3
_

_
vo
.
i cac cong th u
.
c bie

n do

i to
.
a d o
.
x
1
=
2
3
x

1
+
1
3
x

2
+
2
3
x

3
,
x
2
=
1
3
x

2
3
x

2
+
2
3
x

3
,
x
3
=
2
3
x

1
+
2
3
x

2
+
1
3
x

3
.
250 Chu
.
o
.
ng 6. Da
.
ng to`an phu
.
o
.
ng v` a u
.
ng du
.
ng
Vo
.
i phep bie

n do

i do ta c o
() = 2x

1
2
x

2
2
+ 5x

3
2
.
V du
.
7. Du
.
a da
.
ng to`an phu
.
o
.
ng sau day ve
`
da
.
ng chnh ta

c
(x
1
, x
2
, x
3
) = 6x
2
1
+ 3x
2
2
+ 3x
2
3
+ 4x
1
x
2
+ 4x
1
x
3
8x
2
x
3
.
Gia i. 1
+
La
.
p v`a gia i phu
.
o
.
ng tr`nh da
.
c tru
.
ng

6 2 2
2 3 4
2 4 3

= 0
1
=
2
= 7,
3
= 2.
2
+
De

t`m to
.
a do
.
cu a cac vecto
.
rieng ta ca
`
n gia i cac he
.
phu
.
o
.
ng
tr`nh thua
`
n nhat
(6
i
)
1
+ 2
2
+ 2
4
= 0,
2
1
+ (3
i
)
2
4
3
= 0,
2
1
4
2
+ (3
i
)
3
= 0
_

_
la
`
n lu
.
o
.
.
t vo
.
i
1
=
2
= 7,
3
= 2.
a) Gia su
.
= 7. Ha
.
ng cu a ma tra
.
n A7E cu a he
.

1
+ 2
2
+ 2
3
= 0,
2
1
4
2
4
3
= 0,
2
1
4
2
4
3
= 0
_

_
(6.15)
l`a ba
`
ng 1 nen he
.
co hai nghie
.
m co
.
ba n. He
.
(6.15) du
.
o
.
.
c du
.
a ve
`
mo
.
t
phu
.
o
.
ng tr`nh

1
= 2
2
+ 2
3
.
Do do nghie
.
m to

ng quat cu a he
.
(6.15) co da
.
ng
1
= 2 + 2,
2
= ,

3
= :
(2 + 2, , ), (6.16)
6.1. Da
.
ng to`an phu
.
o
.
ng 251
t u
.
c l` a ho
.
cac vecto
.
rieng phu
.
thuo
.
c hai tham so v`a .
Ta lay ra hai vecto
.
tru
.
.
c giao n`ao do cu a ho
.
u = 2(+)e
1
+e
2
+
e
3
. Ch a

ng ha
.
n da
.
t = 0, = 1 th` thu du
.
o
.
.
c vecto
.
rieng
u
1
(2, 0, 1)
v`a sau khi chua

n hoa ta du
.
o
.
.
c
c
1
=
_
2

5
, 0,
1

5
_
.
De

co vecto
.
th u
.
hai u
2
ta ca
`
n cho
.
n v`a sao cho u
1
, u
2
) = 0 t u
.
c
l`a
2 2( +) + = 0 4 + 5 = 0.
Ta co the

cho
.
n = 5, = 4 v`a t` u
.
(6.16) ta co
u
2
(2, 5, 4)
v`a sau khi chua

n hoa ta c o
c
2
=
_
2
3

5
,

5
3
,
4
3

5
_
.
b) Gia su
.
= 2. Ta c o
8
1
+ 2
2
+ 2
3
= 0,
2
1
+ 5
2
4
3
= 0,
2
1
4
2
+ 5
3
= 0
_

_
Ha
.
ng cu a ma tra
.
n cu a he
.
ba
`
ng 2 nen he
.
co
.
ba n ch go
`
m mo
.
t nghie
.
m.
Cha

ng ha
.
n gia i hai phu
.
o
.
ng tr`nh cuoi ta co
2
=
3
v`a
1
=

2
2
v`a
do do
1
=

1
2
=

3
2
.
Da
.
t
1
= ta co ho
.
vecto
.
rieng phu
.
thuo
.
c mo
.
t tham so
u
3
(, 2, 2), R
252 Chu
.
o
.
ng 6. Da
.
ng to`an phu
.
o
.
ng v` a u
.
ng du
.
ng
v`a sau khi chua

n hoa ta du
.
o
.
.
c
c
3
=
_
1
3
,
2
3
,
2
3
_
.
Ro r`ang l`a c
1
, c
2
, c
3
l`a mo
.
t co
.
so
.
tru
.
.
c chua

n cu a khong gian R
3
v`a
ma tra
.
n chuye

n ve
`
co
.
so
.
mo
.
i n`ay l`a ma tra
.
n tru
.
.
c giao da
.
ng
T =
_

_
2

5
2
3

5
1
3
0

5
3

2
3
1

4
3

2
3
_

_
T` u
.
do thu du
.
o
.
.
c da
.
ng to`an phu
.
o
.
ng trong co
.
so
.
mo
.
i c
1
, c
2
, c
3
l`a
B = T
T
AT =
_

_
7 0 0
0 7 0
0 0 2
_

_
t u
.
c l`a
() = 7x

1
2
+ 7x

2
2
2x

3
2
,
trong do
x
1
=
2

5
x

1
+
2
3

5
x

2
+
1
3
x

3
,
x
2
=

5
3
x

2
3
x

3
,
x
3
=
1

5
x

4
3

5
x

2
3
x

3
.
V du
.
8. T`m phep bie

n do

i tru
.
.
c giao du
.
a da
.
ng to`an phu
.
o
.
ng
(x
1
, x
2
, x
3
) = x
2
1
8x
1
x
2
16x
1
x
3
+ 7x
2
2
8x
2
x
3
+x
2
3
ve
`
da
.
ng chnh ta

c.
6.1. Da
.
ng to`an phu
.
o
.
ng 253
Gia i. 1
+
Ma tra
.
n cu a da
.
ng to`an phu
.
o
.
ng l`a
A =
_

_
1 4 8
4 7 4
8 4 1
_

_
T` u
.
do ta co
[AE[ =

1 4 8
4 7 4
8 4 1

= 0
1
= 9,
2
=
3
= 9
2
+
T`m cac vecto
.
rieng
De

t`m to
.
a do
.
cu a cac vecto
.
rieng ta c a
`
n gia i cac he
.
phu
.
o
.
ng tr`nh
thua
`
n nhat
(1
i
)
1
4
2
8
3
= 0,
4
1
+ (7
i
)
2
4
3
= 0,
8
1
4
2
+ (1
i
)
3
= 0
_

_
(6.17)
la
`
n lu
.
o
.
.
t vo
.
i
1
= 9,
2
=
3
= 9.
a) Gia su
.

1
= 9. Khi do t` u
.
(6.17) ta co
10
1
4
2
8
3
= 0,
4
1
+ 16
2
4
3
= 0,
8
1
4
2
+ 10
3
= 0
_

_
hay l`a
5
1
2
2
4
3
= 0,

1
4
2
+
3
= 0,
4
1
+ 2
2
5
3
= 0.
_

_
V` ha
.
ng cu a ma tra
.
n cu a he
.
ba
`
ng 2 nen he
.
co nghie
.
m khac 0. Ta gia i
he
.
hai phu
.
o
.
ng tr`nh da
`
u
5
1
2
2
4
3
= 0,

1
4
2
+
3
= 0
254 Chu
.
o
.
ng 6. Da
.
ng to`an phu
.
o
.
ng v` a u
.
ng du
.
ng
v`a thu du
.
o
.
.
c nghie
.
m to

ng quat l`a
u(2, , 2), R.
Do l`a ho
.
vecto
.
rieng (phu
.
thuo
.
c mo
.
t tham so) u
.
ng vo
.
i gia tri
.
rieng

1
= 9. Sau khi chua

n hoa ta thu du
.
o
.
.
c
c
1
=
_
2
3
,
1
3
,
2
3
_
.
b) Gia su
.

2
=
3
= 9. Khi do t` u
.
(6.17) thu du
.
o
.
.
c he
.
phu
.
o
.
ng tr`nh
thua
`
n nhat
8
1
4
2
8
3
= 0,
4
1
2
2
4
3
= 0,
8
1
4
2
8
3
= 0
_

_
hay l`a
2
1
+
2
+ 2
3
= 0,
2
1
+
2
+ 2
3
= 0,
2
2
+
2
+ 2
3
= 0.
_

_
Ha
.
ng cu a ma tra
.
n cu a he
.
ba
`
ng 1 nen he
.
nghie
.
m co
.
ba n cu a no go
`
m
hai nghie
.
m. Nghie
.
m to

ng quat cu a he
.
co da
.
ng
v(, 2 2, ), , R,
2
+
2
,= 0. (6.18)
T` u
.
nghie
.
m to

ng quat n`ay ta r ut ra hai vecto


.
rieng tru
.
.
c giao v
1
v`a v
2
tu
.
o
.
ng u
.
ng vo
.
i gia tri
.
rieng
2
=
3
= 9. De

co v
1
ta cho = 1, = 0
v`a thu du
.
o
.
.
c
v
1
= (1, 2, 0). (6.19)
De

t`m v
2
ta ca
`
n xac di
.
nh v`a trong (6.18) sao cho thoa man die
`
u
kie
.
n tru
.
.
c giao gi u
.
a v
1
v`a v
2
, t u
.
c l`a v
1
, v
2
) = 0. T` u
.
(6.18) v`a (6.19)
ta co
+ 4 + 4 = 0 =
4
5
.
6.1. Da
.
ng to`an phu
.
o
.
ng 255
Do va
.
y, ta co the

lay = 5 v`a khi do t` u


.
(6.18) suy ra
v
2
= (4, 2, 5).
Sau khi chua

n hoa v
1
v`a v
2
ta thu du
.
o
.
.
c
c

=
_
1

5
,
2

5
, 0
_
c
3
=
_
4
3

5
,
2
3

5
,

5
3
_
.
(Lu
.
u y ra
`
ng c
1
c
2
, c
1
c
3
v` c
1
v`a c
2
, c
3
l`a cac vecto
.
rieng tu
.
o
.
ng
u
.
ng vo
.
i hai gia tri
.
rieng khac nhau nen ch ung tru
.
.
c giao vo
.
i nhau).
3
+
Xac di
.
nh phep bie

n do

i tru
.
.
c giao. Trong co
.
so
.
tru
.
.
c chua

n v` u
.
a
thu du
.
o
.
.
c c
1
, c
2
, c
3
da
.
ng to`an phu
.
o
.
ng da cho du
.
o
.
.
c du
.
a ve
`
da
.
ng chnh
ta

c
() = 9y
2
1
+ 9y
2
2
+ 9y
2
3
nh`o
.
ma tra
.
n tru
.
.
c giao
T =
_

_
2
3
1

4
3

5
1
3

2

2
3

5
2
3
0

5
3
_

_
vo
.
i phep bie

n do

i tu
.
o
.
ng u
.
ng l`a
x
1
=
2
3
y
1
+
1

5
y
2

4
3

5
y
3
,
x
2
=
1
3
y
1

5
y
2

2
3

5
y
3
,
x
3
=
2
3
y
1
+

5
3
y
3
.
256 Chu
.
o
.
ng 6. Da
.
ng to`an phu
.
o
.
ng v` a u
.
ng du
.
ng
B
`
AI T

A
.
P
Trong cac b`ai toan sau day hay vie

t ma tra
.
n cu a da
.
ng to`an phu
.
o
.
ng
co bie

u th u
.
c to
.
a do
.
sau trong khong gian R
3
(1-4) v`a trong R
4
(5-6).
1. x
2
1
+x
2
2
3x
1
x
2
. (DS.
_

_
1 3/2 0
3/2 1 0
0 0 0
_

_)
2. 2x
2
1
+ 3x
2
2
x
2
3
+ 4x
1
x
2
6x
1
x
3
+ 10x
2
x
3
.
(DS.
_

_
2 2 3
2 3 5
3 5 1
_

_
)
3. x
2
1
+x
2
2
+x
2
3
+ 4x
1
x
2
+ 4x
2
x
3
+ 4x
1
x
3
.
(DS.
_

_
1 2 2
2 1 2
2 2 1
_

_
)
4. 4x
2
1
+x
2
2
+ 9x
2
3
4x
1
x
2
6x
2
x
3
+ 12x
1
x
3
.
(DS.
_

_
4 2 6
2 1 3
6 3 9
_

_)
5. 4x
2
1
+x
2
3
2x
2
4
x
1
x
2
+ 8x
1
x
4
5x
2
x
4
.
(DS.
_

_
4
1
2
0 4

1
2
0 0
5
2
0 0 1 0
4
5
2
0 2
_

_
)
6. 2x
1
x
2
6x
1
x
3
6x
2
x
4
+ 2x
3
x
4
.
6.1. Da
.
ng to`an phu
.
o
.
ng 257
(DS.
_

_
0 1 3 0
1 0 0 3
3 0 0 1
0 3 1 0
_

_
)
Trong c ac b`ai toan 7-8, t`m ma tra
.
n cu a mo

i da
.
ng to`an phu
.
o
.
ng
7.
_
x
1
x
2
_
_
1 3
4 5
__
x
1
x
2
_
(DS.
_

_
1
7
2
7
2
5
_

_)
8.
_
x
1
x
2
x
3
_
_

_
1 0 2
2 4 1
3 0 1
_

_
_

_
x
1
x
2
x
3
_

_. (DS.
_

_
1 1
5
2
1 4
1
2
5
2
1
2
1
_

_
)
9. Cho c ac da
.
ng to`an phu
.
o
.
ng sau day du
.
o
.
.
c vie

t du
.
o
.
i da
.
ng ma tra
.
n.
Hay vie

t c ac da
.
ng to`an phu
.
o
.
ng do du
.
o
.
i da
.
ng thong thu
.
`o
.
ng
1)
_
x
1
x
2
x
3
_
_

_
3 0 1
0 2 1
1 1 2
_

_
_

_
x
1
x
2
x
3
_

_
(DS. 3x
2
1
2x
1
x
3
2x
2
2
+ 2x
2
x
3
+ 2x
2
3
)
2)
_
x
1
x
2
x
3
_
_

_
4 1 0
1 3 1
0 1 1
_

_
_

_
x
1
x
2
x
3
_

_
.
(DS. 4x
2
1
+ 2x
1
x
2
+ 3x
2
2
2x
2
x
3
x
2
3
)
10. Vie

t c ac da
.
ng to`an phu
.
o
.
ng sau day du
.
o
.
i da
.
ng ma tra
.
n.
1) 3x
2
1
+ x
2
2
x
1
x
2
. (DS.
_
x
1
x
2
_
_

_
3
1
2

1
2
1
_

_
_
x
1
x
2
_
)
2) x
2
1
+x
2
3
2x
1
x
2
+ 5x
1
x
3
.
258 Chu
.
o
.
ng 6. Da
.
ng to`an phu
.
o
.
ng v` a u
.
ng du
.
ng
(DS.
_
x
1
x
2
x
3
_
_

_
1 1
5
2
1 0 0
5
2
0 1
_

_
_

_
x
1
x
2
x
3
_

_)
3) 2x
2
1
+ 3x
2
2
2x
2
3
+x
1
x
2
+ 2x
1
x
3
+ 3x
2
x
3
.
(DS.
_
x
1
x
2
x
3
_
_

_
2
1
2
1
1
2
3
3
2
1
3
2
2
_

_
_

_
x
1
x
2
x
3
_

_
)
Trong cac b`ai toan sau day (11-14) t`m da
.
ng to`an phu
.
o
.
ng thu
du
.
o
.
.
c t` u
.
da
.
ng da cho bo
.
i phep bie

n do

i da ch ra
11. (x
1
, x
2
) = 3x
2
1
x
2
2
+ 4x
1
x
2
; x
1
= 2y
1
y
2
, x
2
= y
1
+y
2
.
(DS.
1
(y
1
, y
2
) = 19y
2
1
2y
2
2
10y
1
y
2
)
12. (x
1
, x
2
, x
3
) = 2x
2
1
+ 3x
2
2
x
2
3
+x
1
x
2
;
x
1
= y
1
+ 2y
2
; x
2
= 3y
1
+y
2
+y
3
; x
3
= 2y
1
y
2
.
(DS.
1
(y
1
, y
2
, y
3
) = 22y
2
1
+ 12y
2
2
+ 3y
2
3
+ 11y
1
y
2
+17y
1
y
3
+ 8y
2
y
3
)
13.
1
(x
1
, x
2
, x
3
) = 2x
2
2
+ 4x
2
3
2x
1
x
2
+x
2
x
3
;
x
1
= y
1
+y
2
y
3
, x
2
= y
1
y
2
+y
3
, x
3
= y
3
+y
2
.
(DS.
1
(y
1
, y
2
, y
3
) = 7y
2
2
+ 9y
2
3
3y
1
y
2
+ 5y
1
y
3
))
14. (x
1
, x
2
, x
3
) = x
2
1
2x
2
2
+x
2
3
4x
1
x
2
2x
2
x
3
;
x
1
= y
1
+ 2y
2
, x
2
= y
2
, x
3
= y
2
y
3
.
(DS.
1
(y
1
, y
2
, y
3
) = y
2
1
7y
2
2
+y
2
3
)
D` ung phu
.
o
.
ng phap Lagrange du
.
a cac da
.
ng to`an phu
.
o
.
ng sau ve
`
da
.
ng chnh ta

c (15-19)
15. 2x
2
1
+ 3x
2
2
+ 4x
2
3
2x
1
x
2
+ 4x
1
x
3
3x
2
x
3
.
(DS. = 2(x
1

1
2
x
2
+x
3
)
2
+
5
2
(x
2

3
5
x
3
)
2
+
11
10
x
2
3

1
(y
1
, y
2
, y
3
) = 2y
2
1
+
5
2
y
2
2
+
11
10
y
2
3
)
6.1. Da
.
ng to`an phu
.
o
.
ng 259
16. (x
1
, x
2
, x
3
) = 9x
2
1
+ 6x
2
2
+ 6x
2
3
6x
1
x
2
6x
1
x
3
+ 12x
2
x
3
.
(DS. () = (3x
1
x
2
x
3
)
2
+ 5(x
2
+x
3
)
2
(y
1
, y
2
, y
3
) = y
2
1
+ 5y
2
2
+ 0 y
2
3
)
17. x
2
1
+ 5x
2
2
4x
2
3
+ 2x
1
x
2
4x
1
x
3
.
(DS. = (x
1
+x
2
2x
3
)
2
+ 4x
2
2
8x
2
3
(y
1
, y
2
, y
3
) = y
2
1
+ 4y
2
2
8y
2
3
)
18. x
1
x
2
+ 2x
1
x
3
+ 4x
2
x
3
.
(Ch da

n v`a Dap so: D` ung phep bie

n do

i phu
.
_

_
x
1
= y
1
x
2
= y
1
+y
2
x
3
= y
3
v`a thu du
.
o
.
.
c da
.
ng co ch u
.
a b`nh phu
.
o
.
ng
() = (y
1
+
1
2
y
2
+ 3y
3
)
2

1
4
(y
2
2y
3
)
2
8y
2
3

1
= z
2
1

1
4
z
2
2
8z
2
3
)
19. 4x
1
x
2
5x
2
x
3
(Ch da

n v`a Dap so: D` ung phep bie

n do

i phu
.
_

_
x
1
=
1
2
(y
1
y
2
)
x
2
=
1
2
(y
1
+y
2
)
x
3
= y
3
v`a thu du
.
o
.
.
c
() =
_
y
1

5
4
y
3
_
2

_
y
2
+
5
4
y
3
_
2
= z
2
1
z
2
2
+ 0 z
2
3
)
D` ung phu
.
o
.
ng phap Jacobi de

du
.
a cac da
.
ng to`an phu
.
o
.
ng ve
`
da
.
ng
chnh ta

c (20-25)
20. 3x
2
1
+ 4x
1
x
2
2x
1
x
3
+ 2x
2
2
2x
2
x
3
+ 6x
2
3
.
260 Chu
.
o
.
ng 6. Da
.
ng to`an phu
.
o
.
ng v` a u
.
ng du
.
ng
(DS. () = 3y
2
1
+
2
3
y
2
2
+
11
2
y
2
3
nh`o
.
phep bie

n do

i
_

_
x
1
= y
1

2
3
y
2
,
x
2
= y
2
+
1
2
y
3
,
x
3
= y
3
.)
21. 5x
2
1
2x
1
x
2
+ 4x
1
x
3
+ 5x
2
2
+ 4x
2
x
3
+ 3x
2
3
.
(DS. () = 5y
2
1
+
24
5
y
2
2
+y
2
3
nh`o
.
phep bie

n do

i
_

_
x
1
= y
1
+
1
5
y
2

1
2
y
3
,
x
2
= y
2

1
2
y
3
,
x
3
= y
3
.)
22. x
2
1
+ 5x
2
2
+ 2x
3
2
+ 4x
1
x
2
+ 2x
1
x
3
+ 4x
2
x
3
.
(DS. () = y
2
1
+y
2
2
+y
2
3
nh`o
.
phep bie

n do

i:
_

_
x
1
= y
1
2y
2
y
3
,
x
2
= y
2
,
x
3
= y
3
.)
23. 5x
2
1
+x
2
2
+ 3x
2
3
+ 4x
1
x
2
2x
1
x
3
2x
2
x
3
.
(DS. () =
1
5
y
2
1
+ 5y
2
2
+y
2
3
nh`o
.
phep bie

n do

i
_

_
x
1
= y
1

2
5
y
2
y
3
,
x
2
= y
2
+ 3y
3
,
x
3
= y
3
.)
24. x
2
1
+x
2
2
+ 5x
2
3
4x
1
x
2
2x
1
x
3
+ 4x
2
x
3
.
6.1. Da
.
ng to`an phu
.
o
.
ng 261
(DS. () = y
2
1
3y
2
2
+ 4y
2
3
nh`o
.
phep bie

n do

i
_

_
x
1
= y
1
+ 2y
2
+y
3
,
x
2
= y
2
,
x
3
= y
3
.)
25. 3x
2
1
+ 2x
1
x
2
2x
1
x
3
+x
2
3
+ 4x
2
x
3
(DS. () = 3y
2
1

1
3
y
2
2
+ 17y
2
3
nh`o
.
phep bie

n do

i
_

_
x
1
= y
1

1
3
y
2
2y
3
,
x
2
= y
2
+ 7y
3
,
x
3
= y
3
.)
Trong cac b`ai toan sau day (26-35) t`m phep bie

n do

i tru
.
.
c giao du
.
a
mo

i da
.
ng to`an phu
.
o
.
ng da cho ve
`
da
.
ng chnh ta

c v`a vie

t da
.
ng chnh
ta

c do.
26. 2x
2
1
4x
1
x
2
+ 5x
2
2
.
(DS.
x
1
=
2

5
y
1
+
1

5
y
2
x
2
=
1

5
y
1
+
2

5
y
2
_

_
() = y
2
1
+ 6y
2
2
)
27. x
2
1
+x
2
2
+ 4x
1
x
2
.
(DS.
x
1
=
1

2
y
1
+
1

2
y
2
,
x
2
=
1

2
y
1

2
y
2
_

_
() = 3y
2
1
y
2
2
)
28. 5x
2
1
+ 12x
1
x
2
.
(DS.
x
1
=
3

13
y
1

13
y
2
,
x
2
=
2

13
y
1
+
3

13
y
2
_

_
() = 9y
2
1
4y
2
2
)
29. 7x
2
1
+ 3x
2
2
+ 6

5x
1
x
2
.
262 Chu
.
o
.
ng 6. Da
.
ng to`an phu
.
o
.
ng v` a u
.
ng du
.
ng
(DS.
x
1
=
3

14
y
1
+
_
5
14
y
2
x
2
=
_
5
14
y
1

14
y
2
_

_
() = 12y
2
1
2y
2
2
)
30. 2x
2
1
4

5x
1
x
2
+ 3x
2
2
.
(DS.
x
1
=
2
3
y
1
+

5
3
y
2
x
2
=

5
3
y
1
+
2
3
y
2
_

_
() = 7y
2
1
2y
2
2
)
31. (x
1
, x
2
) = 4x
1
x
2
(DS.
x
1
=
1

2
y
1

2
y
2
x
2
=
1

2
y
1
+
1

2
y
2
_

_
(y
1
, y
2
) = 2y
2
1
2y
2
2
)
32. 3x
2
1
+ 6x
1
x
2
+ 3x
2
2
.
(DS.
x
1
=
1

2
y
1

2
y
2
,
x
2
=
1

2
y
1
+
1

2
y
2
_

_
() = 6y
2
1
)
33. 6x
2
1
+ 5x
2
2
+ 7x
2
3
4x
1
x
2
+ 4x
1
x
3
(DS.
x
1
=
2
3
y
1

1
3
y
2
+
2
3
y
3
,
x
2
=
1
3
y
1
+
2
3
y
2
+
2
3
y
3
,
x
3
=
2
3
y
1
+
2
3
y
2

1
3
y
3
_

_
() = 9y
2
1
+ 6y
2
2
+ 3y
2
3
)
34. 2x
2
1
+x
2
2
+ 3x
2
3
4

2x
2
x
3
.
(DS. x
1
= y
1
, x
2
=
1

3
y
2
+
_
2
3
y
3
, x
3
=
_
2
3
y
2
+
1

3
y
3
;
() = 2y
2
1
+ 5y
2
2
y
2
3
)
35. 2x
2
1
+ 5x
2
2
+ 2x
2
3
4x
1
x
2
2x
1
x
3
+ 4x
2
x
3
.
(DS. x
1
=
1

2
y
1
+
1

6
y
2
+
1

3
y
3
, x
2
=
2

6
y
2
+
1

3
y
3
,
x
3
=
1

2
y
1

6
y
2

3
y
3
; () = y
2
1
+ 7y
2
2
+y
2
3
)
6.2. D
-
u
.
a phu
.
o
.
ng tr`nh to

ng quat cu a du
.
`o
.
ng ba
.
c hai v`a ma
.
t ba
.
c hai ve
`
da
.
ng chnh ta

c 263
6.2 D
-
u
.
a phu
.
o
.
ng tr`nh to

ng quat cu

a
du
.
`o
.
ng ba
.
c hai v`a ma
.
t b a
.
c hai ve
`
da
.
ng
chnh ta

c
1

Xet phu
.
o
.
ng tr`nh to

ng quat cu a du
.
`o
.
ng ba
.
c hai
a
11
x
2
+ 2a
12
xy +a
22
y
2
+ 2a
13
x + 2a
23
y +a
33
= 0. (6.20)
To

ng cu a ba so ha
.
ng da
`
u tien
(x, y) = a
11
x
2
+ 2a
12
xy +a
22
y
2
(6.21)
l`a da
.
ng to`an phu
.
o
.
ng cu a cac bie

n x v`a y v`a du
.
o
.
.
c go
.
i l`a da
.
ng to`an
phu
.
o
.
ng u
.
ng vo
.
i phu
.
o
.
ng tr`nh (6.20). Ma tra
.
n cu a da
.
ng to`an phu
.
o
.
ng
n`ay co da
.
ng
A =
_
a
11
a
12
a
12
a
22
_
.
1
+
Ne

u detA > 0 th` (6.20) l`a phu


.
o
.
ng tr`nh cu a du
.
`o
.
ng da
.
ng eliptic.
2
+
Ne

u detA < 0 th` (6.20) l`a phu


.
o
.
ng tr`nh du
.
`o
.
ng da
.
ng hy-
pecbolic.
3
+
Ne

u detA = 0 th` (6.20) l`a phu


.
o
.
ng tr`nh du
.
`o
.
ng da
.
ng parabolic.
Trong tru
.
`o
.
ng ho
.
.
p khi detA ,= 0 th` (6.20) xac di
.
nh du
.
`o
.
ng co tam
die

m. Ne

u detA = 0 th` (6.20) l`a phu


.
o
.
ng tr`nh du
.
`o
.
ng khong co tam
die

m. Hu
.
o
.
ng cu a cac vecto
.
rieng tru
.
.
c giao cu a ma tra
.
n da
.
ng to`an
phu
.
o
.
ng tu
.
o
.
ng u
.
ng vo
.
i phu
.
o
.
ng tr`nh (6.20) go
.
i l`a hu
.
o
.
ng chnh cu a
du
.
`o
.
ng xac di
.
nh bo
.
i phu
.
o
.
ng tr`nh (6.20).
Ngu
.
`o
.
i ta ch u
.
ng minh ra
`
ng to
`
n ta
.
i he
.
to
.
a do
.
Dec ac vuong goc m`a
trong do phu
.
o
.
ng tr`nh to

ng quat (6.20) cu a du
.
`o
.
ng ba
.
c hai co da
.
ng
chnh ta

c.
De

t`m he
.
to
.
a do
.
do ta tie

n h`anh nhu
.
sau.
264 Chu
.
o
.
ng 6. Da
.
ng to`an phu
.
o
.
ng v` a u
.
ng du
.
ng
1
+
T`m phep bie

n do

i tru
.
.
c giao du
.
a da
.
ng to`an phu
.
o
.
ng tu
.
o
.
ng u
.
ng
vo
.
i phu
.
o
.
ng tr`nh da cho ve
`
da
.
ng chnh ta

c.
2
+
Du
.
.
a theo phep bie

n do

i n`ay ta t`m c ac hu
.
o
.
ng chnh cu a du
.
`o
.
ng,
t u
.
c l`a t`m cac vecto
.
rieng tru
.
.
c chua

n c
1
v`a c
2
cu a ma tra
.
n da
.
ng to`an
phu
.
o
.
ng (6.21).
3
+
T`m phu
.
o
.
ng tr`nh cu a du
.
`o
.
ng da cho trong he
.
to
.
a do
.
Oc
1
c
2
.
4
+
Trong phu
.
o
.
ng tr`nh thu du
.
o
.
.
c ta bo

sung de

thu du
.
o
.
.
c b`nh
phu
.
o
.
ng du ro
`
i t`m cac to
.
a do
.
cu a die

m O

l`a goc cu a he
.
to
.
a do
.
ca
`
n
t`m. Trong he
.
to
.
a do
.
t`m du
.
o
.
.
c O

c
1
c
2
phu
.
o
.
ng tr`nh cu a du
.
`o
.
ng da
cho c o da
.
ng chnh ta

c.
2

. Xet phu
.
o
.
ng tr`nh to

ng quat cu a ma
.
t ba
.
c hai
a
11
x
2
+a
22
y
2
+a
33
z
2
+ 2a
12
xy + 2a
13
xz + 2a
23
yz +bx +by +ez +f = 0,
(6.22)
trong do t nhat mo
.
t he
.
so a
ij
,= 0, i = 1, 3, j = 1, 3.
To

ng cu a sau so ha
.
ng da
`
u cu a phu
.
o
.
ng tr`nh
(x, y, z) = a
11
x
2
+a
12
y
2
+a
33
z
2
+ 2a
12
xy + 2a
13
xz + 2a
23
yz
(6.23)
l`a da
.
ng to`an phu
.
o
.
ng ba bie

n x, y, z v`a du
.
o
.
.
c go
.
i l`a da
.
ng to`an phu
.
o
.
ng
tu
.
o
.
ng u
.
ng vo
.
i phu
.
o
.
ng tr`nh (6.22). Ma tra
.
n cu a da
.
ng l`a
A =
_

_
a
11
a
12
a
13
a
12
a
22
a
23
a
13
a
23
a
33
_

_
.
Trong mu
.
c tru
.
o
.
c da ch u
.
ng to to
`
n ta
.
i phep bie

n do

i tru
.
.
c giao du
.
a
da
.
ng to`an phu
.
o
.
ng (6.23) ve
`
da
.
ng chnh ta

c. Do va
.
y vie
.
c kha o sat v`a
du
.
.
ng ma
.
t ba
.
c hai xac di
.
nh bo
.
i phu
.
o
.
ng tr`nh (6.22) du
.
o
.
.
c tie

n h`anh
tu
.
o
.
ng tu
.
.
nhu
.
trong 1

.
6.2. Du
.
a phu
.
o
.
ng tr`nh to

ng quat ... ve
`
da
.
ng chnh ta

c 265
C

AC V

I DU
.
V du
.
1. Du
.
a phu
.
o
.
ng tr`nh
17x
2
+ 12xy + 8y
2
+ 20

5x + 20 = 0
ve
`
da
.
ng chnh ta

c v`a du
.
.
ng du
.
`o
.
ng xac di
.
nh bo
.
i phu
.
o
.
ng tr`nh do.
Gia i. 1
+
Da
.
ng to`an phu
.
o
.
ng
(x, y) = 17x
2
+ 12xy + 8y
2
tu
.
o
.
ng u
.
ng vo
.
i phu
.
o
.
ng tr`nh da cho c o ma tra
.
n
A =
_
17 6
6 8
_
.
No c o c ac so da
.
c tru
.
ng l`a
1
= 20,
2
= 5. Ta t`m to
.
a do
.
cac vecto
.
rieng cu a A ba
`
ng cach gia i he
.
phu
.
o
.
ng tr`nh
(17
i
)
1
+ 6
2
= 0,
6
1
+ (8
i
)
2
= 0
la
`
n lu
.
o
.
.
t vo
.
i
1
= 20 v`a
2
= 5.
Vo
.
i
1
= 20 ta co
3
1
+ 6
2
= 0
6
1
12
2
= 0
_

1
= 2
2
.
Do do vecto
.
rieng u
.
ng vo
.
i
1
= 20 c o da
.
ng
u(2, ), R
v`a sau khi chua

n hoa ta du
.
o
.
.
c
c
1
=
_
2

5
,
1

5
_
.
266 Chu
.
o
.
ng 6. Da
.
ng to`an phu
.
o
.
ng v` a u
.
ng du
.
ng
Vo
.
i
2
= 5 ta co
12
1
+ 6
2
= 0,
6
1
+ 3
2
= 0
_

2
= 2
1
.
Do do vecto
.
rieng tu
.
o
.
ng u
.
ng vo
.
i
2
= 5 co da
.
ng
v(, 2)
v`a sau khi chua

n hoa ta thu du
.
o
.
.
c vecto
.
rieng chua

n cu a ma tra
.
n A:
c
2
=
_

5
,
2

5
_
.
T` u
.
do thu du
.
o
.
.
c ma tra
.
n chuye

n ve
`
co
.
so
.
mo
.
i (ma tra
.
n cu a phep bie

n
do

i tru
.
.
c giao) co da
.
ng
T =
_

_
2

5
1

5
2

5
_

_
v`a do va
.
y phep bie

n do

i tru
.
.
c giao c a
`
n t`m co da
.
ng
x =
2

5
x

5
y

,
y =
1

5
x

+
2

5
y

.
_

_
(6.24)
No du
.
a da
.
ng to`an phu
.
o
.
ng ve
`
da
.
ng chnh ta

1
= 20x

2
+ 5y

2
.
2
+
Cac vecto
.
co
.
so
.
c
1
v`a c
2
thu du
.
o
.
.
c t` u
.
cac vecto
.
co
.
so
.
e
1
, e
2
ba
`
ng phep bie

n do

i tru
.
.
c giao du
.
o
.
.
c cho bo
.
i cong th u
.
c
c
1
=
2

5
e
1
+
1

5
e
2
,
c
2
=
1

5
e
1
+
2

5
e
2
.
_

_
(6.25)
6.2. Du
.
a phu
.
o
.
ng tr`nh to

ng quat ... ve
`
da
.
ng chnh ta

c 267
3
+
Thay (6.24) v`ao phu
.
o
.
ng tr`nh da cho ta thu du
.
o
.
.
c phu
.
o
.
ng tr`nh
cu a du
.
`o
.
ng trong he
.
to
.
a do
.
Oc
1
c
2
:
20x

2
+ 5y

2
+ 40x

20y

+ 20 = 0
v`a t` u
.
do
(x

+ 1)
2
1
+
(y

2)
2
4
= 1 (6.26)
4
+
Thu
.
.
c hie
.
n phep d`o
.
i he
.
to
.
a do
.
Oc
1
c
2
theo vecto
.

OO

= c
1
+2c
2
ta thu du
.
o
.
.
c he
.
to
.
a do
.
O

c
1
c
2
v` a trong he
.
do phu
.
o
.
ng tr`nh (6.26) co
da
.
ng
x

2
1
+
y

2
4
= 1. (6.27)
Nhu
.
va
.
y phu
.
o
.
ng tr`nh da cho xac di
.
nh elip (h`nh 6.1)
H`nh 6.1
T` u
.
l`o
.
i gia i v`a h`nh ve tr`nh b`ay suy ra cach du
.
.
ng elip (6.27) trong
he
.
O

c
1
c
2
. Da
`
u tien du
.
.
ng he
.
to
.
a do
.
Oc
1
c
2
(thay cho c
1
v`a c
2
co the

du
.
.
ng cac vecto
.

OM

1
= 2e
1
+ e
2
,

OM

2
= e
1
+ 2e
2
); tie

p de

n thu
.
.
c
hie
.
n phep ti
.
nh tie

n song song he
.
do mo
.
t vecto
.

OO

= e
1
+ 2e
2
de

n
O

. Sau c` ung l`a du


.
.
ng elip (6.27).
268 Chu
.
o
.
ng 6. Da
.
ng to`an phu
.
o
.
ng v` a u
.
ng du
.
ng
V du
.
2. Du
.
a phu
.
o
.
ng tr`nh du
.
`o
.
ng cong
x
2
2xy +y
2
10x 6y + 25 = 0
ve
`
da
.
ng chnh ta

c v`a du
.
.
ng du
.
`o
.
ng cong do.
Gia i. Da
.
ng to`an phu
.
o
.
ng tu
.
o
.
ng u
.
ng vo
.
i phu
.
o
.
ng tr`nh da cho
(x, y) = x
2
2xy +y
2
co ma tra
.
n l`a
A =
_
1 1
1 1
_
La
.
p phu
.
o
.
ng tr`nh da
.
c tru
.
ng

1 1
1 1

= 0 hay l`a
2
2 = 0.
T` u
.
do
1
= 2,
2
= 0. Ta t`m to
.
a do
.
cu a cac vecto
.
rieng cu a A ba
`
ng
cach gia i he
.
phu
.
o
.
ng tr`nh
(1
i
)
1

2
= 0,

1
+ (1
i
)
2
= 0
_
la
`
n lu
.
o
.
.
t vo
.
i
1
= 2 v`a
2
= 0.
Vo
.
i
1
= 2 ta co

2
= 0,

2
= 0
_

1
=
2
v`a do do hu
.
o
.
ng chnh tu
.
o
.
ng u
.
ng vo
.
i
1
= 2 du
.
o
.
.
c xac di
.
nh bo
.
i vecto
.
rieng
u = (, ), R
6.2. Du
.
a phu
.
o
.
ng tr`nh to

ng quat ... ve
`
da
.
ng chnh ta

c 269
v`a sau khi chua

n hoa ta c o
c
1
=
_
1

2
,
1

2
_
.
Tu
.
o
.
ng tu
.
.
vo
.
i
2
= 0 ta co
1

2
= 0,
1
+
2
= 0
1
=
2
v`a
hu
.
o
.
ng chnh u
.
ng vo
.
i
2
= 0 xac di
.
nh bo
.
i vecto
.
rieng
v(, ), R
v`a chu a

n hoa ta du
.
o
.
.
c
c
2
=
_
1

2
,
1

2
_
.
Nhu
.
va
.
y ta da chuye

n t` u
.
co
.
so
.
e
1
, e
2
de

n co
.
so
.
tru
.
.
c chua

n c
1
, c
2
,
trong do
c
1
=
1

2
e
1

2
e
2
,
c
2
=
1

2
e
1
+
1

2
e
2
bo
.
i ma tra
.
n chuye

n
T =
_

_
1

2
1

2
1

2
_

_
v`a phep bie

n do

i tru
.
.
c giao tu
.
o
.
ng u
.
ng
x =
1

2
x

+
1

2
y

,
y =
1

2
x

+
1

2
y

.
_

_
(6.28)
De

t`m da
.
ng cu a phu
.
o
.
ng tr`nh du
.
`o
.
ng da cho trong he
.
to
.
a do
.
Oc
1
c
2
ta thay (6.28) v`ao phu
.
o
.
ng tr`nh to

ng quat da cho v`a thu du


.
o
.
.
c
2x

2
x

16

2
y

+ 25 = 0 (6.29)
270 Chu
.
o
.
ng 6. Da
.
ng to`an phu
.
o
.
ng v` a u
.
ng du
.
ng
hay l`a
_
x

2
_
2
= 4

2
_
y

2
2
_
.
Sau phep ti
.
nh tie

n song song cac tru


.
c to
.
a do
.
de

n goc mo
.
i O

=
_

2
2
,
3

2
2
_
, phu
.
o
.
ng tr`nh (6.29) trong he
.
to
.
a do
.
O

XY co da
.
ng chnh
ta

c X
2
= 4

2Y . Su
.
.
sa

p xe

p cu a parabon du
.
o
.
.
c ch ra tren h`nh 6.2.
6.2. Du
.
a phu
.
o
.
ng tr`nh to

ng quat ... ve
`
da
.
ng chnh ta

c 271
H`nh 6.2
V du
.
3. Du
.
a phu
.
o
.
ng tr`nh to

ng quat cu a ma
.
t ba
.
c hai
9x
2
+ 20y
2
+ 20z
2
40yz 36x 4

2y + 4

2z + 4 = 0
ve
`
da
.
ng chnh ta

c v`a du
.
.
ng ma
.
t do.
Gia i. Da
.
ng to`an phu
.
o
.
ng tu
.
o
.
ng u
.
ng vo
.
i phu
.
o
.
ng tr`nh da cho co
da
.
ng
(x, y, z) = 9x
2
+ 20y
2
+ 20z
2
40yz
vo
.
i ma tra
.
n
A =
_

_
9 0 0
0 20 20
0 20 20
_

_.
Ma tra
.
n n`ay c o ba so da
.
c tru
.
ng l`a
1
= 9,
2
= 40,
3
= 0. Do do
da
.
ng chnh ta

c cu a da
.
ng to`an phu
.
o
.
ng () l`a

1
() = 9x

2
+ 40y

2
.
Ta ca
`
n t`m phep bie

n do

i tru
.
.
c giao du
.
a da
.
ng to`an phu
.
o
.
ng tu
.
o
.
ng
u
.
ng vo
.
i phu
.
o
.
ng tr`nh da cho ve
`
da
.
ng chnh ta

c. To
.
a do
.
cu a c ac vecto
.
272 Chu
.
o
.
ng 6. Da
.
ng to`an phu
.
o
.
ng v` a u
.
ng du
.
ng
rieng du
.
o
.
.
c t`m t` u
.
he
.
phu
.
o
.
ng tr`nh
(9
i
)
1
+ 0
2
+ 0
3
= 0,
0
1
+ (20
i
)
2
20
3
= 0,
0
1
20
2
+ (20
i
)
3
= 0
vo
.
i
1
= 9,
2
= 40,
3
= 0.
a) Vo
.
i
1
= 9 ta co
0
1
+ 0
2
+ 0
3
= 0,
0
1
+ 11
2
20
3
= 0,
0
1
20
2
+ 11
3
= 0.
T` u
.
do thu du
.
o
.
.
c vecto
.
rieng u
.
ng vo
.
i
1
= 9 l`a
u(, 0, 0), R, ,= 0
v`a sau khi chua

n hoa ta du
.
o
.
.
c
c
1
= (1, 0, 0).
b) V o
.
i
2
= 40 ta co
31
1
+ 0
2
+ 0
3
= 0,
0
1
20
2
20
3
= 0,
0
1
20
2
20
3
= 0
v`a t` u
.
do thu du
.
o
.
.
c vecto
.
rieng u
.
ng vo
.
i
2
= 40:
v(0, , ), R, ,= 0
v`a sau khi chua

n hoa ta du
.
o
.
.
c
c
2
=
_
0,
1

2
,
1

2
_
.
6.2. Du
.
a phu
.
o
.
ng tr`nh to

ng quat ... ve
`
da
.
ng chnh ta

c 273
c) Vo
.
i
3
= 0 ta co vecto
.
rieng tu
.
o
.
ng u
.
ng l`a
w(0, , ), R, ,= 0
v`a sau khi chua

n hoa ta c o
c
3
=
_
0,
1

2
,
1

2
_
.
Ma tra
.
n chuye

n t` u
.
co
.
so
.
e
1
, e
2
, e
3
de

n co
.
so
.
tru
.
.
c chua

n c
1
, c
2
, c
3
co da
.
ng
T =
_

_
1 0 0
0
1

2
1

2
0
1

2
1

2
_

_
.
Nhu
.
va
.
y phep bie

n do

i tru
.
.
c giao du
.
a da
.
ng to`an phu
.
o
.
ng tu
.
o
.
ng
u
.
ng vo
.
i phu
.
o
.
ng tr`nh da cho ve
`
da
.
ng chnh ta

c c o da
.
ng
x = x

,
y =
1

2
y

+
1

2
z

,
z =
1

2
y

+
1

2
z

.
_

_
(6.30)
Phep bie

n do

i n`ay bie

n cac vecto
.
co
.
so
.
e
1
, e
2
, e
3
th`anh
c
1
= e
1
,
c
2
=
1

2
e
2

2
e
3
,
c
3
=
1

2
e
2
+
1

2
e
3
.
_

_
(6.31)
De

t`m phu
.
o
.
ng tr`nh cu a du
.
`o
.
ng da cho trong he
.
to
.
a do
.
mo
.
i
Oc
1
c
2
c
3
ta the

(6.30) v`ao phu


.
o
.
ng tr`nh to

ng quat da cho v`a thu


du
.
o
.
.
c
9x

2
+ 40y

2
36x

8y

+ 4 = 0
274 Chu
.
o
.
ng 6. Da
.
ng to`an phu
.
o
.
ng v` a u
.
ng du
.
ng
hay l`a
(x

2)
2
3, 6
+
(y

0, 1)
2
0, 81
= 1.
Tie

p theo ta thu
.
.
c hie
.
n phep ti
.
nh tie

n song song he
.
to
.
a do
.
Oc
1
c
2
c
3
mo
.
t vecto
.

OO

= 2c
1
+ 0, 1c
2
v`a thu du
.
o
.
.
c he
.
O

c
1
c
2
c
3
, trong he
.
do
phu
.
o
.
ng tr`nh da cho c o da
.
ng
x

2
a
2
+
y

2
b
2
= 1, a =
_
3, 6, b = 0, 9.
Phu
.
o
.
ng tr`nh n`ay (v`a do do phu
.
o
.
ng tr`nh da cho) xac di
.
nh ma
.
t tru
.
eliptic vo
.
i du
.
`o
.
ng sinh | c
3
.
Du
.
.
ng ma
.
t tru
.
eliptic: c` ung vo
.
i he
.
to
.
a do
.
Oe
1
e
2
e
3
ta du
.
.
ng he
.
to
.
a
do
.
O

c
1
c
2
c
3
, trong do thay cho vie
.
c du
.
.
ng cac vecto
.
(6.31) ta co the

du
.
.
ng cac vecto
.

OM
1
= e
1
,

OM
2
= e
2
e
3
,

OM
3
= e
2
+e
3
.
Su
.
.
sa

p xe

p cu a ma
.
t da cho du
.
o
.
.
c ch ro tren h`nh 6.3
H`nh 6.3
6.2. Du
.
a phu
.
o
.
ng tr`nh to

ng quat ... ve
`
da
.
ng chnh ta

c 275
B
`
AI T

A
.
P
Du
.
a phu
.
o
.
ng tr`nh to

ng quat cu a cac du
.
`o
.
ng ba
.
c hai ve
`
da
.
ng chnh
ta

c v`a nha
.
n da
.
ng ch ung.
1. 3x
2
2xy + 3y
2
+ 2x 4y + 1 = 0.
(DS. Du
.
`o
.
ng elip, phu
.
o
.
ng tr`nh chnh ta

c
32
3
x

2
+
16
3
y

2
= 1)
2. x
2
+ 2xy y
2
6x + 4y 3 = 0.
(DS. Du
.
`o
.
ng hypecb on, phu
.
o
.
ng tr`nh chnh ta

c 2

2y

2
2

2x

2
= 1)
3. x
2
2xy +y
2
+ 4x 6y + 1 = 0.
(DS. Du
.
`o
.
ng parabon, phu
.
o
.
ng tr`nh chnh ta

c 2y

2x

= 0)
4. 2x
2
4xy y
2
+ 8 = 0
(DS. Du
.
`o
.
ng hypecb on, phu
.
o
.
ng tr`nh chnh ta

c
x

2
2
2

y

2
(
_
8/3)
2
= 1)
5. 5x
2
+ 4xy + 5y
2
9 = 0.
(DS. Du
.
`o
.
ng elip, phu
.
o
.
ng tr`nh chnh ta

c
x

2
(3/

7)
2
+
y

2
(

3)
2
= 1)
6. 11x
2
+ 24xy + 4y
2
15 = 0.
(DS. Du
.
`o
.
ng hypecb on, phu
.
o
.
ng tr`nh chnh ta

c
x

2
(

3/2)
2

2
(

3)
2
= 1)
7. 2x
2
+ 4xy + 5y
2
24 = 0.
(DS. Du
.
`o
.
ng elip, phu
.
o
.
ng tr`nh chnh ta

c
x

2
(

24)
2
+
y

2
2
2
= 1)
8. x
2
8xy + 7y
2
36 = 0.
(DS. Du
.
`o
.
ng hypecb on, phu
.
o
.
ng tr`nh chnh ta

c
x

2
2
2

y

2
6
2
= 1)
Du
.
a phu
.
o
.
ng tr`nh to

ng quat cu a cac ma
.
t ba
.
c hai ve
`
da
.
ng chnh
ta

c v`a nha
.
n da
.
ng ch ung.
276 Chu
.
o
.
ng 6. Da
.
ng to`an phu
.
o
.
ng v` a u
.
ng du
.
ng
9. 6x
2
2y
2
+ 6z
2
+ 4xz + 8x 4y 8z + 1 = 0.
(DS. Du
.
`o
.
ng paraboloid mo
.
t ta
`
ng;
x

2
(
_
5/4)
2
+
y

2
(
_
5/8)
2

2
(
_
5/2)
2
= 1)
10. 4x
2
+ 3y
2
+ 2z
2
+ 4xy 4yz + 4x 2y 4z 3 = 0.
(DS. Ma
.
t tru
.
eliptic;
x

2
(

2)
2
+
y

2
1
= 1)
11. x
2
+ 2y
2
3z
2
+ 2x + 8y + 18z 54 = 0.
(DS. Hypecboloid 1-ta
`
ng;
x

2
36
+
y

2
18

Z

2
12
= 1)
12. 2x
2
+y
2
4xy 4yz = 0.
(DS. Ma
.
t non,
x

2
4
+z

2
=
y

2
2
)
13. 2x
2
+ 2y
2
+ 3z
2
+ 4xy + 2xz + 2yz 4x + 6y 2z + 3 = 0.
(DS. Ma
.
t paraboloid eliptic, 2x

2
+ 5y

2
5

2z

= 0)
14. 2x
2
+ 2y
2
+ 3z
2
2xz 2yz 16 = 0
(DS. Ma
.
t elipxoid,
x

2
4
2
+
y

2
(2

2)
2
+
z

2
2
2
= 1)

You might also like